Chemestry For JEE PDF

You might also like

Download as pdf or txt
Download as pdf or txt
You are on page 1of 322

Copyright of this book is reserved by Gujarat Secondary and Higher Secondary Education

Board, Gandhinagar. No reproduction of this book in whole or in part, or in any form is


permitted without written permission of the Secretary, Gujarat Secondary and Higher
Secondary Education Board, Gandhinagar.

Gujarat Secondary and Higher


Secondary Education Board,
Gandhinagar

QUESTION BANK
CHEMISTRY

Price : ` 70.00
Published by :
Secretary
Gujarat Secondary and Higher Secondary Education Board,
Gandhinagar

I
Contribution
1 Dr. Hasmukh Adhiya (IAS) Principal Secretary , Education Department Gandhinagar

2 Shri R. R. Varsani (IAS) Chairman , G.S&H.S.E. Bord, Gandhinagar

3 Shri H. K. Patel (G.A.S) Dy. Chairman, G.S&H.S.E. Bord, Gandhinagar

4 Shri M. I. Joshi (G.E.S) Secretary , G.S&H.S.E. Bord, Gandhinagar

Coordination
1 Shri B. K. Patel O.S.D., G.S&H.S.E. Bord, Gandhinagar

2 Shri D. A.Vankar Assistant Secretary (Retd.), G.S&H.S.E. Bord, Gandhinagar

5 Shri M. P. Parmar Assistant Secretary, G.S&H.S.E. Bord, Gandhinagar

Expert Teachers
1. Shri C. I. Patel (Convenor) Shri Vidyanagar High School, Ahmedabad
2. Shri S. B. Gor (Co-Convenor) Ghyanda Girls High School, Ahmedabad
3. Shri A. I. Patel Navchetan High School, Ahmedabad
4. Shri V. R. Patel
5. Shri B. R. Patel Muktjeevan Vidhyalaya, Ahmedabad
6. Shri K. K. Purohit M. K. Higher Sec. School, Ahmedabad
7. Shri M. B. Patel New Vidhyavihar for Girls, Ahmedabad
8. Shri B. A. Nayak Swaminarayan High School, Ahmedabad
9. Shri H. M. Patel
10. Shri S. B. Suthar R.P.T.P. Science School, Vallabh Vidhyanagar
11. Shri R. N. Patel R.P.T.P. Science School, Vallabh Vidhyanagar
12. Shri N. N. Shah Best High School, Ahmedabad
13. Shri J. Y. Mehta
14. Shri I. B. Amlani
15. Smt. M. N. Shethiya
16. Smt. H. N. Nayak
17. Smt. P. S. Thakar R.P.T.P. Science School, Vallabh Vidhyanagar
18. Shri G. S. Patel
19. Shri M. L. Sharma
20. Shri H. K. Patel

II
P R E FA C E
Uptil now , the Students had to appear in various entrance examinations for
engineering and medical courses after std-12. The burden of examinations on the side of the
students was increasing day-by-day. For alleviating this difficulty faced by the students,
from the current year, the Ministry of Human Resource Development , Government of India,
has Introduced a system of examination covering whole country. For entrance to engineering
colleges, JEE(Main) and JEE(Advanced) examinations will be held by the CBSE. The
Government of Gujarat has except the new system and has decided to follow the examinations
to be held by the CBSE.

Necessary information pertaining to the proposed JEE (Main) and


JEE(Advanced) examination is available on CBSE website www.cbse.nic.in and it is requested
that the parents and students may visit this website and obtain latest information guidance
and prepare for the proposed examination accordingly. The detailed information about the
syllabus of the proposed examination, method of entrances in the examination /centers/
places/cities of the examinations etc. is available on the said website. You are requested to
go through the same carefully. The information booklet in Gujarati for JEE( Main) examination
booklet has been brought out by the Board for Students and the beneficieries and a copy of
this has been already sent to all the schools of the state. You are requested to take full
advantage of the same also However, it is very essential to visit the above CBSE website
from time to time for the latest information guidance . An humble effort has been made by
the Gujarat secondary and Higher Secondary Education Boards, Gandhinagar for JEE and
NEET examinations considering the demands of the students and parents , a question bank
has been prepared by the expert teachers of the science stream in the state. The MCQ type
Objective questions in this Question Bank will provide best guidance to the students and we
hope that it will be helpful for the JEE and NEET examinations.

It may please be noted that this Question Bank is only for the guidance of the
Students and it is not a necessary to believe that questions given in it will be asked in the
examinations. This Question Bank is only for the guidance and practice of the Students. We
hope that this Question Bank will be useful and guiding for the Students appearing in JEE and
NEET entrance examinations. We have taken all the care to make this Question Bank error
free, however, if any error or omission is found, you are requested to refer to the text
books.

M.I. Joshi R.R. Varsani (IAS)


Date: 02/ 01/ 2013 Secretary Chairman

III
INDEX
UNIT 1 SOME BASIC CONCEPTS IN CHEMISTRY 1

UNIT 2 STATES OF MATTER 20

UNIT 3 STRUCTURE OF ATOM 44

UNIT 4 CHEMICAL BONDING AND ATOMIC STRUCTURE 79

UNIT 5 CHEMICAL THERMODYNAMICS 96

UNIT 6 SOLUTIONS 120

UNIT 7 EQUILIBRIUM 151

UNIT 8 REDOX REACTIONS & ELECTROCHEMISTY 183

UNIT 9 CHEMICAL KINETICS 207

UNIT 10 SURFACE CHEMISTRY 230

UNIT 11 CLASSIFICATION OF ELEMENTS AND PERIODICITY


IN PROPERTIES 241

UNIT 12 GENERAL PRINCIPLES AND PROCESSES OF


ISOLATION OF METALS 263

UNIT 13 HYDROGEN 283

UNIT 14 S - BLOCK ELEMENTS 301

IV
UNIT : 1 SOME BASIC CONCEPTS IN CHEMISTRY
Important Points
[A] Important formulae :
mass
1. No.of moles
Molar mass
Volume at STP
2. No.of moles of gas
22.4
No.of Particles
3. No.of moles of Particles
6.022 1023
4. No. of moles of solute Molarity Vol ( L )
M.W.of salt
5. Eq. wt. of a salt =
Total + ve charge of metal ion
Atomic Weight
6. Eq. wt.of element
Valency
m a nb
7. Avg. at. mass
mn
where, a + b are atomic masses
and m + n are precentage.
n (at mass of element )
8. % of element in compound = M .W . of compound 100
where, n= No. of atoms of that element
w 1000
9. Molarity
M .W . V (ml )
w 1000
10. Normality
E.W . V (ml )
w 1000
11. Molality
MW Wo( g )
Wo = Weight of solvent
n
12. Mole fraction ( X )
nN
W 100
13. %W W
W Wo

weight (vol ) of solute 106


14. ppm
weight (vol ) of solution

1
15. Molecalar weight = 2 V.D.
Wt. of metal
16. Eq.wt of metal 1.008
wt of H 2 displaced
Wt. of metal 11200
17. Eq.wt of metal
Vol of H 2 displaced at STP (mL )
Wt. of metal 35.5
18. Eq.wt of metal
Wt of Chlorine combined
Wt. of metal 11200
19. Eq.wt of metal
Vol of Cl2 combined at STP (mL )
Wt. of metal 8
20. Eq.wt of metal
Wt of oxygen combined
Wt. of metal 5600
21. Eq.wt of metal
Vol of O2 displaced at STP (mL )
% W W density 10
22. Molority
Molecular weight
23. M 1 V1 M 2 V2 ( Molarity equation)
24. N1 V1 N 2 V2 ( Normality equation)
Molecular weight
25. n
Empirical formula Weight

0 9 0
26. F ( C ) 32
5
27. K 0 C 273.15
28. 1 L 1 dm 3 , 1 mL 1 cm 3
[B] Important Facts :
1. Antoine Lavoisier - Law of conservation of mass
2. Joseph proust - Law of definite proportions
3. John Dalton - Law of Multiple proportions
4. Richter - Law of combining weights.
5. Gay Lussac - Law of combining Volumes.
6. 1 amu = 1.6605 x 10-24 gram
12
7. Mass of C atom 1.9926 1023 gram
8. Avogadro number ( N A ) 6.022 10 23
9. AZT = Azido thymidine,drug used for aids victims.
10. The limiting reagent is the reagent that is entirely consumed when a reaction goes to completion. Its
amount limits the amount of the product formed.

2
[c] Precision and Accuracy.
The term precision refers for the closeness of the set of values obtained form identical measurements
of a quantity.
Accuracy refers to the closeness of a single measurement to its true value.
Let us take an example to illustrute. this. Three students were asked to determine the mass of a piece of
metal where mass is known to be 0.520g. Data obtained by each Student are recorded in table below
mesurements in g.
1 2 3 Average
Student A 0.521 0.515 .0509 0.515
Student B 0.516 0.515 .0514 0.515
Student C 0.521 0.500 .0520 0.520
The data for student A are neither, precise nor accurate. The data for student B are precise but not
accurate. The data for student C are both precise and accurate.

M.C.Q.
1. Identify the wrong statement in the follwing (AIEEE 2008).
(a) CFCs are responsible for ozone layer depletion.
(b) Greenhouse effect is responsible for global warming.
(c) Ozone layer does not permit I.R. radiation from the sun to reach the earth.
(d) Acid rain is mostly because of oxides of N and S.
2. In the reaction
2 Al( s ) 6 HCl( aq ) Al 3 ( aq ) 6Cl ( aq ) 3H 2( g ) (AIEEE 2007)

(a) 6L HC1 ( aq ) is consumed for every 3L, H 2( g ) produced.

(b) 33.6L H 2( g ) is produced regardless of temperature and pressure for every mole of Al that reacts.

(c) 67.2L H 2( g ) at STP, is produced for every mole Al that reacts.

(d) 11.2L H 2( g ) at STP, is produced for every mole HC1 ( aq ) consumed.


3. Consider a titration of potassium dichromate solution with acidified Mohrs salt solution using diphenyl
amine as indicator. The number of moles of Mohrs salt required per mole of dichromate is (IIT JEE
2007)
(a) 3 (b) 4 (c) 5 (d) 6
4. Which has maximum number of atoms ? (IIT JEE 2003)
(a) 24g of C (12) (b) 56g of Fe(56) (c) 27g of Al (27) (d)108g of Ag (108)
5. What volume of hydrogen gas at 273K and 1 atm pressure will be consumed in obtaining 21.6 g of
elemental boron (atomic mass = 10.8) form the reduction of boron trichloride by hydrogen (AIEEE
2003)
(a) 89.6L(b) 67.2L (c) 44.8L (d) 22.4L
6. In an organic compound of molar mass 108g/mol, C,H and N atoms are present in 9:1:3.5 by weight
Molecular formula can be (AIEEE 2002)
(a) C6 H 8 N 2 (b) C6 H10 N
(c) C5 H 6 N 3 (d) C4 H18 N 3

3
7. Number of atoms in 560 g of Fe (atomic mass = 56) is (AIEEE 2002)
(a) twice that of 70 g N. (b) half that of 20 g H
(c) both (a) and (b) (d) None of these
8. In the standardization of Na2 S 2O3 using K 2 Cr2 O7 by iodometry, the equivalent weight of K 2 Cr2 O7 is
(IIT JEE 2001)
Molar mass Molar mass
(a) (b)
2 6
Molar mass
(c) (d) same as molar mass
3
9. Mixture X=0.02 mole of [Co(NH3)5SO4] Br and 0.02 mole of[Co(NH3)5Br]SO4 was prepared in 2L of
Solution
1L of mixture X excess AgNO3 Y

1L of mixture X excess BaCl2 Z

Number of mole of Y and Z are (IIT JEE 2003)


(a) 0.01, 0.01 (b) 0.02, 0.01
(c) 0.01,0.02 (d) 0.02, 0.02
10. How many moles of electron weight one kilogram ? (IIT JEE 2002)
1
(a) 6.023 10 23 (b) 1031
9.108

6.023 1
(c) 1054 (d) 108
9.108 9.108 6.023
11. An Oxide of metal contains 60% of the metal. What will be the equivalent weight of the metal ?

(a) 12 (b) 40 (c) 24 (d) 48


12. A container is filled with 2L of water. What will be the volume of water in m3?
(a) 2 103 (b) 1 103 (c) 2 103 (d) 1 103
13. The mass of carbon -12 atom considered in the definition of a mole is
(a) 0.012Kg (b) 0.12g (c) 120 mg (d) None of these
14. The drug which is used for treating AIDS victims is
(a) Azidothymidine (b) Cis- platin
(c) Taxol (d) All of these
15. Chose the incorrect statement .
(a) The constituents of a compound cannot be separated into simpler substances by physical meth-
ods.
(b) An element is consists of only one type of particles and these particles may be atoms or molecules.
(c) The properties of a compound are same as its constituent elements.
(d) Atoms of different elements are different in nature.

4
16. Which of the following is a pair of physical and chemical property respectively of a substance ?
(a) acidity & combustibility (b) colour & density
(c) basicity & colour (d) density & acidity.
17. What is the symbol of S.I. unit for the amount of substance ?
(a) NA (b) n (c) mole (d) mol
18. What is the symbol of a multiple 109 ?
(a) G (b) E (c) n (d) Z
19. Find the correct relation.
5 o
(a) o F 9 (o C ) 32 (b) C ( F 32)
o

5 9
(c) Both & (a) and (b) (d) Neither (a) nor (b)
20. In chemistry a number is represented in the form N 10n . This method of expressing the number is
called scientific notation. What is the value of N here.
(a) 1 to 10 (b) 0.1 to 9.99
(c) 10 to 100 (d) Any value can be taken
21. What is the correct scientific notation for 0.00016 ?
(a) 1.6 10 4 (b) 16 10 5

(c) 0.16 103 (d)cannot be determined.


22. How many significant digits are there in 0.25 ?
(a) 1 (b) 2 (c) 3 (d) cannot be determined.
23. Which of the following number contains there significant digits ?
(a) 0.200 (b) 0.030 (c) 0.0052 (d) 0.002
2+
24. What is the number of neutrons in Zn ion
(Atomic mass namber = 70) (IITJEE 1979)
(a) 34 (b) 36 (c) 38 (d) 40
25. The same amount of Zinc is treated separately with excess of sulphuric acid and excess of sodium
hydroxide.
What will be the ratio of volumes of hydrogen evolved ? (IITJEE 1979)
(a) 1 : 1 (b) 1 : 2 (c) 2 : 1 (d) 9 :4
26. 2.76g of silver carbonate on being strongly heated yields a residue weighing (IITJEE 1979)
(a) 2.16g (b) 2.48 g (c) 2.32 g (d) 2.64 g
27. Find the total number of electrons in one molecule of carbon dioxide.
(a) 22 (b) 44 (c)66 (d) 88
28. A gaseous mixture contains oxygen and nitrogen in the ratio of 1 : 4 by weight. Therefore, the ratio of
their number of molecules is
(a) 1 : 4 (b) 1 : 8 (c) 7 : 32 (d) 3 : 16
29. Identify the incorrect unit conversion factor.

1cm 3 1cm 60s


(a) (b) (c) (d) None of these
1mL 10mm 1min

5
30. 90 g KClO3 on heating gives 2.96g KCl and 1.92g oxygen. Which of the follwing laws is illustrated by
this statement ?
(a) Law of definite proportion (b) Law of mass conservation
(c) Law of multiple proporation (d) Avogadros law.
31. Match the following property.
A B
(i) Law of Multiple proportions. (p) Richter
(ii) Law of Combining volumes (q) Proust
(iii) Law of Reciprocal proportions. (r) GayLussac
(iv) Law of Constant composition. (s) Dalton
(a) i - s, ii - p, iii - r, iv - q (b) i - s, ii - r, iii - p, iv - q
(c) i - s, ii - r, iii - q, iv - p (d) i - q, ii - r, iii - p, iv - s
32. Two oxides of a metal M contain 27.6% and 30.0% of oxygen respectively. If the formula of the first
oxide is M3O4 , find that of the second.
(a) M 2 O3 (b) M 2 O (c) MO2 (d) M 3O2
33. Naturally occuring Boron consists of two isotopes having atomic masses 10.01 and 11.01 respectively.
Calculate the percentage of both the isotopes in natural Boron (Atomic mass of natural Boron = 10.81)
(a) 20% and 80% (b) 80% and 20%
(c) 25% and 75% (d) 75% and 25%
34. Calculate the mass percent of Na and S in sodium sulphate.
(a) Na = 16.2%, S = 22.54% (b) Na = 32.39%, S = 11.26%
(c) Na = 22.54%, S = 32.39% (d) Na = 32.39%, S = 22.54%
35. Determine the empirical formula of an oxide of iron which has 69.9% iron and 30.1% oxygen by mass.
(a) FeO (b) Fe2O3 (c) Fe3O4 (d) Fe3O2
36. Calculate the amount of carbon dioxide that can be produced when 1 mole of carbon is burnt in 16 g of
dioxygen.
(a) 44g (b) 22g (c) 88g (d) 11 g
37. Calculate the concentration of nitric acid in moles per litre which has a density , 1.41 g/mL. %w/w of
nitric acid is
(a) 15.44M (b) 0.064M (c) 0.077M (d) 12.87M

38. In a reaction : N 2( g ) 3H 2( g ) 2 NH 3( g ) , 2000g N 2 reacts with 1000g H 2


which reactant will left unreacted ? How much ?
(a) N2 , 2428 g (b) H 2 , 428.6 g
(c) N 2 , 571.4 g (d) H 2 , 571.4 g
39. Calculate the number of sulphate ions in 100mL of 0.001M ammonium sulphate solution.
(a) 6.022 1019 (b) 6.022 1019
(c) 6.022 1020 (d) 6.022 1020
40. Calculate the molarity of a solution of ethanol in water in which mole fraction of ethanol is 0.040.
(a) 2.31M (b) 0.213M (c) 0.0213M (d) 23.1M

6
41. Some statements are given below based on the pictures. Identify true and false statements.

(P) (Q) (R)


(i) P and Q both indicates precision and accuracy.
(ii) Q indicates precision and accuracy white R indicates neither precision nor accuracy.
(iii) P indicates precision but not accuracy.
(iv) Q indicates both precision and accuracy
(a) FTTT (b) TTTT (c) TTFT (d)FTFT
42. The normality of 0.3M phosphorous acid is (IITJEE 1999)
(a) 0.1 (b) 0.9 (c) 0.3 (d) 0.6
43. An aqueous solution of 6.3g oxalic acid dihydrate is made upto 250 mL. The volume of 0.1 N NaOH
required to completely neutralize 10 mL of this solution is
(a) 40 mL (b) 20 mL (c) 10 mL (d) 4 mL
44. The pair of the compounds in which both the metals are in the highest possible oxidation state is
3 3
(a) Fe CN 6 , Co CN 6 (b) CrO2 Cl2 , MnO4

3
(c) TiO3 , MnO2 (d) Co CN 6 , MnO3
45. In the analysis of 0.0500 g sample of feldspar, a mixture of the chiorides of sodium and potassium is
obtained, which weighs 0.1180 g. Subsequent treatment of the mixed chlorides with silver nitrate gives
0.2451g of silver chloride. What is the percentange of a sodium oxide and potassium oxide in
feldspar ?
(a) 10.62% Na2O , 3.58% K 2O (b) 3.58% Na2O , 10.62% K 2O

(c) 10.62% Na2O , 35.8% K 2O (d) 35.8% Na2O , 10.62% K 2O


46. 5.5 g of a mixture of FeSO4.7H2O and Fe2 (SO4)39H2O requires 5.4 mL of 0.1N KMnO4 solution for
complete oxidation. Calculate the number of mole of Fe2 (SO4)39H2O in the mixture.
(a) 0.0095 (b) 0.15 (c) 0.0952 (d) 1.52
47. A compound contains 28% of nitrogen and 72% of a metal by weight. Three atoms of the metal
combine with two atoms of nitrogen. Find the equivalent weight of the metal.
(a) 12 (b) 24 (c) 36 (d) 48
48. The density of a 3M Na2|S2O3 solution is 1.25 g per mL, What is the molalities of Na+ and S2O32-ions ?
(a) 3.865, 7.732 (b) 7.732, 3.865 (c) 1.933, 7.732 (c) 7.732, 1.933

7
49. Haemoglobin present in blood contain 3.72% by mass iron. Calculate the number of iron atoms in 2.0g
of haemiglobin.
(a) 4.53 X 1026 (b) 4.53 X 1023 (c) 5.95 X 1019 (d) 8 X 1020
50. How many moles of magnesium phosphate, Mg3(PO4)2 will contain 0.25 mole of oxygen atoms (AEEE
2006)
(a) 0.02 (b) 3.125 x 10-2 (c) 1.25 x 10-2 (d) 2.5 x 10-2
51. The unit J Pa -1 is equivalent to
(a) m3 (b) cm3 (c) dm3 (d) none of these
-3
52. The density of Al metal is 2.7 gcm . An irregularly shaped piece of aluminium weighing 40.0g is added
to a 100mL graduated cylinder containing 50.0mL of water. upto what height the water level will rise in
the cylinder ?
(a) 14.8mL (b) 79.6mL (c) 64.8mL (d) 50mL
53. A sample of clay after drying partially was found to contain 50% silica and 7% water.The original
sample of clay had 12% water, What is the percentage of silica in the original sample?
(a) 50% (b) 5% (c) 43% (d) 47%
54. In which of the following pairs percent compostion of element is not same ?
(a) benzene and ethyne (b) But - 2 - ene and Cyclobutane
(c) glucose and fructose (d) phenol and ethanol
55. What weight of CuO will be required to provide 200Kg copper
(a) 200Kg (b) 79.5Kg (c) 250Kg (d) 100Kg
56. Choose the proper option after studying following statement (T = True, F = False)
1. The percent composition of vinyl chloride and its polymer PVC are same.
2. The perecent compostion of phosphorous trioxide (P2O3)is half than that of its dimer phosphorous
hexoxide (P4O6) for each of the elements present in them.
(a) T, F (b)F,T (c) T, T (d) F, F
57. Impure sample of ZnS contains 42.34% Zn. What is the percentage of pure ZnS in the smaple ?
(a) 67% (b) 63% (c) 58% (d) 37%
58. If the atomic mass of carbon were set at 50 amu, what would be the value of Avog adros number ?
(a) 5.01 x 1024 (b) 6.022 x 1023 (c) 1.66 x 1024 (d) none of these
59. For which of the following compounds molecular weigh cannot be determined from atomic weights ?

(a) Fe4 Fe CN 6 (b) TiO2

(c) TiO1.12 (d) none of these


60. Which one of the following contains greatest number of oxygen atoms ?
(a) 1.0g of O atoms (b) 1.0g of O2
(c) 1.0g of O3 (d) All have same number of atoms

8
61. Which of the following chemical equation is incorrectly balanced ?
(a) Sb2 S3 12 HCl 2H 3 SbCl6 3H 2 S

(b) 3IBr 4 NH3 NI3 3NH 4 Br

(c) 2 KrF2 2 H 2O 2 Kr 2O2 4 HF

(d) PCl3 3 H 2 O H 3 PO3 3 HCl


62. Match the following
Column -I Column -II
(i) Cl2O3 (P) basic anhydride
(ii) Li2O (Q) acid anhydride
(iii) CO2 (R) base
(a) (i) - (q) (ii) - (p) (iii) - (q)
(b) (i) - (r) (ii) - (q) (iii) - (p)
(c) (i) - (p) (ii) - (q) (iii) - (r)
(a) (i) - (p) (ii) - (q) (iii) - (p)
63. How many g of NaOH can be obtained by reaction of 1 Kg of Na2CO3 with Ca(OH)2 ?
(a) 106 g (b) 850 g (c) 755g (d) 943 g
64. How much calcium oxide (CaO) can be obtained by heating 200 Kg of lime stone theat is 95% pure
CaCO3 ?
(a) 56Kg (b) 190Kg (c) 170Kg (d) 107Kg
65. Calculate the amount of NaOH required to neutralize 100 mL 0.1M H2SO4.
(a) 40g (b) 0.4 g (c) 80 g (d) 0.8 g
66. 3g of an oxide of a metal is converted into chloride and it yielded 5 g of chloride. Find the equivalent
weight of the metal.
(a) 33.25 (b) 3.325 (c) 12 (d) 20
67. A compound contains two oxygen atoms, four carbon atoms and number of hy drogen atoms is double
of carbon atoms. What is the density of vapour of this compound ?
(a) 88 (b) 44 (c) 132 (d) 72
68. The number of molecules in 100 mL of each of O2,NH3 and CO2 at STP are
(a) CO2 O2 NH 3 (b) NH 3 O2 CO2

(c) NH3 = CO2 < O2 (d) NH 3 O2 CO2


69. Which of the following represents the formula of a compound which contains 26% nitrogen and
74% oxygen ?
(a) N2O (b) NO (c) NO2 (d) N2O5
-1
70. NKg is the unit of
(a) momentum (b) velocity (c) Pressure (d) accelaration

9
71. Which one of the following statements is incorrect ?
(a) All elements are homogeneous system
(b) Compounds made up of a number of elements are heterogeneous.
(c) A mixture is not always heterogeneous
(d) Smoke is a heterogeneous mixture.
72. A balanced chemical equation is in accordance with
(a) Avogadros law.
(b) Law of constant proportions
(c) Law of conservation of mass
(d) Law of gaseous volumes.
73. The atomic weights of two elements X and Y are 20 and 40 respectively. If a gm of X contains b
atoms, how many atoms are present in 2a gm of Y ?

a
(a) b (b) a (c) 2b (d) ( )
2
74. If the components of air are N2 ,78%, O2, 21% ; Ar, 0.9% and CO2, 0.1% by volume, what will be the
molecular weight of air ?
(a) 28.9 (b) 32.4 (c) 16.4 (d) 14.5
75. Calculate the molarity of a solution obtained by mixing 50mL of 0.5M H2SO4 and 75 mL of 0.25M
H2SO4.
(a) 0.375M (b) 0.35M (c) 0.045M (d) 0.45M
76. Which of the following has the highest normality ?
(a) 1M H2SO4 (b) 1M H3PO3 (c) 1M H3PO4 (d) 1M HNO3
77. In an experiment, 4 gm of M2Ox oxide was reduced to 2.8 gm of the metal. If the atomic mass of the
metal is 56 gm/mol, the number of oxygen atoms in the oxide is (AFMC 2010)
(a) 1 (b) 2 (c) 3 (d) 4
78. Match the following
Column - I Column - II
(i) femto (P) 109
(ii) yotta (q) 10-15
(iii) giga (r) 10-18
(iv) atto (s) 1024
(a) i - q, ii - p, iii - r, iv - s (b) i - s, ii - q, iii - p, iv - r
(c) i - q, ii - s, iii - p, iv - r (d) i - r, ii - s, iii - p, iv - q
79. The total number of atoms of all elements present in mole of ammonium dichromate is
(a) 19 (b) 6.023 x 1023 (c) 114.47 x 1023 (d) 84 x 1023
80. 0.32 gm of a metal on treatment with an acid gave 112 mL of hydrogen at STP. Calculate the equivalent
weight of the metal
(a) 58 (b) 32 (c) 11.2 (d) 24

10
81. For a reaction A + 2B C, the amount of C formed by starting the reaction with 5 moles of A and 8
moles of B is
(a) 5 moles (b) 8 moles (c) 16 moles (d) 4 moles
82. 100 mL of PH3 on heating forms P and H2 . The volume change in the reaction is
(a) an increase of 50 mL (b) an increase of 100 mL
(c) an increase of 150 ml (d) a decrease of 50 mL
83. An organic compound made of C, H, and N contains 20% nitrogen. Its molecular weight is (WBJEE
2009)
(a) 70 (b) 140 (c) 100 (d) 65
84. Volume occupied by one molecule of water (d = 1 gm cm -3) is
(a) 9 x 10-23cm3 (b) 6.02 x 10-23cm3 (c) 3 x 10-23cm3 (d) 5.5 x 10-23cm3
85. Calculate the number of moles in 1m3 gas at STP.
(a) 4.46 (b) 44.6 (c) 446 (d) 4460
86. An ore contains 1.24% of the mineral argentite Ag2S by mass. How many grams of this ore would have
to be processed in order to obtain 1.0 g of pure solid silver ?
(a) 23.15 g (b) 69.45 g (c) 92.6 g (d) 46.3 g
87. Find the electric charge in couloumb of 9.0 gm of A13+ ions.
(a) 9.6 x 104 (b) 6.9 x 104 (c) 2.9 x 105 (d) 4.80 x 10-19
88. Which of the following is not a homogeneous mixture ?
(a) smoke (b) air (c) Brass (d) Aqueous solution of sugar
89. Which of the following has the largest number of atoms ?
(a) 0.5g atom of Cu (b) 0.635g of Cu
(c) 0.25 moles of Cu atom (d) 1 g of Cu
90. 27 g of A1 (at mass = 27) will react with oxygen equal to (IIT 1978)
(a) 24 g (b) 8 g (c) 40 g (d) 10 g
91. Two containers P and Q of equal volumes contain 6 g of O2 and SO2 respectively at 300K and 1
atmosphere. Then
(a) No. of molecules in P is less than that in Q
(b) No. of molecules in Q is less than that in P
(c) No. of molecules in P and Q are same.
(d) cannot be determined
92. Which of the following pairs of substances illustrates the law of multiple proportions ?
(a) CO and CO2 (b) NaCl and NaBr
(c) H2O and D2O (d) MgO and Mg(OH)2
In each of the follwoing questions, two statements are given, one is Assertion (A) and the other
is Reason (R). Examine the statements carefully and mark the correct answer according to
the instructions given below :
(a) If both A and R are correct and R is the correct explanation of A.
(b) If both A and R are correct and R is not the correct explanation of A.
(c) If A is correct R is wrong.
(d) If both A and R are false.

11
93. A : Normality of 0.1M H2SO4 is 0.2N.
R : H2SO4 is a dibasic acid.
94. A : 1 Gram molecule of sulphar also represents 1 gram atom of sulphur.
R : Atomicity of sulphur is one.
95. A : In the equation NH 3 HCl NH 4 Cl , Gay-Lussacs law is not applicable to NH4Cl.
R : NH4Cl is not a gas,.
96. A : Atomic mass of sodium is 23 u.
R : An atom of sodium is 23 times heavier than an atom of 12C.
97. A : Pure water, irrespective of its source always contain hydrogen and oxygen in the ratio 1 : 8 by mass.
R : Total mass of reactants and products remains constant during physical or chemical change.
98. A : Mass numbers of most of the elements are fractional.
R: Mass numbers are obtained by comparing with mass number of 12C.
99. A : The mass of the products formed in a reaction depends upon the limting reactant.
R: Limting reactant reacts completely in the reaction.
100. A : Cinnabar is a chemical compound whereas brass is mixture.
R : Cinnabar always contain 6.25 times mercury than sulplur by weight. Brass can have any proportion
of Cu and Zn.
101. A : 1 L of O2 and 1 L of O3 contains the same number of moles under identical conditions.
R : Under identical conditions, 1 L of O2 and 1 L of O3 contain the same number of oxygen atoms.
102. A : The standard unit for expressing atomic mass is amu.
R : Now a days amu is represented by u.

ANSWER KEY

1c 2d 3d 4a 5b 6a 7c 8b 9a 10 d 11 a 12 c 13 a 14 a 15 c

16 d 17 d 18 a 19 d 20 a 21 a 22 b 23 a 24 d 25 a 26 c 27 a 28 c 29 d 30 b

31 b 32 a 33 a 34 d 35 b 36 b 37 a 38 d 39 b 40 a 41 a 42 d 43 a 44 b 45 b

46 a 47 a 48 b 49 c 50 b 51 a 52 c 53 d 54 d 55 c 56 a 57 b 58 d 59 d 60 d

61 c 62 a 63 c 64 d 65 d 66 a 67 b 68 d 69 d 70 d 71 b 72 c 73 a 74 a 75 b

76 c 77 c 78 c 79 c 80 b 81 d 82 a 83 a 84 c 85 b 86 c 87 a 88 a 89 a 90 a

91 b 92 a 93 a 94 d 95 a 96 c 97 b 98 d 99 a 100 a 101 c 102 b

12
SOLUTIONS/HINTS
3. (d)
The redox reaction between potassiumdichromate and Mohrs salt is :
6 Fe2 Cr2 O7 2 14 H 6 Fe3 2Cr 3 7 H 2 O
4. (a)
Number of particles Number of moles
24
No. of moles of carbon 2
12
5. (b)
2 BC 3 3H 2 2 B 6 HC
2 mol 3 mol 2 mol
= 21.6 g
nRT 3 0.0821 273
V 67.2 L
P 1
6. (a)
9
mass of carbon = 108 72 g
13.5
72
No.of carbon atoms 6
12
similarly, no of H and N atoms are 8 and 2 respectively.
8. (b)
During the reaction, Cr2 O7 2 changes to Cr3+ . Hence the change in oxidation number of Cr is 6.
Molar mass
Equivalent weight
6
9. (a)
In 2L solution, there are 0.02 mol Br - ions and 0.02 mole so4 2
2
1 L of mixture X contains 0.01 mol Br and 0.01 mol SO4 ions.
Hence, Y= 0.01 mol Ag Br
Z= 0.01 mol BaSO4
10 (d)
Mass of an electron 9.108 1031 Kg
1
No. of electron s in 1 Kg
9.108 10 31

1
31
9.108 10 6.023 1023 mol 1
108
mol
9.108 6.023

13
11. (a)
mass of metal = 60 g
mass of oxygen = 40 g
mass of oxygen = mass of metal
40 g = 60g
8 g = (?)
8 60
= 12
40
25. (a)
(i) Zn H 2 SO4 ZnSO4 H 2
(ii) Zn 2 NaOH Na2 ZnO2 H 2
26. (c)

Ag 2 CO3( s ) Ag 2 O( s ) CO2( g )
1 mol 1mol
0.01 mol 0.01 mol
There fore mass of residue (Ag2O) 0.01 molarmass of Ag 2O
0.01 232 2.32g
28. (c)
1
The ratio by weight =
4
1
28 7
32
Ratio of moles 4 4 32 32
28
32. (a)
Let x be the atomic mass of metal M
In the oixde M3O4, the mass of M = 72.4 and that of O = 27.6
M 72.4 O27.6
M 3 O4
x 16

72.4 27.6
: 3: 4
x 16

x 56
For second oxide, the mass of M = 70 and that of O =30
M 70 O30
56 16

M1.25 O1.875
M1 O1.5 OR M 2 O3

14
33. (a)
Let the % of isotope with atomic mass 10.01 be x
% of isotope with atomic mass 11.01 = 100-x
10.01x (100 x)11.01
Avg at mass = 10.81(Given)
100
37. (a)
69% w/w means 100 g nitric acid soution contain 69 g of nitric acid by mass.
69
moles of HNO3 1.095
63
100
Vol. of 100 g nitric acid solution 0.07092 L
1.41
1.095
moles per litre 15.44
0.07092
38. (d) N 2 3H 2 2 NH 3
28 g 6g
2000 g (?)
2000 6
428.6 g
28
But we are given 1000 g H2 There fore 1000 - 4286 =
571.4 g H2 will left.
39. 3(b)
No of moles of (NH4)2SO4 = molarity vol(L)
= 0.001 0.1 = 0.0001
2
No. of SO4 ions 0.0001 6.022 10 23 6.022 1019
40. (a)
n( ETOH ) n( ETOH )
X ETOH 0.04
nETOH n( H 2O ) n( ETOH ) 55.55

n( ETOH ) 2.31
42 (d)
O
phosphorous acid ( H 3 PO3 ) is a dibasic acid. Its structure is as follows :
Normality = basicity x Molarity P OH
H
= 2 x 0.3 = 0.6 OH
43. (a)
Equivalents of H 2 C 2O 4 . 2H 2 O in 10ml = Equivalents of NaOH
6.3 1, 0000
V
63 250 0.1
40 mL

15
44. (b)
The oxidation states of various metals are :
(a) Fe 3, Co 3
(b) Cr 6, Mn 7
(c) Ti 6, Mn 4
(d) Co 3, Mn 6
45. (b)
Suppose amount of NaCl in the mixture = x g
The amount of KCl in the mixture = (0.118 - x) g
NaCl + AgNO3 AgCl + NaNO3
58.5 143.5
143.5 x
x g.........................(i )
58.5
143.5 (0.118 x )
Similarly AgCl obtained from KCl = g .........................(ii )
74.5
But (i) + (ii) = 0.2451 g (Given)
Amount of NaCl = 0.0338 g
Amount of KCl = 0.0842 g
Now, 2NaCl = Na 2 O
117 62
0.0338 62
0.03380.0338 0.0179 g
117
0.0179 10 0
% of Na2 O 3.58% .....
0.5
46. (a)
5.4 0.1 278
Weight of FeSO4 7 H 2O 0.150 g
1000
5.35
Moles of Fe2. (SO4 )3 . 9 H 2O 0.0095
562
47. (a)
Atomic Weight
Equivalent weight
Valency
48. (b)
1000 M
m , M Molarity of solution
1000d MM s
d density of solution
M s Molar mass of solute
1000 3
3.865
1000 1.25 3 158

16
51. (a)
J Work Nm
2
m3
Pa Pr essure Nm
52. (c)
m
d
v
m 40
V 14.8cm3
d 2.7
Water level in cylinder = 50+ 14.8 = 64.8 mL
65. (d)
0.1MH 2 SO4 1000mL solution contains 0.1 mol H 2 SO4
100mL solution contains 0.01 mol H 2 SO4
mass of H 2 SO4 0.01 98 0.98 g

2 NaOH H 2 SO4 Na2 SO4 2 H 2O


2(40) g 98 g
(?) 0.98 g
80 0.98
0.8 g
98
66. (a)
Wt. of metal oxide Eq. wt. of metal Eq. wt. of oxide

Wt. of metal Choride Eq. wt. of metal Eq. wt . of Choride
3 E 8
E 33.25
5 E 35.5
67. (b)
M .F . C4 H 8O2

Molar mass 88
88
Vapour density 44
2
68. (b)
Equal volumes under identical conditions contain equal no. of molecules

69. (d)
N 26 O74 N1.85 O4.625 N 2O5
14 16

70. (d)
F ma
f N
a NKg 1
m Kg
17
73. (a)
a
No of moles of X
20
a
No.of atoms of X N b ( given )
20
20b
a
N
2a
No.of moles of Y
40
2a
No.of atoms of Y N
40
2 20b
N b
40 N
74. (a)
78 28 21 32 0.9 40 0.1 44
Mol. wt. of air
78 21 0.9 0.1
75. (b)
No.of moles of 0.05L H2SO4 = 0.5 x 0.05 = 0.025
No.of moles of 0.075L H2SO4 = 0.25 x 0.075 = 0.01875
Total no. of moles = 0.025 + 0.01875 = 0.04375
Total vol = 0.05L + 0.075L = 0.125L
0.04375
Molarity 0.35M
0.125
77. (c)
1 Mol M2Ox = (2 56 + 16x) gm
Now, (2 56 16 x) gm of oxide 112 gm metal
112 4
4 gm of oxide gm metal
112 16 x
112 4
But 2.8 ( given ) x = 3
112 16 x
79. (c)
Molecular formula of ammonium dichromate is
( NH 4 )2 Cr2O7
80. (b)
wt of metal 11200
Eq. wt of metal
vol.of H 2 in ml displaced at STP

18
82. (a)
2 PH 3( g ) 2 P( s ) 3H 2( g )
2mL 3mL
100ml (?)
100 3
150ml
2
Increase 50mL

85. (b)
1m3 1000 L
At STP, 22.4 L 1mol
1000
1000 L 44.6
22.4

86. (c)

Ag 2 S Ag ore Ag 2 S
248 g 216 g 100 g 1.24 g
(?) 1.0 g (?) 1.148 g
1.148 g 92.58 92.6 g

87. (a)
9
No. of moles of Al 3 0.33
27
No. of Al 3 ions 0.33 6.022 1023 2 1023
Electric charge 3 1.602 1019 10 23 9.6 Coulomb

19
UNIT : 2 STATES OF MATTER
Important Points
The group of molecules is called matter. Matter is made up of small particles. Matter is in three states,
Solid, liquid and gas. The other two states are known as Plasma and Bose, Einstein condensate. The
physical state of the matter changes by changing temperature. The physical properties of a subtance are
changed by changing its physical state but the chemical properties do not change, sometimes the rate of
chemical reaction changes by changing the physical state. During the chemical calculation, it is most
essential to have the information about the physical state of substances (reactant or product) and hence
it is essential to study the physical state of matter, factors affecting and related some important laws. The
deciding factors of the physical state of matter are intermolecular forces, molecular interaction and the
effect of thermal energy on the motion of particles.
The Dutch scientist van der Waals suggested that the weak forces of attraction exist between the
molecules, which cannot be explained by any other chemical attraction is known as van der Waals
0
attractive forces. This force is universal. This force of attraction is exerted upto 4.5 A distance in sub-
stance. van der Waals forces depend upon the shape of molecules, number of electrons present in
molecules, contact surface of molecules and average intermoleculer distance. The van der Waals forces
of attraction are different like (i) Dispersion forces or London forces. (2) Dipole-dipole forces and (3)
Dipole-induced dipole forces.
Dispersion forces of attraction was first of all proposed by the German scientist Fritz London so it is
known as London forces. This type of force of attraction is observed in atoms or molecules, there is a
temporary dispersion in electron density that affect the electron density of nearby atom or molecule so
the force of attraction is developed and so such effect is called dispersion force. The dipole-dipole forces
are observed in permantently dipolar molecules. Such dipolar molecules also have interactive London
forces so the cummulative effect of both forces are observed. The dipole-dipole force is stronger than
London forces. The dipole-induced dipole forces are observed when dipolar molecules come near to non-
polar molecules. This type of molecules also have London forces and hence the cumulative effect of both
forces are observed. The hydrogen bonding is an important intermolecular force. The first elements of
groups 5, 6 and 7 due to their high electronegativity combine with hydrogen to form hydride compounds,
in which hydrogen bond is observed. There also exists an intermoleculer repulsive forces; and based on
that the effect of pressure on solid, liquid and gaseous state explained very easily. The most important
factor which decides the physical, state of matter is the effect of thermal energy, on motion of molecules
due to this motion of molecules or atoms the energy produced is called thermal energy to keep the
molecules near to each other while the thermal energy has tendency to keep the molecules away from
each other. By balancing combination of the two opposite factors, the physical state of matter as solid,
liquid or gas is decided. Due to weak forces of attraction between molecules of gaseous state have some
characteristics. The behaviour of gas is described by the quantitative relation between mass, volume,
temperature and pressure and these relations are discovered by experimental observations and such
relations are called laws of gases. The relation between pressure and volume of a gas was studied
and it is known as Boyles law. At constant temperature for a fixed amount gas, pressure (P) varies
inversely with its volume (V). Mathematically the Boyles law is written as PV = K or P1V1 = P2V2 .
The equation d/P = K devised from Boyles law where d is the density. The Kelvin temparature is
accepted as an SI unit. The relation T = (t + 273.15) K is obtained. On the basis of experimental
observations a relation between absolute temperature and volume is obtained, which is known as Charles

20
V V1 V2
law. Mathematically it is written as = K or = . The relation between pressure and absoulte
T T1 T2
temperature (T) is obtained on the basis of experimental orbservations by scientist Gay Lussac and is
P P1 P2
known as Gay Lussacs law. Mathematically it is written as = K or = . The relation between
T T1 T2
volume of a gas and number of molecules was given by Avogadro, which is known as Avogadros law.
The mathematical form of it is V = K . n. The 00C or 273 K temperature and 1 bar pressure is accepted
as a standared value by SI system and hence these values are known as standard temperature and
pressure (STP). 1 mole of gas at STP is having volume 22.4 litre and number of molecules equal to 6.022
1023 known as molar volume and Avogadros number respectively. Combining Boyles law and Charles
PV P1 V1 P2 V2
law, the relation obtainged = K or = is known as combined gas equation. The ideal
T T1 T2
gas equation, PV = nRT is also known as equation of state and R is called universal gas constant which
has different values in different units. The real gas behaves as ideal gas at high temperature and low
pressure and are called ideal gases. The behaviour of real gas is deviated from ideal gas and its study
came from the study of effect of pressure and temperature and so the ideal gas equation is written as
an 2
P + 2 (V - nb) = nRT and this equation is also known as van der Waals equation. The gas can be
V
liquefied by lowering the temperature and increasing pressure at which gas get liquified is known as
critical temperature (TC) and critical pressure (PC) respectively and at critical temperature and critical
pressure the volume occupied by 1 mole of gas is called critical volume (VC) and this state is called
critical state. The PC, TC and VC values are constant so they are known as critical constants. The
liquefication of gas is explained by isotherm. Maxwell and Boltzmann had studied the distriubution of molecules
between different possible and plotted graph which is known as Maxwells distribution curve.
The total pressure of the mixture of two or more than two gases is obtained by the Daltons law. Total
pressure (P) = pA + pB + pC + pD .... and the partial pressure (p) is calculated from total pressure by
equation p1 = X1 Ptotal. If the % by volume is given then the partial pressure of gas is caculated using
equation.
% by volume of gas A total pressure
Partial pressure p A = . The Grahams law of gaseous diffusion is
100
1
r and using formula the ratio of rate of diffuson of NH3 and HCl gas was obtained practically as
d
1.46 + .01. The application of Grahams law of gaseous diffusion are as given in the text. The Avogadros
hypothesis is useful to calculate the number of molecules, atoms and total number of atoms in given amount
of gas.
The liquid state has its physical properties like fixed volume, fluidity, non- compressibility, diffusion,
evaporation, vapour pressure, surface tension and viseosity.

21
M.C.Q.
1. What is value of Boyles temperature of ethane gas when a= 5.489 dm6 atm mol-2 and b = 0.0638 dm3
atm mol-1
(a) 1048K (b) 104.8K (c) 209.6K (d)290.6K
2. The value of universal gas constant R depends upon the
(a) Temperature of the gas (b) Volume of the gas
(c) Number moles of the gas (d) none of these
3. The Boyles temperature for the ideal gases is given by

a a 2a
(a) (b) (c) (d) none of these
R bR bR

4. The ratio of van der Waals constants a and b has the dimensions of
(a) atm mole-1 (b) L mole-1 (c) atm . L mole-1 (d) atm mole-2
5. A gas expanse through a porons plug and exhibits cooling if its temperature is
(a) More than inversion temperature (b) Less than inversion temperature
(c) Less than critical temperature (d) Less than Boyles temperature
6. A gas behaves like an ideal gas at
(a) High pressure and low temperature (b) High pressure and high temperature
(c) At low pressure and increasing in volume (d) Decreasing velocity by lowering temperature
7. To which of the following gaseous mixture is Daltons law not applicable?

(a) He + Ne + SO2 (b) NH3 + HCl + HBr (c) O2 + N 2 + CO2 (d) N 2 + H 2 + O2

8. The degree of dissociation of cl2 at 1500K is 0.45 according to the reaction Cl2 2Cl assumig that
both Cl2 and Chlorine atoms behave like ideal gases, calculate the density of the mixture if the pressure
of the mixture is 1.5 atm
(a) 0.596 g. l-1 (b) 0.496 g. l-1 (c) 0.696 g. l-1 (d) 0.396 g. l-1
9. A gas is kept at 1 atm pressure. To compress it to 1/4th of its initial volume, the pressure to be applied is
1
(a) 1 atm (b) 2.0 atm (c) 4.0 atm atm (d)
4
10.The density of a gas at 300K and 1 atm is d pressure remaining constant, at which of the following
temperatures will its density become. 0.75 d ?
(a) 200 C (b) 300 C (c) 400K (d) 300K
11. A mixture contains N2O4 and NO2 in the ratio 2 : 1 by volume. The vapour density of the mixture is
(a) 45.4 (b) 49.8 (C) 32.6 (d) 38.3

22
12. At extremely low pressure, the vander waals equation for one mole of a gas may be written as

a a
(a) PV =RT + pb (b) PV = RT - (c) PV = RT (d) ( p + )(v - b) = RT
v v

13. The compressibility of a gas is less than unity at STP Therefore


(a) Vm > 22.4 L (b) Vm < 22.4 Litre (c) Vm = 22.4 L (d) Vm = 44.8 Litre
14. The correct order for magnitude of vanderwaals constant b should be

(a) C2 H 6 < CO < CO2 < He (b) CO < C2 H 6 < He < CO2

(c) C2 H 6 < CO2 < CO < He (d) He < CO < CO2 < C2 H 6

15. The molecular radius of O2 is 2.88 x 10-10 m calculate the excluded volume per mol of O2
(a) 0.24 dm3 (b) 0.48 dm3 (c) 0.024 dm3 (d) 0.048 dm3
16. An ideal gas can not be liquefied because
(a) The intermolecular force of attraction between the gaseous molecules are negligible.
(b) Its critical temperature is very high
(c) The vanderwaals constants a and b are very high
(d) Of all of these
17. The values of a for the gases O2 , CO2, N2 and CH4 are respectively 1.36, 3.64, 1.39 and 2.253 L2 atm
mol-1 which gas can be most easily liquefied ?
(a) O2 (b) CO2 (c) N2 (d) CH4
18. The rate of diffusion of H2 is about

(a) 1 2 that of He (b) 1.4 timesthat of He (c) Twice that of He (d) 4 times that of He

19. Most probable speed, average speed, and RMS (root mean square speed) are related as
(a) 1 : 1.224 : 1.128 (b) 1.128 : 1 :1.224
(c) 1 : 1.128 : 1.224 (d) 1.224 : 1.128 : 1.0
20. At room temperature the mixture of SO2 and O2 gas, compared to theO2 molecule, the SO2 molecule
will hit the wall with..
(a) Smaller avarage speed (b) Greater average speed
(c) Greater kinetic energy (d) Greater mass
21. Which has maximum value of mean free path ?
(a) CO2 (b) H2 (c) O2 (d) N2

23
22. 4.0 gm ideal gas is filled in a bulb having volme 10 dm3 at a constant temperature T & constant
pressure P. If 0.8 gm gas is removed from the bulb to maintain the original pressure at (T + 125)K
temperature, what would be the value of T for a gas having molar mass 40 gm mole-1.
(a) 500K (b) 5000C (c) 773K (d) 7730C
23. What would be vlaue of ratio for RMS and average speed of gaseous molecules at a constant
temperature ?
(a) 1.086 : 1 (b) 1 : 1.086 (c) 2 : 1.086 (d) 1.086 : 2
Hint - 2 2
Pressure and Temperature are constant

So : n1T1 = n2T2
W1 W
T1 = 2 T2 M1 = M 2
M1 M2
W1T1 = 3.2(T + 125)
4T = 3.2T + 400
\ 0.8T = 400
\T = 500 K
24. If temperature T2 > T1, which graph of maxwell Botlzmamn distriburibution of molecular speed is
correct ?

T1
T1 T2
N N T2
(a) (b)

O O
Molecular Speed Molecular Speed

T2 T2
T1
N N

T1
(c) (d)
O O
Molecular Speed Molecular Speed

25. The RMS velocity of an ideal gas at constant pressure varies with density relates as

1
(a) d (b) d (c) d 2 (d)
d

24
26. The ratio of rms (root mean square ) velocities for two different gases is

V1 M2 m1 m2 1 2 M2 M1
(a) = (b) M = (c) M = (d) =
V2 M1 2 M1 2 M1 m1 m2

27. At constant temperature, a gas is filled at 1 atm pressure in a closed container. To compress this gas to
1
th of its intial volume, the pressure to be applied is
4

4 1 th 1
(a) atm (b) 2 atm (c) 4 atm (d) atm
3 4 3
28. What is the pressure of 380 mm Hg column of a gas in pascal ?
(a) 5.05 x 104 Pa (b) 5.06 x 105 Pa (c) 0.505 x 103 Pa (d) 1.013 x 105 Pa
29. The graphs plotted V T for one mole of an ideal gas as follows , which graph represent its ideal
behaviour at atmospheric pressure ?

(a) (b) V (L)


V (L)
) K)
L, 373K 3 73
(22.4L, 273K) (38.8 8.8 L
,
(3
(22.4L, 273K)

T (K) T (K)

(c) V (L) (d) V (L) (14


373K) .2L
, 37
,
.6L 3K)
(30
(22.4L, 273K) (22.4L, 273K)

T (K) T (K)

30. Helium gas is compressed to half of the volume at 303 K. It should be heated to which temperature for
its volume to increase to double of its original volume ?
(a) 303K (b) 606K (c) 1212K (d) 300C
31. When a gas is heated from 298K to 323K at a constant pressure of 1 atm its volume is
(a) Increases from V to 1.8 V (b) Increases from V to 1.08 V
(c) Increases from V to 1.5 V (d) Increases from V to 2 V

25
Hint 1
380
380mm = 380 torr = atm
760
\ 0.987 atm = 105 Pa
380
\ atm = (?) \ 380 mm = 5.05 104 Pa
760

Hint 2
At const pressure according to charles law

V (L) V1 V2
= =K
) T1 T2
, 3 73K
6L
(22.4L, 273K) (30. \
22.4
= 0.082 ............( I )
273
30.6
and = 0.082............( II )
373
T (K)

(I) and (II) constant behaves the gas as an ideal


Hint - 3 : According to Charles law
V1 V2 V
= =K V = 1 at T = 303 K
T1 T2 1 2 1

V T
\T2 = 2 1 V2 = 2V1 at T2 = ?
V1
2V 303
= 1 = 303 4 = 1212K
v1
2
Hint 4 : According to Charles law
V1 V2
=
T1 T2
V1 V
= 2
298 323
323
\V = V = 1.08 V
2 298 1 1

32. At 200 and 760 torr, the sample of air contains 20% O2 & 80% N2 gaseous mixture, find the density of
the air (Molarmass O2 = 32 g/mol , N2 = 28 g/mole R =0.082 liter mole-1 K-1)
(a) 1.918 gL-1 (b) 2.198 gL-1 (c) 1.198 gL-1 (d) 1.394 gL-1
33. The ratio of velocities of diffusion of gas A and B is 1 : 4, if the ratio of their masses in a mixtare is 2:3,
calculate the ratio of their mole fractions (BIT 1990)
(a) 1 : 12 (b) 1 : 24 (c) 1 : 6 (d) 4 : 3

26
34. Under same conditions of temperature and pressure the volumes of 14g N2 and 36g of O3 are
related as :

(a) 2VN2 = 3Vo3 (b) 3VN2 = 2Vo3 (c) 3VN2 = 4Vo3 (d) 4VN2 = 3Vo3

35. Equal masses of Hydrogen and oxygen gases are placed in a closed container, at a pressure of 3.4 atm.
The contribution of hydrogen gas to the total pressure is
(a) 1.7 atm (b) 0.2 atm (c) 3.2 atm (d) 3.02 atm
36. The kinetic energy of 4.0 moles of N2 gas at 1270C is (R =2 cal mole-1K-1)
(a) 4400 cal (b) 3200 cal (c) 4800 cal (d) 1524 cal
37. The critical temperature of H2O is higher than CO2 because (IIT 1997)
(a) fewer electrons than CO2 (b) It contains 2- covalent bonds
(c) Molecular shape is V- shape (d) dipole moment of H2O
38. The compressibility factor of an ideal gas is
(a) zero (b) 1 (c) 2 (d)4
39. A 10 L cylinder of nitrogen at 4.0 atm pressure and 270C developed a leak. When the leak was
repaired 2.36 atm of nitrogen remained in the cylinder at 270C. How many grams of nitrogen escaped?
(a) 18.7g (b) 0.67g (c) 52.6g
(d) 10.0g
40. Calculate percentage of NO2 by weight in N2O4 which has vapour density of 36.
(a) 27.7% (b) 67.7% (c) 52.6% (d) 25.7%
41. Two gases A and B having the same temperature T, same pressure P and same volume V are mixed.
If the mixture is at the same temperature T and occupied a volume V, the pressure of the mixture is ..
(a) 2P (b) P (c) P/2 (d) 4P
42. The density of nitrogen is maximum at
(a) STP (b) 273K and 2 atm (c) 546K and 1 atm (d) 546 K and 2 atm
43. The temperature at which real gases obey the ideal gas laws over a wide range of pressure is called
(a) Critical temperature (b) Inversion of temperature
(c) Boyles temperature (d) Reduced Temperature
44. Which of the fallowing gases with have the highest rate of diffusion
(a) O2 (b) CO2 (c) NH3 (d) N2
45. At a given temperature Qx= 39y and My= 2Mx whaere Q and M stand for density and molarmass
respectively the ratio of Presures px/py would be

(a) 1 4 (b) 4 1 (c) 6 1 (d) 1 6

27
46. The density of methane at 2.0 atmosphere prassure at 270c is
(a) 0.13 gl-1 (b) 0.26 gl-1 (c) 1.30 gl-1 (d) 26.0 gl-1
47. The diffusion of methane at a given temperature is twice that of gas x. The molecular weight and the gas
is
(a) 64.0 SO2 (b) 32.0 (O2) (c) 4.0 (He) (d) 30.0(C2H6)
48. The compressibility factor of an ideal gas is (AIIMS - 1997, IIT91997)
(a) 0 (b) 0.1 (c) 0.2 (d) 0.4
49. As temperature is raise from 200C to 400C, the average kinetic energy of neon atoms changes by a
factor of which of the follwoing ?

1 313 313
(a) (b) 2 (c) (d)
2 293 293

50. If V rms is 30 R 1/2 at 270C calculate the molar mass of the gas in kilogram (DPMT 2005)
(a) 1 (b) 2 (c) 4 (d)0.001
51. Gas eqution PV=nRT is obeyed by (BHV 200)
(a) only isothermal process (b) only adiabatic process (c) both a and c (d) none of these
52. The average kinetic energy of an ideal gas per molecule in SI unit at 250C will be (CBSE 1996)
(a) 6.17 x 10-21 KJ (b) 6.17 x 10-21 J (c) 6.17 x 10-20J (d) 7.17 x 10-20 J
53. The vapour density of pure ozone would be
(a) 48 (b) 32 (c) 24 (d) 16
54. Equal masses of CH4 and H2 are mixed in an empty container at 250C. The fraction of the total pressure
exerted by H2 is

1 16
(a) (b) 8 9 (c) 1 9 (d)
2 17

55. The ratio between the root mean square (rms) vlocity of H2 at 50L that of at 800 K is ?

1
(a) 2 (b) 4 (c) 1 (d)
4

56. A real gas behave more ideally at (IIT flrelining 1993)


(a) Low temperature and low presure (b) Low temperature and high presure
(c) high temperature and low presure (d) high temperature and high presure

28
57. According to the kinetic theory of gases (IIT 1991)
(a) The pressure exerted by a gas is proportional to mean square velocity of the molecules.
(b) The pressure exerted by the gas is proportional to the root mean square velocity of the molenles.
(c) The root mean square velocity is invesly proportional to the temperature
(d) The mean transtational K.E of molecule is direlty propostional to the alsolute temperature
58. The value of vander waals constant a for gses O2, N2, NH3 and CH4 are 1.36, 1.39, 4.37 and
2.253 L2 atm mol-1 respectively the gas which can liquefied most easily will be
(a) O2 (b) N2 (c) NH3 (d) CH4
59. A certain sample of a gas volume 0.24Liter measared at 1.0 atm pressure and 2730C its volume will be
(BHU 2005)
(a) 0.4L (b) 0.8L (c) 27.8L (d) 55.6L
60. If a volume containg gas is compressed to half, how many moles of gas remained in the vessel
(a) Just double (b) just half (c) same (d) more than double
61. At constant Volume for a fixed number of a moles of a gas, the pressure of the gas increases with the rise
in tempreture due to (IIt 1992)
(a) Increase in average molecules speed (b) Increase rate of collisions amongst
(c) Increase in molecular attraction (d) Increase in mean free path
62. Equal mass of methane and oxygen are mixed in an empty container at 2500C the fraction of the total
pressure exarted by O2 is

1 273
(a) (b) 2 3 (c) 1 3 (d) 1 3
2 298

63. Find the true and false statements from the following on the basis of given graph
Fraction of Molecules colliding

0.5
0.4 Maximum probable speed
0.3
0.2
0.1

2 4 6 8 10 12
Molecular Speed cm/s

29
1. Maxwell and Boltz mann had studied the distribution of molecules between different possible
speeds
2. This graph is known as maxwells distribution curve in which kinetic energy and molecular speed
of gas is studdied.
3. The fraction molecues with very high or very low speed is very high
4. Inecresing the speed fraction also increasing Which becomes maximum and then deereases.
5. The top portion of curve indicates maximum fraction of molewles and the speed of molecules is
called most probable speed which is indicated by
6. On increasing temperature the collison of molecules increases and speed of molecules decreases
(a) TTFTTF (b) TFTTFT (c) FTTFFT (d) TTTTFF
64. When a real gas behaves as an ideal gas ?
(a) Inter molecular attraction among molecules are negligible then
(b) At very low pressure and high temperature then
(c) When molecular size is very very small and negligible to the volume of container then
(d) In all the above condition
238 235
65. What is the ratio of diffusion rate of UF6 and UF6 When these gases are diffused under the same
condition of temperature and pressure ?
(a) 0.09953 (b) 1.0047 (c) 1.0 (d) 10487
66. Molecular mass of SO2 gas is 4 times than CH4 therefore
(a) Being so2 and CH4 both gases, they diffuse with same rate
(b) so2 gas will diffuse 4 times factor than that of CH4
(c) Diffusion of SO2 gas is half than that of CH4
(d) CH4 gas found 4 times faster than SO2
67. 50 ml O2 gas diffused in 80 sec, What time will be required to diffuse same volume of He gas?
(a) 22.89 sec (b) 28.29sec (c) 92.82 sec (d) 24.29 sec
68. What is the mass (weight) of 6.022 x 1022 oxygen molcules ?
(a) 32 gms (b) 3.2 gms (c) 16 gms (d) 1.6 gms
69. How many molecules of 502 gas will be present in 5.6 litre volume of STP ?

(a) 1.5 1023 (b) 6.002 1023 (c) 7.52 1023 (d) 3.01 1023

70. Molar mass of CaCO3 = 100 gm/mole, how many moleunles are present in 20 gm (aCO3) ?

(a) 12.44 1023 (b) 1.2044 1023 (c) 1.2044 1025 (d) 1.2044 1022

71. For an ideal gas if pressure is (P), temperature (T) and gas constant is (R) then how many moles of gas
30
will be available in its 10 litre volume ?

P P 1 RT RT 1
(a) 10 (b) (c) 10 (d)
RT RT 10 P P 10
STP
P1 = 1.5 bar P2 = 1
V1 = 200 ml V2 = ?
T1 = 400K V2 = 273

PV PV
1 1
= 2 2
T1 T2

PV T
V2 1 1
2
T1 P2

1.5 200 273


400 1
72. At constant temperature 270C and pressure, 5 litre gas is raised its temperature by 10C, What will be the
change in its volume ?
th
1
(a) part of its original volume will be increased.
234
th
1
(b) The volume at 300K acquired by a gas will be increased of its part
234
th
1
(c) The volume of a gas acquired at O L kmp; will be increased of its
234
th
1
(d) part of volume of a gas at O0C , will be increased.
234

73. At 400 K temperature , pressure of 200 ml N2 gas is 1.5 bar, What is its volume at STP?
(a) 204.7 ml (b) 20.47 ml (c) 402.7 ml (d) 40.27 ml
74. What is correct prediction from the given graph ?

H2
He
PV
Z=
RT

C CH 4
CO 2

1.0

31
(a) H2 gas is less compressible than an ideal gas as its z > 1.0
(b) CO2 gas is more compressive than the ideal gas as Z > 1.0
(c) CH4 gas is higher compressive its Z < 1 than the ideal gas
(d) a and c both
75. It A,B,C, and D are the graphs plotted for H2, He, CH4 and CO2 Q-75 gases which graph is related for fH2
and CH4 compared to an ideal gases

D
B

A
PV
Z=
RT C

1.0

(a) D and A (b) A and B (c) A and D (d) B and C


76. which state of matter whose the intermolecular attractive force do not exist ?
(a) solid (b) liquid (c) gas (d) none
77. Which word of the following does not used for states of matter ?
(a) Bose - Einstin (b) Boyle - Einstein (c) plasma (d) solid, gas, liquid
78. 14.2 kg LPG is diffused in a gas cyllinder at 2.5 atm. If 50% lf LPG gas is used up then what will be the
pressure of gas will remain in cyllinder ?
14.2 2.5
(a) 2.5 atm (b) 1.25 atm (c) 5.0 atm (d)
7.1 V
79. When the unit of R = 8.314 x 10 erg. mole k What are the units of pressure and volume of a gas ?
7 -1 -1 .

(a) P - dyne cm-2, V-cm3 (b) P - paxal, V-cm3


(c) P - newton m-2, V-cm3 (d) P - atm, V-cm3
80. unit of R in CGS system is

(a) 8.314 107 erg K -1mole -1 (b) 8.314 JK -1mole -1

(c) 0.082litre atm mole -1 K -1 (d) 1.987 cal mole -1 K -1

81. What is the value of gas constant R cal.mol-1K-1 ?


(a) 0.082 (b) 1.987 (c) 8.314 (d) 8.314 x 107

32
82. What is not rateted to Z ?
(a) z=1 states the ideal behaviour of the gas
(b) Gases which has z =1 are an ideal gases

PV
(c) z is the ratio of known as compressibility factor
nRT

(d) When z < 1 of z > 1, the gases convert into their liqud state
83. Which equation is the ideal gas equation for the real gases ?
(a) PV = nRT (b) PV = iRT

am 2 am 2
(c) ( P = )(V - nb) = CRT (d) ( P = )(V - nb) = nRT
v2 v2

84. Proportion of O2,U2 and N2 gases are 3 :2: 5, and the Total pressure of this gas mixture container is 50
bar, What are the partial pressure of cl2 and N2 gases respectively?
(a) 10 bar, 25 bar (b) 10 bar, 15 bar (c) 15 bar, 25 bar (d) 25 bar, 15 bar
85. Which factor is the deciding factor of physical state of matter ?
(a) inter molecular forces (b) molecular interaction
(c) effect of thermal energy on the motion of partcles (d) Given all.
86. Whichphysical state is accuired by water in between temperature above than 273 K and below 373 K?
(a) plasma (b) liquid (c) solid (d) Gas
87. Which physical state of water is more compressable applying pressure at constant temperature ?
(a) Ice (b) water (c) Vapour (d) Plasma
88 Which sabstance can be easily poored form one container into the other at room temperature ?
(a) Kerosene (b) Ice (c) salt (d) all
89. Match column I with column II
column I column II
i) Gas (a) Easily povred form one container to the other
ii) Liquid (b) difinite shape and fixed volume like a container is acquired
iii) Solid (c) stats to melt at a definite temperature
(d)At constant pressure the increasing in temperature raises Volume effectively
(A) i) - b, ii) - a, iii) - c (B) i) - a, ii) - b, iii) - c
(C) i) - d, ii) - c, iii) - b (D) i) - a, ii) - c, iii) - b

33
90. Observation on physical state of water at 273K up to 373K are given as below, find the Correct option.
(A) changing the temperature of water above 273K upto 373K, composition of water is changing
gradnally.
(B) When temp is changed to rise from 273K on ward the physical state of water changes fom solid
- liquid to vapour state.
(C) Heating water at 373K temperture, propotion of hydrogen with oxygen is changed
(D) Molar mass of water decreases with changing its physical states solid - liquid gaseous an raising the
temperature
91. What is meant by Bose Einstein condensate ?
[A] It is the specific state of matter
[B] Showing relation E = MC2 for the matter
[C] It is an electronic device developed by Bose and Einstein
[D] It is an energy of radiation
92. Which pheno menon will occur when temperature of the matter is changed ?
[A] Physical state of matter changes.
[B] specific arrangement of molecules in a matter changes
[C] chemical properties are not chaging but density is changing
[D] Given all
93. Physical state of matter depend on...
[A] Inter molecular forces which keeps moleules near to eachother
[B] Thermal energy of kinetic molecules which keeps molecules away from eachother
[C] By balncing of combination of two opposite factors is intermolecular forces and thermal energy
decide the physical state of matter
[D] Given all
94. What is negative electrical charge on F atom in HF the permanent dipole molecule ?
[A] higher than 1.6 x 10-19 C [A] less than 1.6 x 10-10 C
[C] less than 1.6 x 10-19 C [D] higher than 1.6 x 10-10 C
95. State Figure showing dipole - Induced dipole forces in the following ?

d+ d
A B + +d d + d
(a) (b) H Cl + H Cl

34
d d
+ + + +d +d
(c) (d) H O H O
Nonpolar Nonpolar H H
Atom-I Atom-II
96. HCl polar molecule comes closer to He molecule, which effect of vander waals forces will be created ?
[A] Dipole - induced dipole forces [B] Dipole induced dipole forces with London forces
[C] London forces [D] dipole - dipole forces with London forces
97. Melting point of Rhomic Salphar is higher than phosphorus because ...state
[A] size and number of electrons in Rhombic sulphur is more compared to phosphorus
[B] sulphur S8 has metallic properties while P4 is nonmetal
[C] Inter attratcion forces are lower compared to thermal energy in sulphur than that of in phosphorus
[D] Given all
98. Statement (A) : In liquid state molecules are arranged little for form each other compared to its solid
state. hence the effect of pressure is observed in liquid.
statement (R) : At 293 K temperature and 1000 bar pressure applied on water than the volume reduced
by 4%
[A] statement (A) and (R) both correct, statement (R) is the explanation to statement (A)
[B] statement (A) and (R) both correct, statement (R) is not the explanation to statement (A)
[C] statement (A) is correct, statement (R) is wrong
[D] statement (A) is wrong, statement (R) is correct
99. Common physical states and other two physical states of matter are....
[A] plasma, liquid, Gas are common but solid state and Bose Einstein conden sate are spcial.
[B] Bose Einistein condensate and plasma are the other. rtwo states than common physical states gas,
liquid and solid.
[C] solid, liquid and Gas states are the only physical states : no other state is included.
[D] Bose Einistein condensate and plasma are the main rules to decided the common physical states
solid, liquid and Gas
100. Which statement is correct in the following.
[A] Matter is existing as a individual single moleule
[B] A group of matter is called molecule
[C] a group of moleules is called matter
[D] combination of group of different moleules form the same type of matter

35
101. Which of the following statements is false
[A] Matter is made of small particles always exist in solid state
[C] matter is solid state possess fixed volume with definite shape
[B] matter of the same substance in liquid state has more volume compared to its solid state
[D] Beyond solid, liquid and gaseous state, two other physical states are also known as plasma and
Bose Einistein condensate.
102. The density of neon will be maximum at (IIT 1990)
(a) NTP (b) O0C, 2 atm (c) 2730C, 1 atm (d) 2730C, 2 atm
103. Pressure of a mixture of 4 g of O2 and 2g of H2 combined in a bulb o f 1 litre at O0C is (AllMs - 2000)
(a) 25.215 atm (b) 31.205 atm (c) 45.215 atm (d) 15.210 atm
104. The temperature at which real gases obeys the ideal gas laws over a wide range of pressure in called.
(a) critical temperature (b) proyles temperature
(c) Inversion temperature (d) Reduced temperature
105. A bottle of NH3 and a square of dry HCL connected through a long tube are opened simultanuulely at
both ends, the white ammonia chloride ring first fromed will be (IIT - 1988)
(a) At the centre of tuloe (b) near the hydrogen chloride bottle
(c) near the NH3 bottle (d) through out the length of the tube
106. At 1000 C and 1 atm, if the density of liquid water is 1.0 g -3 an and that of water vapour is 0.006g cm-3
, then the volume of water moleules in 1 L of steam at this temperature is (IIT - 2000)
(a) 6 cm3 (b) 60 cm3 (c) 0.6 cm3 (d) 0.06 cm3
107. Two separate bulbs contains gas A and gas B the density of A is twice as that of gas B. The molecular.
mass of gas A is half as that of B If two gases are at same temp, the ratio of the presure of A to that of B
is
1 1
(a) 2 (b) (c) 4 (d)
2 4
108. 50 ml of a gas A diffuse throught a membrane in the same time as for this diffiusion of 40 ml of a gas B
under identical pressure temperature conditions . If the molecular weight of A = 64, that of B would be
(CBSC - 1992)
(a) 100 (b) 250 (c) 200 (d) 80
109. Which of the following statement is false ? (BHD - 1994)
(a) The product of volume pressure of fixed amound of a gas is independent of temperateure
(b) Molecules of differant gases have the same K.E. at a given temperature
(c) The gas equation is not valid at high pressure and low temperature
(d)The gas costant per moleule is known as Boltzmann Constant.

36
110. Density ratio of O2 and H2 gas is 16 : 1 The ratio of their rms velocities will be
(a) 4 : 1 (b) 1 : 16 (c) 1 : 4 (d) 16 : 1
111. at constant temperature and pressure, the rate of diffusion DA and DB of gases A and B having densities
PA and PB are related by the expressions (IIt Screening - 1993)

1 1 1 1
r 2 r 2 r 2 r 2
(a) DA = DB A (b) DA = DB B (c) DA = DB A (d) DA = DB B
rB rA rB rA

112. If 300ml of gas at 270C is cooled to 70C at constant pressure Its final volume will be (AIIMS - 2000)
(a) 135 ml (b) 540 ml (c) 350 ml (d) 280 ml
113. Positive deviation from ideal gas behaviour takes place. because of

PV
a) Molecular interaction between atoms and >I
nRT
PV
b) Molecular interaction between atoms and <I
nRT
PV
c) Finite size of the atoms and >I
nRT
PV
d) Finite size of the atoms and <I
nRT
114. Kinetic energy of 2 moles of N2 at 270C is (R = 8.324 JK-1mol-1)
(a) 5491.6 J (b) 6491.6 J (c) 7491.6 J (d) 8882.4 J
115. The root mean square velocity of one molemass of a monoatomic as having molarmass M is Urms. The
relation between average kinetic energy (E) of the gas and Urms is (IIT screening - 2004)

3E 2E 2E E
(a) m rms = (b) m rms = (c) m rms = (d) m rms =
2M 3M 3M 3M

116. Triple point of water is


(a) 273 K (b) 373 K (c) 203 K (d) 193 K
117. The root mean square velocity of an ideal gas at constant pressure varies with density (d) as (IIT - 2001)
1
(a) d2 (b) d (c) d (d)
d
Read the following paragraph carefully and give correct choice to the option for the questions
asked related to the paragraph
In the following given figure three glass containers X, Y, and Z are connected by values 1 and 2 having
negligible volume at 300K abd 1.0 atmosphere pressure.
When values 1 and 2 are closed, container X contains H2 gas at 8.2 atmosphere pressure in volume 6.0
37
litre, container Y contains N2 gas at 1.64 atm pressure in 10 litre volume, and container Z is evacurated
with pressure Zero atmosphere
external pressure = 1.0 atm, temperature = 300 K

H2 N2
8.2 atm. 1.64 atm. 0.0 atm.
Valve Valve
1 2
6 Litre 10 Litre 4 Litre
Container-X Container-Y Container-Z

118. What relative effect will be observed in average velocity of gseous molecules when valve -1 is oppened
between two containers X and Y ?
(a) Vx =Vy (b) Vx < Vy (c) Vx > Vy (d) Vx = 2Vy
119. What pressure will be exerted in container X when valve -1 is opened
(a) 4.1 atm (b) 8.2 atm (c) 2.05 atm (d) 3.84 atm
120. Keeping valve -1 closed, on opening valve 2 between container Zand Y, on expansion of gas how much
work will be done by process ?
(a) 1.0 litre atm (b) 14 - litre atm (c) -14.0 litre atm (d) zero
121. Opening valve 1 and 2 , on connecting all the three X,Y,Zcontainers with each other, What would be the
kineti cenergy of all. gaseous molecules ? (R = 0.082 litre atm /mol.K = 8.314 J/molk)
(a) 6842 J (b) 9974 J (c) 4988 J (d) 3832 J
122. Connecting all the three containers by opening valves 1 and 2, if internal pressure of containers are
obtained 1.0 atmosphere by lowering temperature of the system, what would be the contribution of
partical pressure of H2 gas and N2 gas respectively ?
(a) 0.85 atm, 0.15 atm (b) 0.15atm, 0.85 atm
(c) 0.75 atm, 0.25 atm (d) 0.25 atm, 0.75 atm
Paragraph - 2
The gases which do not obey general gas equation at all tempratures and pressures are called non ideal
or real gases . But such gases show ideal behaviour at low pressure and high temperatures.
According to vander waals, the following are two faulty assumptions in kinetic theroy of gases.
(1) molecules of gas zero consideraed as point masses and the volume occupied by the gas motecules
is neglihigible in comparison to the total volume of gas.

38
(2) It was also assumed that there are no intermolecular attractive forces and the molecules of gas
move independently.
Hence the vanderwaals equation for non- ideal (real) gases becomes

a
P + 2 ( v - b ) = RT for 1 moles and
V
an 2
P + 2 ( v - nb ) = nRT for n moles
V

here a, and b are vanderwaals constants. a is a measure of intermolecular cular forces in a given gas.It
is a measure of icompressibility volume per mole of gas.

PV
Deviation of gases from ideal behaviour is studded by plotting graph ( z ) vs P here quantity z is
nRT
called compressbility factor.

PV
for H2 and He gases = z is always > 1. They show he deviation but for N2,O2CH4andCo2 gases
nRT
z is < 1 show -ve deviation at low pressure expected them ideal behaviour value of compressbility factor
Z at critiacal always < 1 and real gases show negative deviation
as per vander waals equation at critical point

a
(3b) 3
PV 27b 2
z= c c
= =
Tc 8a 8
R
27 Rb
thus Z is less than 1 at critical point show negative deviation of real gases compare to ideal behaviour
here in the above derivation at critical points

8a a
Tc = , Pc = and Vc = 3b
27 Rb 27b 2

alter natively, constants


Vc 8 Pc Vc
b= , a = 3Pc Vc 2 , R =
3 3Tc

The units of a : atm L2 mole-2


b : l mole-1
question (1)

39
123. Which statements are correct in the following ?
(a) The real gases donot obey the ideal behaviour at all temperatures and pressures
(b) The gases which donot obey general gas equation are called non - ideal gases
(c) Molecules were considered mass and volume less hence They donot occupy volume com
pared to total volume in the derivation of vander waals equation
(d) vander waals proposed the gas equation for 1 note gas is

a
P + 2 ( v - nb ) = RT
n

(A) a,b (B) b,c (C) c,d


(D)a,d
124. (2) question : (2) on what bases the deviation of gases from ideal behaviour is studied ?
(a) by plotting graph PV vs T (b) by plotting graph Z vs T

PV
(c) by plotting graph Z vs P (d) by plotting graph vs P
nRT
(A) a,b (B) b,c (C) c,d (D) a,d
125. When a graph Z plotted against P for CH
4
and CO2 gases then the graph obtained as ...

(a) (b)
CH 4 CH 4

CO 2 CO 2

Z = 1.0
P Z=1 P

(c) (d)
CO 2

CH4
CH 4
CO 2

Z = 1.0 P Z = 1.0 P

40
126. What is indicated by the given graph for the behaviour of H2 and CO2 gases correctly ?

H2

CO 2

Z = 1.0 P

(a) H2 has value z > 1 show positive deviation from ideal behaviour

PV
(b) Value of z = for H2 gas is greater than zero but less then 1
nRt

PV
(c) value of z = for CO2 gas is greater than zero but less than 1 shows its negative deviation of its
nRT
ideal behaviour
(d) value of Z for H2 is greater than 1 and for co2 it is less then one hence at high pressure co2 gas is
more compressible but at low pressure it less compressible than expected from ideal behaviour
compressible than expected from ideal behaviour
(A) a,b,c (B) b,c,d (C) a,c,d (D) a,b,d
127. At critical point the value of Tc,Pc and Cc interms of vanderwaals equation are respectively.

8a a 3a 8a 3b 8a 2 a 3a 8a 3b
(a) , , 3b (b) , , 2 (c) , , 3nb (d) , , 2
27 Rb 27b 2 8 Rb 27b v 27 Rb 27b 2
8 Rb 27b v

128. Match gases under specified conditions listed in column I with their properties in column II
Column I Column II

PV
1) H2 (g) (P = 200 atm, T = 273 K) a) z = 1
RT

2) ( )
H2 (g) P V= O , T =273 K) b) Attractive forces are dominant

3) CO2 (g) ( p = 1 atm , T= 273 K) c) PV = nRT


4) real gas SO2 with bigger size of its volume d) P( V - nb) = nRT

A) 1 - a, 2 - c, 3 - d, 4- b B) 1 - b, 2 - d, 3 - c, 4 - a
C) 1 - a, 2 - d, 3 - b, 4 - c D) 1 - a, 2 - c, 3 - b, 4 - d

41
129. column I column II
a
1) If force of attraition among the a) P + 2 ( v - b ) = RT
v
gaseous molecules are negligible
a
2) If the volume of the gas molecules b) PV = RT -
v
are negligible
3) At STP c) PV = RT + PB
4) At low pressure and high temperature d) PV = RT .
A) 1) - c, 2) - b, 3) - a, 4)- d B) 1 - d, 2 - b, 3 - c, 4 - a
C) 1 - c, 2 - a, 3 - b, 4 - d D)1 - b, 2 - a, 3 - d, 4 - c
130. for a fixed mass of a gas and constant pressure, which of the follownig graphs are releted to V T, the
charles law ?
(a) V (b)
T
V

T(K) T(K)
(c) (d)
V
V

T(C)
T(K)

(A) b,c,d (B) a,b,c (C) a,c,d (D) a,b,d


131. Which of the following statements is / are correct ?
(a) AT high prssure, all real gases are less compressible than ideal gas.
(b) H2 he gases are more compressible than ideal gas for all values of pressure

PV
(c) compressiblity factor z = nRT is less than 1 for all real gases at low pressure except H2 and He

(d) The compressibility factor z of real gases are quite independent of temperature therefore z is not
effected with change in temperature.
(A) a,c (B) b,c (C) c,d (D) a,d

Passage

A gas Undergoes dissociation as M 4( q ) 4 M ( g ) in a closed rigid container having volume 22.4L at


273K If the initialmoles of M4 taken befor dissoliciation is 1 then.

42
132. The total pressure (in attm) after 50% completion of the reaction assuning ideal behaviour is
(a) 0.5 (b) 2.5 (c) 2.8 (d) 3.8
133. If the gases are not ideal at the begining total pressure observed is less than 1 atm then

PV PV
(a) z = of M 4 > 1 (b) z = of M 4 < 1
RT RT

PV PV
(c) z = of M 4 = 1 (d) z = of M > 1
RT RT
134. If the gases are not ideal and after 100 % dissociation total pressure is greater than 4 atm, then
(a) Compressing of M(q) is easiq then an ideal gas
(b) The compression of M(q) is difficult than an ideal gas
(c) The compression of m(g) is the same as an ideal gas
(d) A gas is non compressible

Answer Key

1 a 2 d 3 b 4 c 5 b 6 a 7 b
8 d 9 c 10 c 11 d 12 b 13 b 14 d
15 a 16 a 17 b 18 b 19 c 20 d 21 b
22 a 23 a 24 b 25 d 26 b 27 d 28 a
29 b 30 c 31 b 32 c 33 b 34 b 35 c
36 c 37 d 38 b 39 a 40 a 41 a 42 b
43 c 44 c 45 c 46 c 47 a 48 b 49 c
50 d 51 c 52 b 53 c 54 b 55 c 56 c
57 d 58 c 59 a 60 a 61 a 62 d 63 a
64 d 65 b 66 c 67 b 68 b 69 a 70 b
71 a 72 d 73 a 74 d 75 c 76 c 77 b
78 a 79 a 80 a 81 b 82 d 83 d 84 c
85 d 86 b 87 c 88 a 89 a 90 b 91 a
92 d 93 d 94 c 95 a 96 b 97 a 98 a
99 b 100 c 101 a 102 b 103 a 104 b 105 b
106 b 107 c 108 c 109 a 110 c 111 d 112 d
113 a 114 c 115 c 116 a 117 d 118 c 119 a
120 d 121 b 122 c 123 a 124 c 125 d 126 c
127 a 128 c 129 a 130 c 131 a 132 b 133 b
134 b

43
UNIT : 3 STRUCTURE OF ATOM
Important Points
Composition of atom
Electron (1 eo )
(1) It was discovered by J.J. Thomson (1897) and is negatively charged particle.
(2) Electron is a component particle of cathode rays.
(3) Cathode rays were discovered by William Crookes & J.J. Thomson (1880).
Properties of Cathode rays
(i) Cathode rays travel in straight line.
(ii) Cathode rays produce mechanical effect, as they can rotate the wheel placed in their path.
(iii) Cathode rays consist of negatively charged particles known as electron.
(iv) When cathode rays fall on solids such as Cu, X rays are produced.
(v) The nature of these rays does not depend upon the nature of gas or the cathode material used
in discharge tube.
(vi) The e/m (charge to mass ratio) for cathode rays was found to be the same as that for an e
(1.76 10 8 coloumb per gm). Thus, the cathode rays are a stream of electrons.
Proton ( H+, p)
(1) Proton was discovered by Goldstein
(2) It is a component particle of anode rays.
Goldstein (1886) used perforated cathode in the discharge tube and repeated Thomsons
experiment and observed the formation of anode rays. These rays also termed as positive or canal
rays.
Properties of anode rays
(i) Anode rays travel in straight line.
(ii) Anode rays are material particles.
(iii) Anode rays are positively charged.
(iv) Anode rays may get deflected by external magnetic field.
(v) Anode rays also affect the photographic plate.
(vi) The e/m ratio of these rays is smaller than that of electrons.
(vii) Unlike cathode rays, their e/m value is dependent upon the nature of the gas taken in the tube.
It is maximum when gas present in the tube is hydrogen.
Neutron (on1, n)
(1) Neutron was discovered by James Chadwick (1932) according to the following nuclear
reaction, 4 Be 9 2 He 4 6 C 12 o n1 or 5 B 11 2 He 4 7 N 14 o n1
(2) Neutron is an unstable particle. It decays as follows,
1 1 0 0
0 n 1 H 1 e 0
neutron Proton electon antinutrino

44
Name of constant Unit Electron(e) Proton(p+) Neutron(n)
Amu 0.000546 1.00728 1.00899
Mass (m) Kg 9.109 1031 1.673 1027 1.675 1027
Relative 1/1837 1 1

Coulomb (C) 1.602 1019 +1.602 1019 Zero


Charge(e) Esu 4.8 1010 +4.8 1010 Zero
Relative 1 +1 Zero
Specific charge (e/m) C/g 1.76 108 9.58 104 Zero
Density Gram / cc 2.17 10
17
1.114 10
14
1.5 10
14

The atomic mass unit (amu) is 1/12 of the mass of an individual atom of 6 C 12 , i.e. 1.660 10 27 kg .

Other non fundamental particles


Particle Symbol Nature Charge Mass Discovered by
esu (amu)
1010
Positron e ,1e0 , + + 4.8029 0.00054 Anderson (1932)
86
Neutrino 0 0 < Pauli (1933) and Fermi (1934)
0.00002
Anti-proton p 4.8029 1.00787 Chamberlain Sugri (1956) and
Weighland (1955)
Positive mu + + 4.8029 0.1152 Yukawa (1935)
meson
Negative mu 4.8029 0.1152 Anderson (1937)
meson
Positive pi + + 4.8029 0.1514
meson
Negative pi 4.8029 0.1514
Powell (1947)
meson
Neutral pi 0 0 0 0.1454
meson
Atomic number, Mass number and Atomic species

(1) Atomic number or Nuclear charge

(i) The number of protons present in the nucleus of the atom is called atomic number (Z).

(ii) It was determined by Moseley as, a (Z b) or aZ ab

Where, X rays frequency s 1


Z= atomic number of the metal a & b are constant. Z

(2) Mass number

Mass number (A) = Z + n

45
Different types of atomic species
Atomic Similarities Differences Examples
species
Isotopes (i) Atomic No. (Z) (i) Mass No. (A) 1 2 3
(i) 1 H , 1 H , 1 H
(Soddy) (ii) No. of protons (ii) No. of neutrons 16
O, 17 18
8 O, 8 O
(iii) No. of electrons (iii) Physical properties (ii) 8

(iv) Electronic 35 37
Cl , 17 Cl
(iii) 17
configuration
(v) Chemical properties
(vi) Position in the
periodic table
Isobars (i) Mass No. (A) (i) Atomic No. (Z) 40 40
Ar , 19 40
K , 20 Ca
(ii) No. of nucleons (ii) No. of protons, (i) 18
130
electrons and neutrons (ii) 52 Te, 130 130
54 Xe, 56 Ba
(iii)Electronic
configuration
(iv) Chemical properties
(v) Position in the
perodic table.
Isotones No. of neutrons (i) Atomic No. 30 31
Si, 15 32
P, 16 S
(i) 14
(ii) Mass No., protons 39 40

and electrons. (ii) 19 K , 20Ca


3
(iii) Electronic H , 4 He
(iii) 1 2
configuration 13 14

(iv) Physical and (iv) 6 C , 7 N


chemical properties
(v) Position in the
periodic table.
Isoelectronic (i) No. of electrons At. No., mass No. N 2O, CO2 , CNO (22e )
(i)
species (ii) Electronic
configuration (ii)
CO, CN , N2 (14e )
2
(iii) H , He, Li , Be (2e )
3 2 2
(iv) P , S , Cl , Ar , K and Ca (18e )

Electromagnetic radiations
(1) Light and other forms of radiant energy propagate without any medium in the space in the form of
waves are known as electromagnetic radiations. These waves can be produced by a charged
body moving in a magnetic field or a magnet in a electric field. e.g. rays, rays, cosmic rays,
ordinary light rays etc.
(2) Characteristics
(i) All electromagnetic radiations travel with the velocity of light.
(ii) These consist of electric and magnetic fields components that oscillate in directions
perpendicular to each other and perpendicular to the direction in which the wave is travelling.

46
(3) A wave is always characterized by the following five characteristics,

Crest Wavelength

Vibrating
source
Energy

Trough
Fig. 2.2

(i) Wavelength : The distance between two nearest crests or nearest troughs is called the
w avelength. I t is denoted by (lambda) and is measured is terms of centimeter(cm),
angstrom(), micron( ) or nanometre (nm).
1 10 8 cm 10 10 m ; 1 10 4 cm 10 6 m ;

1nm 10 7 cm 10 9 m ; 1cm 10 10 10 nm
8 4 7

(ii) Frequency : It is defined as the number of waves which pass through a point in one second.
It is denoted by the symbol (nu)
c
distance travelled in one second = velocity =c

(iii) Velocity : It is defined as the distance covered in one second by the wave. It is denoted by
the letter c. All electromagnetic waves travel with the same velocity, i.e., 3 1010 cm / sec .
c 3 1010 cm / sec

(iv) Wave number : This is the reciprocal of wavelength, i.e., the number of wavelengths per
centimetre. It is denoted by the symbol (nu bar). It is expressed in cm 1 or m 1 .
1


(v) Amplitude : It is defined as the height of the crest or depth of the trough of a wave. It is
denoted by the letter A. It determines the intensity of the radiation.
The arrangement of various types of electromagnetic radiations in the order of their increasing
or decreasing wavelengths or frequencies is known as electromagnetic spectrum.
Atomic spectrum - Hydrogen spectrum
Atomic spectrum
Spectrum is the impression produced on a photographic film when the radiation of particular wavelength
is (are) analysed through a prism or diffraction grating.
Types of spectrum
(1) Emission spectrum: Spectrum produced by the emitted radiation is known as emission spectrum.
This spectrum corresponds to the radiation emitted (energy evolved) when an excited electron
returns back to the ground state.
(i) Continuous spectrum: When sunlight is passed through a prism, it gets dispersed into
continuous bands of different colours. If the light of an incandescent object resolved through
prism or spectroscope, it also gives continuous spectrum of colours.

47
(ii) Line spectrum: If the radiations obtained by the excitation of a substance are analysed
with help of a spectroscope a series of thin bright lines of specific colours are obtained.
There is dark space in between two consecutive lines. This type of spectrum is called line
spectrum or atomic spectrum..
(2) Absorption spectrum : Spectrum produced by the absorbed radiations is called absorption
spectrum.
Hydrogen spectrum
(1) All these lines of H-spectrum have Lyman, Balmer, Paschen, Barckett, Pfund and Humphrey
series. These spectral series were named by the name of scientist discovered them.
(2) To evaluate wavelength of various H-lines Ritz introduced the following expression,

1 1 1
R 2 2 z2
c n1 n2
Where R is universal constant known as Rydbergs constant its value is 109, 678 cm 1 .
Plum pudding model of Thomson
(1) He suggected that atom is a positively charged sphere having electrons embedded uniformly
giving an overall picture of plum pudding.
(2) This model failed to explain the line spectrum of an element and the scattering experiment of
Rutherford.
Rutherfords nuclear model
From the observations of ? ray scattering experiments he concluded that, an atom consists of
(i) Nucleus which is small in size but carries the entire mass i.e. contains all the neutrons and
protons.
(ii) Extra nuclear part which contains electrons. This model was similar to the solar system.
(3) Properties of the nucleus
(i) Nucleus is a small, heavy, positively charged portion of the atom and located at the centre
of the atom.
(ii) All the positive charge of atom (i.e. protons) are present in nucleus.
(iii) Nucleus contains neutrons and protons, and hence these particles collectively are also
referred to as nucleons.
(iv) The size of nucleus is measured in Fermi (1 Fermi = 1013 cm).
(v) The radius of nucleus is of the order of 1.5 10 13 cm. to 6.5 10 13 cm. i.e. 1.5 to 6.5 Fermi.
Generally the radius of the nucleus ( rn ) is given by the following relation,
13
rn ro ( 1 . 4 10 cm ) A 1 / 3
This exhibited that nucleus is 10 5 times small in size as compared to the total size of atom.
(vi) The Volume of the nucleus is about 10 39 cm 3 and that of atom is 10 24 cm 3 , i.e., volume of
the nucleus is 10 15 times that of an atom.
(vii) The density of the nucleus is of the order of 10 15 g cm 3 or 10 8 tonnes cm 3 or 10 12 kg / cc . If
nucleus is spherical than,
mass number
mass of the nucleus
Density = 6.023 1023 4 r 3
volume of the nucleus 3
48
Plancks quantum theory
To explain black body irradiation, Max Planck put forward a theory known as Plancks quantum theory.
(i) The radiant energy which is emitted or absorbed by the black body is not continuous but
discontinuous in the form of small discrete packets of energy, each such packet of energy is called
a quantum. In case of light, the quantum of energy is called a photon.
(ii) The energy of each quantum is directly proportional to the frequency ( ) of the radiation, i.e.
hc
E or E hv Where, h Plancks constant = 6.621027 erg. sec. or 6 .62 10 34 Joules sec .

(iii)The total amount of energy emitted or absorbed by a body will be some whole number quanta.
Hence E nh , where n is an integer..
Photoelectric effect
(1) When radiations with certain minimum frequency ( 0 ) strike the surface of a metal, the electrons
are ejected from the surface of the metal. This phenomenon is called photoelectric effect and the
electrons emitted are called photo-electrons. The current constituted by photoelectrons is known
as photoelectric current. This minimum frequency is known as threshold frequency.
(2) The electrons are ejected only if the radiation striking the surface of the metal has at least a
minimum frequency called Threshold frequency. The minimum potential at which the plate
photoelectric current becomes zero is called stopping potential.
(3) The velocity or kinetic energy of the electron ejected depend upon the frequency of the incident
radiation and is independent of its intensity.
(4) The number of photoelectrons ejected is proportional to the intensity of incident radiation.
(5) Einsteins photoelectric effect equation
According to Einstein,
Maximum kinetic energy of the ejected electron = absorbed energy threshold energy
1 2 1 1
mv max h h 0 hc
2 0

Where, 0 and 0 are threshold frequency and threshold wavelength.


Advantages of Bohrs theory
(i) Calculation of radius of Bohrs orbit : According to Bohr, radius of nth orbit in which electron
h2 n2
moves is rn 2 2 .
4 me k Z

Where, Orbit number, Mass number Charge on the electron Atomic number of element, k =
Coulombic constant

Where, n = Orbit number, m = Mass number 9. 1 10 31 kg , e = Charge on the electron 1.6 10 19


Z= Atomic number of element, k = Coulombic constant 9 109 Nm 2c 2

n2
After putting the values of m,e,k,h, we get rn 0.529
Z

49
(iii) Calculation of velocity of electron
1/ 2
2e 2 ZK Ze 2
2.188 10 8 Z
Vn , Vn Vn cm. sec 1
nh mr n
;
(iv) Calculation of energy of electron in Bohrs orbit
kZe 2 kZe 2 kZe 2
Total energy of electron = K.E. + P.E. of electron
2r r 2r

2 2 mZ 2e 4 k 2
Substituting of r, gives us E Where, n=1, 2, 3.
n2 h2
Putting the value of m, e, k, h, we get
Z2
E 21.8 10 12 erg per atom
n2
Z2 Z2
21. 8 10 19 2
J per atom (1J 107 erg) E 13.6 2
eV per atom(1eV 1.6 10-1 9 J )
n n

Z2 1312
13.6 k.cal / mole (1 cal = 4.18J) or Z 2kJmol 1
n2 n2
When an electron jumps from an outer orbit (higher energy) to an inner orbit (lower energy)then
the energy emitted in form of radiation is given by

2 2k 2 me 4 Z 2 1
1
E En2 En1
h2 n2 n 2
1 2

1 1
E 13.6 Z 2 2 2 eV / atom
n1 n2

1 E 2 2 k 2me 4 Z 2 1 1
As we know that E h , c and , 3

2
2

hc ch n1 n2

1 1 1
This can be represented as RZ 2 2 2

n1 n2

2 2k 2 me 4
Where, R 3
; R is known as Rydberg constant. Its value to be used is 109678 cm-1.
ch
(6) Spectral evidence for quantisation (Explanation for hydrogen spectrum on the basis of bohr
atomic model)
(i) The optical spectrum of hydrogen consists of several series of lines called Lyman, Balmar,
Paschen, Brackett, Pfund and Humphrey.
(ii) To evaluate wavelength of various H-lines Ritz introduced the following expression,
1 1 1 2 2me 4
R 2 2 Where, R is = Rydbergs constant
c n1 n2 ch3

Its theoritical value = 109,737 cm1 and Its experimental value = 109,677. 581cm 1
This remarkable agreement between the theoretical and experimental value was great
achievment of the Bohr model.

50
(iii) Comparative study of important spectral series of Hydrogen is shown in following
table.
S.No. Spectral Lies in Transition n 2n 2 n12 l max n 22
l max 2 1 22 lmin
series the n2 > n1 R l min n22 n12
(n2 n1 ) R
region
(1) Lymen Ultraviole n1 1 n1 1 and n2 2 n1 1 and n2
series t region n 2 2, 3, 4.... 4 1 4
l max lmin
3R R 3

(2) Balmer Visible n1 2 n1 2 and n2 3 n1 2 and n2


series region n 2 3, 4,5.... 36 4 9
l max lmin
5R R 5
(3) Paschen Infra red n1 = 3 n1 3 and n2 4 n1 3 and n2
series region n 2 4,5,6.... 144 9 16
l max lmin 7
7R R

(4) Brackett Infra red n1 4 n1 4 and n2 5 n1 4 and n2


series region n 2 5, 6,7.... 16 25 25
lmax
9R 16 9
l min
R
(5) Pfund Infra red n1 5 n1 5 and n 2 6 n1 5 and n2
36
series region n 2 6,7,8.... 25 36 25
l max lmin 11
11R R
(6) Humphrey Far n1 6 n1 6 and n 2 7 n1 6 and n2
49
series infrared n 2 7,8.... 36 49 36
l max lmin 13
region 13R R

(iv) If an electron from nth excited state comes to various energy states, the maximum spectral lines
n(n 1)
obtained will be . n principal quantum number..
2

6(6 1) 30
As n 6 than total number of spectral lines 15.
2 2

BohrSommerfeilds model
It is an extension of Bohrs model. The electrons in an atom revolve around the nuclei in elliptical orbit. The
circular path is a special case of ellipse. Association of elliptical orbits with circular orbit explains the fine
line spectrum of atoms.
Dual nature of electron
(1) I n 1924, the French physicist, Louis de Broglie suggested that if light has both particle and wave
like nature, the similar duality must be true for matter. Thus an electron, behaves both as a material
particle and as a wave.
(2) According to de-broglie, the wavelength associated with a particle of mass m, moving with velocity
h
v is given by the relation , where h Plancks constant.
mv

51
(3) This was experimentally verified by Davisson and Germer by observing diffraction effects with an
electron beam. Let the electron is accelerated with a potential of V than the Kinetic energy is

1 h h
mv 2 eV ; m 2 v 2 2eVm mv 2eVm P ;
2 2eVm 2k .E m

(4) If Bohrs theory is associated with de-Broglies equation then wave length of an electron can be
determined in bohrs orbit and relate it with circumference and multiply with a whole number

2 r h
2 r n or From de-Broglie equation, .
n mv

h 2 r nh
Thus or mvr
mv n 2
(5) The de-Broglie equation is applicable to all material objects but it has significance only in case of
microscopic particles.
Heisenbergs uncertainty principle
This principle states It is impossible to specify at any given moment both the position and momentum
(velocity) of an electron.
h
Mathematically it is represented as , x . p
4
Where x uncertainty is position of the particle, p uncertainty in the momentum of the particle

Now since p m v
h h
So equation becomes, x. mv or x v
4 m
4
In terms of uncertainty in energy, E and uncertainty in time t, this principle is written as,

h
E . t
4
Schrdinger wave equation
(1) Schrodinger wave equation is given by Erwin Schrdinger in 1926 and based on dual nature of
electron.

2 2 2 8 2 m
The Schrodinger wave equation is, 2 2 (E V ) 0
x 2 y 2 z h
Where x, y and z are the 3 space co-ordinates, m mass of electron, h Plancks constant, E
Total energy, V potential energy of electron, amplitude of wave also called as wave function,
2
is mathematical operation to be performed on Y
x 2
(2) The Schrodinger wave equation can also be written as,
8 2 m
2 ( E V ) 0 Where laplacian operator..
h2

52
(3) Physical significance of and 2
(i) The wave function represents the amplitude of the electron wave. The amplitude is thus
a function of space co-ordinates and time i.e. (x, y, z......times)

(ii) For a single particle, the square of the wave function 2 at any point is proportional to the
probability of finding the particle at that point.
(iii) If 2 is maximum than probability of finding e is maximum around nucleus and the place
where probability of finding e is maximum is called electron density, electron cloud or an
atomic orbital. It is different from the Bohrs orbit.
(iv) The solution of this equation provides a set of number called quantum numbers which describe
specific or definite energy state of the electron in atom and information about the shapes and
orientations of the most probable distribution of electrons around the nucleus.
It was Erwin Schrodinger who developed a new model of atom in 1920.He incorporated the idea of
quantisation, and the conclusions of de-Broglies principle and Heisenbergs principle in his model.
In this model, the behaviour of the electron in an atom is described by the mathematical equation

2 2 2 8 m
known as Schrodinger Wave Equation. ) + 2 (E V)
x 2 y 2 z 2 h

(Here x, y and z are three space coordinates, m mass of electron, h Plancks constant, E Total
energy, V Potential energy and 1/J Wave function of electron wave)

8 m
The above expression can also be expressed as 2 (E V)
h2
The permitted solutions of Schrodinger equation are known as wave functions which correspond
to a definite energy state called orbital. Thus, the discrete Bohr orbits are replaced by orbitals i.e.,
three dimensional geometrical olumes where there is maximum probability of locating the electrons.
In simple words, the equation may be interpreted by stating that a body/particle of mass m, potential
energy E, velocity v, has wave like characteristics associated with it, with amplitude given by
wave function.
Probability Distribution
In wave mechanics a moving electron is represented by wave function, j. It has on physical significance and
refers to the amplitude of electron wave. However, j2 is a significant term and give intensity of electrons. An
atomic orbital is a region around the nucleus where there is more probability of intensity of electrons. An
orbital is considered as a diffused electron cloud having more density close to the nucleus. The probability
of finding an electron in a given volume is understood best in the form of radial probability distribution
curves. The probability curves for some orbitals are given in the figure. The distance of maximum radial
probability is radius of an atom. There are two humps for 2p-orbital which means that the 2s electron
penetrates a little closer to nucleus. The point at which radial probability becomes zero is known as nodal
point.

53
The radial probability plots for some orbitals are shown in the given figure.

RADIAL PRO VABILITY


RADIAL PRO VABIL ITY
RADIAL PROVABIL ITY

DISTANCE
DISTANCE
1s
DISTANC E
2s 2p

ORBITAL WAVE FUNCTIONS AND SHAPES OF ORBITALS


According to wave mechanics, atomic orbitals are described bywave functions known as orbital wave
functions. These orbital wave functions can be represented by the product of two wave functions, (i)
radial wave function and (ii) angular wave function.
The radial wave function depends upon distance r from the nucleus. On the other hand, angular wave
function depends upon the direction given by the angles with respect to different co-ordinate axis. It is
found that the wave function for s-orbital is independent of angles and, therefore, s-orbitals do not have
angular dependence. Thus, all s-orbitals are spherically symmetrical. However, all other types of orbitals
(p, d or f) have angular dependence and, therefore, have directional dependence.
Radial Probability Distribution Curves
If we draw a graph between radial wave function, R (radial part of wave functionj) and r (distance from
nucleus), we obtain graphs as shown below. These graphs are for ls, 2s and 2p-orbitals of hydrogen atom.
This type of dependence is known as radial dependence. These plots show radial dependence on only
one side of the nucleus. These plots do not have any direct physical significance, but are useful in molecular
structure because atomic wave functions are needed to build molecular wave functions. It is clear from the
graph, that in ls radial wave function, j, is positive everywhere, but for other s orbitals i.e., for 2s or 3s-
orbitals it is positive in some regions and negative in others. It may be noted that +ve and - ve signs have
only relative significance. During superposition (in the formation of molecular orbitals) relative signs play a
very important part.

We know that square of the wave function*, R2, represents the probability of finding the electron in a unit
volume i.e., probability density. The graphs between square of the radial wave function R2 and r (distance
from nucleus) are known as radial probability density graphs. These graphs differ slightly from the earlier
graphs as R2 is positive throughout.

54
(In case R is not real, IRI2 can be taken in place of R2. In such cases IRI2 R. .)

2 2 2
R R R

1s 2s 2p
r r r
2
Graph between R (radialprobabUity density) and r

2 2 2
R dv R dv R dv

1s 2s 2p
r r
r
Graph between radial probability function R2 dV (or 4pr2R and r2)
As R2 represents the probability density, i.e., probability of finding the electron in a unit volume, R2dV
gives the probability of finding the given electron in a volume dV. The product R2dV is also known as radial
probability distribution function. The graph of R2dV versus r (distance from nucleus) is known as radial
probability distribution function graphs.
If we observe the radial probability distributive graph of 1S, we find it is quite different from the radial
probability density graph. The radial probability density is maximum close to the nucleus, but the radial
probability is least. This is due to the fact that volume of the spherical shell (dV) near the nucleus is very
small resulting in a small value of radial probability R2dV. At the nucleus (i.e., r 0), dV is zero, hence
R2dV as also zero, although R2 is very large at this point. As the distance from nucleus (r) increases, the
volume of the shell dV (4 r2dr) increases while R2 decreases. As a result, the radial probability keeps on
increasing gradually and reaches a maximum value and then decreases gradually. The maximum in the
curve indicates the most probable value and the corresponding distance from the nucleus is called distance
of maximum probability (r0). For hydrogen atom in ground state, this has a value of 53 pm.
It is important to note that Bohr predicted that the electron will always be at a distance of 53pm from the
nucleus for hydrogen atom in ground state. However, according to wave mechanical model the electron is
most likely to be found at this distance and there is probability of finding the electron at distances both
shorter and longer than this.
The radial probability distribution curve for 2s-orbital (n 2, l 0) shows two maxima, a smaller one
nearer the nucleus and a bigger one at a larger distance. Comparing the location of the maxima in the 2s
orbital, we conclude that an electron in the 2s-orbital has greater probability to stay further away than an
electron in the 1s orbital.
The radial probability distribution curves of three 2p-orbitals (n 2, l 1) are identical. It shows only one
maximum. The distance of maximum probability for a 2p-orbital is slightly less than that for a 2s-orbital.
However, in contrast to the curve for 2p-orbital, there is a small additional maximum in the curve for 2s-
orbital. In other words, 2s-orbital penetrates closer to the nucleus, than a 2p-orbital. Thus, an electron in
2s-orbital has a lower energy than an electron in a 2p-orbital.

55
Some Note worth points

1. The radius of maximum probability of 1s electron is 0.529.


2. The number of regions of maximum probability for 1s, 2p, 3d and 4f is one. For 2s, 3p, 4d and 5f
these are two and so on.
3. The small humps indicate that the electron has a tendency to penetrate closer to the nucles.
4. In between the regions of zero electron density called node. More is the number nodes more is the
energy of an orbital.
5. In these curves, the first orbital of cash type (1s, 2p, 3d, 4f) has one region of maximum probability
and no node. Whereas the first orbital of each type (2s, 3p, 4d, 5f) has two regions of maximum
probability and one node so on.
Quantum numbers
Each orbital in an atom is specified by a set of three quantum numbers (n, l, m) and each electron is
designated by a set of four quantum numbers (n, l, m and s).
(1) Principle quantum number (n)
(i) The maximum number of an electron in an orbit represented by this quantum number as 2n 2 .
(ii) It gives the information of orbit K, L, M, N.
(iii) Angular momentum can also be calculated using principle quantum number
(2) Azimuthal quantum number ( )
(i) Azimuthal quantum number is also known as angular quantum number. Proposed by
Sommerfield and denoted by .
(ii) It determines the number of sub shells or sublevels to which the electron belongs.
Value of 0 1 2 3
Name of subshell s p d f
Shape of subshell Spherical Dumbbell Double Complex
dumbbell

(iii) It tells about the shape of subshells.


(iv) It also expresses the energies of subshells spd f (increasing energy).
(v) The value of l (n 1) always.
h
(vii) It represent the orbital angular momentum. Which is equal to l (l 1)
2
(viii) The maximum number of electrons in subshell 2(2l 1)
(3) Magnetic quantum number (m)
(i) It was proposed by Zeeman and denoted by m.
(ii) It gives the number of permitted orientation of subshells.
(iii) The value of m varies from to + through zero.
(iv) It tells about the splitting of spectral lines in the magnetic field i.e. this quantum number
proves the Zeeman effect.
(v) For a given value of n the total value of m is equal to n2 .

56
(vi) For a given value of l the total value of m is equal to (2l 1).
(vii) Degenerate orbitals : Orbitals having the same energy are known as degenerate orbitals.
e.g. for p subshell p x py pz
(4) Spin quantum numbers (s)
(i) It was proposed by Goldshmidt & Ulen Back and denoted by the symbol of s.
(ii) The value of ' s' is 1/2 and - 1/2, which signifies the spin or rotation or direction of electron
on its axis during movement.
(iii) The spin may be clockwise or anticlockwise.
h
(iv) It represents the value of spin angular momentum is equal to s (s 1).
1 2
(v) Maximum spin of an atom (spin multiplicity) 2 number of unpaired electrons
(vi) This quantum number is not the result of solution of Schrodinger equation as solved for H-
atom.
Distribution of electrons among the quantum levels

Designation Number of Orbitals in the


n l m
of orbitals subshell
1 0 0 1s 1
2 0 0 2s 1
2 1 1, 0, +1 2p 3
3 0 0 3s 1
3 1 1, 0, +1 3p 3
3 2 2, 1, 0, +1, +2 3d 5
4 0 0 4s 1
4 1 1, 0, +1 4p 3
4 2 2, 1, 0, +1, +2 4d 5
4 3 3, 2, 1, 0, +1, +2, +3 4f 7

Shape of orbitals
(1) Shape of s orbital
(i) For s orbital l=0 & m=0 so s orbital have only one unidirectional orientation i.e. the
probability of finding the electrons is same in all directions.
(ii) The size and energy of s orbital with increasing n will be 1s 2s 3s 4 s.

1S 2S

(iii) s-orbitals known as radial node or nodal surface. But there is no radial node for 1s orbital
since it is starting from the nucleus.

57
(2) Shape of p orbitals
(i) For p orbital l=1, & m=+1,0,1 means there are three p orbitals, which is symbolised
as p x , p y , p z .
(ii) Shape of p orbital is dumb bell in which the two lobes on opposite side separated by the
nodal plane.
(iii) p-orbital has directional properties.

(3) Shape of d orbital


(i) For the d orbital l =2 then the values of m are 2, 1, 0, +1, +2. It shows that the d
orbitals has five orbitals as d xy , d yz , d zx , d x 2 y 2 , d z 2 .

(ii) Each d orbital identical in shape, size and energy.


(iii) The shape of d orbital is double dumb bell .
(iv) It has directional properties

(4) Shape of f orbital

(i) For the f orbital l=3 then the values of m are 3, 2, 1,0,+1,+2,+3. It shows that the
f orbitals have seven orientation as f x ( x2 y 2 ) , f y ( x2 y 2 ) , f z ( x2 y2 ), f xyz , f z 3 , f yz 2 and f xz 2 .

(ii) The f orbital is complicated in shape.

Rules for filling of electrons in various orbitals


The atom is built up by filling electrons in various orbitals according to the following rules,
(1) Aufbaus principle
This principle states that the electrons are added one by one to the various orbitals in order of their
increasing energy starting with the orbital of lowest energy. The increasing order of energy of various
orbitals is 1s 2s 2p 3s 3 p 4 s 3d 4 p 5s 4d 5 p 6s 4 f 5d 6 p 7 s 5 f 6d 7 p.........

58
(2) (n+ ) Rule
In neutral isolated atom, the lower the value of (n + ) for an orbital, lower is its energy. However,,
if the two different types of orbitals have the same value of (n + ), the orbitals with lower value of
has lower energy.
(3) Paulis exclusion principle
According to this principle no two electrons in an atom will have same value of all the four quantum
numbers.
(4) Hunds Rule of maximum multiplicity
Electron pairing in and orbitals cannot occur until each orbitals of a given subshell contains one
electron each or is singly occupied.
As we now know the Hunds rule, let us see how the three electrons are arranged in orbitals.
The important point ot be remembered is that all the singly occupied orbitals should have electrons
with parallel spins i.e in the same direction either-clockwise or anticlockwise .
2px 2py 2pz 2px 2py 2pz
or

Electronic configurations of elements


On the basis of the electronic configuration principles the electronic configuration of various elements are
given in the following table :
The above method of writing the electronic configurations is quite cumbersome. Hence, usually the electronic
configuration of the atom of any element is simply represented by the notation.
Number of
electrons

number of
principal
nlx symbol of
subshell

Some Unexpected Electronic Configuration


Some of the exceptions are important though, because they occur with common elements, notably chromium
and copper.
Cu has 29 electrons. Its excepted electronic configuration is 1s 2 2s 2 2 p 6 3s 2 3 p 6 4 s 2 3d 9 but in reality the
configuration is 1s 2 2s 2 2 p6 3s 2 3 p 6 4 s 1 3d10 as this configuration is more stable. Similarly Cr has the configuration
of 1s 2 2s 2 sp 6 3s 2 3 p 6 4 s 1 3d 5 instead of 1s 2 2s 2 2 p 6 3s 2 3 p 6 4 s 2 3d 4 .
Factors responsible for the extra stability of half-filled and completely filled subshells,
(i) Symmetrical distribution : It is well known fact that symmetry leads to stability. Thus the electronic
configuration in which all the orbitals of the same subshell are either completely filled or are exactly
half filled are more stable because of symmetrical distribution of electrons.
(ii) Exchange energy : The electrons with parallel spins present in the degenerate orbitals tend to
exchange their position. The energy released during this exchange is called exchange energy. The
number of exchanges that can take place is maximum when the degenerate orbtials (orbitals of same
subshell having equal energy) are exactly half-filled or completely. As a result, the exchange energy
is maximum and so it the stability.

59
M.C.Q.

1. What is wrong about anode rays?


(A) Their e/m ratio is constant
(B) They are deflected by electrical and magnetic field
(C) They are produced by ionisation of molecules of the residual gas
(D) Their elm ratio depends on nature of residual gas.
2. When atoms of the gold sheet are bombarded by a beam of -particles, only a few -particles
get deflected whereas most of them go straight undeflected. This is because
(A) The force of attraction on -particles by the oppositely charged electron is not sufficient
(B) The nucleus occupies much smaller volume as compared to the volume of atom
(C) The force of repulsion on fast moving -particles is very small
(D) The neutrons in the nucleus do not have any effect on -particles.
3. Which of the following is not a characteristic of Plancks quantum theory of radiations?
(A) Radiations are associated with energy
(B) Magnitude of energy associated with a quantum is equal to hv
(C) Radiation energy is neither emitted nor absorbed continuously
(D) A body can emit less or more than a quantum of energy.
4. Which of the following statements is wrong? The probability of finding the electron in px
orbital is
(A) Maximum on two opposite sides of the nucleus along x-axis
(B) zero at the nucleus
(C) same on all the sides around the nucleus
(D) zero on the z-axis
5. In uni electron system, the wave number of any spectral line is directly proportional to
(A) the number particles present in the system
(B) the velocity of electron undergoing transition

(C) (D) the charge on the nucleus and the of light used.
6. The conclusion that every additional electron enters the orbital with lowest possible energy
has been drawn from
(A) Paulis exclusion principle (B) Hunds rule
(C) Aufbau principle (D) de-Broglies equation.
7. Bohrs model of atom is not in agreement with
(A) Line spectra of hydrogen atom (B) Paulis principle
(C) Plancks theory (D) Heisenbergs principle.

60
8. Which of the following statements is correct?
(A) All electromagnetic radiations do not possess the same velocity
(B) Matter waves are associated with electrical and magnetic fields
(C) Matter waves and electromagnetic radiations are alike
(D) The velocityof matter wave is generally less than that of light
9. Which experimental observation correctly account for the phenomenon?
Experimental observation Phenomenon
(A) X-rays spectra Charge on nucleus
(B) -particle scattering Quantized electron orbit .
(C) Photoelectric effect The nuclear atom
(D) Emission spectra Quantizationof energy. .
10. In the Schrodingers wave equation represents
(A) orbit (B) wave function (C) wave (D) radial probability
11. Which of the following gave the idea of a nucleus of the atom?
(A) Oil drop experiment (B) Davissonand Germers experiment
(C) -ray scattering experiment (D) Austens mass spectrogram experiment.
12. Cathode rays have same charge to mass ratio as
(A) -particles (B) -rays (C) Anode rays (D) Protons
13. Which of the following statements is/are correct?
(A) Isotopes have same number of nucleons
(B) Isobars have same number of protons
(C) Isotones have same number of neutrons
(D) Isobars are atoms of different elements with same isotopic number
14. According to Bohrs Model of hydrogen atom
(A) Total energy of the electron is quantized
h
(B) Angular momentum of the electron is quantized and given as l l + 1
2
(C) Both (A) and (B)
(D) None of the above.
15. Rutherfords experiment established that
(A) inside the atom there are positive centres immersed in sea of electrons
(B) nucleus contains protons, neutrons and mesons
(C) most of the space in an atom is empty
(D) all A, B and C.

61
16. Mathematically, Heisenbergs uncertainty principle can best be explained by

(A) (B) (C) (D)

17. The correct Schrodingers wave equation for an electron with E as total energy and V as
potential energy is
2 2 2 2 2 2
(A) 2 + 2 + 2 + ( ) =0 (B) 2 + 2 + 2 + ( ) =0

2 2 2 2 2 2
(C)
(C) 2 + 2 + 2 + ( ) = 0 (D)= 0(D)
(D) 2 + 2 + 2 + ( ) =0

18. In which of the following electron distributions in ground state, only the Hunds rule is
violated?

(A) (B)

(C) (D)

19. Which of the following expressions imposes the condition of quantization of energy of an
electron in an atom?

(A) E mc2 (B) E h v (C) l (D) mvr


20. The total number of electrons in a subshell designated by azimuthal quantum number, l is given
as
(A) 2 l + 1 (B) l2 (C) 4 l + 2 (D) 2l + 2.
21. According to Bohrs model of the atom, an electron can revolve around the atomic nucleus in
a suitable orbit without emitting energy if its orbit
(A) is a perfect circle (B) is a circle with a large radius
(C) houses a whole number of de-Broglie waves
(D) houses odd number of de-Broglie waves.
22. Which of the following concerning Bohrs model is false?
(A) Predicts that probability of electron near nucleus is more
(B) Angular momentum of electron in H atom
(C) Introduces the idea of stationary states
(D) Explains line spectrum of hydrogen.
23. The kinetic energy of the electron in the nth orbit of an atom is given by the relation (A) (B)
(C) (D) none of these
24. According to Somerfields model, only circular orbit is possible for the electron in
(A) K shell (B) L shell (C) M shell (D) N shell.

62
25. According to Bohrs atomic model
(A) electron on H atom can have only certain values of angular momentum
(B) electrons have a particle as well as wave character
(C) atomic spectrum of atom should contain only five lines
(D) all the above statements are correct. .
26. The transition of electrons inH atom that will emit maximum energy is
(A) n3 n2 (B) n4 n3 (C) n5 n4 (D) n6 n5
27. The limiting line in Balmer series will have a frequency of
(A) 3.29 1015s1 (B) 3.65 1014 s1 (C) 8.22 1014s1 (D) 9.22 1014s1
28. The wavelength of a spectral line for electron transition is inversely related to
(A) Z (nuclear charge) (B) velocity of electron
(C) number of electrons undergoing transition
(D) the energy difference between the energy levels involving transition.
29. The phenomenon of splitting of spectral lines under the influence of the electric field is known
as
(A) Photoelectric effect (B) Stark effect
(C) Zeeman effect (D) Electromagnetic effect.
1312
30. If the energy of electron in H atom is given by expression, kJ mol-1, then the energy
n2
required to excite the electron from ground state to second orbit is
(A) 328 kJ (B) 656 kJ (C) 984 kJ (D) 1312 kJ.
31. In the atomic spectrum of hydrogen, the spectral lines pertaining to electronic transition of
n 4 to n 2 refers to
(A) Lyman lines (B) Balmer lines (C) Paschen lines (D) Brackett lines.
32. When electronic transition occurs from higher energy state to a lower energy state with energy
difference equal to E electron volts, the wavelength of line emitted is approximately equal to
12397 12397 12397
(A) 12397 1010 m (B) 1010 m (C) 1010 cm (D) 1010 cm
E E E E

33. If ionising energy of H atom is 13.6 eV, then the second ionising energy of He should be
(A) 13.6eV (B) 27.2eV
(C) 54.4 eV (D) Cannot be predicted.
34. The first line in the Balmer series in the H atom will have the frequency
(A) 4.57 1014S1 (B) 3.29 1015s1 (C) 8.22 1015S1 (D) 8.05 1013S1.
35. The orbital configuration of 24Cr is 3d54s1. The number of unpaired electrons in is
(A) 3 (B) 2 (C) 1 (D) 4.

63
36. How many electrons in 19Khave n 3; l0 ?
(A) 1 (B) 2 (C) 4 (D) 3.
37. The maximum number of electrons in s, p and d-subshells are
(A) 2 in each (B) 2, 4 and 6 (C) 2, 6 and 10 (D) 2, 6 and 12.
38. In an atom which has 2K, 8L, 8M and 2N electrons in the ground state, the total number of
electrons having 1are
(A) 20 (B) 8 (C) 12 (D) 10.
39. The number of electrons that can be accommodated in dxy orbital is
(A) 10 (B) 4 (C) 1 (D) 2.
40. The electronic configuration of Mn can be written as
(A) [Ar] 4s2 (B) [Ar] 3d6,4s2 (C) [Ar] 3d5, 4s1 (D) [Ar] 3d5,4s2.
41. Which of the following sets of quantum numbers is not possible for 23rd electron of Cr( At.
no. 24) ?
(A) 3, 2, +2, - (B) 3, 2, -2, + (C) 3, 2, + 1, + (D) 3, 1, + 1, + .
42. Which d-orbital does not have four lobes?
(A) dx2-y2 (B) dxy (C) dyz (D)
43. The total number of electrons present in any main energy level can be calculated from
(A) (2l + 1) (B) 2n2 (C) (2n+1) (D)n2.
44. Which shape is associated with the orbital designated by n 2; / 1 ?
(A) spherical (B) tetrahedral (C) dumb-bell (D) pyramidal.
45. Which of the following statements about quantum numbers is wrong?
(A) If the value of l 0, the electron distribution is spherical
(B) The shape of the orbital is given by subsidiary quantum number
(C) The Zeemans effect is explained by magnetic quantum number
(D) The spin quantum number gives the orientations of electron cloud.
46. The two electrons have the following sets of quantum numbers . X: 3, 2, -2, +
Y : 3, 0, 0, + . What is true of the following?
(A) X and Y have same energy (B) X and Y have unequal energy
(C)X and Y represent same electron (D) None of the statements is correct.
47. If the value of azimuthal quantum number of an electron is 2, then which of the following
values of magnetic quantum numbers is not permissible,
(A) 3 (B) 2 (C) 0 (D) 1.
48. An isotone of 32Ge76is (i) 32Ge77 (ii) 33As77 (iii) 34Se77 (iv) 34Se78.
(A) only (i) & (ii) (B) only (ii) & (iii) (C) only (ii) & (iv) (D) (ii), (iii) & (iv)

64
49. The fundamental particles which arc responsible for keeping nucleons together is
(A) Meson (B) Antiproton (C) Positron (D) Electron. .
50. The positron is as heavy as
(A) electron (B) neutron (C) alpha particle (D) proton.
51. Atoms may be regarded as comprising of protons, electrons and neutrons. If the mass
attributed to the neutrons were halved and that attributed to electrons were doubled then
atomic mass of would
(A) remain approximately the same (B) be doubled
(C) Approximately be halved (D) be reduced by approximately 25%.
52. How many electrons in an atom with atomic number 105 can have (n +l) 8 ?
(A) 30 (B) 17 (C) 15 (D) Unpredictable.
53. The size of the nucleus is approximately
(A) 1/100th of the atom (B) 1/1000 th of the atom
(C) 1/10000th of the atom (D) 1/l00000th of the atom.
54. The line spectrum ot two elements is not identical because .
(A) they do not have same number of neutrons
(B) they have dissimilar mass number
(C) they have different energy level schemes
(D) they have different number of valence electrons.
55. Bohrs atomic model can explain the spectrum of
(A) hydrogen atoms only
(B) atoms or ions which are uni electron
(C) atoms or ions which have only two electrons
(D) hydrogen molecule.
56. The electronic configuration of a dipositive ion M+2 is 2, 8, 14 and its mass number is 56. The
number of neutrons present is
(A) 32 (B) 42 (C) 30 (D) 34.
57. An electron of mass m and charge -e moves in circular orbit of radius r around the nucleus of
charge + Ze in uni electron system. In C.G.S. system the potential energy of electron is
(A) (B) (C) (D)
58. An atom has 2 K, 8 L, 11 M, 2 N electrons, the total number of s-electrons will be
(A) 6 (B) 8 (C) 10 (D) 4.
59. In an atom with 2K, 8L, 11M and 2N electrons the number of electrons with m 0 ; s +
are
(A) 2 (B) 7 (C) 8 (D) 16.

65
60. For a sub-shell with azimuthal quantum number l, the total values of magnetic quantum number
m can be related to l as
( 1)
(A) m ( +2) (B) m (2 2 + 2) (C) = 2
(D) 2m+1.

61. An orbital with 0 is


(A) symmetrical about X-axis only (B) symmetrical about Y-axis only
(C) spherically symmetrical (D) unsymmetrical.
62. The units for equation are
2 2
3
2 1 2
(A) 1 (B) 1
(C) 2 1 (D)

63. The momentum of a photon of frequency 50 1017S1 is nearly


(A) 1.1 1023 kg ms1 (B) 3.33 1043 kg ms1
(C) 2.27 1040 kg ms1 (D) none
64. A near U.V. photon of 300 nm is absorbed by a gas and then remitted as two photons. One
photon is red with wavelength760 nm. Hence wavelength of the second photon is
(A) 460 nm (B) 1060nm (C) 496nm (D) 300nm
65. The transition in He+ ion that would have the same wave number as the first Lyman line in
hydrogen spectrum is
(A) 21 (B) 53 (C) 42 (D) 64
66. The work function of a metal is 4.2 eV. If radiations of 2000 A0 fall on the metal then the
kinetic energy of the fastest photo electron is
(A) 1.6 1019J (B) 16 1010J (C) 3.2 1019J (D) 6.6 1010J
67. A certain metal when irradiated to light ( 3.2 1016 Hz) emits photoelectrons with twice
kinetic energy as did photo electrons when the same metal is irradiated by light ( 2.0
1016Hz). The of metal
(A) 1.2 1014 Hz (B) 8 1015 Hz (C) 1.2 1016 Hz (D) 4 1012 Hz
68. The ratio of the radil of the first three Bohr orbit in H atom is
1 1
(A) 1: (B) 1:2:3 (C) 1:4:9 (D) 1:8:27
2 3
Hint : rn n2 a0 i.e. r n2 r1 : r2 : r3 12 : 22 : 32 1 : 4 : 9
69. An electron of a velocity is found to have a certain value of de-Broglie wavelength. The
velocity to be possessed by the neutron to have the same de-Broglie wavelength is
1840
(A) (B) (C) 1840 (D)
1840

70. The momentum of a particle associated with de-Broglies wave length of 6 A0 is


(A) 1.1 1024kgms1 (B) 1.1 1034kgms1
(C) 39.6 1034kgms1 (D) 39.6 1024kgms1

66
71. Frequency of matter wave is equal to v frequency, v velocity of particle
(A) (K.E.)/2h (B) 2.(KE.)/h (C) (K.E./h) (D) l
v v v2 2. . 1
= v
v= = = v
=
=
( K.E. = 2 v2 )

72. If threshold wavelength for ejection of electron from metal is 300nm, then work function for
photoelectric emission is
(A) 1.2 1018 J (B) 1.2 1020 J (C) 6 1019 J (D) 6 1012 J

Work function is h 0 = = 6 1019 J
0

73. The mass of one mole of electron is


(A) 9.1 1028g (B) 0.55mg (C) 9.1 1024g (D)1.008mg
74. In two H atoms A and 13 the electrons move around the nucleus in circular orbits of radius r
and 4r respectively. The ratio of the times taken by them to complete one revolution is
(A) 1: 4 (B) 1: 2 (C) 1: 8 (D) 2: 1
75. For the electronic transition from n 2 to n 1 which one of the following will produce shortest
wave length?
(A) H atom (B) D atom (C)He+ ion (D) Li+2ion
76. The energy required to dislodge electron from excited isolated H atom (IE1 13.6eV) is
(A) 13.6eV (B) >13.6eV (C) <13.6eVand >3.4eV (D) 3.4eV
77. Velocity of electron in the first orbit of H-atom is compared to that of velocity of light is
approximately
1 1 1
(A) (B) (C) (D) same
1000 137 10

78. How many electrons in Cu atom have (n + ) 4


(A) 6 (B)7 (C)8 (D)4
79. In any subshell the maximum number of electrons having same value of spin quantum number
is
(A) ( + 1) (B) l + 2 (C) 2l + 1 (D) 4l + 2
80. For a certain particle, it is found that uncertainty 1840 in velocity is reciprocal of uncertainty
in position This implies that
(A) Mass of the particle is > unity (B) Mass of the particle is unity
(C) Mass of the particle < h (D) Mass of the particle > h / 4
81. How many electrons in K (Z19) have n 4 and ?
(A)1 (B)2 (C) 3 (D)4
82. Suggest two transitions in hydrogen spectrum for which wave number ratio is 108:7
(A) 21, 31 (B) 21, 43 (C) 21, 54 (D) 21, 41

67
83. The ratio of K.E. to P.E. of an electron in Bohr orbit of H like species is
(A) 1 : 2 (B) 1 : 2 (C) 1 : 1 (D) 1 : 1
84. For particles having same KE., the de-broglie wavelength is
(A)Directly proportional to its velocity, (B) Inversely proportional to its velocity ,
(C) Independent of its mass and velocity (D) Unpredictible.
85. In one joule of energy, the number of photon with wave number equal to is
(A) ( )1 (B) ( )1 (C) ( ) (D) ( )1
86. Which of the expression given below gives IE of H atoms in terms of Rydbergs constant (RH)?
(A) RH.hc (B) RH.NA.hc (C) RH(2hc) (D) RH.c
87. One unpaired electron causes magnetic moment of 1.1 B.M. The magnetic moment of Fe+2
ion is
(A) 4.4 (B) 5.5 (C) 2.2 (D)0
88. If numerical value of mass and velocity are equal then de-Broglie wave length in terms of K.E.
is

(A) (B) (C) both correct (D) none
2 . . 2 . .
89. If uncertainty in position and velocity are equal then uncertainty in momentum will be


(A) (B) (C) (D)
4 4

90. Number of waves made by a Bohr electron in one complete revolution in its fourth orbit is
(A) 2 (B) 3 (C) 4 (D) infinite
91. Which of the following electronic transitions require that the greatest amount of energy be
absorbed by a hydrogen atom?
(A) n1 to n2 (B) n2 to n4 (C) n 3 to n 6 (D) n to n 1
92. How fast an electron is moving if it has a wavelength equal to the distance it travel in one
second?

(A) / (B) / (C) / (D) /2( . )


93. If uncertainties in the measurement of position and momentum are equal, the uncertainty in the
measurement of velocity is

1 1
(A) (B) (C) (D) none of these
2 2

94. Energy equivalent of 10.00 cm1 is


(A) 19.9 1023 Jper photon (B) 28.6 103 kcal mol1photon
(C)12.0 103 kJ mol1photon (D) all are correct

68
95. The ratio of energy of the electron in ground state of hydrogen to the electron in first excited
state of Be+3 is
(A) 1: 4 (B) 1: 8 (C) 1: 16 (D) 16: 1
96. The shortest wavelength of H-atom in Lyman series is x, then longest wavelength in Balmer
series of He+ is
9 36 5
(A) (B) (C) (D)
5 5 4 9

97. The radius of first Bohr orbit is , then de-Broglie wavelength of electron in 3rd orbit is nearly
(A) 2 x (B) 6 x (C) 9x (D) x/3
98. With increasing principle quantum number, the energy difference between adjacent energy
levels in H atom
(A) decreases (B) increases (C) remains constant
(D) decreases for low value of Z and increases for higher value of Z,
99. The electrons present in K-shell of the atom will differ in
(A) principal quantum number (B) azimuthal quantum number
(C) magnetic quantum number (D) spin quantum number.
100. The ratio of the ionisation energy of H and Be3+ is.............. (Pb. C.E.T. 1996)
(A) 1: 1 (B) 1: 3 (C) 1 : 9 (D) 1: 16.
101. The maximum number of electrons in a subshell for which l 3 is
(Andhra B. Tech 1982)
(A) 14 (B) 10 (C) 8 (D) 4.
102. The number of electrons in the M shell of the clement with atomic number 24 is
(Andhra B. Tech. 1982)
(A) 24 (B) 12 (C) 13 (D) 8.
103. Sodium chloride imparts a yellow colour to the Bunsen flame. This can be interpreted due to
the
(A) low ionization energy of sodium
(B) sublimation of metallic sodium to give yellow vapour
(C) emission of excess energy absorbed as a radiation in the visible region
(D) photosensitivity of sodium.
104. The exact path of electron 2p-orbital cannot be determined. The above statement is based
upon (Delhi P.M. T. 1981)
(A) Hunds Rule (B) Bohrs Rule (C) Uncertainty principle (D) Aufbau principle.
105. The maximum number of electrons in a subshell is given by the expression
(C.B.S.E. 1989)
(A) 4l - 2 (B) 4l + 2 (C) 2l + 1 (D) 2n2.

69
106. If r is the radius of first orbit, the radius of nth orbit of the H atom will be
(C.B.S.E. 1989)
(A) r n2 (B) r n (C) r/n (D) r2n2
107. The energy of hydrogen atom in its ground state is -136 eV. The energy of the level
corresponding to the quantum number n 5 is
(A) 054 eV (B) 540 eV (C) 085 eV (D) 272 eV.
(C.B.S.E. 1990, M.P.C.E.T. 1999)
108. At 200C hydrogen molecules have velocity 105 cm sec1. The de-Broglie wavelength in this
case is approximately
(A) 2 (B) 1000 (C) 100 (D) 10 .
(C.B.S.E. 1991)
109. The number of electrons in 3d shell for element with atomic number 26 is
(A) 4 (B) 6 (C) 8 (D) 10.
(C.E.E. T. Sample Paper 1992)
110. In a set of degenerate orbitals the electrons distribute themselves to retain similar spins as far
as possible. This statement is attributed to
(A) Paulis exclusion principle (B) Aufbau principle
(C) Hunds Rule (D) Slater rules. (Pb. C.E. T. 1989)
111. The ground state configuration of Fe3+ ion in gaseous state is: (At. No. of Fe 26}
(A) [Ar]18 3d3 4s2 (B) [Ar] 18 3d6 4s2 (C) [Ar] 18 3d5 (D) [Ar]18 3d6.
(Karnataka C.E.E.T. 1992)
112. If uncertainty in the position of electron is zero, the uncertainty in its momentum would be

(A) zero (B) > (C) < (D) infinite.


(B.H.U.1992)
113. The radius of second Bohrs orbit is
(A) 0053 nm (B) 0053/4 nm (C) 0053 x 4nm (D) 0053 x 20 nm.
(B.H. U. 1989)
114. For which of the following sets of quantum -numbers an electron will have the highest
energy ?
(A) 3,2,1,1/2 (B) 4,2,-1,1/2 (C) 4, 1,0, -1/2 (D) 5,0,0, 1/2.
(C.B.S.E. 1994)
115. The uncertainty in the position of an electron (mass 9.1 1028 g) moving with a velocity of
3.0 x 104 cms1 accurate up to 0.011% will be
(A) 192 cm (B) 768 cm (C) 0175 cm (D) 384cm.
(C.B.S.E. 1995)

70
116. The radius of hydrogen atom in the ground state is 053 , the radius of 3Li2+ in the similar
state is
(A) 106 (B) 0265 (C) 017 (D) 053 .
(C.BS.E. 1995)
117. Splitting of spectral lines under the influence of magnetic field is called
(A) Stark effect (B) Zeeman effect (C) Photoelectric effect (D) None of these.
(A.F.M.C. 1995)
118. The total number of orbitals in a shell with principal quantum number n is
(A) 2n (B) 2n2 (C) n2 (D) n + 1.
(A.I.I.M.S. 1997)
119. Which of the following expressions gives the de-Brogiie relationship ?

(A) =p (B) l = (C) l = (D) lm = (M.P.P.M.T. 1998)

120. The uncertainty in the momentum of an electron is 10 105 kg ms1. The uncertainty in its
position will be (h 662 x 1034 kg m2s1)
(A) 105 1028 m (B) 105 1026 m (C) 527 1030 m (D) 525 1028 m.
(Pune A.F.M.C. 1998)
121. If the radius of first Bohr orbit be a0, then the radius of third Bohr orbit would be
(A) 3 x a0 (B) 6 x a0 (C) 9 x a0 (D) 1/9 x a0.
(M.P.C.E.T. 1998)
122. The first emission line in the atomic spectrum of hydrogen in the Balmer series appears at
9 7 3 5
(A) cm1 (B) cm1 (C) cm1 (D) cm1
400 144 4 36

(EA.M.C.E.T. 1998)
123. The ratio of specific charge of a proton and an particle is
(A) 2 : 1 (B) 1 : 2 (C) 1 : 4 (D) 1 : 1
(M.P.C.E.T 1999)
124. The de-Broglie wavelength of a particle with mass 1g and velocity 100 m/s is
(A) 663 1033m (B) 663 1034m (C) 663 1035m (D) 665 1035m.
(C.B.S.E. 1999)
125. Which of the following sets of quantum numbers belongs to highest energy ?

(A) n 4, l 0, m 0, s + 12 (B) n 3, l 0, m 0, s + 1
2

1 1
(C) n 3, l 1, m 1, s + 2 (D) n 3, l 2, m 1, s + 2

(C.P.M.T. 1999)

71
126. If wavelength of photon is 22 1011 m, h 66 1034 Js, then momentum of photon is
(A) 3 1023 kg ms1 (B) 333 1022 kg ms1
(C) 1452 1044 kgms1 (D) 689 1043 kg ms1.
(M.P.C.E.T. 1999)
127. According to Bohrs theory, the energy required for the transition of H atom from n 6
to n 8 state is
(A) equal to energy required for the transitions from n 5 to n 7 state
(B) larger than in (A)
(C) less than in (A)
(D) equal to energy for the transition from n 7 to n 9 state (Kerala M.E.E. 2000)
128. An electron has kinetic energy of 214 10 22J.Its de-Broglie wavelength will be nearly
(me 9.1 1031 kg)
(A) 928 104 (B) 928 107m (C) 928 108m (D) 928 1010m.
(M.P.C.E.T. 2000)
129. What will be de-Broglie wavelength of an electron moving with a velocity of 120 105ms1 ?
(A) 6068 109 (B) 3133 1037 (C) 6626 109 (D) 6018 107
(M.P.CE.T. 2000)
130. The de-Broglie wavelength associated with ball of mass 200 g and moving at a speed of 5 m
hour1 is of the order of (h 6625 x 1034 Js)
(A) 1015m (B) 1020m (C) 1030m (D) 1025m
(Kerala P.M.T. 2001)
131. The third line of the Balmer series. in the emission spectrum of the hydrogen atom, is due to
the transition from the
(A) fourth Bohr orbit to the first Bohr orbit (B) fifth Bohr orbit to the second Bohr orbit
(C) sixth Bohr orbit to the third Bohr orbit (D) seventh Bohr orbit to the third Bohr orbit
(Kerala P.M.T. 2001)
132. The highest number of unpaired electrons are w present in (D.C.E. 2001)
(A) Fe (B) Fe4+ (C) Fe2+ (D) Fe3+.
133. Rutherfords atomic model suggests the existence
(A) Atom (B) Nucleus (C) -particle (D) Mesons
(A.EM.C. 2001)
134. Which is not true with respect to cathode rays?
(A) A stream of electrons (B) Charged particles
(C) Move with speed as that of light (D) Can be deflected by magnetic fields
(Kerala C.E.T. 2001)

72
135. A element M has an atomic mass 19 and atomic number 9. Its ion is represented by
(A) M+ (B) M2+ (C) M (D) M2.
(Manipal P.M.T. 2001)
136. Which of the following ions has the maximum magnetic moment ?
(A) Mn2+ (B) Fe2+ (C) Ti2+ (D)Cr2+.
(A.I.E.E.E. 2002)
137. The value of the energy for the first excited state of hydrogen will be
(A) 136eV (B) 340eV (C) 1.51eV (D) 085eV.
(M.P.C.E.T. 2002)
138. In hydrogen atom, energy of first excited state is -34 eV. Find out the K.E. of the same orbit
of hydrogen atom
(A) +34eV (B) +68eV (C) 136eV (D) +136eV
(C.B.S.E. P.M.T. 2002)
139. The energy of the first electron in helium will be
(A) 136eV (B) 544eV (C) 544eV (D)zero.
(Bihar C.EE. 2002)
140. In a hydrogen atom, if the energy of an electron in the ground state is 136 eV, then that in the
2nd excited state is
(A) 151eV (B) 34eV (C) 604cV (D) 136eV.
(A.I.E.E.E. 2002)
141. In the Bohrs orbit, what is the ratio of total kinetic energy and total energy of the electron
(A) 1 (B) 2 (C) 1 (D) +2.
(Rajashan P.M.T. 2002)
142. The ratio between kinetic energy and the total f energy of the electrons of hydrogen atom
according to Bohrs model is
(A) 2 : 1 (B) 1 : 1 (C) 1 : (1) (D) 1 : 2.
(Pb. P.M.T. 2002)
143. In Bohr series of lines of hydrogen spectrum, the third line from the red end corresponds to
which one of the following inter-orbit jumps of the electron for Bohr orbits in an atom of
hydrogen ?
(A) 3 2 (B) 5 2 (C) 4 1 (D) 2 5.
(A.I.E.E.E. 2003)

144. The orbital angular momentum for an electron revolving in an orbit is given by ( + 1) .
2
This momentum for an s-electron will be given by

73
1
(A) + (B) zero (C) (D) 2
2 2 2 2

(A.I.E.E.E. 2003)
145. The atomic number of an element is 35. What is the total number of electrons present in all
the p-orbitals of the ground state atom of that element ?
(A) 6 (B) 11 (C) 17 (D) 23.
(EA.M.C.E.T. 2003)
146. The emission spectrum of hydrogen is found to satisfy the expression for the energy change
E (in Joules) such that E 218 x 1018 J where n1 1, 2, 3 ........ and n2 2, 3, 4,......
The spectral lines correspond to Paschen series it
(A) n1 1 and n2 2, 3, 4 (B) n1 3 and n2 4, 5, 6
(C) n1 1 and n2 3, 4, 5 (D) n1 2 and n2 3, 4, 5
(Kerala Engg. 2003)
147. Among the following series of transition metal ions, the one where all metal ions have same
3d electronic configuration is
(A) Ti2+, V3+, Cr4+, Mn5+ (B) Ti3+, V2+, Cr3+, Mn4+
(C) Ti+, V4+, Cr6+, Mn7+ (D) Ti4+, V3+, Cr2+, Mn3+
(C.B.S.E. P.M.T. 2004)
148. For d-electron, the orbital angular momentum is

(A) 6 / 2 (B) 2 / 2 (C) h / 2 (D) 2h/.


(J & K Med.2004)
149. Time taken for an electron to complete one revolution in the Bohr orbit of hydrogen atom is
4 2 2 2
(A) (B) (C) (D)
4 2 2 2 2

(Kerala P.M.T. 2004)


150. Which of the following sets of quantum numbers is correct for an electron in 4f orbital ?
(A) n 4, 3, m +4, s + (B) n 3, 2, m 2, s +
(C) n 4, 3, m + 1, s + (D) n 4, 4, m 4 s -
(A.I.E.E.E. 2004)
151. The wavelength of radiation emitted when in a hydrogen atom electron falls from infinity to
stationary state 1, would be (Rydberg constant 109 107m1)
(A) 91nm (B) 91 108nm (C) 406 nm (D) 192 nm
(A.I.E.E.E. 2004)
152. The relationship between energy E, of the radiation with a wavelength 8000 and the energy
of the radiation with a wavelength 16000 is
(A) E1 6E2, (B) E1 2E2 (C) E1 4E2 (D) E1 1/2E2
(Kerala Engg. 2005)
74
153. The energy of second Bohr orbit of the hydrogen atom is 328 kJ mol1, hence the energy of
fourth Bohr orbit would be
(A) 41 kJ mol1 (B) 1312 kJ mol1 (C) 164 kJ mol1 (D) 82 kJ mol1
(C.B.S.E. P.M.T. 2005)
154. Which of the following statements in relation to the hydrogen atom is correct?
(A) 3s-orbital is lower in energy than 3p-orbital
(B) 3p-orbital is lower in energy than 3d-orbital
(C) 3s and 3p orbitals arc of lower energy than 3d orbital
(D) 3s, 3p and 3d orbitals all have the same energy. (A.I.E.E.E. 2005)
155. The correct order of number of unpaired electrons in the ions Cu2+, Ni2+, Fe3+ and Cr3+ is
(A) Cu2+ > Ni2+ > Cr3+ > Fe3+ (B) Cr3+ > Fe2+ > Ni2+ > Cu2+
(C) Fe3+ > Cr3+ > Cu2+ > Ni2+ (D) Fe3+ > Cr3+ > Ni2+ > Cu2+.
(Kerala P.M.T. 2005)
156. The most probable radius (in pm) for finding the electron in He+ is
(A) 00 (B) 529 (C) 265 (D) 1058
(A.I.I.M.S. 2005)
157. Ground state electronic configuration of nitrogen atom can be represented by

(A) (B)

(C) (D)

(I.I.T. 1999)
158. Which one of the following sets of quantum number represeni an impossible arrang- ement ?
n I ml ms n I m1 ms
(A) 3 2 2 (B) 4 0 0
(C) 3 2 3 (D) 5 3 0 .
(I.I.T. 1986)
159. The orbital diagram in which Aufbau principle is violated is

(A) (B)

(C) (D)

75
160. If the speed of electron in Bohr first orbit of hydrogen atom be x, then speed of the electron
in 3rd orbit is
(A) x/9 (B) x/3 (C) 3x (D) 9x (I.I.T.1990)
161. If wavelength of photon is 22 10 11m, h 66 1034 Js, then momentum of photon is
(A) 3 1023 kg/s (B) 1452 1044 kg/s (C) 333 1022 kg/s (D) 689 1043 kg/s.
(Roorkee 1990)
162. The wave number of first line of Balmer series of hydrogen is 15200 cm1. The wave number
of the first Balmer line of Li1+ ion is
(A) 15200 cm1 (B) 60800 cm1 (C)76000cm1 (D) 136,800cm1.
(I.I.T. Screening 1992)
163. Which of the following is violation of Paulis exclusion principle ?

(A) (B)

(C) (D)

(I.I.T. Screening 1993)


164. The electrons, identified by quantum number n and (i) n 4, 1 (ii) n 4, 0
(iii) n 3, 2 (iv) n 3, 1 can be placed in order of increasing energy, from the lowest to
highest, as
(A) (iv) < (ii) < (iii) < (i) (B) (ii)< (iv) < (i) < (iii)
(C) (i) < (iii) < (ii) < (iv) (D) (iii) < (i) < (iv) < (ii) (I.I.T. 1999)
165. The wave length associated with a golf ball weighing 200 g and moving at a speed of 5 m/h is
of the order
(A) 1010m (B) 1020m (C) 1030m (D) 1040m.
(I.I.T. Screening2001)
166. If the nitrogen atom had electronic configuration 1s7, it would have energy lower than that of
the normal ground state configuration 1s2 2s2 2p3 because the electrons would be closer to
the nucleus. Yet, 1s7 is not observed because it violates.
(A) Heisenberg uncertainty principle (B) Hunds rule
(C) Paulis exclusion principle (D) Bohr postulates of stationary orbits.
(I.I.T. Screening2002)
167. The radius of which of the following orbits is same as that of the first Bohrs orbit of hydrogen
atom ?
(A) He+ (n 2) (B) Li2+ (n 2) (C) Li2+ (n 3) (D) Be3+ (n 2)
(I.I.T. Screening2004)

76
168. Radial nodes present in 3s and 2p -orbitals are respectively
(A) 0, 2 (B) 2, 0 (C) 2, 1 (D) 1,2.
(I.I.T. Screening2005)
169. Rutherfords experiment, which established the nuclear model of the atom, used a beam of
(A) - particles, which impinged on a metal foil and got absorbed
(B) - rays, which impinged on a metal foil and ejected electrons
(C) helium atoms, which impinged on a metal foil and got scattered
(D) helium nuclei, which impinged on a metal foil and got scattered (I.l.T. Screening2002)

ANSWER KEY

1 a 2 b 3 d 4 c 5 c 6 c 7 d
8 d 9 d 10 b 11 c 12 b 13 c 14 c
15 c 16 c 17 d 18 a 19 d 20 c 21 c
22 a 23 c 24 a 25 a 26 a 27 c 28 d
29 b 30 c 31 b 32 a 33 c 34 a 35 a
36 b 37 c 38 c 39 d 40 d 41 a 42 d
43 b 44 c 45 d 46 b 47 a 48 c 49 a
50 a 51 d 52 b 53 d 54 c 55 b 56 b
57 b 58 b 59 b 60 c 61 c 62 a 63 a
64 c 65 c 66 c 67 b 68 c 69 d 70 a
71 d 72 c 73 b 74 c 75 d 76 d 77 b
78 b 79 c 80 d 81 a 82 b 83 b 84 a
85 a 86 a 87 a 88 c 89 a 90 c 91 a
92 a 93 c 94 d 95 a 96 a 97 b 98 a
99 b 100 d 101 a 102 c 103 c 104 c 105 b
106 a 107 a 108 a 109 b 110 c 111 c 112 d
113 c 114 b 115 c 116 c 117 b 118 c 119 b
120 c 121 c 122 d 123 b 124 a 125 d 126 a
127 c 128 c 129 a 130 c 131 b 132 d 133 b
134 c 135 a 136 c 137 c 138 a 139 b 140 a
141 a 142 c 143 b 144 b 145 c 146 b 147 a
148 a 149 a 150 c 151 a 152 b 153 d 154 d
155 c 156 c 157 c 157 d 158 c 159 b 160 b
161 a 162 a 163 d 164 a 165 c 166 c 167 d
168 b
77
HI NT S

2 2 1
62. Unit of h = J s, J = kgm s , m = kg, =m
1 1 1
64. E -= E1 + E2 now = = +
1 2

2 1 1 1 1 1 1 1 1
65. = 2 2 = 2 2 = (2)2 2 2
1

68. rn = n2 a0 i.e. r n2 r1 : r2 : r3 = 12 : 22 : 32 = 1 : 4 : 9

69. e = n = = =
1840

v v v2 2. . 1
71. =
v
v= = =
v
=

=

( K.E. =
2
v2 )

Ze 2 Ze 2 K .E 1/ 2
83. K .E P.E 1: 2
2r r P.E 1
c 1
85. E hv nh nhcv now E 1, v x n (hcx) 1
hcx
13.6 Z 2
95. En 2
eV atom1 For hydrogen atom in ground state Z=1 and n=1 for Be+3 ion in first
n
excited state Z= 4 and n= 2

RH Z 2 RH 12
100. I .Eo I . E H RH
n2 12
n2
121. Radius of Bohr orbit a0
Z
122. for Balmer series n1 = 2 and n2 = 3 for first line
138. K.E. of e- in nth orbit = - En = 3.4 e.V
13.6 2
139. En Z e.V
n2
1 Ze 2
141. K .E mv 2 P.E
2 r
Electrostatic force = centrifugal force

Ze 2 mv 2
P.E mv 2
r2 r
1 2 1 K .E
Total energy = K.E + P.E = mv mv 2 mv 2 1
2 2 Total energy

78
UNIT : 4 CHEMICAL BONDING AND MOLECULAR STRUCTURE

Important Points
In this unit, the study of chemical bonding and formation of molecule from the atoms are
included. The smallest particle of element is atom and the smallest particle in a compound is molecule.
The force or the binding that keeps the atoms in the molecule combined during the formation of
molecule is called chemical bonding. The concepts like that of Kossel-Lewis, VSEPR principle, va-
lence bond theory, molecular orbital theory have been presented. In chemical bonding, it has more
relation with orbitals around the nucleus and especially the valence orbitals. We do not think about
the nucleus but we take into consideration the effect due to its positive charge. Scientists Lewis and
Kossel have mentioned the approach of chemical bonding. In this, the atom obtains the octet either
by losing or by gaining the electron, which is chemically inert. This is called law of octet. Such bonds
are called ionic bonds e.g. NaCl. Also, some atoms share electrons with each other and obtain octet
structure resulting into stable covalent molecule. e.g. Cl2. To explain the structures of such molecules
he mentioned dot and cross symbols and explained the stability of the molecules. Such a bond is called
covalent bond. The approach of Kossel Law is explained in detail in the unit.
When any bond is formed, the distance between their atoms is called bond length and the angle
is called bond angle. As you know the bond lengths of single ( - ) bond, double (=) bond and triple
() bond are different. The bond angle gives geometrical shapes to molecules viz.1800- linear, 109028'
tetrachedral. You will study in detail about covalent bond which can be of three types. (1) Polar
covalent bond in which the electron remains dragged towards the more electronegative atom and +d
charge on electropositive atom and -d charge on electronegative atoms are shown. As a result the
molecule becomes polar. If the electronegativities of the two atoms are same or the difference
between them is less, than non-polar bonds, formed by both the atoms sharing the electrons. In co-
ordinate covalent bond, one of the two atoms sharing a pair of electrons and the second atom
completes the octet with the help of this gained electron pair. viz. F3B NH3. Bond indicates
co-ordinate covalent bond. Over and above, bond length, bond angle, bond enthalpy (bond energy) is
also an important concept. Shorter the bond length, more will be the stability and so more energy will
be required to break it. Thus, the values of enthalpy may be different according to bond formation.
The number of bonds is called bond order which we have studied in detail and also the formula to
determine it. Born-Haber showed that the enthalpy evolved in formation of compound is the math-
ematical results of the enthalpies of several reactions of atoms. It is explained in the unit by discussing
the formation of compound like NaCl.
Sometimes, it so happens that the electron pair instead of being localised on any molecule it
localises towards other molecule. Thus, the bonds in the molecule can be shown at different positions
in the compounds having same molecular formula. Such structures are called resonance structures and
energy associated with the changes of these resonance forms is called resonance energy. This can
be studied through the molecules of ozone, carbon dioxide, benzene etc.

79
As we have seen earlier, structures like linear, tetrahedral etc, can be obtained on the basis of
bond angle. This study can be used to show the shapes of the molecule by hybridisation of atoms
in them, geometrical structures etc. viz. linear BeCl2 - 1800, trigonal BCl3 - 1200 , tetrahedral CH4 -
109028'.
Lewis approach being insufficient to explain the shapes of molecules, Sidgwick and Powell
proposed one principle which is known as VSEPR principle which was developed by Nyholm and
Gillespie and they proposed certain assumptions. In this it is important to note that when non-bonding
electron pairs are there, then they show deviation in geometrical structure and bond angle due to
repulsion between electron pairs. e.g. Molecule of water has sp3 hybridisation and so its bond angle
must be 109028' but it becomes 1040 30' due to repulsion by two non-bonding electron pairs. Hence,
it is called distored tetrachedral. The polarity of bond is a vector quantity. Hence, if a polar bond is
formed due to difference in electronegativities but another bond of the same type is formed in its
opposite direction, then polar bond will be formed but the resultant polarity of the molecules becomes
zero and molecule will be non-polar.

e.g F B
Be F or
F F

The dipole momentes of polar substances can be calculated for which both the charges +d and
- d and the distance between them is to be utilised. More the value of dipole moment, more will be
the polar bond and more will be the ionic bond. One important aspect is that polar substances dissolve
only in polar solvents and non-polar substances dissolve in nonpolar solvents. e.g. NaCl will dissolve
in water. Napthalene will dissolve in benzene. New hypotheses have been presented affter taking into
consideration the limitations of the principles for the approach of covalent bond. Two are main from
them : (1) Valence Bond Theory and (2) Molecular Orbital Theory. These concepts are based on
quantum mechanics. Heitler and London first of all gave the idea of valence bond theory and it was
developed by Pauling and Slater.
In the assumptions of valence bond theory the attraction - repulsion forces between positively
charged nuclei of two atoms and the electrons arranged in the orbits around them. According to
Coulomb's Law if attractive forces are more than repulsive forces then the bond will be formed and
molecule will be formed. In this theory, on the basis of the overlapping of valence orbitals different
overlaps can be formed. In this type of overlapping the excitation of electrons in valence orbitals can
be shown and then formation of molecule by covalent bond with other atoms. viz. In carbon, the elec-
trons of valence orbital 1s2 2s2 2p2 will be excited to give 1s2 2s1 2px1 2py1 2pz1 containing four orbitals
with one electron in each and four hydrogen atoms, and hence will give stable molecule like CH4 .
The geometrical structure, and bond angle can be expressed from the hybridisation associated with it.
In such valence bonds, two types of bonds-s and p are also observed. s bond is a covalent bond; it

80
attains axial overlap of internuclear axis. The stability of this bond is more than that of p bond. In the
p-bond the axis of the atomic orbitals undergoing overlapping remains parallel to each other and is
perpendicular to internuclear axis. p- bonds are less stable in comparison to s-bonds or they are
weaker. Valence bond theory is based on overlapping of valence orbitals. It explains properties like the
geometrical shapes, the bond angle etc. very simply but cannot explain magnetic properties.
Scientists Mulliken and Hund suggested molecular orbitals like atomic orbitals and proposed
molecular orbital theory. Amongst its important points, the idea that atomic orbitals can also form
molecular orbitals was taken into consideration. As many atomic orbitals take part in the formation,
same number of molecular orbitals, their energy, symmetry etc. were taken into consideration. The
formation of these types of atomic orbitals can be shown in the formation of homonuclear molecules
like H2, Be2, F2 etc. and heteronuclear molecules like CO, NO etc. Molecular orbitals are formed
by linear combination of atomic orbitals-LCAO principle. On the basis of these types of combination
two types of molecular orbitals are formed which are known as Bonding Molecular Orbitals (BMO)
and Anti-Bonding Molecular Orbitals (ABMO). In the formation of rules these types of BMO and
ABMO the principles like Hund's rule of maximum spin, Pauli's exclusion principle, Aufbau principle
etc. which are applicable in formation of atomic orbital are also obeyed and maintained. In the unit
the molecular orbital diagrams of construction of molecular orbitals from the atomic orbitals for
formation of homonuclear molecules from H2 to Ne2 elements as well as for formation of heteronuclear
molecules like CO, and NO are shown. From these diagrams, important property like bond order can
be calculated. Bond order

1
= {electron in bonding molecular orbitals - electrons in anti - bonding orbital} viz. for N2 mol-
2

1
ecule bond order = (10 - 4 ) = 3 i.e. there will be triple bond N N. In the same way, in NO
2

1
molecule bond order will be = [10 - 5] = 2.5. Here, we will note that if the value of bond order
2
becomes zero, the bond will not be formed e.g. He2. If the value of bond order is integer, the bond
will be formed and according to the integer 1, 2, or 3, there will be single (- ), double (=) or triple
() bonds respectively. If the value of bond order is fraction, then molecule will attain unstable
structure. The molecular orbital theory can explain the magnetic properties e.g. In O2 molecule
two unpaired electrons are there and so it is paramagnetic and in N2 molecule, all the electrons are
paired and so it is diamagnetic. Thus molecular orbital theory is superior to valence bond theory in
this matter.

Above this, one important phenomenon is hydrogen bond. The first element of 15, 16, and 17,
groups N, O, F being higer electronegative than the other elements of the group it can form covalent
mo lecu les like NH3 , H2 O and HF with h yd r ogen . Af terw ar ds th e mo lecu le lik e
H F H F combines with each other H - F molecule through hydrogen bond

81
H F H F where H ........ (dotted line) indicates hydrogen bond formation. HF,,
NH3, H2O possess hydrogen bonds and so their properties are different from those of other elements
in the group. Hydrogen bond is of two types (1) Intermolecular and (2) Intramolecular hydrogen
bonds. When hydrogen bond is formed between two molecules it is called intermoleculer hydrogen
bond e.g. p-chlororphenol and between two groups in the same molecule, it is called intramoleculer
hydrogen bond e.g. o-chlorophenol. Intermolecular hydrogen bond is stronger than intramolecular
hydrogen bond. The presence of hydrogen bond is the reason for specific properties of the com-
pounds. Viz the retaining of water in the soil, drying of terrylene clothes is faster than that of cotton
clothes.

After knowing about ionic bond, covalent bond, co-ordinate covalent bond, we shall study the
special type of bond present in metals which is called metallic bond. As there are 1, 2 or 3 electrons
in the outermost orbit of the metals, they are not able to form covalent bonds. Their ionisation energy
is less and attraction of electron towards the nucleus is less. One, two or three electrons are arranged
around the nucleus of the atom. Hence, the positively charged nucleus or kernel is there. The
electrons around it have attraction towards other nuclei of the atoms in the lattice. Thus, the electron
instead of being localised for any one atom, remains delocalised in the whole metal crystal. For this
theory Electron Sea model was proposed. In this, the atomic kernel is imagined as floating in the sea,
delocalised electrons are arranged around kernel possessing positive charge. Because of this type of
metallic bonds, the specific properties of metals, like density, ductility, malleability etc. are different.
Co-ordinate covalent bond is a type of covalent bond as seen earlier. The characteristic in it is
that from the two atoms undergoing sharing of electrons, only one of the atoms provides a pair of
electrons, and is shared by both the atoms. Hence, it is called co-ordinate covalent bond. e.g. In BF3 ,
three F atoms were bonded with B-atom through three covalent bonds but the octet of B is not
complete. Similary in NH3 molecule, three H atoms are bonded with N through three covalent bonds.
But N has one non-bonding pair of electrons, which it gives to BF3 molecule and is shared by both
the molecules. Hence F3B NH3 Co-ordinate covalent bond is formed. In this, the molecule which
gives pair of electrons is shown by arrow () from the molecule which donates it towards the
molecule or atom which accepts and shares gained electron pair. You will study more about co-
ordinate covalent bond in the unit of complex salts in Standard-12.

82
M.C.Q.
(1) Which of the following is ionic ?
(a) HCl (b) CHCl3 (c) IF 5 (d) KI
(2) When molecule is form by chemical bonding between atoms then
(a) nucleous of combining atoms are participate
(b) valence electrons and inner cell electrons are participate
(c) only valence electrons of combining atoms are participate
(d) only inner cell electrons of combining atoms are participate
(3) Which factor is not responsible for the formation of ionicbond?
(a) crystal lattice energy (b) density
(c) ionisation enthalpy (d) electron gain enthalpy
(4) According to valence-bond theory which magnetic property oxygen possess ?
(a) Paramagnetic (b) Ferrimagnetic (c) Diamagnetic (d) Anti Ferromagnetic
(5) Who was proposed valence-bond theory ?
(a) Mulliken (b) Lenus Pauling (c) Hittler and Londan (d) Hund
(6) In H - C C - CH = CH2 molecule C 3 - C 2 single bond carbons has which type of
hybridization ?
(a) sp2 - sp 3 (b) sp - sp2 (c) sp3 - sp (d) sp3 -sp 3
(7) Which of the following pair of species is isoelectronic and same structure ?
(a) NO3- , SO 3 (b) SO3, CO 32- (c) CO32- , ClO3- (d) NO3-, CO 32-
(8) Which of the following sentence is incorrect for covalent bond ?
(a) Strenght of covalent bond depenas upon overlapping at atomic orbitals.
(b) Covalent bond is not directional.
(c) There is sharing of electrons between atoms bonded by covalent bond
(d) Covalent bond is formed between atoms having less difference in their electronegativity.
(9) Which of the following compound possesses covalent bond ?
(a) MgCl2 (b) NaH (c) BF 3 (d) CsCl
(10) Which of the following molecule possesses polar and nonpolar covalent bond ?
(a) NH4Cl (b) CCl4 (c) H2O 2 (d) HCN
(11) Which of the following compound does not possesses coordinate covalent bond ?
(a) CO (b) SO 2 (c) HNO2 (d) HNO3

83
(12) Which of the following characteristic is not for covalent compound ?
(a) They do not possesses particular geometical structure
(b) They may be polar or nonpolar
(c) Their boiling and melting point is low
(d) Generally they are insoluble in water
(13) Which of the following possesses ionic and covalent bond ?
(a) CO 2 (b) H2SO 4 (C) NH4Cl (D) NaI
(14) Whhat is Geometrical Structure of ClF3 molecule ?
(a) Triogonal bipyramid (b) Corn shpae (c) sea-saw (d) T-shape
(15) Which of the following molecule possesses linear structure ?
(a) SO2 (b) CO 2 (c) H2O (d) C2 H 4
(16) Correct structure of SF4 is

(17) Numbers of possible resonating structure of carbonate iong is....


(a) 9 (b) 6 (c) 3 (d) 2
(18) Which of the following molecule has not zero dipol movement ?
(a) NF 3 (b) BF 3 (c) CO 2 (d) BeF2
(19) Which of the following molecule possesses highest dipolspace movement ?
(a) CCl4 (b) CHCl3 (c) CHCl2 (d) CH3Cl
(20) Which of the following molecule possesses dipol movement ?
(a) trans - 1, 2 - dichloro ethene (b) trans pent - 2 - ene
(c) 2, 2- dimethyl propane (d) 2, 2, 3, 3- tetra methyl butane
(21) Which of the following molecule has lowest bond space angle ?
(a) NH 3 (b) SO 2 (c) H2O (d) H2S
(22) Which orbital has highest energy ?

(a) s(2Px) (b) p * (2Py) (c) s(2S) (d) s * (1S)

84
(23) Which is the paramagnetic species ?
- -
(a) C N (b) O 2 (c) NO + (d) CO

(24) Which of the following statement is incorrect when N2 and O2 are convert into N2+ and O2+
respectively ?

(a) In O +2 , O - O bond order increases. (b) In N +2 , N - N bond become weaker..

(c) N +2 become paramagnetic (d) Increasing dimagnetism in O +2

(25) According to VSEPR theory geomety of which block elements can be explain ?
(a) s (b) p (c) d (d) f
(26) Atoms complete octet in valence shell electron during the bond formation. This postulate was
proposed by which scientist ?
(a) Powel (b) Lewis (c) Sigdwick (d) Mulliken
(27) Crystal formation is which type of reaction ?
(a) endothermic and exothermic (b) endothermic
(c) exothermic (d) no heat change occurs
(28) Lattice energy of ionic compound depends upon which factor ?
(a) Size of ion (b) Size of ion and charge
(c) charge on ion (d) Arrangement of ion

(29) Which is correct order for C - O bond length in CO, CO 23 - , CO 2

(a) CO 3-2 < CO 2 < CO (b) CO 2 < CO 32- < CO

(c) CO < CO 2 < CO 32- (d) CO < CO 23 - < CO 2

(30) Maximum how many numbers of hydrogen bond can be form by H2O molecule ?
(a) 2 (b) 4 (c) 3 (d) 1
(31) In buta 1, 3 - diene
(a) only one sp hybridised carbon atom
(b) only sp2 hybridised carbon atoms
(c) Two sp3 and two sp2 hybridised carbon atoms
(d) sp, sp2 and sp3 hybridized carbon atoms

85
(32) Which of the following statement is irrelevant for sigma bond ?
(a) strength of sigma bond is not related with overlapping of atomic orbitals.
(b) s - bond can form by overlapping of S - P orbitals.
(c) s - bond can form by overlapping of end of atomic orbitals of inner center axis.
(d) This type of overlapping is also known as axial overlaping
(33) In which molecule inter molecular hydrogen bond can be form ?
(a) methanol (b) ethelene glycol (c) p - nitrophenol (d) phenol
(34) In which molecule intra molecular hydrogen bond can be form ?
(a) o - nitro phenol (b) aniline (c) ethylene glycol (d) all of these
(35) Which of the following pair possesses very strong H - bond ?
(a) CH3 COCH3 and CHCl3 (b) HCOOH and CH3 COOH
(c) H2O and H2 (d) SiH4 and SiCl4
(36) Which of the following relation is correct ?
(a) Bond order Bond energy Bond length stability

1 1
(b) Bond order stability
Bond length energy

1
(c) Bond order Bond energy stability
Bond length

1 1
(d) Bond order stability
Bond length Bond energy

(37) Molecule : H2 F2 Cl2 Br2


Bondlength : 74pm 144pm 199pm 228pm
Mention more stable molecule from above
(a) Cl2 (b) H2 (c) Br2 (d) F2
(38) In water bond angle is 104o 30 because
(a) Oxygen atom is sp3 hybridised
(b) Repulsion between lone pair election and bonding pair electron
(c) Oxygen has high electronegetivity.
(d) H2O molecule possesses ''V'' - shape.

86
(39) In which of the following strong H-bond is present ?
(a) F - H.....F (b) O - H.....N (c) O - H.....O (d) O - H.....F

(40) Which is correct order for bond dissociation energy in O2 , O +2 , O -2 and O 22-

(a) O 2 > O +2 > O 22- > O -2 (b) O +2 > O 2 > O -2 > O 22-

(c) O -2 > O 22 - > O +2 > O 2 (d) O -2 > O 22 - > O 2 > O +2

(41) O, P, Q, R elements electronic configuration is given below

O = 1s 2 P = 1s 2 , 2s 2 , 2p2

5
Q = 1s 2 , 2s 2 , 2p R = 1s 2 , 2s2 , 2p 6

Which atom has strong behaviour of electrovalent bond ?


(a) O (b) P (c) Q (d) R
(42) In which molecule bond distorsion is more according to VSEPR theory ?
(a) SO 2 (b) NH 3 (c) O3 (d) H2O
(43) Which of the following species is more stable ?

(a) O -2 (b) Ne +2 (c) O 2 (d) F2

(44) Number of nonbonding electron pair in XeF6, XeF4 and XeF2 respectively
(a) 2, 3, 1 (b) 1, 3, 2 (c) 3, 2, 1 (d) 1, 2, 3
(45) On keeping two cube of ice on each other which become one cube which factor is responsible
for it ?
(a) Van-der waals attraction (b) Hydrogen bond (c) Dipole attraction (d) Covalent bond
(46) Determine lattice energy of LiF(S) according to given data.

(i) Li(S) Li(g) 155.2 KJ mol -1 ( DsH)



75.2 KJ HD
(ii) F2(g) 2F (g)
2
-
(iii) Li(g) Li (+g ) + e 520.0 KJ mol-1 H( i )

(iv) F( g ) + e F( g ) -33.0 KJ ( H
eg )
1
(v) Li(s) + F2 ( g ) LiF(S) -504.1 KJ mol -1
( Df H )
2
(a) - 86.7 KJ mol -1 (b) 86.7 KJ mol -1 (c) -867 KJ mol -1 (d) 867 KJ mol -1

87
(47) Which of the following statment is incorrect for metallic bond ?
(a) There is attraction between delocalised electrons and atomic karnel
(b) Directionl property is shown by metal
(c) Delocalised electron can change their position easily in crystal
(d) Explanation of metallic bond can be given by 'electron sea model' S - Orbital
(48) Why lattice energy of NaCl > KBr ?
(a) When size of negative ion decrease in ionic crystal then lattice energy increases.
(b) When volume of positive and negative ion is small than then interionic attraction become
more and hence latice energy increases.
(c) In ionic crystal when size of positive ion decrease, then lattice energy increases.
(d) All of given
(49) Number of H - bond form by unpaired electrons of liquid NH3 , H2O and HF respectively are
(a) 3, 4, 2 (b) 4, 4, 2 (c) 3, 2, 1 (d) 1, 2, 1
(50) Which of the following pair is not in order for boiling point for 14, 15, 16 and 17 group ?
(a) H2O > H2S (b) HF > HCl (c) CH4 > SiH 4 (d) NH3 > PH 3
(51) Which of the following compound possesses ionic bond ?
(a) CH 4 (b) SiCl4 (c) BF 3 (d) MgCl2
(52) Which of the following relation between BMO and ABMO electrons is correct for stability of
diatomic malecule or ion ?
(a) Na > N b (b) Nb > N a (c) Na + Nb = 0 (d) Na = N b
(53) At what distance van-derwaals attraction exist ?

(a) 4.5 10 -10 m (b) 0.45nm (c) 4.5 Ao (d) Given all
(54) What is bond energy of H-bond ?
(a) 40 J mol-1 (b) 40 KgJ mol-1 (c) 40 cal. mol-1 (d) 40 Kg cal mol-1
(55) In which molecule inter molecular H-bond is possible ?

(a) CH 3COCH 3 (b) CH 4 (c) SiH4 (d) NH 3

(56) Which of the following characterstic does not possesses by metal ?


(a) luminus (b) ductility
(c) increase in conductance by increase in temperature (d) malleability

88
(57) On which factor conductance of metals responsible ?
(a) ions (b) delocalised (c) atomic kernel (d) number of atoms
(58) Which of the following figure shows electron-sea model ?

(59) According to which group, hydrogen bond is form in protein molecule present in musecls of
living organism ?
(a) -CO- (b) -COOR (c) -CONH- (d) -COOH
(60) On which factor van-der waalls attraction force does not depend ?
(a) numbers of molecules (b) contact surface area of molecules
(c) shape of molecules (d) numbers of electron in molecules
(61) Practicol dipal movement of HCl is 1.03D. If bond length of HCl is 1.275 Ao than what will
be the pereentage of ionic nature in HCl ?
(a) 7 (b) 17 (c) 43 (d) 21
(62) Which sentence is correct with respect to bond enthalpy ?
(a) As bond order is more, then bond dissociation enthalpy is less
(b) As atomic volume is more, then bond energy is more.
(c) As bond enthalpy is more, then stability of molecule or ion is less.
(d) As number of nonbonding election pair on bonded atom then bond enthalpy is less.
(63) which of the following orbitals form bonding orbital ?

(a) (b) (c) (d)


S - Orbital P - Orbital P - Orbital a f - Orbital P - Orbital P - Orbital
S - Orbital P - Orbital

(64) Mention number of bonding electron pairs and nonbonding electron pairs in NO 3- ion

(a) 3, 1 (b) 2, 2 (c) 4, 0 (d) 1, 3


(65) How many numbers of bonding and nonbonding electron pairs in CO2 ?
(a) 4, 4 (b) 2, 4 (c) 4, 2 (d) 2, 2

89
(66) Mention proper order of bond length given below.

(a) N 2 < N 22- < N 2- (b) N 22- < N -2 < N 2 (c) N -2 < N 2 < N 22- (d) N 2 < N 22- < N 2

(67) Show paramagnetic compound given below.


(a) O3 (b) KO 2 (c) N2O (d) Na2O 2
(68) Which species possesses pyramidal shape ?

(a) OsF2 (b) SO 3 (c) BrF3 (d) SiO23-

(69) Which of the following does not possesses bond order as CO ?


+ - -
(a) N O (b) N O (c) N2 (d) C N

(70) Which rule is violated in the given electronic configuration ?

2P 2P

(a) Aufbau (b) Pauli (c) Hund (d) Given all


(71) In which of the following molecule double bond possesses two pispace bond ?
(a) S2 (b) O2 (c) C2 (d) H2C = CH 2
(72) Mention AB4F2 type molecule.
(a) BrF5 (b) XeF4 (c) SF 6 (d) XeOF4
(73) Which of the following is the correct order for lone pair and bonding pair electrons ?
Lp = Lone pair and Bp = Bonding pair
(a) Lp - Lp > Lp - Bp > Bp - Bp (b) Lp - Bp > Lp - Lp > Bp - Bp
(c) Bp - Bp > Lp - Lp > Lp - Bp (d) Lp - Lp > Bp - Bp > Lp - Bp
(74) Which theory is useful to determine geometrical structure of molecules ?
(a) molecular orbital theory (b) VSEPR theory
(c) Resonance theory (d) Quantam mechanics
(75) The one outermost electron present in Na element at
(a) one corner of simple cube (b) eight corner of simple cube
(c) center of simple cube (d) each corner of simple cube

90
(76) In which molecules / ion have not all the equal bonds ?

(a) SF 4 (b) BF4- (c) XeF4 (d) SiF4

(77) Which of the following has maximum bond angle ?


(a) NH 3 (b) CH 4 (c) CO 2 (d) H2O
(78) Which of the following have equal bond order ?
+
(a) O -2 (b) CN (c) N O (d) B and C
(79) The type of bond present in CuSO4 .5H2O
(a) covalent and co-ordinate covalent (b) electrovalent and covalent
(c) electrovalent and co-ordinate covalent
(d) electrovalent, covalent and co-ordinate covalent
(80) Which of the following statement is wrong
(a) sp2 hybrid orbitals are formed from two p - atomic and one s - orbitals
(b) hybridization is the mixing of atomic orbitals prior to their combing into molecular orbitals
(c) d2p2 hybrid orbitals are all at go to one an other
(d) d2sp3 hybrid orbitals are directed towords the corners of a regular tetrahedron
(81) CO2 is isostructual with
(a) SnCl2 (b) HgCl2 (c) C2H 2 (d) SO 2
(82) NH3 has a higher boiling point than expected because
(a) its density decreases on freezing
(b) with water it forms NH4OH
(c) it has strong inter molecular covalent bonds ?
(d) it has intermolecular hydrogen bonds.
(83) The molecule with zero dipole moment is
(a) chloroform (b) methyl chloride
(c) carbon tetrachloride (d) methylene chloride
(84) Molecular shaper of SF4 , CF4, XeF4 are
(a) the same with 1, 1 and 1 lone pairs of electrons respectively
(b) different with 1, 0 and 2 lone pairs of electrons respectively
(c) different with 0, 1 and 2 lone pairs of electrons respectively
(d) different with 2, 0 and 1 lone pairs of electrons respectively

91
(85) Which of the following has the regular tetrahedral structure ?

(a) SF 4 (b) [ Ni (CN ) 4 ] 2- (c) BF4- (d) XeF4

(86) In OF2, number of bond pairs and lone pairs of electrons are respectively
(a) 2, 8 (b) 2, 6 (c) 2, 9 (d) 2, 10

(87) In O -2 , O2 , O 22- molecular species the total number of antibonding electrons respectively are

(a) 7, 6, 8 (b) 1, 0, 2 (c) 6, 6, 6 (d) 8, 6, 8


(88) Match the following and choose the correct Answer
Column - I Column -II

(i) sp3 d 2 (a) Ni [ (CN)4 ] 2-

(ii) sp3 d (b) SnCl2

-
(iii) dsp2 (c) IC l4

(iv) sp2 (d) TeCl4


(a) i d, ii a, iii c, iv b (b) i c, ii d, iii a, iv d
(c) i b, ii c, iii d, iv a (d) i a, ii b, iii c, iv d
(89) Among the following compounds, the one that is polar and has the central atom with sp2
hyridisation is
(a) HClO2 (b) BF 3 (c) H2CO 3 (d) SiF4
(90) Match the following
Set A Set B
(1) stability of bond (p) Bond energy
(2) Molecular orbital theory (q) Bond order
(3) octet rule (r) Variable Valency
(4) Valence bond theory (s) Electronic concept of valency
(a) 1 q, 2 p, r, 3 p, 4 s (b) 1 p, q, 2 p, 3 r, 4 s
(c) 1 p, q, 2 r, 3 s, 4 r (d) 1 p, q, 2 q, 3 s, 4 r
(91) Bond strength increases with
(a) Bond length increasing
(b) Antibonding eletrons being higher in number
(c) Bond order increasing (d) Bond angle increasing

92
(92) O 22+ will have

(a) Bond order lower than O


2

(b) Bond order higher than O2


(c) Bond order lower than H2
(d) Bond order higher than N2
(93) In a molecule number of electrons in BMO are more as compared to ABMO, hence
(a) a bond will be formed
(b) no bond will be formed
(c) information is not sufficient
(d) none of the above
(94) The bond angle in the ammonium ion is equal to
(a) 90o (b) 104o (c) 120o (d) 109o.281
(95) The correct order of dipole moment is
(a) CH4 < NF3 < NH3 < H2O (b) NF3 < CH4 < NH3 < H2O
(c) NH3 < NF3 < CH4 < H2O (d) H2O < NH3 < NF4 < CH 4
(96) The correct order of the O O bond length in O2 , H2O2 and O3is
(a) O2 > O3 > H2O 2 (b) O3> H2O2 > O 2
(c) H2O2 > O3 > O 2 (d) O2 > H2O 2 > O 3

(97) The bond order of O -2 is

(a) 1.0 (b) 1.5 (c) 2.5 (d) 0.5


(98) Choose the incorrect statement.
(a) s bond is weaker than p bond
(b) p bond is weaker than s bond
(c) p bond is present along with a s bond
(d) s bond can be present alone
(99) Which of the following is not paramagnetic ?

(a) NO (b) S2- (c) O -2 (d) N -2

(100) Which one of the following compound has sp2 hybridization ?


(a) CO 2 (b) SO 2 (c) CO (d) N2O

93
Answer key
1d 2c 3c 4b 5c 6b
7d 8b 9c 10 c 11 c 12 a
13 c 14 d 15 b 16 a 17 c 18 a
19 d 20 b 21 d 22 b 23 b 24 c
25 b 26 b 27 c 28 b 29 b 30 b
31 b 32 a 33 c 34 c 35 b 36 c
37 b 38 b 39 a 40 b 41 c 42 d
43 c 44 d 45 c 46 a 47 b 48 d
49 d 50 c 51 d 52 b 53 d 54 b
55 d 56 c 57 b 58 d 59 c 60 a
61 b 62 d 63 d 64 c 65 a 66 b
67 b 68 a 69 b 70 d 71 c 72 b
73 a 74 b 75 a 76 a 77 c 78 d
79 c 80 a 81 c 82 d 83 c 84 b
85 c 86 a 87 a 88 b 89 c 90 d
91 c 92 b 93 a 94 d 95 a 96 c
97 d 98 a 99 b 100 b

Hints
1. Electronegative difference in KI is more
2. ACC to V.B.T, O2 contain all e- paired. So they are diamagnetic
6. Here C - C contain double and triple bond

7. NO 3- and CO 3-2 contains 32 e- so they are iso electric ions Both possess sp2 hybridization

10. In H2O2 , O - H is polar


O - O is non-polar
11. H-O-N = 0
16. Struchure A is stable
18. In NF3 , polarity of N F bond and non-bonding e- pair are in opposite direction
19. In CH3Cl, dipole moment is due to C-Cl and C-H

94
20.

trans - pent - 2 - ene


possess magnetic moment

21. Compound SO 2 H2O H2S NH 3


Bond angle 119.5o 104.5o 92.5o 106.5o

29. CO O=C=O

30. In H2O, O Contain two lone pair of e-


H Contain positive change

40. Bond order O +2 = 2.5


O2 = 2
O -2 = 1.5
O -22 = 1

42. In H2O, O contain two lone pair of e- so repulsion is more

46. D f H o = D sub H o + D D H o + D i H o + D eg H o + D u H o

61. Theorectical dipole momentum


H =qd 1 D = 1 10 esa cm
-18

= 4.8 10 -10 1.275 10 -8


= 6.12 10 -10 e.s.u.cm
= 6.12 D

Pr actical (m)
% of ionic character = Theoretical (m)

1.03
= 100
6.12
= 16.83
17 %

95
UNIT : 5 - CHEMICAL THERMODYNAMICS
Important Points
DE =Ep - Er = qv DE = q - PDV DE = q + w

w = -nRT loge VV12 DH = DE + PDV DH = DE + DngRT

Hess Law H steps = H1 + H 2 + H3

Heat absorbed
Heat Capacity = Temperature difference
Heat absorbed
Specific heat capacity = (Temperature difference) (Weight of subs ta nce in gram)
Heat absorbed
Molar heat capacity = (Temperature difference) (Molecular weight)

Cp - C = R
Cp
= g (gamma)
C
w T2 - T1
= = Thermodynamic efficiency
q2 T2
q rev
DS = SFinal state SInitial state =
T
DSsystem + DSsurrounding > 0 . The reaction will occur spontaneously
DSsystem + DSsurrounding < 0 . The reaction will be non spontaneous
DSsystem + DSsurrounding = 0 . The reaction will be in equilibrium
Entropy change for an ideal gas : In going from initial to final state, the entropy change, DS for an
ideal gas is given by the following relations,
T2 V2
(i) When T and V are two variables, S = nC v ln + nR ln . Assuming Cn is constant
T1 V1
T p2
(ii) When T and p are two variables, S = nCP ln 2 - nR ln . Assuming Cp, is constant
T1 p1
V2 p2
(a) Thus, for an isothermal process (T constant), S = nR ln or = - nR ln
V1 p1
T2
(b) For isobaric process (p constant), S = n Cp ln
T1
T2
(c) For isochoric process (V constant), S = n C v ln
T1
(d) Entropy change during adiabatic expansion : In such process q = 0 at all stages. Hence
DS = 0. Thus, reversible adiabatic processes are called isoentropic process.

96
V2
DS = R ln V DG = DH TDS
1

DfGoreaction = SDfGoProduct SD fGoReactant

total s tan dard free energy of total s tan dard free energy of
DfGoreaction = -
formation of product formation of reac tan t
P2
DG = nRT ln P
1
DGo = 2.303RTlog K
DG = nFE Cell
H T2 - H T1
Kirchhoffs equation. = CP
T2 - T1
E T2 - E T1
Kirchhoffs equation at constant volume, = C
T2 - T1
Effect of Temperature on Spontaneity of Reactions
Dr H0 D rS 0 D rG0 Description*
+ Reaction spontaneous at all temperature
(at low T) Reaction spontaneous at low temperature
+ (at high T) Reaction nonspontaneous at high temperature
+ + + (at low T) Reaction nonspontaneous at low temperature
+ + (at high T) Reaction spontaneous at high temperature
+ + (at all T) Reaction nonspontaneous at all temperature

T
Joule thomson coeffient =
p H
For cooling m = + ve
For Heating m = ve
Neither cooling nor heating m = 0
The temperature at which a real gas shows neither cooling nor heating effect on adiabatic expansion
(i.e., m = 0) is called inversion temperature.
Hydrogen has highest calorific value.
13.7 Kcal/mol = 57 KJ/mol (be cause of 1cal = 4.2 Joule)
Enthalpy of fusion of ice per mole is 6KJ.
Order of bond energy in halogen Cl2 > Br2 > F2 > I2.
Heat of vapourisation of water per mole is 10.5 KCal.
The heat of reaction is independent of the time consumed in the process.
T
Joule thomson coeffient =
p H

97
M.C.Q.
1. The temperature of the system decreases in an
(A) Adiabatic compression (B) Isothermal compression
(C) Isothermal expansion (D) Adiabatic expansion
2. If a refrigerators door is opened, then we get
(A) Room heated (B) Room cooled
(C) More amount of heat is passed out (D)No effect on room
3. The cooling in refrigerator is due to
(A) Reaction of the refrigerator gas (B) Expansion of ice
(C) The expansion of the gas in the refrigerator (D) The work of the compressor
4. The process, in which no heat enters or leaves the system, is termed as
(A) Isochoric (B) Isobaric (C) Isothermal (D) Adiabatic
5. Warming ammonium chloride with sodium hydroxide in a test tube is an example of :
(A) Closed system (B) Isolated system (C) Open system (D) None of these
6. Out of boiling point (I), entropy (II), pH (III) and e.m.f. of a cell (IV), intensive properties are
(A) I, II (B) I, II, III (C) I, III, IV (D)All the above
7. A thermodynamic state function is
(A) one which obeys all the laws of thermodynamics
(B) a quantity which is used in measuring thermal changes
(C) one which is used in thermo chemistry
(D) a quantity whose value depends only on the state of the system.
8. In thermodynamics, a process is called reversible when
(A) surroundings and system change into each other
(B) there is no boundary between system and surroundings
(C) the surroundings are always in equilibrium with the system
(D) the system changes into the surroundings spontaneously
9. Which one of the following statement is false
(A) work is a state function (B) temperature is a state function
(C) change in the state is completely defined when the initial and final states are specified
(D) work appears at the boundary of the system.
10. A mixture of two moles of carbon monoxide and one mole of oxygen, in a closed vessel is ignited
to convert the carbon monoxide to carbon dioxide. If DH is the enthalpy change and DE is the
change in internal energy, then
(A) DH < DE (B) DH > DE (C) DH = DE
(D) The relationship depends on the capacity of the vessel
11. At constant T and P, which one of the following statements is correct for the reaction,
1
CO(g) + O2 (g) CO2 (g)
2

(A) DH is independent of the physical state of the reactants of that compound


(B) DH < DE (C) DH > DE (D) DH = DE

98
(A) The randomness of the universe decreases
(B) The randomness of the surroundings decreases
(C) Increase is randomness of surroiundings equals the decrease in randomness of system
(D) The increase in randomness of the surroundings is greater as compared to the decrease in
randomness of the system.
21. The enthalpy change for a given reaction at 298 K is x J mol1 (x being positive). If the reaction
occurs spontaneously at 298 K, the entropy change at that temperature
(A) can be negative but numerically larger than x/298
(B) can be negative but numerically smaller than x/298
(C) cannot be negative (D) cannot be positive
22. Spontaneous adsorption of a gas on a solid surface is exothermic process because
(A) enthalpy of the system increases. (B) entropy increases.
(C) entropy decreases. (D) free energy change increases.
23. Identify the correct statement regarding entropy :
(A) At absolute zero, the entropy of a perfectly crystalline substance is +ve.
(B) At absolute zero, the entropy of a perfectly crystalline substance is zero.
(C) At 0C the entropy of a perfectly crystalline substance is taken to be zero.
(D) At absolute zero of temperature the entropy of all crystalline substances is taken to be zero.
24. Identify the correct statement regarding a spontaneous process :
(A) Exothermic processes are always spontaneous.
(B) Lowering of energy in the reaction process is the only criterion for spontaneity.
(C) For a spontaneous process in an isolated system, the change in entropy is positive.
(D) Endothermic processes are never spotaneous.
25. DS will be highest for the reaction
(A) Ca(s) + l/2 O2(g) CaO(s) (B) CaCO3(s) CaO (s) + CO2(g)
(C) C(s) + 02(g) CO2(g) (D) N2(g) + O2(g) 2NO (g)
26. The spontaneous flow of heat is always
(A) unidirectional from higher temperature to lower temperature
(B) from high to low pressure
(C) unidirectional from lower temperature to higher temperature
(D) from low to high pressure.
27. Which of the following is zero during adiabatic expansion of the gas
(A) DT (B) DS (C) DE (D) All the above
28. The occurrence of a reaction is impossible if
(A) DH is +ve; DS is also + ve but DH < TDS (B) DH is ve; DS is also ve but DH > TDS
(C) DH is ve; DS is + ve (D) DH is + ve; DS is ve

100
29. Identify the correct statement regarding entropy
(A) At 00C, the entropy of a perfectly crystalline substance is taken to be zero
(B) At absolute zero of temperature, the entropy of a perfectly crystalline substance is +ve
(C) At absolute zero of temperature, the entropy of all crystalline substances is taken to be zero
(D) At absolute zero of temperature, the entropy of a perfectly crystalline substance is taken to
be zero
30. A container has hydrogen and oxygen mixture in ratio of 4 : 1 by weight, then
(A) Internal energy of the mixture decreases (B) Internal energy of the mixture increases
(C) Entropy of the mixture increases (D) Entropy of the mixture decreases
31. The second law of thermodynamics says that in cyclic process.
(A) Work cannot be converted into heat (B) Heat cannot be converted into work
(C) work cannot be completely converted into heat
(D) Heat cannot be completely converted into work
32. A heat engine absorbs heat Q1 at temperature T1 and heat Q2 at temperature T2. Work done by
the engine is (Q1 + Q2). This data
(A) Violates Ist law of thermodynamics
(B) Violates Ist law of thermodynamics if Q1 is ve
(C) Violates Ist law of thermodynamics if Q2 is ve
(D) Does not violate Ist law of thermodynamics
33. The molar neutralization heat for and as compared to molar neutralization heat of NaOH and
HCl
(A) Less (B) More (C) Equal (D) Depends on pressure
34. If the enthalpy of B is greater than of A, the reaction AB is
(A) Endothermic (B) Exothermic (C) Instantaneous (D) Spontaneous
35. Which of the following fuels will have the highest calorific value (kJ/kg)
(A) Charcoal (B) Kerosene (C)Wood (D) Dung
36. Which is the best definition of heat of neutralization
(A) The heat set free when one gram molecule of a base is neutralized by one gram molecule of
an acid in dilute solution at a stated temperature
(B) The heat absorbed when one gram molecule of an acid is neutralized by one gram molecule
of a base in dilute solution at a stated temperature
(C) The heat set free or absorbed when one gram atom of an acid is neutralized by one gram
atom of a base at a stated temperature
(D) The heat set free when one gram equivalent of an acid is neutralized by one gram equivalent
of a base in dilute solution at a stated temperature
37. Compounds with high heat of formation are less stable because
(A) High temperature is required to synthesise them
(B) Molecules of such compounds are distorted
(C) It is difficult to synthesis them (D) Energy rich state leads to instability

101
38. When a gas undergoes adiabatic expansion, it gets cooled due to
(A) Loose of kinetic energy (B) Fall in temperature
(C) Decrease in velocity (D) Energy used in doing work
39. For a reaction to occur spontaneously
(A) (DH TDS) must be negative (B) (DS + TDS) must be negative
(C) DH must be negative (D) DS must be negative
40. If for a given substance melting point is TB and freezing point is TA, then correct variation shown
by graph between entropy change and temperature is

(A) (B) (C) (D)

41. A Beckmann thermometer is used to measure


(A) High temperature (B) Low temperature (C) Normal temperature (D) All temperature
42. The calorific value of fat is .
(A) less than carbohydrates and protein
(B) less than that of protein but more than carbohydrates
(C) less than that of carbohydrates and more than that of protein
(D) more than thant of carbohydrates and protein
43. Which of the following processes is accompanied by an increase in entropy ?
(A) Normal rubber band to stretched rubber band (B) Normal egg to hard boiled egg
(C) Decomposition of N2O5 to N2O to O2 (D) Formation of NH3 for N2H2.
44. Which of the following does not exhibit zero entropy at absolute zero
(A) Benzene (B) Glass (C) Pyridine (D) CCl4
45. The favourable conditions for a spontaneous reaction are
(A) TDS > DH, DH = +ve, DS = +ve (B) TDS > DH, DH = +ve, DS = ve
(C) TDS = DH, DH = ve, DS = ve (D) TDS = DH, DH = +ve, DS = +ve.
46. A block of ice at 10 C is slowly heated and converted into steam at 100C .Which of the
following curves represents the phenomenon qualitatively ?

(A) (B) (C) (D)

47. On passing CO2 gas in water, its entropy


(A) Remains constant (B) Decreases (C) Increases (D) May increase or decrease.
48. When does the reaction occur spontaneously on the basis of the relation DG = RT/nK?
(A) K = 0 (B) K = 1 (C) K > 1 (D) K < 1

102
49. In thermodynamics, a process is called reversible when,
(A) Surroundings and system change into each other
(B) The surroundings are always in equilibrium with the system
(C) The system changes into the surroundings spontaneously.
(D) There is no boundary between system and surroundings.
50. Under certain conditions, the value of DG for a hypothetical reaction, X + Y Z is greater
than zero, then
(A) The reaction has tendency to proceed towards Z
(B) The reaction has attained equilibrium
(C) increase in temperature increases the yield of product Z
(D) X and Y predominate in the final mixture
51. For which of the following processes will energy be absorbed
(A) Separating an electron from an electron (B) Separating proton from a proton
(C) Separating a neutron from neutron (D) Separating an electron from neutral atom
52. For the combustion of 1 mole of liquid benzene at 25C, the heat of reaction at constant pressure
is given by, C6H 6(l) + 7 O2 (g) 6CO2 (g) + 3H2O (l); DH = 780980 cal.
What would be the heat of reaction at constant volume?
(A) 780090 cal (B) 780890 cal (C) 780000cal (D) 780900 cal
53. Calculate heat of the following reaction at constant pressure,
F2O(g) + H2O(g) O2 (g) + 2HF(g)
The heats of formation of F2O (g), H2O(g) and HF (g) are 5.5 kcal, 57.8kcal and 64.2 kcal
respectively.
(A) 76.1 Kcal (B) 11.9 Kcal (C) 71.6 Kcal (D) 91.1 Kcal
54. Calculate the heat of formation of benzene from the following data, assuming no resonance.
Bond energies :
C C = 83 kcal = 140 kcal C H = 99 kcal
Heat of atomisation of C = 170 .9 kcal
Heat of atomisation of H = 52.1 kcal
(A) 70 Kcal (B) 75 Kcal (C) 75 Kcal (D) 70 Kcal
55. Calculate DH at 358 K for the reaction
Fe2O 3(s) + 3H2(g) 2Fe (s) + 3H2O(l)
Given that, DH298= 33.29 kJ mole1 and Cp for Fe2O 3 (s), Fe (s), H2O (l) and H2 (g) are
103.8, 25.1, 75.3 and 28.8 J/K mole.
(A) 22.22 KJ/mole (B) 25. 123 KJ/mole (C) 28.136 KJ/mole (D) 30.135 KJ/mole
56. Ka for CH3COOH at 25C is 1.754 105 . At 50C, Ka is 1.633 105 What will be value of
D S for the ionisation of CH3COOH?
(A) 94.44 J/mole K (B) 96.66 J/mole K (C) 96.44 J/mole K (D) 90.44 J/mole K

103
57. C2H6 (g) + 3.5 O2 (g) 2CO2 (g) + 3H2O (g)
DSvap (H2O, l) = x1 cal K1 (boiling point + T1)
DHf (H2O, l) = x2
DHf (CO2) = x3
DHf (C2H6) = x4
Hence DH for the reaction is
(A) 2x3 + 3x2 x4 (B) 2x3 + 3x2 x4 + 3x1T1
(C) 2x3 + 3x2 x4 3x1T1 (D) x1T1 + X2 + X3 x4
58. C (s) + O2 (g) CO2, (g); DH = 94.3 kcal/mol
CO (g) + O2(g) CO2 (g); DH = 67.4 kcal/mol
O2(g) 2O (g); DH = 117.4 kcal/mol
CO (g) C (g) + O(g) ; DH = 230.6 kcal/mol
Calculate DH for C (s) C (g) in kcal/mol.
(A) 171 (B)154 (C)117 (D)145
59. The difference between DH and DE on a molar basis for the combustion of noctane at 25C
would be : 25C
(A) 13.6 kJ (B) 1.14 kJ (C) 11.15 kJ (D) + 11.15 kJ
60. What is the work done against the atmosphere when 25 grams of water vaporizes at 373 K
against a constant external pressure of 1 atm ? Assume that steam obeys perfect gas laws. Given
that the molar enthalpy of vaporization is 9.72 kcal/mole, what is the change of internal energy in
the above process ?
(A) 1294.0 cals, 11247 cals (B) 921.4 cals, 11074 cals
(C) 1029.4 cals, 12470.6 cals (D) 1129.3 cals, 10207 cals
61. In the reaction CS2 (l) + 3O 2 (g) CO2 (g) + 2SO2 (g) DH = 265 kcal
The enthalpies of formation of CO2 and SO2 are both negative and are in the ratio 4 : 3. The
enthalpy of formation of CS2 is + 26 kcal/mol. Calculate the enthalpy of formation of SO2.
(A) 90 kcal/mol (B) 52 kcal/mol (C) 78 kcal/mol (D) 71.7 kcal/mol
62. The bond dissociation energy of gaseous H2, Cl2 and HCl are 104, 58 and 103 kcal mol1
respectively. The enthalpy of formation for HCl gas will be
(A) 44.0 kcal (B) 22.0 kcal (C) 22.0 kcal (D) 44.0 kcal
63. AB, A2 and B2 are diatomic molecules. If the bond enthalpies of A2, AB & B2 are in the ratio
1 : 1 : 0.5 and enthalpy of formation of AB from A2 and B2 is 100 kJ/mol1. What is the bond
enthalpy of A2.
(A) 400 kJ/mol (B) 200 kJ/mol (C) 100 kJ/mol (D) 300 kJ/mol
64. Given the following data :
Substance DH (kJ/mol) S(J/mol K) DG (kJ/mol)
FeO(s) 266.3 57.49 245.12
C (Graphite) 0 5.74 0
Fe(s)0 27.28 0
CO(g) 110.5 197.6 137.15

104
Determine at what temperature the following reaction is spontaneous ?
FeO(s) + C (Graphite) Fe(s) + CO(g)
(A) 298 K (B) 668 K
(C) 966 K (D) DG is +ve, hence the reaction will never be spontaneous
65. One mole of a gas occupying 3 dm3 expands against constant external pressure of 1 atm to a
volume of 13 dm3. The work done is
(A) 10 atm dm3 (B) 20 atm dm3 (C) 39 atm dm3 (D) 48 atm dm3
66. The enthalpy change in the oxidation of glucose is 2880 kJ mol1. Twenty five per cent of this
energy is available for muscular work. If 100 kJ of muscular work is needed to walk one
kilometre, then the maximum distance that a person will be able to walk after eating 120 g of
glucose will be
(A) 4.8 km (B) 2.4 km (C) 8.4 km (D) 9.8 km
67. The heat of formation of liquid methyl alcohol is kilojoule per mole using the following data will
be [Heat of vaporisation of liquid methyl alcohol = 38 kJ/mol. Heat of formation of gaseous atoms
from the elements in their standard states : H, 218 kJ/mol; C, 715 kJ/mol; O, 249 kJ/mol.
Average bond energies : C H, 415 kJ/mol; C O, 356 kJ/mol O H, 463 kJ/mol.]
(A) 46.0 kJ/mole (B) 50.0 kJ/mole (C) 73.3 kJ/mole (D) 266 kJ/mole
68. 10 g of argon gas is compressed isothermally and reversibly at a temperature of 27C from 10 L
to 5 L. q, W, DE and DH for this process are [R = 2.0 cal K1 mo l1, log102 = 0.30.
[Atomic wt. of Ar = 40.]
(A) W = 106.635 cal, q = 103.635 cal, DE 0 & DH = 0
(B) W = 53.635 cal, q = 53.635 cal, DE 0 & DH = 0
(C) W = 53.635 cal, q = 63.635 cal, DE & DH 0
(D) W = 103.635 cal, q = 103.635 cal, DE & DH = 0
69. Molar heat capacity of water in equilibrium with ice at constant pressure is
(A) zero (B) infinity () (C) 40.45 kJ1 mol1 (D) 75.48 J K1 mol1
70. Diborane is a potential rocket fuel which undergoes combustion according to the reaction,
B2H6 (g) + 3O2 (g) B2O3 (s) + 3H2O(g)
from the following data, the enthalpy change for the combustion of diborane will be
2B(s) + O2 (g) B2O3(s); DH = 1273 kJ
H2(g) + O2 (g) H2O(l); DH = 286 kJ
H2O(l) H2O(g) ; DH = 44 kJ
2B(s) + 2H2 (g) B2H6 (g); DH = 46 kJ
(A) 2079 kJ mol1 (B) 1091 kJ mol1 (C) 2035 kJ mol1 (D) 762 kJ mol1
71. A sample of argon gas at 1 atm pressure and 27C expands reversibly and adiabatically from
1.25 dm 3 to 2.50 dm3. The enthalpy change in this process will be.
[Cv.m. for argon is 12.48 jK1 mol1].
(A) 114.52 J (B) 114.52 J (C) 57.26 J (D) 57.26 J

105
72. Find DG and DH for that the reaction CO(g) + O2 (g) CO2 (g) at 300 K respectively are,
when the standard entropy change is 0.094 kJ mol1 K1. The standard Gibbs free energies of
formation for CO2 and CO are 394.4 and 137.2 kJ mol1, respectively.
(A) DG = 257.2 kJ/mol, DH = 285.4 kJ/mol
(B) DG = 514.4 kJ/mol, DH = 570.8 kJ/mol
(C) DG = +514.4 kJ/mol, DH = 570.8 kJ/mol
(D) DG = 257.2 kJ/mol, DH = 285.4 kJ/mol
73. DH = 30 kJ mol1, DS = 75 J / k / mol. Find boiling temperature at 1 atm.
(A) 400 K (B) 300 K (C) 150 K (D) 425 K
74. Spontaneous adsorption of a gas on a solid surface is exothermic process because
(A) enthalpy of the system increases. (B) entropy increases.
(C) entropy decreases. (D) free energy change increases.
75. There is 1 mol liquid (molar volume 100 ml) in an adiabatic container initial, pressure being 1 bar
Now the pressure is steeply increased to 100 bar, and the volume decreased by 1 ml under
constant pressure of 100 bar. Calculate DH and DE. [Given 1 bar = 105 N/m2]
(A) DE = 0 J, DH 0 J (B) DH = 0 J, DE = 10 J
(C) DE = 20 J, DH = 890 J (D) DE = 0 J, DH = 10 J
76. The ratio of P to V at any instant is constant and is equal to 1, for a monoatomic ideal gas under
going a process. What is the molar heat capacity of the gas
(A) 3R
2 (B) 4R
2 (C) 5R
2 (D) 0
77. The entropy values (in J K1 mol1) of H2 (g) = 130.6 Cl2(g) = 223 and HCl(g) = 186.7 at
298 K and 1 atmpressure are given. Then entropy change for the reaction.
(A) + 540.3 (B) +727.3 (C) 166.9 (D) +19.8
78. A mixture of 2 mole of CO(g) and one mole of O2 in a closed vessel, is ignited to convert the
carbon monoxide to carbon dioxide. If DH and DU are enthalpy and internal energy change.
Then
(A) DH > DU (B) DH < DU
(C) DH = DU (D) the relationship depends on the capacity of the vessel.
79. For the reaction of one mole zinc dust with one sulphuric acid in a bomb calorimeter, DU and
w correspond to :
(A) DU < 0, w = 0 (B) DU < 0, w < 0 (C) DU > 0, w = 0 (D) DU > 0, w > 0
80. If the enthalpies of formation of Al2O3 and Cr2O3 are 1596 kJ and 1134 kJ respectively,
then the value of DH for the reaction ; 2Al + Cr2O3 2Cr + Al2O3 is :
(A) 462 kJ (B) 1365 kJ (C) 2530 kJ (D) +2530 kJ
81. The internal energy change when a system goes from state A to B is 40 kJ/mole. If the system
goes from A to B by a reversible path and returns to state A by an irreversible path what would
be the net change in internal energy
(A) < 40 kJ (B) Zero (C) 40 kJ (D) > 40 kJ
82. DG for the reaction x + y z is 4.606 kcal. The value of equilibrium constant of the reaction
at 227C is : (R = 2.0 cal K1 mol1 )
(A) 100 (B) 10 (C) 2 (D) 0.01

106
83. The latent heat of vaporisation of a liquid at 500 K and 1 atm pressure is 10 kcal/mol. What will
be the change in internal energy (DE) of 3 moles of liquid at the same temperature?
(A) 13.0 kcal (B) 13.0 kcal (C) 27.0 kcal (D) 27.0 kcal
84. The work done in ergs for a reversible expansion of one mole of an ideal gas from a volume of
10 litres at 25C is :
(A) 3.43 KJ (B) 3.43 Kcal (C) 3.43 J (D) 3.43 cal
85. Reaction, H2(g) + I2 (g) 2HI; DH = 12.40 kcal.
According to this, heat of formation of HI will be
(A) 12.40 kcal (B) 12.4 kcal (C) 6.20 kcal (D) 6.20 kcal
86. The heat of combustions of yellow phosphorus and red phosphorus are 9.91 kJ and 8.78 kJ
respectively. The heat of transition of yellow phosphorus to red phosphorus is :
(A) 18.69 kJ (B) +1.13 kJ (B) +18.69 kJ (D) 1.13 kJ
87. The heat of formation of CO(g) and CO2 (g) are 26.4 kcal and 94.0 kcal respectively. The
heat of combustion of carbon monoxide will be :
(A) + 26.4 kcal (B) 67.6 kcal (C) 120.6 kcal (D) +52.8 kcal
88. The heats of combustion of rhombic and monoclinic sulphur are 70960 and 71030 calorie
respectively. What will be the heat of conversion of rhombic sulphur to monoclinic sulphur?
(A) 70960 cal (B) 71030 cal (C) 70 cal (D) 70 cal
89. An ideal gas expands in volume from 1 10 m to 1 10 m at 300 K against a constant
3 3 2 3

pressure of 1 105 Nm2 . The work is :


(A) 900 J (B) 900 kJ (C) 270 kJ (D) + 900 kJ
90. The enthalpies of combustion of carbon and carbon monoxide are 393.5 and 283 kJ mol1
respectively. The enthalpy of formation of carbon monoxide per mole is
(A) 110.5 kJ (B) 676.5 kJ (C) 676.5 kJ (D) 110.5 kJ
91. If the bond dissociation energies of XY, X2 and Y2 (all diatomic molecules) are in the ratio of
1 : 1 : 0.5 and DHf for the formation of XY is 200 KJ mol1. The bond dissociation energy of
X2 will be
(A) 100 KJ mol1 (B) 200 KJ mol1 (C) 300 KJ mol1 (D) 800 KJ mol1
92. Consider the reaction, N2(g) + 3H2(g) 2NH3(g); carried out at constant temperature and pressure.
If DH and DU are enthalpy change and internal energy change respectively, which of the following
expressions is true ?
(A) DH = 0 (B) DH = DU (C) DH < DU (D) DH > DU
93. An ideal gas is allowed to expand both reversibly and irreversibly in an isolated system. If Ti is
the initial temperature and Tf is the final temperature, which of the following statements is
correct ?
(A) Tf > Ti for reversible process but Tf = Ti for irreversible process
(B) (Tf)rev = (Tf)irrev (C) Tf = Ti for both reversible and irreversible processes
(D) (Tf)irrev > (Tf)rev

107
94. Identify the correct statement regarding a spontaneous process :
(A) Exothermic processes are always spontaneous.
(B) Lowering of energy in the reaction process is the only criterion for spontaneity.
(C) For a spontaneous process in an isolated system, the change in entropy is positive.
(D) Endothermic processes are never spontaneous.
95. In conversion of limestone to lime, CaCO3(s) CaO(s) + CO2(g) the values of DH0 and DS 0
are +179.1 kJ mol1 and 160.2 J/K respectively at 298 K and 1 bar. Assuming that DH and
DS do not change with temperature, temperature above which conversion of limestone to lime
will be spontaneous is :
(A) 845 K (B) 1118 K (C) 1008 K (D) 1200 K
96. For a reversible process at T = 300 K, the volume is increased from Vi = 1 L to Vf = 10 L.
Calculate DH if the process is isothermal
(A) 11.47 kJ (B) 4.98 kJ (C) 0 (D) 11.47 kJ
97. If at 298 K the bond energies of CH, CC, C=C and HH bonds are respectively 414, 347,
615 and 435 kJ mol1, the value of enthalpy change for the reaction ;
H2C = CH2(g) + H2(g) H3CCH3(g) at 298 K will be
(A) +125 kJ (B) 125 kJ (C) +250 kJ (D) 250 kJ
98. Considering entropy(s) as thermodynamic parameter, the criterion for the spontaneity of any
process is :
(A) DS system + DS surroundings > 0 (B) DS system DS surroundings > 0
(C) DSsystem > 0 only (D) DS surroundings > 0 only
99. Assuming that water vapour is an ideal gas, the internal energy change (DU) when 1 mol of water
is vapourisedat 1 bar pressure and 100C, (Given : Molar enthalpy of vapourization of water at
1 bar and 373 K = 41 kJ mol1 and R = 8.3 J mol1 K1) will be :
(A) 37.904 kJ mol1 (B) 41.00 kJ mol1 (C) 4.100 kJ mol1 (D) 3.7904 mol1
100. The standard enthalpy of formation (DHf) at 398 K for methane, CH4(g) is 74.8 kJ mol1. The
additional information required to determine the average energy for C H bond formation would
be.
(A) the dissociation energy of H2 and enthalpy of sublimation of carbon
(B) latent heat of vapourisation of methane
(C) the first four ionization energies of carbon and electron gain enthalpy of hydrogen
(D) the dissociation energy of hydrogen molecule, H2
101. Standard entropy of X2, Y2 and XY3 are 60, 40 and 50 JK1 mol1, respectively.
For the reaction,1/2 X2 + 3/2 Y2 XY3 DH = 30 kJ. To be at equilibrium the temperature
will be :
(A) 500 K (B) 750 K (C) 1000 K (D) 1250 K
102. On the basis of the following thermochemical data : (DGH (aq) = 0)
+

H2O(l) H+ (aq) + OH (aq.) ; DH = 57.32 kJ


H2(g) + O2(g) H 2O(l); DH = 286.20 kJ
The value of enthalpy of formation of OH ion at 25C is :
(A) 228.88 kJ (B) +228.88 kJ (C) 343.52 kJ (D) 22.88 kJ

108
103. In a fuel cell methanol is used as fuel and oxygen gas is used as an oxidizer. The reaction is
CH3OH(l) + 3/2O2 (g) CO 2(g) + 2H2O(l)
At 298 K, standard Gibbs energies of formation for CH3OH(l), H2O(l) and CO2 (g) are 166.2,
237.2 and 394.4 kJ mol1 respectively. If standard enthalpy of combustion of methanol is
726kJ mol1, efficiency of the fuel cell will be :
(A) 87% (B) 90% (C) 97% (D) 80%
104. The standard enthalpy of formation of NH3 is 46.0 kJ mol . If the enthalpy of formation of
1

H2 from its atoms is 436 kJ mol1 and that of N2 is 712 kJ mol1, the average bond enthalpy of
N H bond in NH3 is
(A) 964 kJ mol 1 (B) + 352 kJ mol 1 (C) + 1056 kJ mol 1 (D) 1102 kJ mol 1
105. For a particular reversible reaction at temperature T, DH and DS were found to be both +ve.
If Te is the temperature at equilibrium, the reaction would be spontaneous when.
(A) Te > T (B) T > Te (C) Te is 5 times T (D) T = Te
106. Choose the correct option about the following sentnences [T= True , F =False]
(i) Ice in contact with water constitutes a homogeneous system.
(ii) The process is known as isochoric in which the pressure remains constant throughout the
change, i.e., dP = 0.
(iii) A spontaneous process is reversible in nature.
(iv) In an isolated system, one form of energy cannot be converted into another, i.e., internal
energy remains constant.
[A] FFFF [B] TTTT [C] FTFT [D] FFFT
107. Choose the correct option about the following sentnences [T= True , F =False]
(i) Molar heat capacity at constant pressure = Molar heat capacity at constant volume + PDV.
(ii) A spontaneous process is accompanied by a decrease in entropy.
(iii) DHsub = DHfusion + DHvap.
(iv) The standard heat of formation represents the formation of the compound from its elements
at 25C and one atmospheric pressure.
(v) Whenever an acid is neutralised by a base, the net reaction is
H+ (aq) + OH (aq) H2O(l) ; DH = 13.7 kcal
[A] TFTTF [B] TFTTF [C]TFTFTF [D] TFFFF
108. Match the following :
Column I Column II
(i) A process carried out infinitesimally slowly (A) Adiabatic
(ii) A process in which no heat enters or leaves the system (B) DG = 0
(iii) A process carried out at constant temperature (C) Sublimation
(iv) A process in equilibrium (D) DE = 0, DH = 0
(v) A(s) A(g) (e) Reversible
(vi) Cyclic process (f) Isothermal
(A) (i e, ii a, iii f, iv b, v c, vi e) (B) (i e, ii a, iii f, iv e v c, vi d)
(C) (i e, ii a, iii f, iv b, v c, vi d) (D) (i e, ii a, iii c, iv b, v f, vi d)

109
109. Match the following
ColumnI ColumnII
A. Isothermal process P. q = DU
B. Adiabatic process Q. w = PDV
C. Isobaric process R. w = DU
D. Isochoric process S. w = n RT ln (V2/V1)
(A) A= S B= R C= Q D= P (B) A= P B= Q C= R D= S
(C) A= R B= S C= Q D= P (D) A= S B= R C= P D= Q

Question 110 to 115 are reasoning question choose the correct statement.
(A) Statement 1 is true, Statement 2 is true.Statement 2 is the correct explanation for
statement 1
(B) Statement 1 is true,Statement 2 is true.Statement 2 is not the correct explanation for
statement 1
(C) Statement 1 is true, Statement 2 is False
(D) Statement 1 is false Statement 2 is True
110. Statement1 : The enthalpy of formation of H2O(l) is greater than of H2O (g).
Statement2: Enthalpy change is negative for the condensation reaction H2O (g) H2O(l)
(A) A (B) B (C) C (D) D
111. Statement1 : Heat of neutralisation of perchloric acid, HClO3, with NaOH is same as that of
HCl with NaOH.
Statement2: Both HCl and HClO4 are strong acids.
(A) A (B) B (C) C (D) D
112. Statement1 : When a gas at high pressure expands against vacuum, the work done is maximum.
Statement2: Work done in expansion depends upon the pressure inside the gas and increase
in volume.
(A) A (B) B (C) C (D) D
113. Statement1 : In the following reaction :C(s) + O2 (g) CO2 (g); DH = DU RT
Statement2: DH is related to U by the equation, DH = DU Dng RT
(A) A (B) B (C) C (D) D
114. Statement I : The chemical reaction, 3H2(g) + N2(g) 2NH3 shows decrease in entropy.
Statement II: The process passes into equilibrium state when DGT,P becomes zero.
(A) A (B) B (C) C (D) D
115. Statement I : Both H and U are state functions.
Statement II: Absolute values of H or U can be determined.
(A) A (B) B (C) C (D) D
116. 1 mole of NH3 gas at 27 C is expanded adiabatic condition to make volume 8 times (g = 1.33).
Final temperature and work done respectively are
(A) 150 K, 900 cal (B) 150 K, 400 cal (C) 250 K, 1000 cal (D) 200 K, 800 cal

110
117. Calculate the work performed when 2 moles of hydrogen expand isothermally and reversibly at
25C form 15 to 50 litres.
(A) 1438 Cal (B) 1436 cal (C) 1348 cal (D) 1346 cal
118. The DHf0 for CO2(g), CO(g) and H2O (g) are 393.5, 110.5 and 241.8 kJ mol1 respectively.
The standard enthalpy change (in kJ) for the reaction CO2(g) + H2(g) CO(g) + H2O (g) is
(A) 524.1 (B) 41.2 (C) 262.5 (D) 41.2
119. For reaction carried out in automobiles, what is the value of D H, DS and DG ?
2C8H 18(g) + 25O2(g) 16CO2(g)+ 18H2O(g)
(A) +, , + (B) , + , (C) , +, + (D) +, +,
120. At 25 temperature equilibrium constant Kp for given reaction is = 1.8 107 Then what is the
value of DG ? PCl5 D PCl3 + Cl2
(A)+9197.5 (B) 9197.5 (C) +771 (D) 771.6
121. The conversion A to B is carried out by the following path :
Given : DS(A C) = 50 e.u. , DS(C D) = 30 e.u. , DS(B D) = 20 e.u.
Where e.u. is entropy unit then DS(A B) is
(A) +100 e.u. (B) + 60 e.u. (C) 100 e.u. (D) 60 e.u.
122. For the chemical reaction A + B P + Q two paths are given in the diagram. Which of the
following relationship is correct

(A) DH1 + DH2 = DH3 + DH 4 (B) DH1 + DH2 = DH3 DH 4


(C) DH3 DH1 = DH4 DH 2 (D) DH1 DH2 = DH3 + DH 4

123. For the reaction, + H H bond energies are given as under

(i) CC, 346 kJ/mol (ii) CH, 413 kJ/mol


(iii) HH, 437 kJ/mol and (iv) C = C, 611 kJ/mol
What will be the value of DH 25C for the above reaction ?
(A) 289 kJ mol1 (B) 124 kJ mol1 (C) + 124 kJ mol1 (D) +289 kJ mol1
124. The value of DHf of U3O 8 is 853.5 KJ mol1. Also DH for the reaction
3UO2 + O2 U3O8, is 76.00 KJ. The value of DHf of UO2 is approx
(A) 259.17 KJ (B) 310.17 KJ (C) + 259.17 KJ (D) 930.51 KJ
125. The heat procuced by complete neutralisation of 100 ml of HNO3 with 300 ml of decimolar
KOH solution is 1.713 kJ. The molarity of HNO3 solution will be
(A) 0.1 (B) 1 (C) 0.3 (D) 0.5

111
Hints
V2 20
(16) W = 2.303 nRT log = 2.303 1 8.314 107 298 log
V1 10
= 298 107 8.314 2.303 log 2
(20) As dew formation is spontaneous process therefore entropy or randomness of the universe will
increase. As randomeness of the system has decreased but randomness of the surrounding will
increase larger so that change is positive.
(21) It I is because of the fact that for spontaneity, the value of DG = (DH TDS) should be <
0. If DS is ve, the value of TDS shall have to be less than DH or the value of DS has to
be less than that of x
(24) In an isolated system, there is no exchange of energy or matter between the system and
surrounding. For a spontaneous process in an isolated system, the change in entropy is
positive, i.e. DS > 0
(52) DH = DE + Dng RT
Here, Dng = 6 7.5 = 1.5.
Thus, DE = DH + Dng RT = 780980 (1.5 ) 2 298 = 780090 calories.
(53) F2O (g) + H2O (g) O2 (g) + 2HF(g); DH = 76.1 kcal.
(55) DCp = 2 25.1 + 3 75.3 [103.8 + 3 28.8] = 85.9 J/K mole.
DH 2 - DH1
We have, T2 - T1 = DCp

DH 358 - (-33290)
= 85.9
358 - 298
DH358 = 28136 J/mole = 28.136 kJ/mole.
(56) (DG)298 = 2.303RT log K = 2.303 8.314 298 log (1.754 10-5) = 27194 J.
(DG)323 = 2.303 8.314 323 log ( 1.633 10-5) = 29605 J.
DG = DH TDS
27194 = DH 298 DS
29605 =DH 323 DS
DS = 96.44J/mol.K
(59) DH DE = 4.5 8.315 298 J = 11.15 kJ
(61) Cs2 (l) + 3O2 (g) Co2 (g) + 2SO2 + 2SO2 DH = -265Kcal
Let DHf (CO2, g) = 4x and DHf (SO2 , g) = 3x
DHreaction = DHf (CO2 ,g) = 2 DHf (SO2 .g) DHf (CS2 l)
265 = 4x + 6x 26
x = 23.9
\ DHf (SO2,g) = 3x = -71.7 Kcal / mol.

115
(62) Given
H2 (g) HCl (g); DH = 104 kcal ...(1)
Cl2 (g) 2Cl(g); DH = 58 kcal ...(2)
HCl (g) H(g) + Cl(g); DH = 103 kcal ...(3)
Heat of formation for HCl
H2 (g) + Cl2 (g) HCl (g); DH = ?
Divide equation (1) and (2) by 2, and then add
H2 (g) + Cl2 (g) H(g) + Cl(g); DH = 81 kcal ...(4)
Subtracting equation (3) from equation (4)
HCl (g) H(g) + Cl(g) ; DH = 103 kcal ...(3)

H2 (g) + Cl2 (g) HCl(g) ; DH = 22.0 kcal
\ Enthalpy of formation of HCl gas = 22.0 kcal
(76) From first law of Thermodynamics, DE = q + w nCvdT = nCdT PdV .....(1)
Now according to process, P = V and according to ideal gas equation, PV = nRT
We have, V2 = nRT
nRdT
On differentiating, 2VdV = nRdT and PdV = VdV =
2
nRdT
So from first equation we have, nCvdT = nCdT
2
R 4R
So, Cv = C Hence C =
2 2
(89) W = PDV
= 1 105 (1 102 1 103) = 1 105 9 103 = 900 J.
(90) DH = 393.5 kJ mol1 .... (1)
CO(g) + O2(g) CO2(g) ; DH = 283 kJ mol1 .... (2)
On subraction equation (2) from equation (1), we get C(s) + O2(g) CO(g) ; DH =
110.5 kJ mol1.
The enthalpy of formation of carbon monoxide per mole = 110.5 kJ mol1.
DH = 393.5 kJ mol1 .... (1)
CO(g) + O2(g) CO2(g) ; DH = 283 kJ mol1 .... (2)
(91) Let the bond dissociation energy of XY, X2 and Y2 be x,x and x, KJ/mol respectively,
1 1
X 2 + Y2 XY; DH = 200 KJ mol1.
2 2
DHreaction = [(sum of bond dissociation energy of all reactants) (sum of bond dissociation
energy ofproduct)]

1 1 x 0.5x
= DH x 2 + DH y2 - DH xy = + x = 200 \ x = = 800 KJ mol1.
2 2 2 2

116
Second method

XY
X (g) + YH
(g) =a + kJ / mole ; .(i)

X 2
2XH =a+ kJ / mole ; . (ii)

Y2
2YH =0.5
+ a kJ / mole ; . (iii)

1 1 1 1
(ii) + (iii)-(i), gives X 2 + Y2
XY ;
2 2 2 2
(92) N2 + 3H2 2NH3 Dn = 2 4 = 2
DH = DU + DnRT = DU 2RT. \ DH < DU.
(93) In isolated system, the expansion of gas is carried out adiabatically. Since heat exchange
between system and surrounding is not possible i.e. q = 0 and secondary wrev is always
greater than wirr- therefore for reversible process there must be comparatively higher decreases
in internal energy i.e. DU for reversible process will be more negative. Hence, final temperature
in reversible process will be smaller than irreversible process.
\ (Tf)irrev > (Tf)rev
(94) In an isolated system, there is no exchange of energy or matter between the system and
surrounding. For a spontaneous process in an isolated system, the change in entropy is
positive, i.e. DS > 0.
Most of the spontaneous chemical reactions are exothermic. A number of endothermic reaction
are spontaneous e.g melting of ice (an endothermic process) is a spontaneous reaction.
The two factors which are responsible for the spontaneity of process are
(i) tendency to acquire minimum energy
(ii) tendency to acquire maximum randomness.
(95) DG = DH TDS
for a spontaneous process DGo < 0
DHo TDSo < 0
TDSo > DHo

DH o 179.1 1000
T> T>
DSo 160.2
T > 1117.9 K 1118 K.
(99) DU = DH DnRT
= 41000 1 8.314 373 = 41000 3101.122
= 37898.878 J mol1 = 37.9 kJ mol1.
(100) C + 2H2 CH4; DHo = 74.8 kJ mol1
In order to calculate average energy for C H bond formation we should know the followng data.
C(graphite) C(g); DH o = enthalpy of sublimation of carbon
H2 (g) 2H(g) ; DHo bond dissociation energy of H2.

117
(101) DS reaction = 50 1/2 (60) 3/2 (40) = 40 JK1
For reaction to be at equilibrium
DG = 0
DH 30000
DH TDS = 0 T = = = 750 K
DS 40
(102) H2(g) + O2(g) H2O (l) DH = 286.20 kJ
DHr = D Hf (H2O,l) D Hf (H2 , g) DHf (O2 , g)
286.20 = DHf (H2O (l))
So DHf (H2O, l) = 286.20 KJ/mole
H2O (l) H+ (aq) + OH (aq) DH = 57.32 kJ
DHr = DHf (H+, aq) + DHf(OH, aq) DHf (H2O, l)
57.32 = 0 + DHf (OH, aq) (286.20)
DHf (OH, aq) = 57.32 286.20 = 228.88 kJ.
(103) CH3OH(l) + 3/2 O2 (g) CO2(g) + 2H2O(l)
DGr = DGf (CO2, g) + 2DGf (H2O, (l)) DGf (CH3OH, (l)) DGf (O2, g)
= 394.4 + 2 (237.2) (166.2) 0 = 394.4 474.4 + 166.2 = 868.8 166.2
DGr = 702.6 kJ
702.6
% efficiency = 100 = 97%.
726
(105) DG = DH TDS
For spontaneous reaction DG must be negative
At equilibrium temperature DG = 0
to maintain the negative value of DG
T should be greater than Te.
V2 50
(117) W = 2.303 n RT log V = 2.303 2 2 298 log = 1436 calories.
1 15
(120) DG = RT ln Kp
= 2.303RT log Kp
= 2.303 1.987 298 log1.8 107
= 2.303 1.987 298 ( 6.7447) = 9197.5 Cal

(131) 3 5
Cv = RT; Cp = RT for monoatomic gas
2 2
5 7
Cv = RT; Cp = RT for diatomic gas
2 2 3 5 5 7
RT + RT RT + RT
Thus for mixture of 1 mole each, C v = 2 2 and Cp = 2 2
2 2
Therefore, Cp / C v = 3RT = 1.5 .
2RT

118
(132) 78g of benzene on combustion produces heat = 3264.6 kJ
-3264.6
\ 39g will produce = = 1632.3 kJ.
2
(137) G = -2.303 RT log K , Here R = 2 cal, T = 300K

10 15
K = = 10; G = -2.303 2 300 log10 10
3 5
= = -2.303 2 300 1 = -1381.8 cal
(140) For 2 moles of water vapour, Absorbed energy by system is
DHvap = 2 x 9720 = 19440 Cal.
DH Vap
DSvap =
T b

19440
= (100 + 273)

= 52.12 Cal. K1. mole1


= 52.12 4.184
= 217.6 joule.K1. mole1
(142) DG= 2.303 RT logk
= 2.303 x 1.987 298 log4
= 1363.7 log 4
= 1363.7 0.6021
= 821.1 Cal.
(143) DG = nFEcell = 2 96500 1.20J = 231.6 KJ
(144) DG = DH TDS
O = 4000 T 10 because (DG = 0)
T = 400 k
i.e. At 400 k, temperature reaction will be in equilibrium. But at temperature higher than 400
k the value of DG will be negative.
Thus at 500 k temperature reaction will be spontaneous.
(145) DGo= DGoP DGoR
= DGoC [DGoA + DGoB]
= 25 [10 15]
= 25 + 25 = 0
DGo= 0 so K = 1
13.95 44
(149) H = = 278.7 kcal
2.2016

119
UNIT : 6 SOLUTIONS
Important Points
When two or more than two substance mix and from a uniform or homogeneous mixture, Such a
mixture is called solution.
Type of solutions
The solutions can be found in three states; Solid, Liquid and Gas. The solute and solvent can also be
in three states. The physical state of the resulting solution can be decided on the basis of physical state
of solute and solvent.
Sr No. Type of solution Physical state Examples
Solute Solvent

1 Solid solution Solid Solid Alloy formed from copper


and zinc (Brass).
Liquid Solid Zinc amalgam-Zinc
Gas Solid dissolved in mercury (Zn/Hg
adsorption of H2 gas on Pd.

2 Liquid solution Solid Liquid Homogeneous mixture of


sugar and water.
Liquid Liquid Homogeneous mixture of
water and ethanol.
Gas Liquid Homogeneous mixture of
CO2 gas in water.

3 Gaseous solution Solid Gas Homogeneous mixture of


camphor in N2 gas.
Liquid Gas Air containing moisture
Gas Gas Mixture of H2 and O2 gas.

1000 mass of solute ( gram )


Formality ( F ) =
Formula mass of solute volume of solution ( ml )

100 volume of solute 100 volume of solute ( ml )


%V V= =
volume of solute + volume of solvent volume of solvent ( ml )

100 mass of solute ( gram )


%W V=
volume of solution ( ml )

120
amount of solute ( mg )
parts per million by mass to volume =
amount of solution ( litre )

Factors which effect the solubility of gaseous solute in liquid solution formed by homogeneous
Mixture of gaseous solute and are given as under.
(i) Nature of gaseous solute and the solvent (ii) Effect of temperature (iii) Effect of pressure
Henrys law : p = KH .X where, KH is Henrys constant.
When solid solute is dissolved in solid solvent is gives solid Solution. The molecules are arranged
in two ways:
(1) Substituted solid solution (2) Interstitial solid solution
Solution-Colligative Properties
When solute substances are dissolved in pure solvent, the solutions are obtained. Some properties
of solvent change viz. the vapour pressure of a solution prepared from a solvent is less than that
of pure solvent, while the boiling point increases and freezing point decreases. The osmotic
pressure also changes. The change in these properties depend in number of molecules of solute
but not on nature of solute. Such properties are called colligative properties of solution.
Raoults Law (For Non-volatile Solute)
If dilute and ideal solution is prepared by dissolving non-volatile solute in a volatile solvent, the
relative lowering of vapour pressure of the solution is equal to the mole fraction of the dissolved
solute.

P10 - P1 n2
= = X2
0
P1 n1 + n 2

i.e. mole fraction of solute. Where, n1 and n2 are the moles of solvent and solute respectively.
|For Very dilute solution n2<<n1, hence putting n1+n 2 in equation

P10 - P1 n2 w W
0
= but = n 2 = 1 and n1 = 1 ; putting this values in eqution
P1 n1 M2 M1

P10 - P1 W2 M1
=
0
P1 M 2 W1 where, W1 = mass of solvent, M1 = molecular mass of solvent
W2 = mass of solute, M1 = molecular mass of solute
Raoults Law (For Volatile Solute and Volatile Solvent)
Suppose in a binary solution XA is the mole-fraction of solute A and its partial pressure is p and
XB is the mole fraction of the solvent B and its partial pressure is PB then according to Raoult
law PA X A and PB XB .
According to experimental observations Raouls law can be proved that if the vapour pressure
of solute is PA0 and vapour pressure of pure solvent is PB0 then,

P = p 0A X A and p 0B X B
Total Pressure P = p A + p B Total pressure p = p 0A X A + p 0B X B
121
In vapour If at equilibrium stage, YA and YB are the
mole-fractions of component A and component B
respectively the at equilibrium the partial pressure PA
and P B of each component can be obtained by
equations given below.
PA =YA. P total
PB=YB . Ptotal

K b 1000 w 2
Tb = where K b = Molal elevation constant W1 = mass of solvent
M 2 w1

DTb w 1 M 2
\ Kb =
1000 w 1 W2 = mass of solute M2 = molecular mass of solute

K f 1000 w 2
D Tf = DTb = elevation in boiling point.
M 2 w1

DT f w1 w 2
Kf = where, Kf = molal depression constant w1 = mass of solvent
1000 w 2
W2 = mass of solute M2 = molar mass of solute
D T f = depression in freezing point.
Laws of osmotic pressure
(i) Boyles-vantt Hoff Law :
n
C= = molar concentration \ V = K
V
(ii) Gay-Lussac vant Hoff Law :
= KT where, K = proportionality conctant
(iii) Avogadros-vantt-Hoff law
nRT
pan p = where, R is proportionality constant and its value is equation
V
that of gas Constant
T (Boyle-vant-Hoff Law). \ V = nRT
T (Gay-Lussac-vant-Hoff Law) where = Osmotic pressure in bar
V = volume of solution in life
n = number of moles of solute
R = gas constant
T = absolute temperature in Kelvin.
122
Then, solution whose osmotic pressure is less it is known as hypotonic with comparison to higher
pressure solution.
A solution whose osmotic pressure is higher, in comparison to solution having less osmotic
pressure is called hypertonic solution.
Less than 0.91% W/V pure NaCl solution is hypotonic compared to fluid inside human blood
NaCl solution having concentration more than 0.91% W/V is hypertonic with respect to fluid
inside human body.
Normal molar mass of solute
Vant Hoff factor (i) =
Abnormal moral mass of solute
Theorical molar mass of solute
=
Experimental moral mass of solute

Observed colligative property


=
Theoritical collingative property
By introducing vant Hoff factor (i) the formula to obtain molar mass can be written as follows
p10 - p1 n2 n
Raoults law : = i i 2
p10 n1 + n 2 n1

Molal elevation : Tb = i K b .m
Molal depression : Tf = i K f .m

nRT
Osmotic pressure of solution : = i
V
1000 Molarity
Molality = (1000 density) - (molar mass of solute molarity)

M.V = M1.V1+M2.V2+M3.V3, V = V1+V2+V 3


a2 i - 1 i = DTob pob
=
M Tf Kf
Ka = Molarity 1 - a a = =
DTcal pcal M ob = Theoretical molar mass Tb = Kb
n -1

n
Degree of association (X) = (1-i)
n -1
i -1
Degree of dissociation ( a ) =
n -1
[(i ) p = p10 . X + pB0 . X B (ii ) p = p A0 + ( pB0 - p A0 ) X B
UR = QY.VU+VW.QU
UR = QY+(VW-QY)QU
(iii ) p = pB0 + ( p A0 - pB0 ) X A
UR = VW+ [(QY-VW)VU]

123
M.C.Q.
1. Which of the following possesses physical state of solute and solvent are liquid and solid
respectively ?
(a) solution of sugar in water (b) zinc amalgam
(c) solution of Naphthelene in benzene (d) brass
2. Which of the following alternative is correct for physical state of solute and solvent for a solution
of camphor in N 2 gas ?
(a) solid, gas (b) solid, liquid (c) gas, solid (d) gas, gas
3. Which of the following is an example of solid solution ?
(a) interstitial compound of hydrogen and Pd (b) WC
(c) Zn / Hg (d) Given all
4. Which of the following pair of solution having same physical state of solute ?
(a) homogenous mixture of chloroform in N2 gas, solution of CO2 in water
(b) brass, sodium amalgam
(c) homogenous mixture of camphor in N2 gas, solution of H2 gas in Pd metal
(d) moist air, solution of ethanol in water
5. Which of the following pair of solution having same physical state of solvent ?
(a) homogenous mixture of chloroform in N2 gas, solution of CO2 in water
(b) brass, solution of salt in water
(c) homogenous mixture of camphor in N2 gas, moist air
(d) solution of H2 gas in Pd metal, solution of ethanol in water
6. Which of the following pair of solution having different physical state of solute ?
(a) homogenous mixture of chloroform in N2 gas, solution of CO2 in water
(b) brass, homogenous mixture of camphor in N2 gas
(c) sodium amalgam, moist air
(d) solution of H2 gas in Pd metal mixture of N2 and O 2
7. Which of the following pair of solution having different physical state of solvent ?
(a) homogenous mixture of N2 gas in chloroform, mixture of N2 and O2
(b) brass, solution of H2 gas in Pd metal
(c) sodium amalgam, solution of urea in water
(d) solution of CO2 in water, solution of Naphthelene in benzene
8. Which of the following compound possesses maximum solubility in water ?
(a) pentan 1 - ol (b) Pentane 2, 3 diol
(c) pentane 1, 2, 3 triol (d) Pentane 1, 2, 3, 4 tetraol

124
9. Which of the following given compound is least soluble in water ?
(a) Hexan 1 ol (b) glycerol
(c) Propane 1, 3 diol (d) ethylene glycol
10. To convert molarity into which of the following unit of concentration, does not require density
of the solution ?
(a) molality (b) normality (c) mole-fraction (d) % w/w
11. To convert molality into which of the following unit of concentration require density of the
solution ?
(a) percentage weight by weight (b) percentage by volume
(c) mole-fraction (d) given all
12. Which of the following unit of concentration does not depend on temperature ?
(a) formality (b) molarity (c) molality (d) normality
13. Which of the following unit of concentration depends on temperature ?
(a) molality (b) normality (c) mole-fraction (d) given all
14. What would be the formality of solution prepared by dissolving 9.48 gm. Potash-alum dissolved
in 5 litre water ? [M.W of Potash-alum = 948 gm/mole]
(a) 0.04 F (b) 0.02 F (c) 0.002 F (d) 0.004 F
15. What quantity of potash-alum is required to prepare 500ml solution having strength 1.5 F
(a) 711gm (b) 355.5 gm (c) 35.55 gm (d) 71.1 gm
16. What would be the concentration in % w/v of aqueous solution in which 80 ml ethanol is
dissolved in 4 lt. Solution ?
(a) 4% v/v (b) 10% v/v (c) 2% v/v (d) 8% v/v
17. What would be the volume of acetone required to prepare 10% v/v acetone solution in 5 lt.
Solution ?
(a) 50 ml (b) 5 ml (c) 100 ml (d) 500 ml
18. 15 % w/v solution of sugar is prepared by dissolving 1200 gm sugar in water, then, what would
be the volume of the solution ?
(a) 8000 ml (b) 4 lt (c) 800 ml (d) 5000 ml

19. 0.004 gm O 2 is dissolved in an aqueous solution of 50 litre, then, what would be the ppm of
solution by weight-volume ?
(a) 0.04 (b) 0.008 (c) 0.004 (d) 0.08
20. The concentration of Ca2+ion in a sample of water is 0.0002 M then what would be the
concentration of Ca2+ in ppm by weight-volume ? (Atomic wt. of ca = 40 gm/mole)
(a) 4 (b) 8 (c) 0.08 (d) 0.4

125
21. The concentration of F-ion in a sample of water is 10 ppm; then, concentration of F-ion in a
solution in % w/v is
(a) 10-3 (b) 10-2 (c) 10 (d) 10-4
22. What would be the weight of O2 gas in gram dissolved in an aqueons solution of 500ml having
strength 5 ppm ?
(a) 0.025 (b) 2.5 10-4 (c) 2.8 10-3 (d) 2.8 10-5
23. What would be the molarity of solution prepared by taking a mixture of 1400 ml 0.3 M, 700
ml 0.4 M and 500 ml 1.2 M aqueous solutions ?
(a) 0.5 M (b) 0.8 M (c) 0.6 M (d) 0.7 M
24. What quantity of KOH is required to prepare 10 % w/w KOH solution having weight 1000 gm
?
(a) 50 gm (b) 25 gm (c) 100 gm (d) 150 gm
25. What amount of water is added in an aqueous solution of 5000 ml having concentration 1.5 M
to prepare 0.5 M solution ?
(a) 15 litre (b) 5 litre (c) 10 litre (d) 20 litre
26. On Which factors, the solubility of gaseous solute in liquid depends ?
(a) temperature (b) Pressure of the gas
(c) Nature of gaseous solute and solvent (d) Given all
27. At 293 K temperature, if partial pressure of all given gases are same, then, which of the following
gas possesses maximum solubility in water ?
(a) He (b) N2 (c) H2 (d) O2
28. At 298 K temperature, if partial pressure of all given gases are same, then, which of the following
gas possesses least solubility in water ?
(a) carbon dioxide (b) formaldehyde (c) methane (d) vinyl chloride
29. At 298 K temperature, if partial pressure of all given gases are same, then, which of the following
is the correct ascending order of solubility of gases in water ?
(a) Ar < HCHO < CH4 < CH2 = CH Cl
(b) Ar < CH2 = CH Cl < CH 4 < HCHO
(c) Ar < CH4 < HCHO < CH2 = CH Cl
(d) Ar < HCHO < CH2 = CH Cl < CH4
30. At 293 K temperature, if partial pressure of all given gases are same, then, which of the following
is the correct descending order of solubility of gases in water ?
(a) H2 > N 2 > O 2 > He (b) N2 > H 2 > O 2 > He
(c) O2 > N 2 > H 2 > He (d) O2 > H 2 > N 2 > He
31. At 293 K temperature, for solubility of all given gases, in water, which gas possesses higher value
of KH ?
(a) He (b) N2 (c) H2 (d) O 2
126
32. At 293 K temperature, for solubility of all given gases, in water, which gas possesses lower value
of KH ?
(a) Carbon dioxide (b) formaldehyde (c) methane (d) vinyl chloride
33. At 298 K temperature, for solubility of all given gases, which of the following is the correct
ascending order of values of Henrys constant ?
(a) CH4 <Ar < HCHO < CH2 = CH Cl
(b) HCHO < CH2 = CH Cl < CH4 <Ar
(c) HCHO < CH4 < CH2 = CH Cl < Ar
(d) CH2 = CH Cl < HCHO < CH4 < Ar
34. At 293 K temperature, for solubility of all given gases, which of the following is the correct
descending order of values of Henrys constant ?
(a) H2 > N 2 > O 2 > He (b) N2 > H 2 > O 2 > He
(c) He > N2 > H2 > O 2 (d) O2 > H2 > N 2 > He
35. At constant temperature, on the basis of the given graph, which gas possesses higher solubility?
(a) A
(b) B
(c) C
(d) D

36. In which of the following specific condition, CO 2 gas is filled in cold drinks, and in soda water ?

(a) at high temperature and high pressure (b) at low temperature and high pressure
(c) at low temperature and low pressure (d) at high temperature and low pressure
37. In which condition, Henrys law is applicable ?
(a) ideal behaviour of gaseous solute at high pressure and low temperature
(b) gaseous solute neither associate nor dissociate in solution
(c) gaseous solute react with solvent
(d) applicable in given all conditions
38. Now a days, divers uses the cylinder having gaseous mixture contains -
(a) 2 % O2 and 98 % He (b) 11.7 % He, 56.2 % N2 and 32.1 % O2
(c) 11.7 % N2, 56.2% He and 32.1% O2 (d) 11.7 % He, 56.2 % O2 and 32.1 % N2

127
39. Due to which reason, O
2
gas liberates from the blood of tissues of animal bodies
(a) less temperature of tissues (b) partial pressure of oxygen gas is more in tissues
(c) partial pressure of carbon dioxide is less in tissues
(d) partial pressure of oxygen gas is less in tissues
40. Which of the following is not a substitutional solid solution ?
(a) wc (b) brass (c) steel (d) monel metal
41. Which of the following is a substitutional solid solution ?
(a) wc (b) bronze (c) steel (d) monel metal
42. Solute + solvent solution; H > O. what would be the change in solubility of substance on
increasing the temperature at equilibrium ?
(a) increases (b) decreases (c) remains constant(d) cant be predicted
43. Which of the following is a colligative property ?
(a) vapour pressure (b) boiling point (c) freezing point (d) osmotic pressure
44. What will be the ratio of any colligative properties of 1.0 m aqueons solutions of Nacl, Na2So 4
ad K4 [Fe(CN)6] (Assume that solute completely (100%) dissociates in the solution)
(a) 2:3:4 (b) 1:2:4 (c) 2:3:5 (d) 1:3:5
45. At constant temperature, vapour pressure of aqueous solutions of Na2so 4, urea and AlCl3 are
equal with the vapour pressure of aqueous solution of 1.2 m kcl solution; then molality of an
aqueons of Na 2So 4, urea and AlCl3 are respectively
(a) 3.6 .2.4 m,4.8 m (b) 0.8 m,2.4 m, 0.6 m
(c) 0.6 m, 3.6 m, 0.8 m (d) 3.6 m, 1.2 m, 2.4 m
46. Mention the correct value of y in the Reference of given below
(a) 62.5 Torr
(b) 37.5 Torr
(c) 60 Torr
(d) 16.33 Torr

47. In the reference of given graph, The value of RU YQ UV is

Vapour
Pressure

Mole fraction
(a) YQ.UV (b) QU. UW (c) VW. QU (d) ST.YQ

128
48. Ionic substances are completely dissociates in the given solutions, then which of the following
solutions possesses highest freezing point ?
(a) 0.01m Urea (b) 0.01m NaCl
(c) 0.01m BaCl2 (d) 0.01m Al2(SO4)3
49. In Binary ideal solution forms by liquid A and B, at constant temperature, mole-fraction of liquid
A in vapour state is 0.4 and its partial vapour pressure is 400 mm, then what will be the partial
vapour pressure of B ?
(a) 600 mm (b) 300 mm (c) 500 mm (d) 200 mm
50. 1.0 molal aqueous solution of a substance boils at 100.55 oC ; then, at what approximate
temperature, it freezes ? (Kb = 0.51 oC kg - mole-1 and Kb = 1.86 oC kg - mole-1)
(a) 272 K (b) 271 K (c) 375 K (d) 274 K
51. Ionic substances are completely dissociates, then aqueous solution of which of the following
substances having least freezing point ?
(a) glucose (b) NaCl (c) Al2(SO4)3 (d) CaCl2
52. 0.2 M aqueous solution of NH4Cl is isotonic with which of the following aqueous solution ?
(a) 0.1 M Na3PO 4 (b) 0.2 M K2SO 4 (c) 0.1 M Al2(SO4)3 (d) none of thses
53. At constant temperature, osmotic pressure of an aqueous solution of 1.5 M NH4NO 3 and xN
Al2(SO4)3 are equal, then mention the value of X. (Assume that ionic solid substances completely
dissociates in the solution)
(a) 0.1 (b) 3.6 (c) 1.2 (d) 0.6
54. At constant temperature, vapour pressure of an aqueous solution of 1.5M NH4NO 3 and xM
Al2(SO4)3 are equal; then calculate the molality of an aqueous solution of Al2(SO4)3. (Assume
that ionic solid substances completely dissociates in the solution)
(a) 0.3 m (b) 2.1 m (c) 3.75 m (d) 0.6 m
55. Boiling point of the aqueous solution prepared by dissolving 1.5 mole substances in 1000 gm
water at 1 atmosphere pressure is 100.5oC; then which of the following alternative is correct for
the solution ? (Kb = 0.152oC kg - mole-1)
(a) i = 1 (b) 1 < i < 2 (c) i < 1 (d) i > 2
56. Which of the following aqueous solution is isotonic with 0.2 m Na4[CoF6] solution ? (Assume
that ionic solid substances completely dissociates in the solution)
(a) 0.2 m urea (b) 0.25 m AlCl3 (c) 0.15m CaCl2 (d) 0.2 m CuSo4
57. Which type of solution, moist air is ?
(a) gas (b) liquid (c) solid (d) colloidal
58. Aqueous solutions are separated by semipermeable membrane. For which pair of the given
solution having maximum osmotic pressure ? (Assume that ionic solid substances completely
dissociates in the solution)
(a) 0.5 m NaCl | 0.1 m Na2SO 4 (b) 0.3 m NaCl | 0.1 m Na2SO 4
(c) 0.5 m NaCl | 0.1 m FeCl3 (d) 0.5 m NaCl | 0.1 m sugar
129
59. At constant temperature, the vapour pressure of an aqueous solution of Na2SO4 and 0.3 m
Na3PO 4 are approximately equal; then, what would be the molality of an aqueous solution of
Na2SO 4 ? (Assume that ionic solid substances completely dissociates in the solution)
(a) 0.5 m (b) 0.6 m (c) 0.4 m (d) 1.2 m
60. What will be the ratio of elevation in boiling point of aqueous solution of 1 m sugar, 1 CsCl, and
1 m Na2SO 4 ? (Assume that ionic solid substances completely dissociates in the solution)
(a) 3 : 2 : 1 (b) 1 : 3 : 2 (c) 1 : 2 : 3 (d) 3 : 1 : 2
61. At constant temperature, solubility of which of the following substances decreases with increase
in temperature ?
(a) aqueous solution of sugar (b) aqueous solution of salt
(c) aqueous solution of CO2 (d) aqueous solution of KNO3
62. At constant temperature, in a closed vessel, an ideal solution is formed by liquid A and liquid
B; and mole-fraction of A and B are 0.6 and 0.4 respectively. If vapour pressure of pure
liquids are 125.0 and 62.5 mm respectively, then their mole-fraction in vapour state are
respectively (In vessel, no other component is in gaseous form)
(a) 0.6 and 0.4 (b) 0.4 and 0.6 (c) 0.25 and 0.75 (d) 0.75 and 0.25
63. At constant temperature, two liquids having osmotic pressure p1 and p2 are seperated by
semipermeable membrane, then, what will be the osmotic pressure of the system ?

p1 + p 2 1 - 2
(a) p1 + p2 (b) p1 p2 (c) (d)
2 2
64. Which of the following pair of solutions forms ideal solution ?
(a) Chloro benzene, chloro ethane (b) benzene-toluene
(c) acetone-chloroform (d) water, HCL
65. Which of the following pair forms true solutions ?
(a) Hexane, heptane (b) chloro benzen, bromo benzen
(c) chloro ethane, bromo ethane (d) phenol, aniline
66. What state does point s indicate ?

(a) Mole-fraction and partial vapour pressure of both the liquids are same
(b) Mole-fraction of the both the liquids are same, but their partial vapour pressures are different
(c) Mole-fraction and partial vapour pressures of both the liquids are different
(d) Mole-fraction of both the liquids are different, but their partial pressures are same.

130
67. What would be the elevation in boiling point of 0.1 m NaCl solution ? (Assume that Nacl
dissociates completely)
(a) kb/10 (b)10 kb (c) kb/5 (d) k b/20
68. Which of the following semipermeable membrane is best one ?
(a) parchment paper (b) copper frrocyanide
(c) butter paper (d) cello phane
69. At constant temperature, binary ideal solution is formed by two liquids A and B. At equilibrium,
mole-fraction of liquid A is 0.7 and in vapour state mole-fraction of A is 0.4 Poa + Pob =90 mm
then at the same temperature, what will be the vapour pressure of pure liquid A and B ?
(a) 40 mm, 50 mm (b) 30 mm, 60 mm (c) 50 mm, 40 mm (d) 20 mm, 70 mm
70. At constant temperature, binary ideal solution is formed by two liquids A and B. At equilibrium,
mole-fraction of liquid B is 0.4 and vapour state mole-fraction of B is 0.25. PoB=40 mm, then
at the same temperature, what will be the vapour pressure of pure liquid A ? (a) 80 mm
(b) 60 mm (c) 40 mm (d)50 mm
71. Choose correct alternative for True and False statements for given diagram. (For correct
statement T and for wrong statement F) (Assume that ionic solid substances completely dissociates
in the solution) (For Correct- Stutement T and for Wrory worry Srut ment F)

(i) Osmotic pressure of the system increase by adding H2O in an aqueous solution of CuSO 4
(ii) The concentration of solution of glucose increases with the passage of time.
(iii) Osmotic pressure of the system decreases by adding glucose in the solution of glucose.
(iv) The concentration of solution of CuSO 4 increases with the passage of time.
(a) FTTF (b) TFFT (c) FFTT (d) TTFF
72. Choose correct alternative for True and False statements for given figure (For correct statement T and
for wrong statement F) (Assume that lomic solid substances completely dissociates in the solution)
(I) concentration of an aqueous solution of FeCl3 increases
with the pressure of time.
(ii) aqueous solution of FeCl3 gradually turns reddish
(iii) concentration of an aqueous solution of KCNS increases
with the passage of time
(iv) aqueous solution of KCNS remains colourless.
(a) FTTF (b) TFFT (c) FFTT (d) TTFF

131
73. What would be the osmotic pressure of the system at 300 k temperature ? (R=8.314 x 10-2
litre-bar-mole-1 k-1) (Assume that lomic solid substances completely dissociates in an aqueous
solution)
(a) 25 bar
(b) 10.0 membrane bar
(c) 24.9 bar
(d) 19.95 bar

74. At constant temperature, 2 litres aqueous solution of each 0.2 M kcl and 0.3 M AlCl3 are in
contact with each other by semipermeable membrane. When osmosis stops, then, what mililitre
water diffuses from semipermeable membrane to the other side ? (Assume that ionic solids
dissociates completely in the aqueous solution)
(a) 500 ml (b) 600 ml (c) 800 ml (d) 1000 ml
75. Choose the correct alternative for the given diagram for correct and wrong statements. (T is for
false statement) (Assume that ionic solid substances dissociates completely in the aqueous
solution)
(I) concentration of solution of NaCl increases with the
passage of time
(ii) concentration of solution of urea decreases with the
passage of time
(iii) concentration of solution of NaCl decreases with the
passage of time
(iv) concentration of solution of urea increases with the
passage of time
(a) FTTF (b) TFFT (c) FFTT (d) TTFF
76. Which of the following solution is hypotonic with fluids in RBC ?(Assume that ionic solid
substances completely dissociates in the solution)
(a) 0.2 M NaCl (b) 0.1 M NaCl (c) 0.18 M NaCl (d) given all
77. At constant temperature, Which of the following solution is hypotonic in the comparison with
fluids in RBC ? (Assume that ionic solid substances completely dissociates in the solution)
(a) 0.17 M NaCl (b) 0.12 M NaCl (c) 0.1 M NaCl (d) given all
78. X M NaCl is isotonic with fluids present in RBC (Red Blood Corpusceles), then what would be
the value of x ? (M.w. Of NaCl =58.5 gm/mole) (Assume that ionic solid substances completely
dissociates in the solution)
(a) 0.15 (b) 0.05 (c) 0.18 (d) 0.78
79. Which of the following solution is hypotonic in comparison with the solution of 0.4 M glucose?
(a) 0.1 M CaCl2 (b) 0.2 M NaCl (c) 0.15 M FeCl3 (d) 0.3 M urea

132
80. Which of the following solution is hypotonic in comparison with 0.15 M kCl solution ?(Assume
that ionic solid substances completely dissociates in the solution)
(a) 0.1 M CaCl2 (b) 0.08 M FeCl3 (c) 0.2 M urea (d) 0.12 BaCl2
81. Which of the following solution is isotonic with fluid of RBC ? (For NaCl, i=2)
(a) 5.6% w/v glucose (b) 2.8% w/v glucose
(c) 1.5% w/v urea (d) 0.91% w/v urea
82. Which of the following solution is isotonic with fluid of RBC ? (For NaCl, 2=2)
(a) 2.02 % w/v glucose (b) 4.02% w/v glucose
(c) 8.0% w/v urea (d) both b and c
83. In Which of the following solution, RBC get burst ? (Molecular wt, CaCl2=111, FeCl3=162.5,
glucose=180 and urea=60 gm/mole) (Assume that ionic solid completely dissociates in aqueous
solution)
(a) 1.2% w/v urea (b) 0.2 MkCl (c) 0.09% M FeCl3 (d) 6.0% w/v
glucose
84. In Which of the following solution, RBC get shrinks ? (Molecular wt, CaCl2=111, FeCl3=162.5,
glucose=180 and urea=60 gm/mole) (Assume that ionic solid completely dissociates in aqueous
solution)
(a) 0.1% w/v CaCl2 (b) 2% w/v urea (c) 1.0% w/v FeCl3 (d) 5.0% w/v
glucose
85. FeCl3 ionizes 80% in their aqueous solution, then what will be the value of Vant Hoff factor i ?
(a) 4 (b) 2.7 (c) 3.4 (d) 3.1
86. CaCl2 ionices 80% in their aqueons solution of 0.2 m CaCl2, then, molality of solution is -
(a) 0.48 m (b) 0.52 m (c) 0.6 m (d) 2.6 m
87. A substances associates in their solution as dimer (or bimolecule), then what will be the value
of Vant hoff factor i ?
(a) 0.2 (b) 0.4 (c) 0.6 (d) given all
88. A substance associates as trimer in their solution; then, what would be the value of Vant Hoff
factor i ?
(a) 0.4 (b) 0.3 (c) 0.2 (d) 0.25
89. A substance associates as bimolecule in their solution; then what would be the value of Vant
Hoff factor i ?
(a) 0.4 d i < 1 (b) 0.5 d i < 1 (c) 0 < i < 1 (d) 0.6 d i < 1
90. The solute remains as dimer in the m-molal solution; then elevation in boiling point irrelevant with
the solution is -

mkb 3mkb 3mkb mkb


(a) (b) (c) (d)
2 5 4 3

133
91. Which of the following ratio is irrelevant formula mass and experimental molecular weight obtained
from colligative properties of solution of ionic solid of AB type ?
(a) 3 : 2 (b) 5 : 3 (c) 4 : 3 (d) 5 : 2
92. Which of the following ratio of correct molecular wt (formula weight) and experimental molecular
weight obtained from colligative properties of solution of ionic solid of AB type is possible ?
(a) 5 : 3 (b) 4 : 1 (c) 7 : 3 (d) 5 : 2
93. A substance associates as trimer in their solution, then which of the following ratio is irrelevant
for real molecular weight and experimental molecular weight obtained from colligative properties
of solution ?
(a) 2 : 3 (b) 2 : 5 (c) 1 : 4 (d) 1 : 2
94. A substance associates as trimer in their solution, then which of the following alternative is
possible for depression in freezing point of m molal solution ?
mkf mkf mkf mkf
(a) (b) (c) (d)
2 4 5 8
95. Which of the following unit of concentration is common in the field of pharmacy ?
(a) formality (b) molarity (c) molality (d) normality
96. A substance associates as trimer in their solution, then what would be the maximum freezing
point of their m molal solution is positive ?

mkf mkf 2mkf


(a) Tf - (b) Tf - (c) Tf - (d) Tf 2mkf
2 3 3
97. Boiling point of the 0.2 m aqueons solution of a substance is 100.4 oC ; then what would be the
freezing point of th solution ? (Kb = 0.513o, K+ = 1.86
(a) 0.372 C (b) 0.37 C (c) 1.45 C (d) 0.5 C
98. Aqueous solution of 0.5 m H2So4 is more concentrated then 0.5 m H2So4 solution; then what
will be the possible density of that solution ?
9m
(a) 1.07 - (b) 1.06 9m/m1 (c) 1.05 9m/m1 (d) 1.02 9m/m1
m1
99. Which of the following is irrelevant with the boiling point of an aqueous solution of xm AlCl3 ?

7 xkb 5 kb
(a) Tb + 3 kb (b) Tb + 5 kb (c) Tb - (d) Tb -
2 2
100. Which of the following is suitable alternative for density of the solution, when molarity (m) and
molality (m) of an aqueous solution of urea is same at fixed temperature ? (molecular wt of
urea = 60 gm/mole ?

3M M 50 + 3m 25 + 2m
(a) 1 - (b) 1 - (c) (d)
50 25 50 25

134
101. Choose the correct option for true and false statement. (For true statement T and for false
statement F is used)
(i) solubility of gas in liquid increases with increase in partial pressure of the gas.
(ii) solubility of gas in liquid increases with increase in temperature
(iii) solubility of gas in liquid isKH is less
(iv) solubility of gas in liquid increases, as partial pressure of gas decreases and
temperature increases.
(a) TTFF (b) FTTT (c) TFFF (d) FFTT
102. Boiling point of an aqueous solution of 0.05m FeCl3 is 100.087 oC; then, what will be the value
of Vant Hoff factor i ? (Kb = 0.513 oC kg - mole-1)
(a) 4 (b) 3.4 (c) 2.5 (d) 2.8
103. Difference of boiling point and freezing point of an aqueous solution of glucose is 104 oC at 1
bar pressure; then what will be the molality of the solution ? (Kb = 0.513o and K+ = 1.86 oC
kg - mole-1)
(a) 2.373 m (b) 1.05 m (c) 2.151 m (d) 1.68 m
104. Difference of boiling point and freezing point of 0.2 m acetic acid prepared in benzene is 75.7
oC; then, state the value of Vant Hoff factor i ? (For benzene, Kb = 2.65 OC kg mol1,
Kf = 5.12 oC kg mol1, Tb = 80 oC, Tf = 5.5 oC)
(a) 1.44 (b) 0.64 (c) 0.83 (d) 0.77
105. Difference in boiling point and freezing point of 10 kg aqueous solution of urea is 100.2372 OC;
then what quantity of urea dissolved in the solution ? (Kb = 0.513o and K+ = 1.86 oC kg
- mole -1)
(a) 59.64 (b) 38.946 (c) 51.65 (d) 40.5
106. 500 ml solution of HCl is prepared by dissolving 14.6 gm HCl in water. What will be the molarity
of HCl in the solution ? (Molecular weight of HCl = 36.5 gm/mole )
(a) 0.4 M (b) 0.3 M (c) 0.8 M (d) 0.3 M
107. What would be the molality of the solution prepared by dissolving 60 gm NaOH in 1.5 kg water
? (Moleculea weight of NaOH = 40 gm/mole)
(a) 0.5 m (b) 1.0 m (c) 0.8 m (d) 0.4m
108. What would be the molarity of 3.0 N H2SO 4 solution ?
(a) 6 M (b) 1.5 M (c) 3 M (d) 1 M
109. What quantity of NaOH is needed to prepare 1.2 m, 800 ml NaOH solution ?
(a) 3.84 (b) 60 (c) 42 (d) 38.4
110. What would be the molarity and normality of solution prepared by dissolving 19.6 gm H2SO 4
in dissolved water to prepare 800 ml solution ? (Molecular weight of H2SO4 is 98 gm/mole)
(a) 0.5 M, 0.25 N (b) 0.25 M, 0.125 N
(c) 0,25 M, 0.5 N (d) 0.125 M, 0.25 N

135
111. What amount of H2SO4 required to prepare 2 litre of 0.5 N H2SO4 solution ? (Molecular weight
of H2SO4 is 98 gm/mole)
(a) 98 gm (b) 24.5 gm (c) 49 gm (d) 73.5 gm
112. What will be the concentration of solution prepared by dissolving 50 gm glucose in 200 gm water
?
(a) 25 % w/w (b) 20 % w/w (c) 35 % w/w (d) 15 % w/w
113. What quantity of urea required to prepare 20 % w/w solution having weight 150 gm ?
(a) 20 gm (b) 40 gm (c) 10 gm (d) 30 gm
114. What is the normality of an aqueous solution of 0.5 Al2(SO4)3 ?
(a) 3 N (b) 1 N (c) 1.5 N (d) 2.5 N
115. What will be the mole-fraction of ethanol in solution, prepared by dissolving 9.2 gm ethanol in
900 gm water ? (Molecular weight of water and ethanol are 18 and 48 gm/mole respectively)
(a) 0.04 (b) 0.004 (c) 0.4 (d) 0.0004
116. What will be the mole-fraction of water and NaOH respectively, when 260 gm NaOH dissolved
in 1.8 kg water ? (M.W of watll Naoh = 18440)
(a) 0.96, 0.4 (b) 0.0962, 0.38 (c) 0.0962, .038 (d) 0.962, 0.0038
117. What would be the mole-fraction of solute in an aqueous solution of a substances heaving
strength 4.5 m ?
(a) 0.75 (b) 0.075 (c) 0.45 (d) 0.045
118. The density of 98% w/w H2SO 4 solution is 1.8 gm/mole then, molarity of the solution is -
(a) 20 M (b) 10 M (c) 18 M (d) None of these
119. Molarity and molality of an aqueous solution of H2SO4 are 1.56 (M) and 1.8 (M) respectively;
then, waqht will be the density of the solution ?
(a) 1.835 gm/ml (b) 1.55 gm/ml (c) 1.02 gm/ml (d) 1.725 gm/ml
120. A solution is preparesd from A, B, C and D mole-fraction of A, B and C are 0.1, 0.2 and 0.4
respectively then, mole-fraction of D is -
(a) 0.2 (b) 0.1 (c) 0.3 (d) 0.4
121. The density of 4 M H2SO4 solution is 1.992 gm/ml then, what will be the molality of the solution
? (Molecular weight of H2SO4 is 98 gm/mole)
(a) 3 M (b) 3.5 M (c) 1.2 M (d) 0.4 M
122. Molarity of 1.2 N aqueous solution of AlCl3 is -
(a) 3.6 m (b) 2.4 m (c) 1.2 m (d) 0.4 m
123. What will be the molality of the solution prpared using 500 gm of 25 % w/w NaOH and 500
gm of 15 % w/w NaOH solution ? (Molecular weight of NaOH = 40 gm/mole)
(a) 12.74 m (b) 6.25 m (c) 9 m (d) 5 m

136
124. What wiil be the molality of solution prepared by taking 25 % w/w NaOH and 15 % w/w NaOH
solution ? (Molecular weight of NaOH = 40 gm/mole)
(a) 12.74 m (b) 5.5 m (c) 9 m (d) 4 m
125. The density of 2.5 M NaOH solution is 1.15 gm/ml; then, which of the following alternative is
correct for molarity and molality ?
(a) M > m (b) M < m (c) M = m (d) can t be
predicted
126. Which of the following is correct for an ideal solution ?
(a) H = 0, V = 0, S = 0 (b) H ` 0, V = 0, S = 0
(c) H = 0, V = 0, S ` 0 (d) H = 0, V ` 0, S = 0
127. Boiling point of Aqueous solutions of 0.05 m ABC and 0.02 m X2Y3 are same at 1 bar pressure;
then, what will be the values of Vant Hoff factor (i) for solute in both the solutions ?
(a) 1.04, 5.1 (b) 1.9, 4.75 (c) 1.2, 3.4 (d) 1.5, 3.7
128. Which of the following substances having concentration of aqueous solution 1% w/w, possesses
higher boiling point ? (Molecular weight of Kcl, BaCl2, glucose and Al2(SO4)3 are 74.5, 208, 342
gmmole respectively) (Assume that inonic solids dissociates completely in their aqueous solution)
(a) KCl (b) BaCl2 (c) glucose (d) Al2(SO4)3
129. Molecular weight of biomolecules such as protein can be determined by ______ method.
(a) osmotic pressure measurement (b) Depression in freezing point measurement
(c) Elevation in boiling point measurement (d) Vapour pressure measurement
130. In the references of the following graph, UR -Oy = __________.

(a) (SU -VW)QU


(b) VU(VW QY)
(c) (VW QY)QU
(d) VW(QV VU)

131. What will be the elevation in boiling point of an aqueous solution of 0.5 m NaCl ? ( i = 1.8)
(a)2.04, 5.1 (b) 1.9, 4.57 (c) 1.2, 3.4 (d) 1.4, 3.7
132. When 2 gm phenol id dissolve in 100gm benzene; then depression in freezing point is 0.69 K.
If its association is dimeric, then calculate its degree of association (X). Molal depression constant
for solvent is 5.12 K - kg - mole-1.
(a) 0.0734 (b) 0.374 (c) 0.00734 (d) 0.734
133. At 353 K temperature, the Vapour pressure of pure liquids A and B are 600mm and 800 mm
respectively. If mixture of liquids A and B boils at 353 K and 1 bar pressure, then mole
proportion of B in percent is -
(a) 80% (b) 60% (c) 20% (d) 40%

137
134. 90 gm glucose and 120 gm urea dissolved in 1.46 kg aqueous solution, then what will be the
boiling point of the solution at 1 bar pressure ? (Kb = 0.512oC -kg mole-1, molecular weight
of glucose and urea are 180 and 60 gm/mole respectively)
(a) 100.876oC (b) 101.024oC (c) 100.248oC (d) 100.007oC
135. pH of 0.2M dibasic acid H2A is 1.699; then, what will be its osmotic pressure at T K
temperature ?
(a) 0.22 RT (b) 0.02 RT (c) 0.4 RT (d) 0.1 RT
136. Boiling point of an aqueous soultion of 0.4m AlCl3 is 100.7oC; then what would be the pressure
of ionization of AlCl3 ? Kb 0.512oC - kg - mole-1.
(a) 80.67% (b) 60.5% (c) 76.54% (d) 84.75%
137. The vapour pressure of homogenous mixture of 10 mole of liquid X and 30 mole of liquid Y at
constant temperature is 550 mm. In this solution, 10 mole of liquid Y increases, hence, increase
in vapour pressure is 10 mm. Then, find the vapor pressure of pure liquid X and Y at that
temperature.
(a) Pox = 200 mm, Poy = 500 mm (b) Pox = 400 mm, Poy = 600 mm
(c) Pox = 600 mm, Poy = 300 mm (d) Pox = 350 mm, Poy = 500 mm
138. What amount of urea dissolved in 1 kg water at constant temperature, so that vapour pressure
of the solution reduced by 2% ? ( M.W of urea = 60 gm/mole)
(a) 68 gm (b) 60 gm (c) 50 gm (d) 75 gm
139. What would be tne volume of 15% w/v and 5% w/v NaOH solution required to prepare 1 litre
aqueous solution of 2M NaOH ? (M.w of Naoh = 40 gram/mole)
(a) 300 ml, 700ml (b) 250 ml, 750ml (c) 400 ml, 600ml (d) 280 ml, 720ml
140. At constant temperature, vapour pressure of an aqueous solution of 1.5 kg glucose decreases
to 0.98% in comparision with vapour pressure of pure water then, what quantity of glucose in
gram dissolved in the solution ? (Molecular weight of glucose = 180 gm/mole)
(a) 148.5 gm (b) 14.85 gm (c) 125 gm (d) 135 gm
141. At constant pressure, 0.5 m NaCl aqueous solution is diluted by adding water in it. Which of
the following statement is correct in this reference ?
(a) Vant Hoff factor (i) and boiling point of the solution both decreases
(b) Vant Hoff factor (i) and boiling point of the solution both increases
(c) Vant Hoff factor (i) decreases while boiling point of the solution both increases
(d) Vant Hoff factor (i) increases while boiling point of the solution both decreases
142. Boiling point of an aqueous solution of 0.5 m ionic solid substance is 100.5OC; then state the
value of i ? (Kb = 0.512oC -kg mole-1)
(a) 1.95 (b) 1.85 (c) 1.25 (d) 0.85
143. Aqueous solution of substance boils at 100.5oC at 1 bar pressure; then at what temperature it
freezes ? (Kb = 0.512oC -kg - mole-1, Kf = 1.86oC - kg mole-1)
(a) 11.2oC (b) 29.84oF (c) 271.8oK (d) -1.2oC

138
144. 1.4 m aqueous solution of a weak electrolyte AB2 ionizes 20%, then, state boiling point and
freezing point of the solution respectively.
(a) 100.86
o
C, -3.12oC (b) 101oC, -3.65oC
(c) 274oC, -3.65oC (d) 374oC, -3.65oC
145. Solute substance in a 1.4 m aqueous solution associates by 25%, then, find the boiling point and
freezing point of solution; where, solute exists as trimer in the solution; thus, n = 3. (Kb =
0.512oC kg mole-1, Kf = 1.86oC kg mole-1)
(a) 100.448oC, -2.28oC (b) 373.58oK, -3.65oC
(c) 100.59oC, 2.17oC (d) 213oF, 270.83ok
146. Molecular mass of a weak acid HA is 60gm/mo1 Ifs experimental molecular mass in its 0.7 M aqueous
solution obtained from colligative properties is 50gm/mo1. Then calculate ionistion consteint of weak
acid HA.
(a) 0.023 (b) 0.0085 (c) 0.035 (d) 0.085
147. At constant temperature, the total pressure of a homogeneous mixture of gas-A and gas-B in a closed
container collected on water is 2.0 bar. Their ratio of mole frachion is 1.6 If the values of their KH are
2.4 10+4 bur and 4.8 104 bar respectively then calcwate its ration of mole fraction when dissolhed
in H2O.
(a) 1:2 (b) 2:1 (c) 3:1 (d) 1:3
148. If one of the colligative property of 0.3m aqueous ti sdn. of Na cl and x m aqueows solution of H2So4
then what would be the approximate vulwe of aqueows solution of H2So4 ? (Density of m H2So4
solution = 1.185 gm/ml)
(a) 0.464 N (b) 0.928 N (c) 0.232 N (d) 0.53 N
149. At one bar pressure the value of ratio of mole fraction of O2 and N2 gas in air is 1:4 The values of KH
of O2 and N2 are 3.3 107 Torr and 6.60 107 Torr respectively. Then calculate the value of ratio of
mole fractions of O2 and N2 gases will be. (1 Torr = 1mm)
(a) 3:1 (b) 2:1 (c) 1:2 (d) 1:3
150. According to Boyle-vant - Hoff law at a constelnt temperature, osmotic pressure of a solution is directly
proportional to its molarity. It means p a C, where C = molarity of solution
\ p = Kc Then calcalate the value of k in sI unit at 24o c temperatuve. (R = 8. 314 J/mole k)
(a) 24.942 J/mol (b) 2494.2 J/mol (c) 0.024942 J/mol (d) 2.4942 J/mol
151. Boiling point of an aqueous solution of urea at one bar pressure is 373.41 k. Then at a constant
temperature, calculate the percentage decrease in vapoure pressure of a solution compared to
(kb = 0.512 k.kg. mol-1)
(a) 1.42 % (b) 2.56 % (c) 4.17 % (d) 3.44 %
152. Calculate PH of a solution prepared by mixing equal volume of an aqueous solution of HCI having
PH = 2 and PH = 5 at 298 k temp.
(a) 3.5 (b) 3.0 (c) 7.0 (d) 2.3

139
153. Which of the followingis the correct formula for Rqoults law when non-volatile solute is mixed with liquid
solveut. where n=mole traction of solute N= mok fraction of solveet, P= vapoure pressure of solution
Po = vapoure pressure of pure sovent and DP=Decrease in vapoure pressure.

Dp n Dp n Dp n Dp N
(a) = (b) p0 = N (c) p = n + N (d) p = n + N
p N

154. Decrease in Freezing point of 75.2 gm pnenol when dissolved in a solvent having kf = 14 k. kg. mole-1.
is 7 k. Calculate the percentege of association of phenol if it is forms a dimev in the solutio.
(a) 62.5 % (b) 80.5 % (c) 70 % (d) 75 %
155. Which of the following is correct option when kcl is dissolved in H20. ?
(a) H = +ve, S = +ve, G = +ve (b) H = +ve, S = ve, G = ve
(c) H = +ve, S = +ve, G = ve (d) H = ve, S = ve, G = +ve
156. Naphthalene is soluble in ether or benzene because. ?
(a) dipole-dipole attraction is equal (b) London forces are equal
(c) Hydrogen bond (d) Ionic attraction
157. Four Liauids are given.
(i) Water : more polar and capacity to torm H-bond.
(ii) Hexanol : moderatly pdar and partial capacity to form H-bond.
(iii) Chloro form : moderatly polar and does not capable to form H-bond.
(iv) Octane : non polar and does not capable to form H-bond.
which of the following pair of liquids mixed with each other in very less proportion.
(a) I, IV (b) I, II (c) II, III (d) III, IV
158. Which of the following is applicable for the solubility of gases in liquid.
(a) Increases with increase in temperature and pressure.
(b) decreases with increase in temp and pressure.
(c) Increases with decrease in temp and increase in pressure.
(d) Decreases with decrease in temp and increase in pressure.
159. Eoncentration of lead metal in a blood of any person is more than that of 10 microgram. dm-1, than that
person is considered as on effect of poision sectrion. Then calculate its concentration in ppb (parts per
billion)
(a) 1 (b) 10 (c) 100 (d) 1000
RT
160. The ratio of of 6 % w v and 9 % w v is one for both. what would be the value of atomic weight of
p
A and B respectively. (AB2 and A2B are electrolytes)
(a) 60, 90 (b) 40, 40 (c) 40, 10 (d) 10, 40
161. Decrease in vapour pressure of an aqueous soln. of an electrolyte is 4% what would be the percentage
increase in elevation in Boiling point ? (kb = 0.512 k. kg. mol-1)
(a) 0.55 % (b) 0.02 % (c) 5.5 % (d) 2 %

140
ANSWER KEY

1 b 26 d 51 c 76 b 101 c 126 c 151 a


2 a 27 d 52 a 77 a 102 b 127 d 152 d
3 d 28 a 53 b 78 a 103 d 128 a 153 a
4 d 29 b 54 d 79 c 104 d 129 a 154 d
5 c 30 d 55 c 80 c 105 a 130 c 155 c
6 a 31 a 56 b 81 a 106 c 131 a 156 b
7 c 32 b 57 a 82 a 107 b 132 d 157 a
8 d 33 c 58 d 83 a 108 b 133 a 158 c
9 a 34 c 59 c 84 b 109 d 134 b 159 c
10 b 35 d 60 c 85 c 110 c 135 a 160 c
11 b 36 c 61 c 86 b 111 c 136 a 161 a
12 c 37 b 62 d 87 c 112 b 137 b
13 b 38 b 63 b 88 a 113 d 138 a
14 d 39 d 64 b 89 b 114 a 139 a
15 b 40 a 65 d 90 d 115 b 140 d
16 c 41 a 66 d 91 d 116 c 141 d
17 d 42 a 67 c 92 a 117 b 142 a
18 a 43 d 68 d 93 c 118 c 143 c
19 d 44 c 69 a 94 a 119 c 144 b
20 b 45 b 70 a 95 a 120 c 145 d
21 a 46 b 71 a 96 b 121 c 146 c
22 c 47 c 72 c 97 c 122 d 147 d
23 a 48 a 73 b 98 d 123 b 148 a
24 c 49 a 74 d 99 b 124 b 149 c
25 c 50 b 75 d 100 c 125 a 150 b

142
(70) XA = 1 XB = 1 0.4 = 0.6
YA = 1 YB = 1 0.25 = 0.75
pB = p0B XB pB = PTotal YB
pB = 40 0.4 16 = PTotal 0.25
= 16 mm PTotal = 64 mm
pA = PTotal YA = 64 0.75 = 48 mm
Now pA = p0A XA
48 = p0A 0.6
p0A = 80 mm
(73) Whiten Suppose m1 H2O is transferred from aqueous solution of KCI to aqueous
solution of Alcl3 then the phenomenon osmosis stops. So morality of soluble particles in
both solutions becomes equal at this time.
M1 = modality of kcl solution
M2 = modality of kcl solution when x ml water is reduced.
M1 V1 = M2 V2
0.2 2000 = M2 (2000 x)
400
\ M2 =
2000 X
Similarly volume of Alcl3 Solution increases by addition of x m1 H2O.
600
morality of Alcl3 solution =
2000+ X
Now morality soluble particles in kcl solution = morality of soluble particles in Alcl3 soln.
n1M1 = n2M2
2 400 4 600
=
2000 X 2000+ X
x = 1000
(74) Liquid present in Red Blood cells is isotonic with 0.91 % wv solution of Nacl
morality of soluble particles in 0.91 % wv Nacl solution
2 1000 0.91
=
58.5 100
= 2 0.1555
= 0.311 M
(75) = MRT where M = effective molarity of solutions kept in contact with semi-permeable
membrane.
= 0.2 M (molarity of soluble particles in Naclsdn)
= 3 0.2 - 2 0.1
=0.4 M.
o dissociation i1
(86) lo =
100 n 1
Fecl3 ionises 80% in its aqueous solution
80 i1
= i = 3.4
100 4 1
(87) when association of any substance takes place in a solution then degree of associationis < 1.
i1
0<1 1
n
1
1i
0< 1 1
1 n
1
0 < 1 i 1
n
1
1< i
n
1
1> i
n
Here n = 2 (given) 145
1
1 > i means 0.5 i < 1
2
1
1
n
1i
0< 1 1
1
n
1
0 < 1 i 1
n
1
1< i
n
1
1> i
n
Here n = 2 (given)
1
1 > i means 0.5 i < 1
2
(90) For association (given) n=2
1
i
2
mKf
imKf
2
mKf
Tf
2
T ob ob M
(91) i= = =
T cal cal M ob
where M = Actual molecular mass of solute
Mob = experimental molecular mass of solute.
For AB Type ionic compound i 2
M
2 5:2>2:1
M ob
(96) The substance forms trimer in a solution due to association.
1
i<1
3

mKf
imKf < mKf
3

mKf
Tf < mKf
3

mKf
mKf < Tf
3

mKf mKf
Tf0 mKf < Tf0 Tf Tf0 Tf0 mKf < Tf Tf0
3 3

(98) If x molar solution of any substance is more concentrated than x molul solution so its
molarity value is less than the molality value.
molarity
<1
molarity

molarity 1000 W MW 0
Now =
molarity MV 1000W

molarity W 0 gm W0 gm
= < 1
molarity V ml V ml

W0 W 0 + W W W0+ W W gm W
Now = = = density of soln (d )
V V V V ml V

W0 W 1000 W M
= d + = d+
V V MV 1000

W 0 gm gm molarity M
= d + (M = mol. mass of solute)
V ml ml 1000

gm molarity M gm gm gm molarity M
d + <1 d <1 +
ml 1000 ml ml ml 1000

0.5 98 gm gm
Here for 0.5 M H2So4 aqueour solution d < (1 + ) d < 1.049
1000 ml ml

146
molarity 1000 W MW 0
(100) =
molarity MV 1000W

molarity W 0 gm
=
molarity V ml

Now molarity = Molality


W0 gm
=1
V ml

gm gm molarity M
d =1 +
ml ml 1000
gm
For urea molecular mass (M) = 60
ml
molarity 60
d=1+
1000
50+3M
d=
50
(103) Tb - Tf = 104
Tb0 + Tb (Tf0 Tf ) = 104

100 + Tb ( 0 Tf ) = 104

Tb + Tf = 4

For glucose i = 1
mKb + mKf = 4
4 4
m= =
Kb + Kf 0.513 + 1.86
m = 1.68
(104) Tb Tf = 100.2372

Tb0 + Tb (Tf0 Tf ) = 100.2372

100 + Tb ( 0 Tf ) = 100.2372

Tb + Tf = 0.2372

For urea i = 1 mKb + mKf = 0.2372


0.2372 0.2372
m= =
Kb + Kf 0.513 + 1.86

m = 0.1

(105) Tb Tf = 75.7

Tb0 + Tb (Tf0 Tf ) = 75.7

80 + Tb ( 5.5 Tf ) = 75.7

Tb + Tf = 1.2
1.2 1.2
imKb + imKf = 1.2 i = = i = 0.77
m(Kb + Kf ) 0.2(2.65 + 5.12)

147
m
(117) X = X = mole fraction of solute
55.55+ m
m = molality = 4.5 m
4.4
X= = 0.075
55.55+ 4.5
gm
(118) Density of 98 % W/W H2SO4 = 1.8
ml

w = 98 gm
W + W0 W + W0
d= V=
V d

100 1000w
V= ml molarity (m) =
1.8 MV
1000981.8
=
98100
= 18 M
(119) and (121)
100 molrity
molality =
(1000 density ) (mol .mass of solute molrity )
use above formula to solve the question
(123) mass of NaOH in 500gm 25% W/W NaOH solution
= 5 25 = 125gm. and
mass of H2O = 5 75 = 375gm.
mass of NaOH in 500gm 15% W/W NaOH = 5 15 = 75gm.
and mass of H2O = 5 85 = 425gm.
mass of NaOH in a mixed solution when both solutions are
mixed W = 125 + 75 = 200gm.
and mass of H2O wo = 375 + 425 = 800gm.
1000w 1000200
Now molality of mlxed solution = = = 6.25 m
MW 0 40800

(124) molality of 25% W/W NaOH


1000w 100025
= = = 8.33 m
MW 0 4075

1000w 100015
molality of 15 % W/W NaOH = = = 4.41 m
MW 0 4085

when two different wncentration containing solutions of same substances are mixed then

conc of dil. solution < concentration of mixed solution < conc. of concentration soln.

4.41 m < conc. (molality) of mixed solution < 8.33 in

(128) From graph

p = p0A + (p0B p0A ) XB

UR = QY + ( VW QY)QU
n % W /W
(129) =X
molecular mass (formula weigh

If volue of x is hignest than solution hare148


highest Boiling point.

(n = no. of ions in a formula)


n % W /W
(129) =X
molecular mass (formula weigh

If volue of x is hignest than solution hare highest Boiling point.

(n = no. of ions in a formula)

(134) mass of solvent in a solution wo = 1460 - (90 + 120) = 1250gm.


90
mole of glucose = = 0.5
180

120
mole of urea = =2
60

Total moles of solute in a solution = 0.5 + 2 = 2.5


1000n 10002.5
molality = = = = 2.0 m
W0 1250

Tb = mKb = 2 0.512 = 1.024 0C.

Tb = 100 + 1.024 = 101.024 0C.

(135) pH = 1.669 [H3 O+] = 0.02 M, [H2 A] = 0.2,


0.02 i1
Degrce of dissociation = = 0.1 = , (n = 3)
0.2 n1

= iMRT = 1.2 0.2 RT i = 1.2 = 0.22RT

(137) nX = 10 , nY = 30

Total mok = 40

XX = 0.25 XY = 0.75

Total uapour pressure = P = pX + pX

nX p0X + nY p0Y = P 0.25p0X + 0.75p0Y = 550 (1)

If mole of liqulid y is in creased by 10 then its uapour pressure is increased by 10 mm.

nX = 10 , nY = 40 Total mole = 50

XX = 0.2, XY = 0.8 and total vapor pressure P = pX + pX = 560 mm

0.2p0X + 0.8p0Y = 560 (2)


By soloing eqn. (1) and (2) we get p0X = 400 mm and p0Y = 600 mm

149
2p 0 p 1
(138) p = = = X (mole fraction of urea)
100 p0 50

m
=
55.55+m

m = 1.134 (molality of urca)

mass of urea (W2 ) = 1.134 60 = 68 gm.

(139) Suppose V litcr 15 % WV NaOH and V1 liter 5 % WV NaOH solution is requirecl to pre
1

paec one litev 2 M NaOH solution.

80 gm. NaOH is required to prepare one liter 2m NaOH solution.

V1 + V2 = 1 liter..........(1) .

150V1 + 50V2 = 80 gm. (2)

By soluing (i) and (ii)

are get V1 = 300 ml. and V2 = 700 ml.

150
UNIT : 7 EQUILIBRIUM
Important Points
It is said that equilibrium is established when number of molecules moving from liquid state to
vapour state and number of molecules moving from vapour state to liquid state are same and it is
dynamic. Equilibrium is established in both physical and chemical types of reactions. At this point of
time the rates of forward and reverse reactions become equal. Equilibrium constant Kc is expressed
as the ratio of the multiplication of concentration of products to the multiplication of concentration of
reactants; concentration of each can be expressed as power of their stoichiometric coefficient.

For reaction aA + bB cC + dD

[C]c [D]d
Kc = a b
[A] [B]

Equilibrium constant has constant value at constant temperature and at this stage macroscopic
properties like concentration, pressure, etc become constant. For gaseous reaction Kp is taken instead
of Kc and partital pressure of gaseous reactants and products are expressed instead of concentrations.
The relation between Kp and Kc is expressed as Kp = Kc(RT)ng. In which direction reaction will
occur (forward or reverse) can be expressed by reaction quotient Q c which is equal to Kc at
equilibrium. Le Chatelier's principle, mentions that if the equilibrium gets disturbed by change in factors
like concentration, temperature, pressure etc., then equilibrium will move in the direction whereby the
effect has been minimised or made negligible and the value of equilibrium constant will not change.
This can be used in industries to know how equilibrium can be obtained by study of changes in factors
like concentration, pressure, temperature, inert gas etc. In industries, we can change or control factors
accordingly so that reaction shifts from reactants to products (left to right). If catalyst is used, only
the rate of required reaction will increase but no change will occur in amounts of reactants or
products because the effect on forward and reverse reactions will be the same and so equilibrium
constant will not change.
The substances which allow the electric current to pass through their aqueous solutions are
called electrolytes. Acid, base and salt are electrolytes because their aqueous solutions conduct electric
current. The reason for the conduction of electric current in aqueous solution of electrolyte is the
formation of ions due to dissociation or ionisation which conducts electric current. While the weak
electrolytes are incompletely dissociated and so the equilibrium is established between its ions and
undissociated molecules. This is called ionic equilibrium.

According to Arrhenius ionisation theory, acid is called a substance which gives hydrogen ion

(H+) and base is called a substanec which gives hydroxyl ion (OH ) on ionisation. According to
Bronsted - Lowry theory, acid is defined as a proton donor and base is defined as proton acceptor.
Each acid has its conjugute base and each base has its conjugate acid. Hence, it is known as
conjugate acid - base or proton - transfer theory. Proton is tranferred between acid and base.

151
Bronsted - Lowry is more general than Arrhenius definition. According to Lewis' definition, acid
means a substance which accepts a pair of electrons and base is a substance which donates a pair
of electrons. This definition can be applied to organic chemistry, complex compounds chemistry in
addition to acid-base. Hence, it is considered universally acceptable. Ionisation constant is also an
equilibrium constant. The ionisation constants of weak acid (Ka) and weak base (Kb) can be deter-
+
mined. Concentration of acid can be expressed as pH = log10 [H3O ]. Hence, pH scale is determined
for acid - base. Similary, concentration of OH can be expressed as pOH = log10[OH ], ionisation
[OH ] can be calculted by the use of
+
constant of water as pKw = log10Kw [H3O ] and
relation pKw = pH + pOH. If pH < 7 solution will be acidic, pH > 7 solution will be basic and pH
= 7 solution will be neutral.
Different salts can be obtained by neutralisation of strong or weak acid and strong or weak
base. In such salts, acidic, basic and neutral salts are included. When such salts react with water,
hydration (hydrolysis) reaction occurs and solution obtained can be acidic, basic or neutral. This is also
an equilibrium reaction and so corresponding equilibrium constant for it can be determined. Hydrolysis
constant is expressed as Kh. pH or pOH can be calculated from the values of Ka and Kb and the
value of Kh characteristic for the particular salt. Some solutions are such whose pH does not change
by addition of small amount of acid or base or in case they are being diluted. Such solutions are called
buffer solutions which can be acidic, basic or of neutral type. The control of pH is useful in the
control of biological reactions in our body and chemical reactions in analytical chemistry, industries etc.
Sparingly soluble salts (whose solubility is less than 0.01M in water) dissolve in water depending
on their solubility and equilibrium is established. Hence, equilibrium constant for this can be
obtained which is known as solubility product constant or solubility product of the sparingly soluble salt.
The study of effect of common ion, acid, etc. on the solubility of sparingly soluble salt can be carried
out by application of Le Chatelier's principle. Generally, the solubility of sparingly soluble salt de-
creases due to effect of common ion. This is used in qualitative analysis. By mixing two solutions,
whether precipitates will be obtained or not, can be predicted by comparing concentration product Ip
with the solubility product Ksp. If Ip > Ksp precipitation will occur and if Ip < Ksp the precipitation
will not occur and if Ip = Ksp, the precipitation will not occur but solution will remain in saturated state.

152
M.C.Q.
1. Three gaseous equilibria have values of equilibrium constants as k1 , k 2 , k3. resp.

K1
(i) A + B
C
K2
(ii) B +C
P+Q
K3
(iii) A + 2B
P+ Q
What is the relation between k1 ,k2, k3. (?)
K
(a) K 3 1 K (b) K 3 K1 K 2 (c) K1 K 2 K 3 (d) K1 K 2 K 3 1
2

2. Three gaseous equilibria have value of equilibrium constants as K1 , K 2 , K 3 respectively..


(i) N 2 O 2 2 NO (K 1 )
(ii) N 2 2O 2 2 NO 2 (K 2 )
(iii) 2NO O2 2 NO2 (K3 )
What is the relation between K1 , K 2 and K 3 (?)
(a) K 3 K1 K 2 (b) K1 K 2 K 3 (c) K 2 K1 K 3 (d) K K1 K2 K3
3. The equilibrium constant for the reaction,
N2(g) O2(g) 2 NO(g)
4
is 4.4 10 at 2000k temp
In presence of a catalyst, equilibrium is attained ten times faster. Therefore the equilibrium constant,
in presence of catalyst at 2000K is........
(a) 4.4 10 4 (b) 4.4 10 5 (c) 4.4 10 3 (d) difficult to compute
4. For the following reaction in gaseous phase
CO 1 2 O2 CO2

K C K P is
1 1
(a) RT 2 (b) RT 2
(c) RT (d) RT 1
5. For the reaction equilibrium, N 2 O 4 (g ) 2NO ( g ) the concentration of N 2O 4 and NO 2
at equilibrium are 4.8 102 and 1.2 102 molL1 respectively. The value of K C for the reaction
is :
(a) 3.3 10 2 mol L1 (b) 3 101 mol L1 (c) 3 10 3 mol L1 (d) 3 103 mol L1
6. Which one of the following statements is not true ?
(a) The conjugate base of H 2 PO 4 is HPO 24
(b) P H P OH 14 for all aqueous solutions at 298 k
(c) P H of 1 10 8 M HCl is 8
(d) NH 3 is a lewis base

153
7. What is the equilibrium expression for the reaction.
P4 (s) 5O 2 (g) P4 O10(s)

P4O10 1 P4O10
(a) Kc P O 5 (b) Kc O 5 (c) Kc 5P O (d) Kc O 2 5
4 2 2 4 2

8. For the reaction


CO (g) Cl2(g) COCl2(g) then

Kp
is equal to
Kc
1
(a) (b) RT (c) RT (d) 1.0
RT
9. The equilibrium constant for the reaction
N 2(g) O 2(g) 2 NO (g)
at temperature. 300 k is 4 10 4 The value of Kc for the reaction
NO (g) 12 N 2 (g) 1 2 O 2(g) at the
same temperature is:
(a) 2.5 10 2 (b) 4 10 4 (c) 0.02 (d) 50

10. Hydrogen ion concentration in mol L in a solution of P H 5.4 will be,


(a) 3.98 10 6 (b) 3.68 10 6 (c) 3.88 10 6 (d) 3.98 108
11. The equilibrium constants Kp1 and Kp 2 for the reactions, X 2Y and Z P Q respectively..
are in the ratio of 1:9. If the degree of dissociation of x and z are equal then the ratio of total pressure
at these equlibria is.
(a) 1:36 (b) 1:1 (c) 1:3 (d) 1:9
12. In a reaction, CO(g) 2H2(g) CH3OH(g) HO 92 KJ mol1 concentration of hydrogen,
carbon monoxide and methanol become constant at equilibrium. what will happen if an inert gas is
added to the system.?
(a) reaction becomes fast (b) reaction becomes slow
(c) equilibrium state disturbs (d) equilibrium state remains undisturbed
13. At 473 k, equilibrium constant KC for the reaction
PCl5(s) PCl3(g) Cl2(g) is 8.3 103
what is the value of KC for the reverse reaction at the same temperature ?
(a) 8.3 10 3 (b) 120.48 (c) 16.6 10 3 (d) 4.15 103
14. At 473 k, equilibrium constant KC for a reaction,
PCl5 s PCl3(g) Cl 2(g) is 8.3 10 3 (H 124 KJmol-1 )
What would be the effect on KC if
(i) more PCls is added

154
(ii) the pressure is increased
(iii) the temp is increased
(a) KC remains unchanged, unchanged, increase.
(b) KC increases, increases, decreases.
(c) KC remains unchanged, increase, unchanged.
(d) KC decreases, increases, unchanged.
15. The equilibrium constant for the reaction,
N2(g) O2(g) 2NO(g) is 4 104 at 2000 k
temperature. what is the value of kc for the reaction
3 N 3 O 3 NO
2 2(g) 2 2(g) (g)

(a) 4 10 4 (b) 8 106 (c) 8 104 (d) 16 10 4


16. For the reversible reaction
N 2(g) 3H 2(g) 2NH 3(g) at 500 c the value of
KP is 1.44 10 5 . what would be the value of KC for the same reaction
(a) 1.44 10 5 / 0.082 500 2 (b) 1.44 10 5 / 8.314 7732
(c) 1.44 10 5 / 0.082 7732 (d) 1.44 10 5 / 0.082 7732
17. The following equilibria are given
N 2 3H 2 2NH3 K1
N 2 O 2 2NO K 2

H 2 1 2 O 2 H 2O K 3

2 NH3 5 2 O 2 2 NO 3H 2O The equilibrium constant of the reaction

in terms of k1 , k 2 and k 3 is
(a) k 1 k 2 / k 3 (b) k1 k 32 / k 2 (c) k 2 k 33 / k1 (d) k 1 k 2 k 3
18. The P Ka of a weak acid HA is 4.80 The pKb of a weak base BOH is 4.78 The P H of an aqueous
solution of the corrosponding salt BA will be
(a) 9.58 (b) 4.79 (c) 7.01 (d) 9.22
19. The equilibrium constant for the reaction
SO3(g) SO2(g) 1 2 O 2(g) Kc 4.9 102
The value for the KC of the reaction
2SO2(g) O2(g) 2SO3(g) will be
(a) 416 (b) 2.40 10 3 (c) 9.8 10 2 (d) 4.9 10 2
20. P H of 0.1 M solution of weak acid is 3. The value of ionisation constant Ka of acid is
(a) 3 101 (b) 1 10 3 (c) 1 10 5 (d) 1 1 0 7

155
21. A vessel contains CO2 at 1000k temperature with a pressure 0.5 atm.some of CO2 is converted in
to CO on addition of graphite. If total pressure at equilibrium is 0.8 atm the value of KP is:
(a) 3 atm (b) 0.3 atm (c) 0.18 atm (d) 1.8 atm
22. Three reactions involving H 2 PO 4 are given below..
(i) H3 PO 4 H 2 O H 3 O H 2 PO 4
(ii) H 2 PO -4 H 2 O HPO 42 H 3 O
(iii) H 2 PO 4- OH - H 3 PO 4 O 2
In which of the above does H 2 PO-4 act as an acid.
(a) (i) only (b) (ii) only (c) (i) and (ii) (d) (iii) only
23. For the reaction
2 NO2(g) 2 NO(g) O2(g)
Kc 1.810 6 at 184c R 0.0831 KJ/mol.K
When KP and KC are compared at 184 C it is found that :
(a) whether Kp is greater than less than or equal to Kc depends upon the total gas pressure
(b) Kp=Kc
(c) Kp<Kc
(d) Kp>Kc
24. Water is a
(a) Protophobic solvent (b) Protophilic solvent
(c) Amphiprotic solvent (d) Aprotic acid
25. Ammonium ion is
(a) Conjugate acid (b) Conjugate base
(c) Neither an acidnor a base (d) both an acid and a base.
26. Species acting both as bronsted acid and a base is
(a) HSO 4 (b) Na 2 CO 3 (c) NH 3 (d) OH -
27. A solution of an acid has P H 4.70 find out the concentration of OH . pK w 14
(a) 5 10 10 M (b) 4 10 10 M (c) 2 10 5 M (d) 9 10 4 M
28. The conjugate base of H 2 PO 4 is
(a) PO34 (b) HPO 4 (c) H 3PO 4 (d) HPO 24
29. What is the conjugate base of OH ?
(a) O 2 (b) H 2 O (c) O (d) O 2
30. An example for lewis acid is
(a) Ammonia (b) Aluminium chloride
(c) Pyridine (d) Amines.
31. Which of the following molecule act as a lewis acid ?
(a) CH 3 2 O (b) CH 3 3 P (c) CH 3 3 N (d) CH 3 3 B

156
32. In a given system, water and ice are in equilibrium. If pressure is applied to the above system, then,
(a) More ice is formed (b) Amount of ice and water will remain same
(c) More ice is melted (d) Either (a) or (c)
33. In 2 HI H 2 I 2 H 0 the forward reaction is affected by change in
(a) Catalyst (b) Pressure (c) Volume (d) Temp
34. In which case KP is less than KC (?)
(a) PCl5(g) PCl3(g) Cl2(g) (b) H2(g) Cl2(g) 2HCl(g)
(c) 2 SO2 O2 2SO3(g) (d) all of these.
35. If K1 and K 2 are respective equlibrium constants for two reaction,
XeF6(g) H2O(g) XeOF4(g) 2HF(g)
XeF4(g) XeF6 XeOF4(g) XeO3 F2(g)
the equilibrium constant for the reaction
XeF4(g) 2HF(g) XeO3F2(g) H2O(g) will be
(a) K1 K 2 (b) K 2 K1 (c) K 2 / K1 (d) K1 / K 2
36. For a homologous reaction,
4 NH 3 5O 2 4 NO 6H 2 O
the dimensions of equilibrium constant KC is
(a) conc.10 (b) conc.1 (c) conc.1 (d) It is dimensionless
37. For a reversible reaction, if the concetration of the reactants are doubled, the equilibrium constant
will be
(a) The same (b) Halved (c) Doubled (d) One fourth
38. The molar solubility of a sparingly soluble salt AB4 is 's' mol lit The corrosponding solubility product
KSP is given in terms of KSP by the relation
1
4
K 1 1 1
(a) S SP 128 (b) S 128 K SP 4 (c) S 256 K SP 5
(d) S K SP / 256 5


39. The solubility product of a salt having general formula MX2 in water is 4 10 12 The concentration
of M 2 ions in an aqueous solution of the salt is:
(a) 4.0 10 10 M (b) 1.6 104 M (c) 1.0 104 M (d) 2.0 10 6 M
Ka
40. P H of 0.005 M calcium acetate ( P of CH 3COOH 4.74 ) is
(a) 7.04 (b) 9.37 (c) 9.26 (d) 8.37
41. One of the following equilibria is not affected by change in volume of the flask.
(a) PCls(g) PCl3(g) Cl2(g) (b) N2(g) 3H2(g) 2 NH3(g)

(c) N2(g) O2(g) 2 NO(g) (d) SO2 Cl2(g) SO2(g) Cl2(g)

157
42. Equal volmes of two solution of P H 3 and 4 are mixed The P H of the resulting solution will be
(a) 7 (b) 3.5 (c) 2.96 (d) 3.26
43. P H of 10 8 M solution of Hcl in water is
(a) 8 (b) -8 (c) between 7 and 8 (d) between 6 and 7

44. A certain buffer solution contains equal concentration of X and HX . Ka for HX is 10 8 . The P H
of buffer is
(a) 3 (b) 8 (c) 11 (d) 14
45. The solubility product of AgCl is 4 10 10 at 298 k The solubility of AgCl in 0.04 M CaCl2 will be
(a) 2 10 -5 M (b) 1 10-4 M (c) 5 10-9 M (d) 2.2 10-4 M
46. Calculate concentration of sodium acetate which should be added to 0.1 M solution of

CH 3 COOH P K a 4.5 to give a solution of P H 5.5
(a) 1.0 M (b) 0.1 M (c) 0.2 M (d) 10.0 M
47. Which of the following is a base according to lowry-bronsted concept ?
(a) I (b) H 3O (c) HCl (d) NH 4
48. According to lowry-bronsted concept which one of the following is considered as an acid ?
(a) H 3O (b) BF3 (c) OH (d) Cl
49. The conjugate acid of NH 2 is
(a) NH 4 (b) NH 3 (c) N 2 H 4 (d) NH 2OH
50. conjugate base of hydrazoic acid is
(a) HN 3 (b) N 2 (c) N 3 (d) N 3
51. In which of the following reaction NH 3 acts as acid ?
(a) NH 3 HCl NH 4 Cl (b) NH 3 H NH 4
1
(c) NH 3 Na NaNH 2 H 2 (d) NH 3 cannot act as acid.
2
52. Consider the following reactions.
(i) CO 32 H2O HCO3 OH
(ii) CO 2 H 2O H 2CO3
(iii) NH 3 H 2 O NH 4OH.
(iv) HCl H 2 O Cl H 3 O
Which of the pairs of reaction proves that water is amphoteric in character ?
(a) (i) and (ii) (b) (ii) and (iii) (c) (iii) and (iv) (d) (i) and (iii)
53. One of the following is a bronsted acid but not a bronsted base :
(a) H 2S (b) H 2 O (c) HCO 3 (d) NH 3

158
54. The conjugate base in the following reaction
H 2SO 4 H 2 O H 3 O HSO 4 are
(a) H 2 O, H 3O (b) HSO 4 , H 2 O (c) H 3 O , H 2SO 4 (d) H 2SO 4 , HSO 4
55. With increase in temperature, ionic product of water
(a) Decreases (b) Increases
(c) Remains same (d) May increase or decrease.
56. EDTA is a/an
(a) Arrhenius acid (b) Bronsted base.
(c) Lewis base (d) All of above
57. The units of lonic product of water (kw) are :
(a) mol 1 L1 (b) mol 2 L2 (c) mol 2 L1 (d) mol 2 L2
58. Whichof the following weakest ?
(a) C6 H 5 NH 2 : K b 3.8 1010 (b) NH 4 OH : K b 1.6 10 5
(c) C 2 H 5 NH 2 : K b 5.6 10 4 (d) C9 H 7 N : K b 6.3 1010
59. On adding ammonia to water,
(a) Ionic product will increase (b) Ionic product will decrease

(c) [H 3 O ] will increase (d) [H 3 O ] will decrease
60.
According to lowry and bronsted system, the chloride ion Cl in aqueous solution is a
(a) Weak base (b) Strong base
(c) Weak acid (d) Strong acid
61. The P of a buffer containing equal molar concentration of a weak base and its
H

chlorides is K b for weak base 2 10 5 log 2 0.3010


(a) 5 (b) 9 (c) 4.7 (d) 9.3
62. Solution of 0.1 N NH 4OH and 0.1N NH 4Cl has P H 9.25.Then PK b is
(a) 9.25 (b) 4.75 (c) 3.75 (d) 8.25
63. The solubility of product barium sulphate is 1.5 10 at 18 C Its solubility in water at 18 C is
9

(a) 1.5 10 9 mol L1 (b) 1.5 10 5 mol L1


(c) 3.9 10 9 mol L1 (d) 3.9 10 5 mol L1
64. The least soluble compound (salt) of the following is
(a) ZnS Ksp .2 10 23 (b) OH >Cl CH3COO

(c) CsCl Ksp 1 10 12 (d) PbCl 2 Ksp 1.7 10 5


65. What is the value of P H of 0.01 M glycine solution ? For glycine Ka 1 4.5 10 3 and
Ka 2 1.7 10 10 at 298k.
(a) 3.0 (b) 10.0 (c) 6.1 (d) 7.06

159
66. Solid Ba NO 3 2 is gradually dissolved in 1.0 10 4 M Na 2 CO 3 solution. At what concentration of
will precipitate ?
Ksp of BaCO3 5.110 9
(a) 4.110 5 M (b) 5.1 105 M (c) 8.1 10 5 M (d) 8.1107 M

67. What is the OH in the final solution prepared by mixing of 20.0 ml of 0.05 M HCl with 30.0
ml of 0.1 M Ba OH 2 (?)
(a) 0.4 M (b) 0.05 M (c) 0.12 M (d) 0.1 M
68. The ionisation constant of NH 4OH is 1.77 105 at 298 k. Hydrolysis constant of it is
(a) 6.5 10 12 (b) 5.65 10 13 (c) 5.65 10 12 (d) 5.65 10 10
69. The dissociation consant of a substituted benzoic acid at 25 C is 1.0 10 4 The P H of 0.01 M
solution of its sodium salt is
(a) 0 (b) 1 (c) 7 (d) 8
70. Number of H ions present in 500 ml of lemon juice of P 3 is
H

(a) 1.506 10 22 (b) 3.012 10 20 (c) 3.102 10 22 (d) 1.506 1020


71. Equimolar solution of the following were prepared in water separately. Which one of the solutions
will record the highest P H (?)
(a) SrCl 2 (b) BaCl2 (c) MgCl2 (d) CaCl2
72. Solubility products constants (KSP) of the salt types MX, MX 2 and M 3X at temp T. are
4 10 8 ,3.2 1014 and 2.7 1015 respectively. Solubility of the salts at temp. T are in the order,,
(a) MX M 3 X MX 2 (b) MX 2 M 3 X MX
(c) M 3 X MX 2 MX (d) MX MX 2 M 3 X
73. When H ion concentration of a solution increases
(a) PH increses (b) PH decreases (c) no change in PH (d) POH decreases
74. At 25 C temp. the value of pk b for NH 3 in aqueous solution is 4.7. what is the value of P H of 0.1
M aqueous solution of NH 4Cl with 0.1 M NH 3 (?)
`(a) 8.3 (b) 9 (c) 9.5 (d) 10
75. The aqueous solution of HCOO Na, C 6 H5 NH 3 Cl, and KCN are respectively
(a) Acidic, acidic, basic (b) Acidic, basic, neutral
(c) Basic, acidic, basic (d) Basic, neutral, basic
76. KSP of AgIO3 is 1 10-8 at a given temperature what is the mass of AgIO3 in 100 ml of its
saturated solution ?
(a) 1.0 10 4 gm (b) 28.3 102 gm (c) 2.83 103 gm (d) 1.0 10 7 gm

160
77. PH of a solution containing 50 mg of sodium hydroxide in 10 dm3 of the solution is
(a) 9 (b) 3.9031 (c) 10.0969 (d) 10
78. Which one of the following has the lowest P H value ?
(a) 0.1 M HCl (b) 0.1 M KOH (c) 0.01 M HCl (d) 0.01 M KOH
79. Equal volumes of three basic solutions of 11,12 and13 are mixed in a vessel. what will be the H+
ion concentration in the miture ?
(a) 1.11 10 11 M (b) 3.7 10 12 M (c) 3.7 10 11 M (d) 1.11 10 12 M
80. Columm I Column II
(Buffer Solution) PH
(A) O.1M CH3 COOH + 0.01M (p) 3.8
CH3 COONa
(B) 0.0 1M CH3COOH+0.1M CH3COONa (q) 5.8
(C) 0.1M CH3COOH+0.1M CH3COONa (v) 7.0
(D) 0.1M CH3COONH4 (s) 4.8
(Pka Of CH3COOH=Pkb of NH4OH=4.8)
(a) A-r, B-q, C-s, D-p
(b)A-q, B-p, C-s, D-r
(c)A-q, B-p, C-r, D-s
(d)A-r, B-q, C-s, D-p
81. PH of a soda water bottle is
(a) > 7 (b) = 7
(c) < 7 (d) unpredictable.
H -8
82. Statement :1 P of 10 HCl solution is not equal to 8
Statement :2 HCI does not dissociate properly in very dilute solution.
(a) Statement -1 is true.
Statement -2 is true.
Statement-2 is a correct explanation for statement-1
(b) Statement- 1 is true.
Statement-2 is true.
Statement-2 is not a correct explanation of statement-1
(c) Statement-1 is true.
Statement-2 is false.
(d)Statement-1 is false.
Statement-2 is true.
83. For preparing a buffer solution of PH 6 by mixing sodium acetate and acetic acid , the ratio of
concentration of salt and acid should be, (Ka=10-5)
(a) 1:10 (b) 10:1
(c) 100:1 (d) 1:100
84. The KSP of CuS, Ag2S and HgS are 10 , 4 10 45 and 10-54 respectively. The solubility of
-32

these sulphides are in order of


(a) Ag 2S HgS CuS (b) Ag 2S CuS HgS
(c) HgS AgS2 CuS (d) CuS > Ag2S > HgS

161
85. The PH of neutral water is 6.8 .Then the temperature of H 2 O
(a) is 250C (b) is more than 250C
(c) is less than 250C (d) can not be predicted.
86. On adding 0.1 M solution each of Ag ,Ba 2 and Ca 2 ions. in a Na 2SO 4 solution. species first
precipitated is
K SP of BaSO4 1011 , KSP of CaSO4 106 , KSP of Ag 2 SO4 105
(a) BaSO4 (b) CaSO4 (c) Ag 2SO4 (d) all of these
87. The solubility of A 2 B3 is x mol L-1. It solubility product is
(a) 6 x 5 (b) 64 x 5 (c) 36 x 5 (d) 108 x 5
88. How much volume of 0.1 M CH3COOH should be added to 50ml of 0.2 M CH3COONa If we
want to prepare a buffer Solution of P H 4.91. given pKa for acetic acid is 4.76
(a) 80.92 ml (b) 100 ml (c) 70.92 ml (d) 60.92 ml
89. The ionzation constant of formic acid is 7.8 10 . Calculate ratio of sodium formate & formic acid
4

in a buffer of P H 4.25
(a) 9.63 (b) 3.24 (c) 6.48 (d) 3.97
90. The ionization constants of HF is 6.8 10 4 . Calculate ionization constant of corresponding conjugate
base.
(a) 1.9 1010 (b) 1.7 10 10 (c) 1.5 10 11 (d) 2.9 10 11
91. The ionization constant of formic acid is 1.8 10 4 around what P H will its mixture with sodium
formate give buffer solution of highest capacity
(a) 3.74 (b) 7.48 (c) 4.37 (d) 3.96
92. What is PH of our blood ? why does it remains constant inspite the variety of the foods and spices
we eat ?
(a) of blood is 5.4. It remains constant because.it is acidic.
(b) of blood is 7.4. It remains constant because.it is buffer.
(c) of blood is 10.8.It remains constant because.it is basic.
(d) of blood is 7.0. It remains constant because it is neutial.
93. The precipitate of CaF2 (K SP 1.7 10 10 ) is obtained when equal volumes of the following are
mixed.
(a) 10 4 M Ca 2 10 4 M F (b) 10 2 M Ca 2 10 3 M F
(c) 10 5 M Ca 2 10 3 M F (d) 10 3 M Ca 2 10 5 M F
94. A certain buffer solution contains equal concentrations of X and HX The Ka for HX is10 8 The
P H of buffer is
(a) 8 (b) 3 (c) 7 (d) 4

162
95.
The correct order of increasing H 3O in the following aqueous solution is.
(a) 0.1 M H2S 0.1 M H2SO4 0.1 M NaCl 0.1 M NaNO2
(b) 0.1 M H 2SO4 0.1 M NaCl 0.1 M NaNO 2 0.1 M H 2S
(c) 0.1 M NaNO2 0.1 M NaCl 0.1 M H 2S 0.1 M H 2SO4
(d) 0.1 M NaNO2 0.1 M H2S 0.1 M H2SO 4 0.1 M NaCl
96. What is the % hydrolysis of NaCN in N/80 solution when dissociation constant for HCN is
1.310 9& KW 11014
(a) 2.48 (b) 8.2 (c) 5.26 (d) 9.6
97. The KSP of AgCl is 4.0 10 10 at 298 k.solubility of AgCl is 0.04 M CaCl2 will be
(a) 2 10 5 M (b) 1 10 4 M (c) 5 10 9 M (d) 2.2 10 4 M
98. How much sodium acetate should be added to 0.1 M solution of CH3COOH to give a solution of

P H 5.5 pKa of CH 3CooH 4.5


(a) 0.1 M (b) 1.0 M (c) 0.2 M (d) 10.0 M
99. The P of solution obtained by mixing 50ml 0.4 N HCl & 50ml 0.2 N NaOH is
H

(a) log 2 (b) 1.0 (c) log 0.2 (d) 2.0


100. Ionisation constant of CH3COOH is 1.7 105 and concentration of H ions is 3.4 10 4 The initial
concentration of CH 3COOH molecules is
(a) 3.4 10 4 (b) 3.4 10 3 (c) 6.8 10 3 (d) 1.7 10 3
101. In the reversible reaction A B C D , the concentration of each C and D at equilibrium was
0.8 mollitre, then the equilibrium constant KC will be.
(a) 6.4 (b) 0.64 (c) 0.16 (d) 16.0
102. 4 moles of A are mixed with 4 moles of B.At equilibrium for the reaction A B C D 2 moles
of C and D are formed . The equilibrium constant for reaction will be
1 1
(a) (b) 4 (c) (d) 1
4 2
103. A reversible chemical reaction having two reactants in equilibrium. If the concentration of the
reactants are doubled, then equilibrium constantwill
(a) become double (b) become half
(c) become 4 times (d) remains same
104. Two moles of PCls are heated in a closed vessel of 2L capacity. At equilibrium, 40 % of PCl5 is
dissociated in to PCl3 & Cl2. The value of equilibrium constant is,
(a) 0.266 (b) 0.53 (c) 2.66 (d) 5.3
105. The dissociation constant for acetic acid and HCN at 25 C are 1.5 105 and4.5 10 10 respectively..
the equilibrium constant for reaction CN CH 3COOH HCN CH 3COO
(a) 3 105 (b) 3 105 (c) 3 104 (d) 3 10 4

163
106. A B C D if finally the concentration of A and B are both equal but at equilibrium concentration
of will be twice of that of A. Then what will be equilibrium constant of the reaction.

(a) 4 9 (b) 9 4 (c) 19 (d) 4


107. If in the reacton N2O4 2NO2 , is that part of N 2O 4 will dissociate, then the number of moles
at equilibrium will be,
(a) 3 (b) 1 (c) 1 (d) 1
108. 4.5 moles each of hydrogen and iodine heated in a sealed ten litre vessel. At equilibrium, 3 moles of
HI were found. The equilibrium constant for H 2(g) I 2(g) 2HI(g) is
(a) 1 (b) 5 (c) 10 (d) 0.33
109. The rate constant for forward and backward reaction of hydrolysis of ester are 1.1 10 2 &1.5 10 3
per minute respectively. Equilibrium constant for reaction is,
CH 3 COOC 2 H 5 H 2 O CH 3 COOH C 2 H 5OH
(a) 4.33 (b) 5.33 (c) 6.33 (d) 7.33
110. Two moles of NH 3 ,when put in to a previously evacuated vessel(1 Litre), partially dissociate in to
. N2 and H2 If at equlibrium one mole of NH 3 is present, the equilibrium constant is,

(a) 3 4 mol Litre (b) 27 64 mole Litre


2 2 2 2

(c) 27 32 mole Litre (d) 27 16 mol Litre


2 2 2 2

111. What is equilibrium expression for the reaction


P4 5O 2 P4O10
(b) KC P4O10 / 5 A O2
5
(a) K C O 2
5 5
(c) K C P4O10 / P4 O 2 (d) K C 1 / O 2
112. Partial pressure of O 2 in
2 Ag 2 O (g) 4 Ag(s) O 2(g)is

(a) kp (b) kp (c) 3 kp (d) 2 kp


113. For reaction H 2(g) CO2(g) CO(g) H 2 O(g) ,If the initial concentration of H 2 CO 2 and x
moles / litre of hydrogen is consumed at equilibrium, the correct expression of KP is
x2 1 x 2 x2 x2
(a) (b) (c) (d)
1 x 2 1 x 2 2 x 2
1 x2
114. Consider the imaginary equlibrium
4A 5B 4x 6y
The equilibrium constant K C has unit
(a) mole 2 Litre 2 (b) Litre mole 1 (c) mole litre 1 (d) Litre 2 mole 2

164
1 Kp
115. For reaction CO(g) O2(g) CO2(g) is equivalent to
2 Kc

(a) 1 (b) RT (c) 1 RT (d) RT 1 2


116. For the reaction
CH4(g) 2O2(g) CO2(g) 2H2O(L) H 170.8KJ mol1
Which of following statement is not true
(a) Adding of CH 4 ( g ) on O 2 ( g ) at equilibrium will cause a shift to the right
(b) Thee reaction is exothermic
(c) At equilibrium, concentrations of CO2(g) and H2O(L) are not equal.

CO2
(d) Equilibrium constant for the reaction is given by KP
CH4 O2
117. The reaction Quetient (Q) for the reaction
N 2(g) 3H 2(g) 2NH 3
2

Q
NH3
is given by 3 The reaction will proceed from right to left is:
N2 H2
(a) Q 0 (b) Q K C (c) Q < KC (d) Q KC
118. If concentration of reactants is increased by 'x' then k become
(a) ln( k / x ) (b) k/x (c) k+x (d) k
119. Which of following is not favourable for formating SO3 formation
2SO2(g) O2(g) 2SO3(g) H 45.0 Kcal.
(a) High pressure (b) Decreasing SO3 concentration
(c) High temperature (d) Increasing reactants concentration
120. The most important buffer in blood consists of
(a) HCl andCl (b) H 2 CO 3 and Cl (c) H2CO3 and HCO3 (d) HCl and HCO3
121. Given that dissociation constant for H 2 O is K W 1 10 14 moles 2 / Litre2 What is P H of 0.001 M
NaOH
(a) 10-11 (b) 10 3 (c) 11 (d) 3
122. Select the pKa value of strongest acid from following
(a) 1.0 (b) 3.0 (c) 2.0 (d) 4.5
123. The degre of hydrolysis equilibrium
A H 2 O HA OH

at salt concentration of 0.001 M is Ka 1 10 5
(a) 1 10 3 (b) 1 10 4 (c) 6.75 10 4 (d) 5.38 10 2

165
124. If pK b for fluoride ion at 250C is 10.83, the ionisation constant of hydrofluoric acid in water at this
temperature is
(a) 1.74 10 3 (b) 3.52 10 3 (c) 6.75 10 4 (d) 5.38 10 2
H Salt
125. Henderson's equation is P pka log
Acid If acid gets half neutralised the value of P H will
be [pka=4.30]
(a) 4.3 (b) 2.15 (c) 8.60 (d) 7
126. The P H of a 0.01 M solution of acetic acid having degree of dissociation 1.25% is
(a) 5.623 (b) 2.903 (c) 3.723 (d) 4.503
127. By adding 20 ml 0.1 N HCl to 20 ml 0.1 N KOH the P H of obtained solution will be :
(a) 0 (b) 7 (c) 2 (d) 9
128. If the Kb value in the hydrolysis reaction
B H 2 O BOH H is 1.0 10 6 then hydrolysis constant of salt would be
(a) 1.0 10 6 (b) 1.0 10 7 (c) 1.0 10 8 (d) 1.0 10 9
129. For a sparingly soluble salt ApBq relation ship of its solubility product(KSP) with its solubility(s) is.
p q
(a) K SP Sp q P p q q (b) K SP Sp q P q q p (c) K SP S pq P p q q (d) KSP Spq pq
130. How many grams CaC2O 4 (mw 128) on dissolving in disttile water will give saturated

solution K SP CaC 2 O 4 2.5 10 9 mol 2 l 2


(a) 0.0064gm (b) 0.1280gm (c) 0.0128gm (d) 1.2800gm
131. If the concentration of CrO ion in a saturated solution of silver chromate is 2 104 solubility
4
2

product of silve chromate will be


(a) 4 10 8 (b) 8 10 12 (c) 12 10 12 (d) 32 10 12
132. According to bronsted- lowry concept. correct order of relative strength of bases follows the order
_
(a) Cl CH 3COO OH (b) Cl OH CH 3COO
_
(c) OH CH 3COO Cl (d) OH >Cl CH 3COO
133. HSO 4 OH SO 24 H 2O Which is correct about conjugate acid base pair ?
(a) HSO 4 is Conjugate acid of base SO 42
(b) HSO 4 is Conjugate base of acid SO 42
(c) SO 24 is Conjugate base of acid HSO 4
(d) None of these
134. Which of following base is weakest
(a) NH 4 OH:K b 1.6 10 6 (b) C 6 H 5 NH 2 : K b 3.8 10 10
(c) C 2 H 5 NH 2 : K b 5.6 10 4 (d) C 2 H 7 N : K b 6.3 10 10

166
135. HClO is a weak acid. concentration of H + ion in 0.1 M solution of HClO K a 5 10 -8 will be
(a) 7.07 10 5 M (b) 5 10 9 M (c) 5 10 7 M (d) 7 10 4 M


136. Upto what P H must a solution containing a precipitate of Cr OH 3 be adjusted so that all
precipitate dissolves
(a) upto 4.4 (b) upto 4.1 (c) upto 4.2 (d) upto 4.0
137. NH 4Cl is acidic because
(a) On hydrolysis NH 4Cl give weak base NH 4OH and strong acid HCl
(b) Nitrogen donates a pair of e -
(c) It is a salt of weak acid and strong base
(d) On hyrdolysis NH 4Cl gives strong base and weak acid
138. 100 ml of 0.04 N HCl aqueous solution is mixed with 100 ml of 0.02 N NaOH solution. The P H of
resulting solution is
(a) 1.0 (b) 1.7 (c) 2.0 (d) 2.3

ANSWER KEY
1 B 26 A 51 C 76 C 101 D 126 B
2 C 27 A 52 C 77 C 102 D 127 B
3 A 28 D 53 A 78 A 103 D 128 C
4 A 29 D 54 B 79 B 104 A 129 A
5 C 30 B 55 B 80 B 105 D 130 A
6 C 31 D 56 D 81 C 106 D 131 D
7 B 32 C 57 D 82 C 107 B 132 C
8 A 33 D 58 A 83 B 108 A 133 C
9 D 34 C 59 D 84 B 109 D 134 B
10 A 35 C 60 A 85 B 110 B 135 A
11 A 36 B 61 D 86 A 111 D 136 D
12 D 37 A 62 B 87 D 112 A 137 A
13 B 38 D 63 D 88 C 113 A 138 C
14 A 39 C 64 B 89 B 114 C
15 B 40 D 65 D 90 C 115 C
16 D 41 C 66 B 91 A 116 D
17 C 42 D 67 D 92 B 117 D
18 C 43 D 68 D 93 B 118 D
19 A 44 B 69 D 94 A 119 C
20 C 45 C 70 B 95 C 120 C
21 B 46 A 71 B 96 A 121 C
22 B 47 A 72 A 97 C 122 A
23 D 48 A 73 B 98 B 123 A
24 C 49 B 74 A 99 B 124 C
25 A 50 C 75 C 100 C 125 A

167
Hint : Chemical equilibrium .
Hint :
1. Addition of (1) and (2) gives (3) then

K1 K2
C PQ
A B BC

PQ K
AB2 3
K 3 K1 K 2
when the addition of equilibria leads to another equilibria then the product of their equilibria constants
gives the equilibria constant of the resultant equilibrium.
3. The value of equilibrium constant does not change in presence of catalyst.
4. Formula :
n (g)
Kp Kc(RT)
n (g) n p - n r

6. H 2 O is also present in 1 10-8 M HCl solution. So due to self ionisaion of H 2 O ,


H 2 O H OH
1 10 7 M at 298 K
so conc. of H ion in solution increase due to self ionisation of H 2 O Hence P H of HCl solution
decreases and its value is less than 7.
9. N 2(g) O 2(g) 2 NO (g)

P 2NO
Kp
PN 2 PO2
For reaction,
1
N O (g ) 1 O 2 (g ) N 2
2 2
1 1
2 2
PO 2 PN 2 1 1
K'P = Kp 1 2 50
PNO
4 10 4
1
2

10.

P H 5.4
P H log H
5.4 log H
6.6 3.981 10 6 H

168
11 For a reaction X 2Y
initialmol 1 0
at equilibrium mol 1-x 2x
Total moles = 1-x+2x = 1+x
2
2x
2 P1
PY 1 x 4x 2 p1
Kp1
Px 1 x
p1
1 x 1 x ------(1)
1 x
For a reaction Z P Q
Initial 1 0 0
at eqm. 1-x x x
Total moles = 1+x
x P2 x p 2 x2
Kp 2 P2
2 1 x
1 x P2
1 x 1 x -------(2)
1 x
Kp1 1

Kp 2 9
4 P1 1

P2 9
P1 1
P1 : P2 1 : 36
P2 36
18. For salt of weak acid and weak base
1
P H log Ka log kw log K b
2
1 1 1
pKa pkw pk b
2 2 2
1 1 1
4.8 14 4.78
2 2 2
7.01
19. Equilibrium constant for the reaction
SO 2(g) 1 2 O2(g) SO3(g)

1
KC
4.9 10 2
and for reaction 2SO2(g) O2(g) 2SO3(g)
2
1
KC 2
416.49
4.9 10

169
20.
P H 3 3 O 1 103

Ka
H O 110

3
2 3 2
110 5
C 0.1
21. CO2(g) C(g) 2CO(g)
initial pressure 0.5 atm 0
final pressure(0.5-x) 2x
Total pressure = 0.5 -x+2x=0.5 +x = 0.8 atm
x=0.3 atm.
2

KP
p CO (0.6) 2 1.8 atm.
PCO 2 0.2
22. According to lowry-bronsted acid base theory in (ii) reaction H 2 PO 4 donates H ion to H 2 O so
it acts as an acid.
23. Kp Kc(RT) n(g) n( g ) 1
Kc 0.0831 457 It means Kp > Kc
24. Water is an amphiprotic solvent as it can accept protons as well as give protons.
25.
Ammonium ion NH 4 is a conjugate acid of NH 3

NH3 H2O NH4 OH


Bronsted base Conjugate acid
26. HSO 4 can act as a bronsted acid as well as bronsted base.
HSO 4 H 2 O SO 42 H 3 O
Acid
HSO 4 H 2 O H 2SO 4 OH
Base
27. P H 4.7 H 3 O 1.995 10 5 pK w 14 K W 1 10 14

log H3O NowKW H3O OH

110 14

OH 1.995 10 5
5 10 10 M

28. conjugate base of H 2 PO 4 is HPO 24


H 2 PO 4 H 2 O HPO 42 H 3O
Acid
29. conjugate base of OH

OH H 2 O O 2 H 3 O

170
30. Lewis acid always accepts a pair of e here AlCl 3 accepts a pair of e.
31. CH 3 3 B accept a pair of e so lewis acid
32. Pressure on equilibrium system increses ,so volume decreses. volume of ice is more than liquid
H 2 O so more ice is malted
33. For endothermic H O reaction, change in temperature affects the equilibrium system and forward
reaction takes place . by increasing temp .

K1
XeOF4 HF2 K XeOF4 XeO3F2
34.
XeF6 H 2 O 2
XeF4 XeF6
K
XeO3 F2 H 2O
and XeF4 HF2 is obtained by

K2
K1

K K2
K1

Kc
NO H 2 O
4 6
(conc.)46(45)
36.
NH3 4 O 2 5
conc1
37. According to Le -chatelliers principle, if conc. of reactant become doubled, then forward reaction
takes place and concentration of product also increases. so equilibrium constant also remains same.
38. AB4(s) A 4(aq) 4 B (aq)
_ 4
K SP A 4 B

S(45)4 256S5
1
K 5
S SP
256
2
39. KSP M2 X 4S3

1 1
3 4 1012 3
M S KSP
2
1104
4 4

40. CH3COO 2 Ca Ca 2 2CH3COO
0.005 2 0.005 0.01M.

PH 1 log KW logKa log C


2

171
1
log K W 1 log K a 1 log C
2 2 2
1
pkw 1 pka 1 log 1 10 -2
2 2 2

1 2 14 1 2 4.74 1 2 (2)
= 7 2.37 1 8.37
41. change in volume affects number of moks per unit voulme. In a reaction
N2(g) O2(g) 2 NO(g)
no. of moles of reactants and product are equal so volume change does not affects the equilibrium.
42. P H 3 means H 103 M

P H 4 means H 104 M
-3
10 4 1
After mixing equal volume total H 110

2
0.110-4 110 4

2
1.110 3
= 5.5 10 4
2
H 5.5 104 M

P H log H log(5.5 10 4 ) 4 0.7404 3.26


43. P H of HCl should be less than 7. due to self ionisation of H 2 O
from acid H 10 8 M from H 2 O H 10 7 M

Total H 10 8 10 -7 10 8 (1 10) 11 10 8 M

PH log H log(11108 ) (1.0414 8)


= 6.96
H
P 6.96
44. P of buffer solution
H

pK a log
salt
Acid
log K a log1
log108 0 8

172
45. if x is the solubility of AgCl in 0.04 M cacl2 ,

then Ag x mol L1

Cl 2 0.04 x 0.08 x 0.08 M


KSP of AgCl Ag Cl

4 1 0 10 A g 5 10 9 M
0.08

P H pka log
CH3COONa
46.
CH3COOH
5.5 4.5 log
CH3COONa
0.1
5.5 4.5 logCH 3CooNa 1
logCH 3CooNa 0
CH 3CooNa 1 M
47. I can accept protons and hence is a base.
50. N3H N3 H Hydrazoic acid N3 H
53. H 2S can donate proton but can't accept proton.
55. With increase in temperature, ionic product increases. because self ionisation of is endothermic
process
56. EDTA is Arrhenius acid as it can give H ions in aqueous solution, bronsted base. as it can accept
protons and lewis base because N and O in it can donate lone paris of electrons.
57. KW H OH

mol L1 mol L1 mol 2 L2


58. Smaller the Kb value, weaker is the base.
59. On adding NH 3 to water, OH - will increse,
Kw H3O OH is constant.

Therefore H3O will decrease

POH P K b log
Salt
61. [Salt] = [Base]
Base
= log(2 10 5 ) log 1
= 0.3010 5.000 0 4.6990
P H 14 p OH 14 4.6990 9.3010

173
P OH pK b log
Salt P OH 14 P H 4.75
62.
Base
4.75 p kb log 1
pk b 4.75 log 1 0
63. BaSO4 (s) Ba (aq)
2
So24(aq)
x mol/lit x mol/lit
K SP Ba 2 So 24

K SP x 2
1 1
x (ksp) 2 (1.5 10 9 ) 2 3.9 10 5 molL1
64 Hint : As value of KSPis less , solubility is also less.
65. glycine (NH2CH2COOH) is more acidic than basic.
overall ionisation constant
K K a1 K a 2 4.5 10 3 1.7 10 10
7.65 10 13
H

K C 7.65 10 13 0.01
0.87 10 7 M
P H log(8.7 10 8 ) 7.0605
66. KSP of BaCo3 Ba 2 CO32

5.110 9

Ba 2 110 4
5.110 5 M

67. Ba(OH)2 2HCl BaCl2 2H2 O


2 mol HCl neutralize1 mole Ba(OH)2
1 mol HCl neutralize 0.5 mole Ba(OH)2
Ba(OH)2 Ba 2 2OH
1 2
no.of moles of Ba(OH)2 3. 1 2
Ba(OH)2 left 3 0.5 2.5
2.5
Ba(OH) 2 0.05 M
50

or OH 2 0.05 M 0.1 M

174
K W 1.0 1014
68. Kh 5
5.65 1010
K b 1.7 10
1 K
69. PH
2

P W PKa log C
1 2 14 1 2 4 1 2 log10 2
=7+2-1=8
70.
P H 3 means H 10 3 M
1000 ml juice contains 10 3 mole H ions
no.of H ions 10 3 6.022 10 23 in 1000 ml
10 3 6.022 10 23 500
500 ml juice contains H ions
1000
= 3.011 10 20
71. All alkaline earth metal chlorides MCl 2 on hydrolysis will produce acidic solution
MCl 2 H 2 O M(OH) 2 2HCl
because M(OH)2 is a weak base and HCl is a strong acid. but as we go down the group, basic
character of hydroxides increses. Hence acidic character decreses. So BaCl2 will have the highest P H .
72. MX (s) M X
1 1
K SP S 2 S (K SP ) 2
(4 10 8 ) 2
2 10 4 M.
1
3
K
MX 2(s) M 2X K SP
2 3
4S S SP 2 105 M.
4
M3X (s) 3M X 3
1 1
KSP 4 2.7 1015 4
4
KSP = 27S S 1104 M
27 27
2 10 4 1 10 4 2 10 5
MX M 3X MX 2 .

P OH pk b log
Salt
74. weak base
0. 1
4.7 log 5.7
0.01
P H 14 5.7 8.3

175
75. HCOONa is a Salt of weak acid (HCOOH) and Strong base (NaOH) So it is basic. C 6 H 5 NH 3Cl
is a Salt of weak base C 6 H 5 NH 2 and strong acid (HCl) so it is acidic. KCN is a Salt of Strong
base (KOH) and weak acid (HCN) so it is basic.
76. AgIO3(s) Ag (aq)

IO3 (aq)

K SP S2
1 1
S (K SP ) 2
(1.0 108 ) 2
1104 mol
lit

S 110-4 283 283 10 4 gm


lit
1000 ml contains 283 10 4 gm of AgIO 3
100 ml contains 28.3 10 4 gm of AgIO 3

50 10 3 gm
77. Molar concentration of NaOH -4
40 gm mol 1 10dm 3 1.25 10 M
O H 4
P lo g (1 .2 5 1 0 )
0.0969 4.0 3.9031
H
P 14 3.9031 10.0969
78. 0.1 M HCl m eans H 10 1 P H 1

0.1M KOH means OH 10 1 P OH 1 P H 13

0.01 M HCl means H 10 2 P H 2

0.01M KOH means OH 102 POH 2 PH 12

79. H ion concentrations are 10 11 , 10 12 and 1013 on mixing equal volumes, H in final solution

10 11 10 12 10 13 10 10 12 110 12 0.110 12
=
3 3
11.110 12
3.7 10 12 M.
3
H Salt 4.8 log 0.1 4.8 1 5.8
80. (A) P pka log
acid 0.01

H 0.01
(B) P 4.8 log 4.8 log 10-1 4.8 1 3.8
0.1
H 0.1
(C) P 4.8 log 4.8 0 4.8
0.1
H 1 1
(D) P 7 (pka pkb) 7 (4.8 4.8) =7.0
2 2
176
81. Soda water contains weak acid H 2 CO 3 So its P H 7

P H p K a log
Salt
83. acid

6 5 log
Salt
acid
log
Salt 1 or
Salt 10.
acid acid
1
1
84. Solubility of Cus (ksp) 2
(110 32 2
) 11016 M
1
1
ksp 3 4 10-45 3 15
Solubility of Ag 2S 110 M
4 4
1
1

Solubility of HgS ksp 110 2
4 10-54 5 27
M

85. At 25 C temp P of H O 7 H 10 M
H
2
7

P 6.8 means P 7 H is more than 10 M


H H 7

Self ionisation of H 2 O is endothermic so by increasing temp H ion increases.


86. ksp for BaSo 4 Ba 2 SO 42
10 11 0.1 SO 24

SO 24 10 10 M

ksp for CaSO 4 Ca 2 SO 24

10 6
0.1
10 5 M SO 24
2
ksp for Ag 2So 4 Ag SO 42

10 5
(0.1)2
10 3 M SO 24

As SO 24 10 10 M in BaSO 4 (least value)
it can be precipitated first
87. A2 B3(s) 2A3 3B2
2x 3x
ksp A B
3 2 2 3

(2 x) 2 (3 x) 3
4 x 2 27 x 3 108x 5

177
P H P K a log
Salt
88. Acid

4.91 4.76 log


Salt
Acid
log
Salt 0.15
Salt antilogof0 .15 1.41
Acid Acid
0.2
50
1000 1.41 V 70.92 ml
0.1
V
1000
89. pka log(1.8 10 4 ) 3.74

log
Salt P H pka 4.25 3.74 0.51
Acid

Salt antilog of0.51 3.24
Acid
14
90. K b kw 10 4 1.47 10
11
kc 6.8 10
91. Buffer Solution of highest capacity is formed at which
P H pka log(1.8 10 4 ) 3.74
93. Ionic Product of CaF2


in (i) IP Ca 2 F 10 ksp
2 12

(ii) IP 10 2 10 10 ksp
3 2 8
ppt. obtain
(iii) IP 10 5 10 10 ksp
3 2 11

(iv) IP 10 3 10 10 ksp
5 2 13

P H pka log
Salt Salt Acid
94.
Acid
= 8 + log 1 = 8
95. H 2S weak acid H 2SO 4 Strong acid
NaCl neutral NaNO 2 basic
Hence H3 O will be in the order of

NaNO2 NaCl H 2S H 2SO4

178
96. NaCN is Salt of weak acid (HCN) and Strong base (NaOH) Hence

kw 10 14
h 2.48 10 2
k aC 9 1
(1.3 10 )
80
Percen tan ge Hydrolysis (2.48 10 2 )100
= 2.48
97. If x is Solubility of AgCl in 0.04 M CaCl2 , then
A g x m ol L 1

C l (0.04 2) x 0.08M

0.08(x) 4 10 10
x 5.0 10 9 M
log CH 3COONa
98. P H pka
1 CH 3COOH

5.5 4.5 log


CH 3COONa
0.7
4.5 + log CH3COONa 1
log CH 3 COONa 0

CH 3 COONa 7 M

0.4
99. 50 ml of0.4 N HCl 50 0.02 g eq.
1000
0.2
50 ml of 0.2 N NaOH 50 0.01g eq.
1000
0.01 g eq of NaOH will Neutalilise 0.01 g eq of HCl
HCl left unneutralised = 0.01 g eq
vol of Sol. =50+50
=100ml
0.01
HCl 1000 0.1N
100
or H 0.1M

p H log(0.1) = 1.0
100. CH 3 COOH CH 3COO H
at eq (a 3.4 10 4 ) 3.4 10 4 3.4 10 4

3.4 10 3 .4 1 0 1 .7 1 0
4 4
5
(G iven) a 6.8 10 3
a 3.4 10 4

179
101.

A B C D
initial 1 1 0 0
conc.
Ateqm(10.8) 0.8 0.8
conc.(10.8)
Kc
CD 0.8 0.8 16.0
0.2 0.2 AB 0.2 0.2
102.

A BCD
initial 4 4 0 0
conc.
Ateqm(4 2) 2 2 Kc
CD 2 2 1
conc.(4 2) AB 2 2
2 2

103. KC remains same beacause KC is a characteristic constant.


104. PCl5 PCl3 Cl2
2 0 0
2 60 2 40 2 40
100 100 100
moles 1.2 0.8 0.8
mol 1.2 0.8
conc. 2 2 0.8
2
lit

Kc
PCl 3 Cl 2 0.4 0.4 0.266
PCl 5 0.6
105. Dissociation of CH3COOH
CH3COOH H CH3COO k a1 1.5 103
Dissociation of HCN: HCN H CN k a 2 4.5 10 3
for a reaction
CN CH3COOH CH3COO HCN is

Ka 1 1.5 10 3
Ka 3.33 10 4
Ka 2 4.5 10 10

180
106. A B C D
x x 0 0

kc
CD 2x 2x 4
2x 2x
AB x x
107. N 2 O 4 2 NO 2
1 0
(1- ) 2 Total moles = 1- +2 =1+
108. H 2 I 2 2 HI from equation 2x=3
Initial 4.5 4.5 0 x 3 2 1.5
conc.
at eqm. (4.5-x)(4.5-x) 2x
H 2 4.5 1.5 3

Kc
HI2 3 3 1
I 2 4.5 1.5 3 H 2 I 2 3 3
109. K f 1.1 10 2 K b 1.5 10 3

kf 1.1 10 2
kc 7.33
k b 1.5 10 3
110. 2 NH3 N 2 3H 2
initial 2 0 0
conc.
At eqm.
conc. 1 1 3
2 2
3

KC
N 2 H 2
3 1 3
2 22
27
NH 3 (1) 64

111. P4(S) 5O2(g) P4 O10(s)

KC
P4O10
5
P4(s) O2(g)
we know that concentration of a solid component is always taken as a unity
1
Kc
O 2 5

181
112. 2Ag 2O(s) 4 Ag (s) O2(g )
for this reaction Kp Po 2
113. H2(g) CO2(g) CO(g) H2 O(g)
initial
conc. 1 1 0 0
At eqm. (1-x) (1-x) x x
PC O PH 2 O x2
Kp
PH 2 PC O (1 x ) 2
2

129. ApBq (s) PA (aq)


q p
q B(aq)
Solubility is PS qS mol/lit
K SP (PS) P (qs) q
S( p q ) P P q q
135. Co 0.1 M Ka 5 10 8
H
Ka Co
1 1
(5 10 8 0.1) 2
(50 10 10 ) 2

7.07 10 5 M.

182
0.059 C 0.059
at equilibrium E cell = - log 1 T = 298K E cell = log K C
n C2 n

nFE 0Cell
log K C =
2.303RT
Faraday's laws of Electrolysis:
(i) First law: The amount of products produced at the electrodes by electroysis are directly
proportional to the quntity of the electricity passed through the electroytic cell. if w is the mass of the
product produced and Q is value of the quntity of electricity passed, than W a Q
(ii) Second law : If the different electroytic cells, containing different electrolysis are joined in series
and same quntity of electricity os passed through them, than the amount of products obtained at the
electrodes are directly proportional to thier equivalent weight.
Wa Eq, where W = Mass of product obtained and Eq = Equivalent weifht of product. the modern
presentation of Faraday's law was made as follows:
"The products, obtained at the electrodes by oxidation and reduction half reactions have the relation
with the moles of the products and stoichiometry of the reaction and the quntity of electricity." " The
products, obtained at the electrodes by oxidation and reduction half reactions have the relation with the
moles of the products and stoichiometry of the reaction and the qunantity of electricity."
The quantity of electricity passed by 1 more electrons is called one Faraday.

1 Faraday (F) = 1.602 10 -19 6.022 10 23 electron mole-1

= 96487 (@ 96500 ) Coulomb mole-1 (electron)


I t
F=
96500
Emperimental value of product
Efficiency of cell(%) = 100
Theoretical value of product
Gibbs Free Energy and cell Potential
G 0 = - nFE 0 Cell G = Welectrical = - nFE Cell

The resistance of a uniform counductor is directly proportional to its lengh (l) and inversely proportions
to its area of cross section (A)
1
R= p where, R = Resitance, A= Area of cross section, l = Lengh, p = proportionality constant.
A
Conductivity: The inverse of resistance R is called conductivity G

1 A A 1 l
G= - K where K = K = G
R pl l p A

Specific conductivity K = observed conductivity G x cell constant


K 1000
Molar conductivity M =
C
Where, K = (Kappa) specific conductivity, c = concentration if solutin in unit of molarity.

184
Conductivity of strong electrolytes :
1

Molar conductivity of a strong ionic electrolyte M = M - AC 2


0

The value of degree of dissociation for such an electrolyte at the given concentratin will be as below.
Molar conductivity of the solution of a given concentration m
Degree of dissociation ( a ) = 0
Molar conductivity of the solution at infinite diletion m
Ca 2
Ka = where, Ka = Dissociation constanto of weak electrolyte,
(1 - a )
C = concentration of solution, a = degree of dissociation of weak electrolyte
+
Kohlrausch's law " The morar conductivity of an electroyte at infinite dilution 0 m is the sum
o o
if the values of the molar conductivities of positive ion an negative ion present in them l m + and l m-
+ -
0 m = v + m + v - m (where 0 m + and 0 m - respectively are ionic molar conductivity of positive and
negative ion.)
(1) Primary cell: The cell which is dead after a long use and which cannot be regenerated i.e. which
cannot be reproduced, is called primary cell. e.g. Dry cell.
(2) Secondary cell : The cell which can be regenerated or reproduced is called secondary cell. e.g
lead storage cell and Ni Cd storage cell.
Dry cell:
Reaction at cathode: 2MnO 2 + 2NH +4 + 2e - M n O3 + 2NH3 + H 2 O
2

Reaction at anode : Zn( s ) Zn( ag ) + 2e


The potential of this cell is about 1.5 volt.

Mercury cell :
Reaction at cathode: HgO + H 2O + 2e - Hg ( e ) + 2OH -
Reaction at anode : Zn( Hg ) + 2OH - ZnO ( s ) + H 2O + 2e -
The complete equation of these reactions is as follows: Zn( Hg ) + MgO ZnO ( s ) + Hg (l )
The cell potential of this cell is about 1.35 volt and during the whole life of this cell, no ions an produced in
the complate reaction.
Lead storage cell.
The discharging rections taking place in lead storage cell are as follows :
Reaction at cathode: PbO2 ( s ) + 4 H + (aq ) + SO42 - (aq ) + 2e - PbSO4 ( s ) + 2 H 2O(l )
Reaction at anode : Pb( s ) + SO42- (aq ) PbSO4 ( s ) + 2e -
The discharging reactions taking place in lead storage cell are as follows:
Cathode: PbSO2 ( s ) + 2e - Pb( s ) + SO42- (ad )
Anode : PbSO4 ( s ) + 2 H 2O(l ) PbO2 ( s ) + 2 H + (aq ) + SO42 (aq ) + 2e-
The cell potential of this cell is about 2 volt.
NiCd storage cell: Cd ( s ) + 2 Ni (OH ) 3 ( s ) CdO ( s ) + 2 Ni (OH ) 2 ( s ) + H 2O(l )
Hydrogen Fuel cell
Cathode : O2 ( g ) + 2 H 2O(l ) + 4e - 4OH - (aq )
Anode : 2 H 2 ( g ) + 4OH - (aq ) 4 H 2O(l ) + 4e -
Cell reaction : 2 H 2 ( g ) + O2 ( g ) 2 H 2O(l ) + 571.7 kj
185
M.C.Q.
1. Reduction reaction means ________
(a) a process of adding oxygen (b) a process of removing hydrogen
(c) a process of adding electron (d) a process of removing electrons
2. Which substance is oxidizing agent ?
(a) a substance donates hydrogen or accepts oxygen
(b) a substance donates oxygen or accepts hydrogen
(c) a substance experience oxidation
(d) a substance donates electron
3. Which substance is called reducing agent ?
(a) a substance donates hydrogen or accepts oxygen
(b) a substance accepts hydrogen or donates oxygen
(c) a substance expereince reduction (d) a substance gains electron
4. Oxidation reaction means _____________
(a) a process of removing electron (b) a process of adding hydrogen
(c) a process of removel of oxygen (d) a process of adding electrons
5. Which of the following is the characterictic of reducing agent ?
(a) it experience oxidation. (b) it experience reduction
(c) it gains electrons (d) it gives oxygen
6. Which of the following is the characteristic of oxidizing agent ?
(a) it experience oxidation. (b) it experience reduction
(c) it gains oxygen (d) it donates electrons
7. Which of the following statement is true ?
(a) there is always reduction occur of oxidizing agent
(b) there is always oxidation occur of reducing agent
(c) oxidation and reduction are supplimentary processes
(d) Given three statements are wrong.
8. Which of the following statement is wrong ?
(a) there is always reduction occur of oxidizing agent
(b) there is always oxidation occur of reducing agent
(c) oxidation and reduction are supplimentary processes
(d) Given three statements are wrong.
9. Which of the following does not occur, when a rod of Zn metal is dipped in an aqueous
solution of CuSO4 ?
(a) blue colour of the oxygen fades gradually. (b) weight of Znmetal rod decreases.
(c) weight of metal strip of zinc increases.
(d) colour of the surface of Zn road become saffronred.

186
10. Which of the following observation obtained, when rod of Cu metal is dipped in an aqueous
solution of AgNO3?
(a) No change in the weight of metal rod of Cu occurs.
(b) weight of rod of copper metal decreases
(c) solution become bluish gradually
(d) colour of the surface of rod of Cu metal does not change
11. Which substance get oxidized in the reaction : 2Al + Cr2O3 Al2O3 + 2Cr ?
(a) Al (b) Cr2O3 (c) Al2O3 (d) Cr
12. Which substance is a reducting agent in the following reaction ?
Reaction : 2Al + Cr2O3 Al2O3 + 2Cr
(a) Al (b) Cr2O3 (c) Al2O3 (d) Cr
13. In the reaction, 2Na + S Na2S, which sustance acts as oxidizing agent ?
(a) Na (b) S (c) Na2S (d) None of these.
14. Which of following elements does not possess positive oxidation no. in any of its compound ?
(a) O (b) F (c) Cl (d) I
15. Which of the following oxidation no. does not possess by Cl, Br and I, when they conbines with
oxygen forming chemical bond ?
(a) +1 (b) +3 (c) +5 (d) 1
16. Oxygen conbines with which of the element by forming chemical bond, then it possesses posi
tive oxidation no. ?
(a) F (b) Cl (c) Br (d) given all
17. Which of the following element always possesses +1 oxidation state in any of its compound ?
(a) F (b) Ca (c) Cs (d) O
18. Which of the following oxidation no. does not possess by oxygen in any of its compound ?

(a) 2 (b) 1 (c) 12 (d) + 12

19. Which of the following oxidation no. does not possess by carbon in any of its compound ?
(a) 2 (b) 0 (c) 4 (d) +5
20. Which of the following oxidation no. does not possess by nitrogen in any of its compound ?

1
(a) - (b) 3 (c) 4 (d) +5
3
21. Which of the following oxidation no. possesses by oxygen in its compounds ?
(a) 1 (b) +3 (c) 4 (d) +5
22. Which type of metal compounds are nomenclate according to stock notation nomenclature method ?
(a) Metal compound having fixed oxidation no. (b) Comppounds of alkali metals.
(c) Metallic compounds having more than one oxidation no.
(d) Compounds of nonmetal.

187
23. Molecular fromula of sodium chromate (VI) is _________
(a) Na2Cr2O 7 (b) Na2Cr2O4 (c) Na2CrO4 (d) NaCrO4
24. What is the name of K2Cr2O7 according to stock notation nomenclature method ?
(a) Potassium dichromate (VI) (b) Potassium chromate (VI)
(c) Potassium dichromate (III) (d) Potassium dichromate (IV)
25. What is the name of TiO2, according to stock notation nomenclature method ?
(a) Titanium (II) oxide (b) Titanium oxide (IV)
(c) Titanium (IV) oxide (d) Titanium (V) oxide
26. Which of the following is the true for Fe3O4 ?
(a) oxidation no. of each Fe atom in Fe3O4 is +3.
(b) oxidation no. of each Fe atom in Fe3O4 is +2.
(c) The no. of atoms having oxidation no. +2 and +3 is 1:2 in Fe3O4.
(d) oxidation no. of each Fe atom in Fe3O4 is + 83
27. Which of the following is the wrong for Fe3O4 ?
(a) it is a combination of FeO and Fe2O3.
(b) Oxidation no. of each Fe atom in Fe3O4 is not + 83
(c) The no. of atoms havinf oxidation no. +2 and +3 is 1 : 2 in Fe3O4.
(d)The proportion of FeO and Fe2O3 is 1 : 2
28. The no. of peroxy ring in CrO5 is
(a) 1 (b) 2 (c) 3 (d) None of these
29. Which of the following compound possesses peroxide (O22 or [OO]2) in?
(a) kO2 (b) TiO2 (c) BaO2 (d) SiO2
30. Which of the following compound possesses super oxide (O21) ion ?
(a) kO2 (b) TiO2 (c) BaO2 (d) SiO2
31. The oxidation no. of sulphur in H2SO3, H2SO4, H2SO5 or sulphuric acid, Caros acid
(permono sulphuric acid) is respectively________
(a) +4, +6, +7 (b) +3, +6, +6 (c) +4, +6, +6 (d) +5,+6, +6
32. The oxidation no. of sulphur in H2S2O8, H2S2O3, H2S2O7, H2S2O6 or marshal acid
(perdisulphuric acid), thiosulphuric acid, oleum, Dithianic acid is respectively
(a) +6, +3, +6, +7 (b) +7, +2, +6, +5 (c) +6, +2, +6, +5 (d) +6. +3, +7, +5
33. What is the oxidation No. of Cr in CrO5, K2Cr2O7, K2CrO4 or in chromium p e n t o x id e ,
potassium dichromate, potassiam chromate respectively
(a) +4, +5, +6 (b) +5, +6, +6 (c) +4, +6, +6 (d) +6, +6, +6
34. What is the oxidation no. of phosphorus in H3PO3, H3PO4, H3PO2 or phosphorus, acid,
Phosphoric acid, phosphinic acid respectively ?
(a) +3, +5, +1 (b) +3, +5, +2 (c) +4, +5, +1 (d) +2, +5, +1

188
35. What is the oxidation no. of phosphorus in H P O7, H5P3O10, (HPO3)3 or pyrophosphoric acid,
4 2
penta phosphoric acid, triphosphoric acid respectively ?
(a) +4, +5, +3 (b) +6, +5, +5 (c) +5. +5, +5 (d) +4, +5, +5
36. The oxidation no. of chlorine in HClO, HClO2, HClO3, HClO4 or Hypochlorus acid, chloric
acid, chloric acid and perchloric acid respectively are
(a) +1, +3, +5, +7 (b)+1, +2, +3, +4 (c) +1, +2, +4, +5 (d) +4, +3, +2, +1
37. What is the oxidation no. of iodine in ICl3, CsI3 respectively ?
(a) +3, 1 (b) +1, 1 (c) +1/3, 1 (d)+3, 1/3
38. What is the maximum positive oxidation state of halogen element in any of its compound ?
(a) +1 (b) +3 (c) +7 (d) +5
39. What is the maximum positive oxidation state of chalcogen element in its compound ?
(a) +6 (b) +3 (c) +7 (d) +5
40. What is the the oxidation no. of nitrogen in N3H, H2N2O2, HNO3 or hydrazoic acid,
hyponitrons acid, nitrus acid, nitric acid respectively ?
(a) 1, +1, +3, +5 (b) 1/3, +1, +3, +5 (c) +1, +1, +3, +5 (d) +1/3, +2, +2, +5
41. What is the oxidation no. of silicon in zeolite (Na2Al2Si4O12) and tremolite [(Ca2 Mg5(OH)2(Si4O11)2]
respectively ?
(a) +4, +3 (b) +4, 4 (c) +2, +2 (d) +3, +4
42. The value of n in AlFxOyn is
(a) +3 xy (b) +3x2y (c) +3+x+2y (d) +3+xy
43. The value of n in AlFXOyn, if x=1 and y=1 ?
(a) +1 (b) +2 (c) 0 (d) +3
44. The value of n in AlFXOyn, if x=2 and y=3 ?
(a) 2 (b) 0 (c) 5 (d) 4
45. What will be the value of x and y respectively in AlFXOy6 ?
(a) 1, 4 (b) 3, 2 (c) 2, 2 (d) 4, 3
46. How many moles of elements are added when 2.5 mole Cr2O72 reduced in Cr3+ ?
(a) 12.5 (b) 15 (c) 7.5 (d) 10
47. What moles of Cr2O72 reduced in Cr3+ by the addition of 12 moles of electrons ?
(a) 6, (b) 5 (c) 2 (d) 12
48. How many mole ferrous (Fe2+) ion oxidized in ferric (Fe3+) ion by the required no. of electrons
the oretically to reduced 4 mole Cr2O72 in to Cr3+ ?
(a) 8 (b) 24 (c) 48 (d) 12
49. Theoretically what gram ferrous (Fe2+) ion oxidized in to ferric (Fe3+) ion by passing
2.4125 105 coulomb electric charge ? (Atomic mass of Fe=56 gram/mole)
(a) 70 gram (b) 140 gram (c) 14 gram (d) 280 gram

189
50. What mole of MnO4 reduced in Mn2+ by the addition of 7.5 mole electrons in MnO4?
(a) 2.5 (b) 5 (c) 1.5 (d) 7.5
51. How many electrons required to add for the reduction of one mole of MnO4 in Mn2+ ?
(a) 1.8066 x 1024 (b) 3.011 x 1024 (c) 6.022 x 1024 (d) 1.2044 x 1024
52. When 3.11 x 1024 Cr2O72 ion reduced in Cr3+, then how many ferrous (Fe2+) ionoxidised in ferric
(Fe3+) ion ?
(a) 1.8066 x 1024 (b) 1.2044 x 1024 (c) 6.022 x 1025 (d) 1.8066 x 1025
53. Theoretically, how many moles of iodide (I) ion oxidized into iodate (IO3) in using the no of
electrons required for the reduction of 24 moles of MnO4 ion into Mn2+ ion ?
(a) 30 (b) 60 (c) 40 (d) 20
54. What is the oxidation no. of underlined C in HCHO, CH3CHO, CH3COOCH3 or formalehyde
acetaldehyde, acetone respectively ?
(a) 0, +1, +2 (b) 0, +1, +3 (c) +1, 0, +3 (d) 0, +2, +1
55. What is the oxidation no. of underlined C in HCHO, CH3CHO, CH3COOCH3 or methanol,
acetic acid and methyl acetate respectively ?
(a) +2, +3, +3 (b) 2, +3, +3 (c) +2, +3, +4 (d) 2, +3, +4
56. What is the oxidation no. of underlined C in CH3COOCH3 respectively ?
(a) 3, +3, 3 (b) 3, +3, 2 (c) +3, +3, +3 (d) 2, +3, 2
57. In which of the following compound, oxidation no of all C atoms are same ?
(a) ethane (b) cyclohexane (c) benzene (d) given three compounds
58. What are the values of b, d and f in the balanced state of the following reaction?
Reaction : aMnO4+bAs2O3+ cH2O dMn2++eASO43+fH+(PH<7)
(a) 4, 5, 18 (b) 4, 10, 9 (c) 5, 4, 18 (d) 5, 4, 9
59. In the balanced state of reaction, aBr2 + bOH + CH2O dBrO4 + eHBr (basic medium),
if c=9, then what will be the change in oxidation no. (change in no. of electrons).
(a) 21 (b) 7 (c) 14 (d) 9
60. What will be the chage in the oxidation no. (change in no. of electrons) inbalanced eqation given
below ? Reaction : 2S + bHNO3 CH2SO4 + dNO (acidic medium)
(a) 6 (b) 3 (c) 12 (d) 9
61. What will be the chage in the oxidation no. (change in no. of electrons), if no. of e = 10 in
balanced eqation given below ? Reaction : aP4 + bNO3 + cH+ dPO43 + eNO2 + fH2O
(acidic medium)
(a) 20 (b) 10 (c) 5 (d) 15
62. What is the ratio of value of band c in the balanced equation given below ?
Reaction: aP4 + bNO3 + CH+ dPO43 + eNO2 + fH2O (acidic medium)
(a) 5 : 2 (b) 10 : 3 (c) 2 : 5 (d) 1 : 1

190
63. In the reaction, R CHO + 2cuO R COOH + Cu2O which substance is oxidising agent?
And which substance is oxidizes ? (mention it respectively)
(a) CuO, RCOOH (b) RCHO, CuO (c) CuO, RCHO (d) Cu2O, RCHO
64. In which of the following reaction, H2O2 acts as oxidizing agent ?
(a) HOCl + H2O2 H3O+ + Cl + O2 (b) I2 + H2O2 + 2OH 2I + 2H2O + O2
(c) PbS + 4H2O2 PbSO4 + 4H2O (d) In given all three.
65. In which of the following reaction, H2O2 as reducing agent ?
(a) HNO2 + H2O HNO3 + H2O
(b) 2KMnO4 + 2H2O2 2MnO2 + 2KOH + 2H2O + 3O2
(c) H2SO3 + H2O2 H2SO4 + H2O
(d) 2Fe2+ + H+ + H2O2 2Fe3+ + H+ + H2O
66. Which of the following is a redox reaction ?
(a) As2O3 + 3H2S As2S3 + 3H2O (b) 2NH3 + H2O + CO2 (NH4)2CO3
(c) CaO +3C CO + CaC2 (d) Be(OH)2 + 2HCl BeCl2 + 2H2O
67. Which of the following is a correct ascending order, when oxidation no. of sulp hur of H2SO 3,
H2S2O3, H2S2O7, H2S2O8or sulphurus acid, thio sulphuric acid, oleum, dithianic acid is arranged in
ascending order ?
(a) H2SO3 < H2S2O3 < H2S2O7 < H2S2O6 (b) H2SO3 < H2S2O3 < H2S2O6 < H2S2O7
(c) H2S2O3 < H2SO3 < H2S2O7 < H2S2O6 (d) H2S2O3 < H2SO3 < H2S2O6 < H2S2O7
68. When 0.25 mole I oxidised in IO3 then what coulomb of electric charge relateswith the reaction
theoretically ?
(a) 5.79 105 C (b) 4.82500 105 C (c) 2.895 105 C (d) 1.4475 105 C
69. In the sample of ironoxide, the no. of Fe2+ ion is 90% and no. of is 10% then what is the molecular
formula ?
(a) Fe0.905O (b) Fe0.952O (c) Fe0.825O (d) Fe0.857O
70. In a balanced equation, aLiH + bH2O2 CLi2O + dH2O (acidic medium), if d=6 then, what
would be the change in no. of electrons and the value of a, b & c respectively
(a) 8, 4, 4, 2 (b) 4, 2, 2, 1 (c)4, 2, 1, 2 (d) 4, 4, 4, 2
71. The oxidation no. of sulphur in Al2(SO4)3 is
(a) +8 (b) +7 (c) +5 (d) +6
72. In a balanced equation, aP(S) + bH2O + CO2 dH3PO4 +CO2,oxidation no. of oxygen decreases
30, then ce=
(a) 3 (b) 1 (c) 2 (d) 4
73. Zn rod is dipped in a 1.5 litre aqueous solution of 0.1 M CuSO4 ;then choose the correct alternate
for given statements bassed on theoretical calculations. ( Atomic weight of Zn = 65
gram mole1, Cu = 63.5 gram mole1) T= correct statement and F = false statement.
(i) 0.3 gram wt. of Zn rod decreases, then 13 gm Zn metals in the solution

191
(ii) 1.95 gram Zn metal dissolve in the solution, then concentration of Cu2+ ion is 0.08 M.
(iii) when molarity of Cu2+ and Zn2+ becomes equal in the solutoion then no. of Zn2+ ions in the
solution is 4.5165 1024.
(iv) when 3.6132 1022 atoms of Cu deposited then concentration of Zn2+ ions in the solution
becomes 0.04 M
(a) TTTF (b) FFFT (c) TTFT (d) FFTT
74. In balanced equation, aCu2S + bNO3 cH+ 12Cu2+ + eSO42 + fNO + gH2O then what will be
the change in oxidation no. and the value of b and g respectively ?
(a) 30, 10, 8 (b) 121, 20, 16 (c) 24, 10, 8 (d) 60, 20, 16

75. Mg | Mg (c2+1 ) P Ag (C
+
2)
| Ag which of the following Nernst equation is correct for the given
electrochemical cell ?

1
RT [Mg 2+ ] RT [Mg 2+ ] 2
(a) E cell = Ecell - .ln (b) E cell = Ecell - .ln
2F [Ag + ]2 F [Ag + ]

RT [Mg 2+ ] RT [Ag + ]2
(c) E cell=Ecell- .ln (d) E cell = Ecell - .ln
2F [Ag + ] 2F [Mg 2+ ]

2+ 2+
76. For an electro chemical cell, Zn| Zn (C1)
|| Cu (0.5m) |Cu , at constant temperature, reducing the
concentrations of both the solutions, then value of cell potential increases, then, what would be
the value of C1 ?
(a) 0.7 M (b) 0.5 M (c) 0.4 M (d) None of these

E Cu E Cu 2+ E Cu
77. = -0.34 volt and = +0.16 volt , then the value of is ___.
Cu 2+ Cu + Cu +
(a) 0.12v (b) 0.48v (c) +0.48v (d) 0.52v

78. What will be the oxidation potential of Pt| H 2


(a)
| H + (PH =11) half cell 25 C temperature ?
(1bar)

(a) 0.177v (b) 0.649v (c) 0.282v (d) 0.616v

79. For an electrochemical cell, Mg| Mg 2+ (C1) || Br - (C2 ) | Br2(l) | Pt , what wiil be thechange in cell
potential, when conc of solution of cathode increase at constant temperature ?
(a) increases (b) decreases (c) No change occurs(d) E cell=Eo cell
80. Aqueous solution of salt of metal B is stored in a vesselof metal A and aqueous solution of solt
of metal c can be storedin the vessel of metal B, then, which of the following is the
correct descending order of their strength of reducing agent of A, B and c ?
(a) A > B > C (b) A > C > B (c) C > B > A (d) C > A > B

192
81. Standard oxidation potential of half cells of A/A2+, B/B2+, C/C2+ and D/D2+ are in
increasing order, then which of the following statement is correct ?
(a) solution of salt of A2+ can be stored in the vessel of metal B.
(b) solution of salt of D2+ cant be stored in the vessel of metal C.
(c) solution of salt of D2+ can be stored in the vessel of metal B.
(d) given all three statements are wrong.
82. For which of the following compound, a graph of mular conductivity and (molarity)1/2 is obtained
straight line ?
(a) CsCl (b) NH4OH (c) HCOOH (d) given all three
83. Electrolytic cells having molten NaCl, CaCl2 and AlCl3 solutions are connected in series and same
electricity is passed then, which of the following ratio of moles of metal obtained at cathode is
correct ?
(a) 1:2:3 (b) 3:2:1 (c) 6:2:3 (d) 6:3:2
84. Eored for Fe/Fe2+ and Fe2+/Fe3+ half cells are 0.44volt and +0.77volt respectively then what will
be the value of Eoox for Fe/Fe3+ half cell ?
(a) 0.037v (b) 0.33v (c) 0.33v (d) o.11v
85. 5 faraday electric charge is passed during electrolysis of molten cacl2 solution, then what moles of
Ca obtained at cathode experimentally ?
(a) 2.5 mole (b) less then 2.5 mole (c) more then 2.5 mole (d) 5 mole
86. When same electric charge is passed through electrolytic cells containing aqueous solutions of
CuSO4, AgNO3 and NiSO4, then what would be prportion of moles of metal obtained at different
cathodes respectively ?
(a) 2:1:2 (b) 2:2:1 (c) 1:1:2 (d) 1:2:1
87. What would be the change in PH of the solution, when electrolysis of aqueous solution of CuSO4
is carried out in presence of inert electrodes ?
(a) PH increasses (b) PH decreases (c) no change in PH (d) cant be predicted
88. Which of the following formula is true to calculate the molar conductivity in conventional
symbols ?

(a) ^m= 1000Rl


CA
(b) ^m = 1000 GA
C l

^
(c) m =
1000Gl
CA
(d) ^m = 1000Cl
RA


89. ^KCl- ^ NaCl=23.4 Mho (cm) mole 2 -1
and ^ NaSr- ^ NaCl=1.8 Mho -(cm) -mole
2 -1
then,
which of the following is the correct order of molar conductivity at intimite dilution in an ascending
pattern?
(a) NaCl < KCl < NaBr < KBr (b) NaCl < NaBr < KCl < KBr
(b) KBr < NaCl < NaBr < KCl (d) KCl < NaCl < NaBr < KBr
193
90. In an experiment of electroplating, 4 ampere electric current is passed for 2 minutes. Hence, m
gram Ag is deposited at cathode. If 6 ampere current is passed for 40 second, then what amount
Ag is deposited at cathode ?
(a) 2m (b) 4m (c) m/2 (d) m/4
91. Which of the following is correct form of the nernst equation to determine the oxidation
potential of Cu/Cu2+ half cell ?
2+ 2+ RT
(a) E Cu|Cu =E Cu|Cu - ln [Cu 2+ ]
2F
2+ 2+ RT
(b) E Cu|Cu =E Cu|Cu + ln [Cu 2+ ]
2F
2+ 2+ RT
(c) E Cu|Cu =E Cu|Cu + ln [Cu 2+ ]
2F
2+ 2+ RT 1
(d) E Cu|Cu -E Cu|Cu = ln
2F [Cu 2+ ]

92. The values of for ^ m A2B, X3Y2 and A3Y are 2.4, 1.5 and 1.8 respectively. Then what will be the

value of ^ m for XB ?
(a) 1.7 mho (cm)2 mole1 (b) 8.1 mho (cm)2 mole1
(c) 2.1 mho (cm)2 mole1 (d) 0.7 mho (cm)2 mole1
93. What amount of electric charge required for the reduction of 1 mole Cr2O72 into Cr3+ theoretically ?
(a) 6 (b) 3 (c) 1 (d) 4
94. What does the potential of the electrochemical cell become zero ?
(a) Eox of anode and Ered of cathode become equal
(b) Eored of anode and Eored of cathode become equal
(c) Ered of anode and Ered of cathode become equal
(d) Concentration of both the half cell become same
95. In an electrochemical cell for which of the following alternative shows Ecell = Eocell ?
(a) k = 1, then... (b) cell reaction is in equilibrium, then
(c) concentration of both the half cells become equal, then
(d) None of these

96. For the electrochemical cell, Mg|Mg 2+ (0.04m)||Ag + (C2 ) |Ag ; Ecell = Eocell, then what will
be the value of C2 ?
(a) 0.04 M (b) 0.02 M (c) 0.2 M (d) 0.0016M
2+ 2+
97. In an electrochemical cell, Fe|Fe(C1)
||Cu (0.4m) |Cu , at constant temperature, concentration of both
the solutions increases equally, then cell potential increases. Thus which od the following will
be the value of C1 ?
(a) 0.8 M (b) 0.1 M (c) 0.4 M (d) None of these

194
98. Which of the following possesses maximum equivalent conductance at infinite dilution?
(a) K+(aq) (b) Na+(aq) (c) Cs+(aq) (d) Rb+(aq)
99. The potential of a cell is 2.0 volt. change in free energy (SG) occur for the reaction of cell is
50k.cal then what coulomb electric charge obtained from the cell?
(a) 1.08 c (b) 104600 c (c) 25000 c (d) 0.26 c
100. A+ 25 c temperature and 1 bar pressure, oxidation potential of hydrogen half cell is 0.118
volt, then what would be the POH of the solution?
(a) 2 (b) 3 (c) 5 (d) 12
2+ 2+
101. For the electrochemical cell, Zn|Zn (C1)
||Cu (C 2)
|Cu , C2 > C1, then what change occurs
in cell potential when concentration of solutions of both the half cells increases equally ?
(a) increases (b) decreases (c) No change occurs (d) Ecell=Eocell

102. What will be the oxidation potential of Pt | H 2 (g) | H + (aq) half cell at 25oC temperature ?
(1 bar) (P H = 4)

(a) 0.118 (b) 0.649 v (c) 0.236 v (d) 0.118v


2+ 2+
103. For the electrochemical cell, Zn|Zn (C1)
||Cu (C 2)
|Cu ; Ecell > EoCell then., what change occurs
in the value of cell potential, when concentration of both the solution decreased equally ?
(a) Increases (b) decreases
(c) no change occurs (d) cant be predicted
104. Which equation is suitable to calculate the value of Eocell ?

(a) E cell=E red+E red
(b) E cell=E

oxi+E oxi
(anode) (cathode) (cathode) (anode)


(c) E cell=E oxi-E oxi
(d) E cell=E

oxi+E red
(anode) (anode) (anode) (cathode)

+ 2+
105. At 25oc temperature, potential of Mg (2xM ) | Mg and Cu( yM ) | Cu and half cells are 2.365 v and

0.3415 v respectively. What will be the cell potential of the cell formed by these two half cells at
250 c temperature ?
(a) 2.0235 v (b) 2.0235 v (c) 2.7065 v (d) 2.7065 v

195
+
(106) Cell potential of the cell formed by connecting half cells Fe |Fe2+
( x M) and Ag | Ag ( y M) is 1.295
V. if at 250C potential of the | Ag is 0.84 V. then what is the potential of the | at C?
(a) 0.455 V. (b) 2.135 V. (c) 2.135 V. (d) 0.455 V.
0
(107) IF ECo 2+ < ECo 2+ |Co then which value of x is possible?
(x M ) |Co

(a) 1.2 M (b) 0.2 M (c)1.0 M (d) can not be decided


(108) If cell potential of standard cell is 0.59 V then equilibrium constant for the cell reaction
occuring in the cell at 250 C is _______. (n = 1)

(a) 1.0 (b) 10.0 (c) 1010 1


(d) 10

(109) Which of the following cell is different?


(a) Daniel cell (b) lead storage cell (c) laclanche cell (d) electrolytic cell
(110) If equilibrium constant of a cell for reaction occuring in the electrochemical cell is
1.9413 1037 at 250C then what is the std. cell potential of the cell (n=2)
(a) 2.2 V. (b) 1.1 V. (c) 0.0085 V. (d) 0.013 V.
(111) On which of the following cell potential of the cell does not depend?
(a) temperature (b) concentration fo the solution of salt bridge
(c) concentration of the solutioin related with cell reaction
(d) nature of electrodes
2.303RT
(112) What is the value of term in nernst equation at 800C ?
F

(a) 0.007 V. (b) 0.01587 V. (c) 0.01857 V. (d) 0.07 V.


(113) Select correct option for the statements given with reference to electrochemical cell
(where T= true and F = false)
(i) in external circuit e flow from cathode to anode.
(ii) in solution electricity conducted through ions.
(iii) in external circuit electric current flow from anode to cathode.
(iv) anions move from anode to cathode through salt bridge.
(a) TFTF (b) FTFF (c) FFFT (d) FTTF
(114) Mention n and Q for the cell reaction taking place in the Pt Cl2(g,1.0 ) Cl(C 1 ) Au3+
(C 2 ) Au
cell.
[Cl ]6
(a) n = 6 , Q = (b) n = 3 , Q = [Au3+ ]2 [Cl ]6
[Au 3+ ]2
1
(c) n = 3 , Q =[Au ] [Cl]3
3+
(d) n = 6 , Q =
[Au 3+ ]2 [Cl ]6

(115) Mention the oxidation reaction takes place in half cell Pt H2(g,1.0 ) OH(aq ).

(a) H2(g) + 2OH(aq ) 2H2 O(l) + 2e (b) 2OH(aq ) H2(g) + O2(g) + 2e

(c) 2H2 O(l) + 2OH(aq ) 3H2(g) + 2e (d) H2(g) + 2OH(aq ) 2H2 O(l) +O2(g) + 2e
(116) Which of the following relation is correct for Faradays 2nd law? where and is quanitity of
substance while and are equivalent mass of the substance.
(a) m1E1 = m2E2 (b) m1E2 = m2E1 (c) m1 + m2 = E1 + E2 (d) E1E2 = m 1m 2

196
(117) Which of the following cell representation is in correct with reference to cell reaction taking
0 0
place in the cell at 250C ? (EPb 2+|Pb = 0.13 V. , ESn 2+|Sn = 0.14 V.)
(a) Sn | Sn2+ 2+
(0.1 M) Pb(0.01 M) | Pb (b) Sn | Sn2+ 2+
(0.02 M) Pb(0.2 M) | Pb
(c) Pb |Pb2+ 2+
(0.01 M) Sn(0.1 M) | Sn (d) Sn | Sn2+ 2+
(0.05 M) Pb(0.5 M) | Pb

(118) A2+ | A, B2+| B, C2+ | D is increasing order of std. reduction potential then choose correct
option for the given state ments. (T = true and F = false )
(i) rod of metal A dissolve in the solution of metal B2+
(ii) solution of A2+ ion can not be filled in the container of the metal C.
2+
(iii) reaction D(s) + B(aq 2+
1.0 M) D(aq 1.0 M) + B(s) spontaneously in forward direction.

(iv) atoms of metal C can not displace B 2+ ioin from its solution.
(a) TFFT (b) FTTF (c) FTFT (d) TFTF
(119) Select correct option in reference to e.m.f. series .
(a) from top to bottom tendency to accept e increases .
(b) from bottom to top strngth as oxidising agent decreases
(c) top to bottom temdency to get oxidised increases .
(d) if rod of the metal present at lower positon is the series is dipped in the solution of the
metal ion present in the upper part of the solution then rod does not get dissolve
0
(120) What is the value of EMg | Mg 2+ at 250C ? ( EMg |Mg 2+ = 2.36 V. )
( 0.5 M )

(a) 2.3689 V. (b) 2.3689 V. (c) 2.88 V. (d) 2.38 V.


(122) If potential of the cell formed by connecting half cell Ag | Ag +( 0.4M) and Al | Al3+
( 0.1 M) is
2.546V then what is the std. cell potential of the cell at 25 C?
0

(a) 1.46 V. (b) 2.46 V. (c) 2.64 V. (d) 2.76 V.


(123) What is the unit of constant 0.059 in Nernst equation?
(a) colulomb (b) volt (c) faraday (d) no unit

(124) If n = 3 is taken for the cell Al | Al3+ 2+


(0.02 M) Pb(0.2 M) | Pb then what is the value of Q ?

(a) 0.05 M1 (b) 0.2247 M1/2 (c) 0.01 M (d) 0.05 M1/2
(125) For cell Mg | Mg 2+ +
(0.005 M) Ag (0.04 M) | Ag which of the following option is correct for the
value of Q related to n ?
(a) n = 1, Q = 3.125 M1/2 (b) n = 2, Q = 1.7677 M
(c) n = 3, Q = 5.5243 M3/2 (d) n = 4, Q = 0.156 M2
(126) For the cell Co | Co2+
(0.04 M) | Cl(0.1 M) | Cl2(g,1.0 | ) Pt which of the following option is correct for
the value of Q related to the value of n?

(a) n= 1 (1) Q = 4.0 104 M 3 (5) Q = 4 M1 (a) a 4, b 5, c 2, d 8


3
(b) n=2 (b) a 7, b 5, c 2, d 6
(2) Q = 8 M 2 (6) Q = 1.6 107 M6
(c) n= 3 9 3 (c) a 7, b 1, c 3, d 6
(d) n= 4 (3) Q = 8.0 106 M 2 (7) Q = 0.02 M2 (d) a 4, b 8, c 7, d 1
1
(4) Q = 2 M2 (8) Q = 16 M 2

197
3+
Pt | Br2g,1.0 | Br(0.5
(127) For the cell Pt M) || Au(0.4 M) | Au
Au value of Q = 20M-4 then what is the
appropriate value of n?
(a) n = 6 (b) n = 2 (c) n = 3 (d) n = 4
(128) If temperature of the cell MgMg | Mg 2+ +
Ag increases then what changes
(0.005 M) || Ag (0.04 M) | Ag
observed in the cell potential of the cell?
(a) increases (b) decreases (c) do not change (d) cannot predict
Ni | Ni2+
(129) If temperature of the cell Ni 2+
(0.05 M) || Cu(0.08 M) | Cu
Cu increases then what changes observed
in the cell potential of the cell?
(a) increases (b)decreases (c) do not change (d) cannot predict
(130) If temperature of the cell Fe | Fe2+ 2+
(x M) || Cu(0.08 M) | Cu decreases then theoritically cell potential

of the cell increases then which value of x is possible?


(a) x = 0.5 (b) x = 0.02 (c) x = 0.05 (d) x = 0.04

(131) If temperature of the cell Zn | Zn2+


(C 1 ) || Sn(C 2 ) | Sn (C1 < C2 ) decreases then theoritically cell
2+

potential what changes are observed in the cell potential of the cell?
(a) increases (b)decreases (c) do not change (d) cannot predict
(132) What is the equilibrium constant of the reaction taking place in the cell Zn | Zn2+ 2+
(C 1 ) || Cu(C 2 ) | Cu
0 0
at 250C? ( EZn 2+ |Zn = 0.76 V., ECu 2+ |Cu = 0.34 V.)

(a) 3.3791 (b) 1.9413 1037 (c) 4.406 1018 (d) 5.15 1038
(133) If for the reaction taking place in one electrochemical cell Kc = 1.3141 10107 M 1
and n = 2 then for that cell value of E0Cell is ________.
(a) 6.32 V. (b) 0.1186 V. (c) 0.32 V. (d) 3.16 V.
(134) If for the reaction taking place in an electrochemical cell Kc = 1.3141 10307 and n = 6.
if standard reduction potential of the cathode is 1.36 V then what is the standard oxidation
potential of the anode?
(a) 6.32 V. (b) 1.66 V. (c) 2.71 V. (d) 3.16 V.
(135) What is the pH of the solution of HCl if cell potentila of the cell
0
Pt | H2(g,1.0 Zh^) | HCl(X M) || Au3+
(0.001 M) | Au is 1.6655 V ? (E Au 3 +| Au = 1.4 V.)

(a) 3.5 (b) 4.17 (c) 5.5 (d) 4.83


(136) Mention the value of n for the reaction taking place in the cell if its cell potential is 3.16V
and equilibrium constant is 1.727 10214 M 2 at 250 temperature.
(a) 1 (b) 2 (c) 3 (d) 4
(137) If for the cell Mg | Mg 2+ +
(0.2 M) || H(X M) H2(g,1.0 | ) Pt values of ECell and E Cell are 2.3629 V..
0

and 2.36 V. respectively at 250C temperature then value of x is ______.


(a) 0.4 M (b) 0.5 M (c) 0.1 M (d) 0.25 M
+ 0
(138) If standard potential of the reaction 2H2 O() O2(g,1 bar ) + 4H(aq ) + 4e is -1.23V and [H+] = 107 at 25 C
at 250C temperature then what is the value of potential for the reaction? OR What is the
oxidation potential of the pure water?
(a) 0.41 V. (b) 2.62 V. (c) 0.817 V. (d) 0.41 V.

198

(139) If for the reaction 2H2 O() + 2e H2(g,1 b ) + 2OH(aq 0
,10 7 M) E = 0.417 V at 25
temperature what is the value of E 0 for the reaction? OR what is the standard reduction
potential of the pure water at 250 if potential is 0.417 V.
(a) 0.83 V. (b) 2.62 V. (c) 0.817 V. (d) 0.41 V.
3+ 2+ 3+ 3+ 2+ 3+
(140) If Co | Co , Co | Co , Fe | Fe , Fe | Fe std. oxidation potential are
0.4167 V., 1.81 V., 0.0367 V., 0.77 V. respectively then what is the std. potential of
the cell Fe | Fe2+ || Co2+| Co ?
(a) 0.72 V. (b) 0.11 V. (c) 1.0 V. (d) 0.16 V.
2+
(141) If cell potentisl of the cell Pt | H2(g,1.0 b | ) HCl(X M) Cu(0.01 M) | Cu 0.635 V. at 250C
0
then what is the pH of the HCl solution? (ECu 2+ |Cu = 0.34 V.)

(a) 2.0 (b) 5.0 (c) 6.0 (d) 4


2+
(142) If cell potentisl of the cell Mg | Mg (0.01 M) HCl(X M) | H2(g,1.0 Zh^) | Pt 2.09V. at 250C
0
then what is the pH of the HCl solution? (EMg 2+ |Mg = 2.36 .)

(a) 6.58 (b) 5.58 (c) 3.58 (d) 4.58


(143) What is the value of E cell and equilibrium constant respectively for the reaction at 250C
Fe(s) + 2Ag +(aq ,0.5 M) Fe2+
(aq ,0.1 M) + 2Ag (s) ?
(a) 1.25 V., 1.04 1021 M 1 (b) 1.23 V., 1.507 1012 M 1
(c) 1.23 V., 1.264 106 M 1 (d) 1.25 V., 1.081 1042 M 1
(144) Which reaction would takes place spontaneously in forward direction at 250C if standard
oxidation potenitials of Pt | I2(s) | I, Cu | Cu2+, Cr | Cr 3+, Zn | Zn2+, Sn | Sn2+, Pb |
Pb2+, Ag | Ag +, Fe | Fe2+ are 0.54 V., 0.34 V., 0.74 V., 0.76 V., 0.14 V., 0.13 V.,
0.80 V., 0.77 V. respectively.
2+
(a) I2(s) + Cu(s) 2I(aq ,0.1 M) + Cu(aq ,0.05 M)
3+ 2+
(b) 2Craq ,0.001 M + 3Zn(s) 2Cr(s) + Zn(aq ,0.1 M)
(c) Pb2+
aq ,0.02 M + Sn(s) Pb(s) + Sn2+
(aq ,0.2 M)
(d) 3Ag + 3+
aq ,0.1 M + Fe(s) 3Ag (s) + Fe(aq ,0.01 M)
(145) How many gram of Ag will be obtained if 5.0 F quantity of electricity is passed through
aqueous solution of AgNO3 ?
(a) 270 g (b) 540 g (c) 180 g (d) 135 g
(146) If 15 Faraday quantity of electricity is passed through Al3+
() solution then how many
gram of Al metal will be obtained ? ( cell efficiency is 80 % .) (At.wt Al = 27 gm mol-1)
(a) 135 gm (b) 121.5 gm (c) 108 gm (d) 94.5 gm
(147) If 10 Faraday quantity of electricity is passed through molten NaCl during electrolysis 84
L of Cl2 gas is obtained at STP then what is the efficiency of the cell?
(a) 80 % (b) 75 % (c) 50 % (d) 90 %
(148) During electrolysis oif acqueous solution of NaCl with inert electrodes then at cathode
+
instead of Na(aq ) reduction of H2 O takes place because .....................
(a) Compare to H2 O std. reduction potential of Na+
(aq ) is more
(b) Standard oxidation potential of H2 O is very high
(c) of inert electrodes
(d) reduction potential of Na+
(aq ) is very less

199
(149) Select correct option for the given statements (where T = true anf F= false )
i) if electrolysis of aqueous solution of CuSO4 is done with inert electrodes then blue
colour of solution become faint.
ii) if electrolysis of aqueous solution of CuSO4 is done with inert electrodes then pH of
the solution increases.
iii) if electrolysis of aqueous solution of CuSO4 is done with active electrodes of Cu
then weight of anode decreases.
iv) if electrolysis of aqueous solution of CuSO4 is done with active electrodes of Cu then
precious metals release from cathode.
(a) FTFT (b) TFFF (c) TFTT (d) TFTF
3+
(150) On passing necessary quantiy of elecricity through Al() solution 4.5 g Al deposited then
+
on passing same quantity of electricity through the solution of H(aq ) having sufficient
concentration then what volume of hydrogen gas will be obtained at STP.? (at.wt Al = 27
g mol1 )
(a) 44.4 L (b) 222.4 L (c) 11.2 L (d) 5.6 L
(151) Select correct option for the given statements (where T = true anf F= false )
i) if electrolysis of dilute aqueous solution of NaCl is done with inert electrodes then pH
of the solution increases.
ii) if electrolysis of concentrated aqueous solution of NaCl is done with inert electrodes
then at anode chloricne gas is obtained.
iii) if electrolysis of dilute aqueous solution of NaCl is done with inert electrodes at
cathode hydrogen gas is obtain.
iv) if electrolysis of concentrated aqueous solution of NaCl is done with inert
electrodes then on adding phenolphthalien to the solution becomes pink.
(a) TFTT (b) FTTT (c) FFTT (d) TFTF
(152) Select correct option for the given statements (where T = true anf F= false )
i) if electrolysis of dilute aqueous solution of NaCl is done with inert electrodes then pH
of the solution increases.
ii) if electrolysis of dilute aqueous solution of NaCl is done with inert electrodes then
on adding methyl orange to the solution it becomes orange coloured.
iii) if electrolysis of concentrated aqueous solution of NaCl is done with inert electrodes
pOH of the solution increases.
iv) if electrolysis of concentrated aqueous solution of NaCl is done with inert electrodes
then the solution obtained turns red litmus to blue.
(a) TFTF (b) FTTT (c) FTFT (d) FFTT
0 0
(153) If at 50 C temperature value of Eox is + 0.83 V for the half cell Pt H2(g,1.0b ) OH(aq )
then what is the value of ionic product of water at 500 ?
(a) 1.12 1013 (b) 1.102 1012 (c) 1.105 1012 (d) 1.0 1014
(155) What quantity of electricity is required for complerte reduction of all Ag + from 1.0 M
AgNO3 is 250 ml aqueous solution ?
(a) 2412.5 C. (b) 24125 C. (c) 4825 C. (d) 25250 C.
(156) Using 2 g Hg cathode Cd-Hg amalgam is obtained by electrolysis of CdCl2 then how
much ampere electric current should be passed for 100 seconds to obtain Cd-Hg having
20% Cd ? (At.wt Cd = 112.5 g mol-1 )
(a) 34.32 A (b) 17.16 A (c) 4.29 A (d) 8.58 A

200
201
202
203
204
205
JEE-UNIT-8 ANSWER KEY
1 c 50 c 99 b 148 d 197 d
2 b 51 b 100 d 149 d 198 b
3 a 52 d 101 b 150 d 199 c
4 a 53 d 102 c 151 b 200 a
5 a 54 a 103 a 152 c 201 b
6 b 55 b 104 d 153 a 202 d
7 d 56 b 105 c 154 a 203 b
8 d 57 d 106 d 155 b 204 b
9 c 58 c 107 a 156 d 205 d
10 c 59 a 108 c 157 c 206 a
11 a 60 c 109 d 158 a
12 a 61 a 110 b 159 a
13 b 62 b 111 b 160 a
14 b 63 c 112 a 161 a
15 d 64 c 113 d 162 d
16 a 65 b 114 d 163 c
17 c 66 c 115 a 164 d
18 d 67 d 116 b 165 c
19 d 68 d 117 a 166 c
20 c 69 b 118 a 167 b
21 a 70 a 119 c 168 c
22 c 71 d 120 a 169 a
23 c 72 a 121 d 170 c
24 a 73 c 122 b 171 d
25 c 74 d 123 b 172 b
26 c 75 b 124 b 173 d
27 d 76 c 125 c 174 a
28 b 77 d 126 c 175 d
29 c 78 a 127 c 176 c
30 a 79 b 128 b 177 b
31 c 80 c 129 a 178 b
32 c 81 c 130 a 179 b
33 d 82 a 131 b 180 a
34 a 83 d 132 b 181 d
35 c 84 a 133 d 182 c
36 a 85 b 134 b 183 a
37 d 86 d 135 c 184 c
38 c 87 b 136 a 185 b
39 a 88 c 137 b 186 d
40 b 89 b 138 c 187 c
41 b 90 c 139 a 188 b
42 b 91 c 140 d 189 c
43 c 92 a 141 c 190 a
44 c 93 a 142 b 191 c
45 a 94 c 143 d 192 d
46 b 95 d 144 a 193 b
47 c 96 c 145 b 194 c
48 b 97 a 146 c 195 a
49 a 98 c 147 b 196 d

206
UNIT - 9 CHEMICAL KINETICS
Important Points
Chemical Kinetics : The branch of chemistry which deals with the study of the rate of reaction and
the factor affecting them.
Kinetics - Greek word kinesis = movement
classification of reaction on the basis of rates:
Very fast reaction : ionic reactions (10-9 sec)
very slow reaction : rusting of iron, radiation from uranium.
Slow reaction : reaction by combining dinitrogen and dihydrogen under certain conditions.
Factors Affecting rates of Reaction:
(i) The state of substance and the area of surface
(ii) concentration of solution. (iii) temperature of system.
(iv) Pressure of system (v) Effect of catalyst
(vi) Presence of light
(If there is any impurity which tries to decrease the rate of reaction then it is calledc a t a l y t i c
poison)
Rate of Reaction :
The rate of reaction is the change in the concentration of any one of the reactants or products per
unit time.
Average rate of reaction
D[R] D[P]
rav = - =+
Dt Dt
Instantaneous Rate of Reaction
d[R] d[P]
rinst = - =+ as D t 0
dt dt
Rate of reaction is always positive. The minus sign is used simply to show that the
concentration of the reactant is decreasing.
Rate determination :
In the reactions the stoichiometric coefficients may be different. Rate of reaction can be
determined but the determination must be consistent. In chemical kinetics, the following
method is accepted.
1 d[R] 1 d[P]
Rate = - =+
VR dt vp dt
For any reation n1A + n2B n3C + n4D
1 d [A] 1 d [ B] 1 d [ C ] 1 d [ D ]
Rate = - =- = =
n1 dt n 2 dt n 3 dt n 4 dt

5Br - (aq) + BrO3- (aq)


+
+ 6H (aq) 3Br2(aq) + 3H 2 O()

1 d Br d BrO3- 1 d H 1 d [ Br2 ]
- +

Rate = - =- =- =+
5 dt dt 6 dt 3 dt
207
In aqueous solution, there is negligible change in concentration of water and so the change
in its concentration is not expressed.
Rate Law :
The presentation of the rate of reaction with reference of concentration of reactants is called
rate law.
This rate law in the wide range of concentration of reactants or products is studied and the
law that is established is called differential rate equation or Rate expression. Viz
H 2(g) + I 2(g) 2HI(g) Rate = K [ H 2 ] [ I 2 ]
Rate constant and order of Reaction :
In most of the reaction carried out, the simple rate equation can be obtained in which rate
is proportional to exponents of the concentration of reactant. The exponent is called order
of reaction.
H 2 + I 2 2HI Rate a [ H 2 ] [ I 2 ]

\ Rate = k [ H 2 ][ I 2 ]

H 2 + Br2 2HBr Rate a [ H 2 ][ Br2 ]


1
2

\ Rate = K [ H 2 ][ Br2 ] 2
1

2N 2 O5 4NO 2 + O 2 Rate a [ N 2 O5 ]

\Rate = K [ N 2 O5 ]
Where K is rate constant and it is called specific rate constant when concentration of
reactant is 1 M. At that time reaction rate = rate constant.
It is necessary to note that the order of reaction has no relation with the stoichiometric
cofficient of reactant.
For the reaction n1A + n2B n3C + n4D
Rate = k [A]x[B]y
Order of reaction with reference to reactant A is x and with that of reactant B is y.
Hence total order of reaction = x + y. Thus the total order of reaction is equal to the
sum of exponents of concentration of all reactants.
5Br- + BrO3- + 6H+ 3Br2 + 3H2O
Rate = K [Br-][BrO3-][H+]2
Order of reaction = 1 + 1 + 2 = 4
The total order of reaction can be positive, zero or even a fraction
Examples of different order.
(I) 2NH3(g)
Mo
N 2(g) + 3H 2(g) Rate = k[NH 3 ]o

1
(ii) H 2 O 2 H 2 O + O 2 Rate = k [ H 2 O ]
2

208
(iii) 2NO 2 + F2 2NO 2 F Rate = k [ NO 2 ][ F2 ]

Rate = k [ NO ] [ O 2 ]
2
(iv) 2NO + O 2 2NO2

Rate = k [ CH 3CHO ]
1.5
(v) CH 3CHO CH 4 + CO

Rate = k [ CO ] [ Cl 2 ]
2 1.5
(vi) CO + Cl 2 COCl 2
Unit of Rate constant

dx dx 1 conc 1
Rate = = k (con) n \K = =
dt dt ( conc ) time ( conc )n
n

Order of reaction Unit of K


0 M s-1 Mole litre-1 time-1
1 s-1 time-1
2 M-1 S-1 litre mole-1 time-1
n M1-n S-1 litre1-n molen-1 time-1
Molecularity :
The number of atoms, ions or molecules of the reactant that take part in the reaction and
which experience collision with each other so that the reaction results, it is called molecularity
Hence, the orders of reactions and molecularities of bimolecular, trimolecular and elementary
reaction are same.
The possibility of collision of three or more molecules with one another and to result in the
reaction is less.
The molecularity more than three is not seen.
Molecularity Order of the reaction
It is equal to the sum of the number of It is equal to the exponents of the molar
reactant particles taking part in a single concentration of the reactants in the experimentally
step reaction determined rate equation.
It is always a whole number It can be fractional
It cannot be zero It can be zero
Molecularity applies to one step reaction or Order of reaction applies to a reaction as a
a single step of reaction which involves wholes, irrespective of the intermediate steps
several steps for its completion. The involved for its completion.
molecularity of an overall complex
or multistep reaction has no significance
or meaning.
It can be obtained from simple balanced It cannot be obtained from a simple balanced
equation for the single step reactions. equation. It is obtained only from experimentally
determined rate equation.
It does not help in elucidation of It is helpful in the elucidation of reaction
reaction mechanism mechanism

209
Zero order reaction :
The rate of the reaction is proportional to zero exponent of the concentration of reactant.

-d [ R ]
= k [R ]

- \[ R ] = - K.t + [ R ]o
dt
Graph of [R] versus t then straight line.
Slope = - k and intercept = [R]o
First order reaction :
The rate of the reaction is proportional to exponent one of concentration of reactant.

d [R ] [ R ]o
- = k [R ] kt = n
dt [R ]
[ R ] = [ R ]o .e- kt
K
log [ R ]t = - .t + log [ R ]o
2.303
Graph of log[R]t versus t then straight line.
K
Slope = - and intercept = log[R]o
2.303

0.693
t1 =
2 k
Pseudo unimolecular or Pseudo first order reaction :
Those reaction which are not truly of the first order but under certain conditions become
reactions of the first order. Examples : (i) Hydrolysis of ethyl acetate. (ii) Hydrolysis of cane
sugar.
Determination of order of reaction :
Methods : (i) Intial rate method
(ii) Integrated rate equation method or Graphical method
(iii) Half life method
(iv) Ostwalds isolation method
Ostwalds Isolation method :
In certain reaction there is involvement of more than one reactants. In this method, the
concentration of other reactants in comparison to one reactant is taken in very large proportion.
The reaction rate will be indicative with respect to reactant with less concentration because
the concentration of other reactants remain almost constant.
Half life method :
The time taken for half of the reaction to complete.
It is very simple method.

210
For zero order reaction t[R]
1 o
2

first order reaction t 1 2 is independent of initial concentration

second order reaction t 1 2 a [ R ]o


-1

1
nth order reaction t 1 2 a [ R ]o or t 1 2 a
1- n

[ R ]o
n -1

Arrhenius equation :
Ea
K = A.e - Ea /RT \nk = nA -
RT
Ea 1
og k = og A - .
2.303R T
1
Graph of log K versus = straight line
1
Ea
Slope = -
2.303R

K2 Ea 1 1 Ea. DT
og = - =
K1 2.303R T1 T2 2.303R.T1.T2

Threshold energy :
Arrhenius equation shows that rate constant increases exponentially with temperature.
Increasing temperature from 300 to 310 K, the Kinetic energy increases only by 3% because
it is proportional to temperature.
The reaction rates have almost doubled by increase in temperature by 10 K.
The explanation for this can be given that there must be some pushing energy or threshold
energy required for the reaction of molecules.
Arrhenius factors :
Ea = NA.E* where Ea = Activation energy
Na = Avogadro number
E* = Kinetic energy
K = Ae-Ea/RT where A = pre-exponential factor or frequency factor
Ea = Energy of activation
Both A & Ea = Arrhenius factors
Theory of collision :
Max Trauz and William Lewis (1916-18) = Theory of collision.
In Chemical reaction, the number of collision per second per unit volume is called collision
frequency (z).

211
For biomolecular reaction A + B Products
Rate = ZAB . e-Ea/RT
Rate = P . ZAB . e-Ea/RT
ZAB = collision frequency of A and B whose energy is equal to or more than activation
energy.
P = Probability factor OR steric factor
Those collision in which molecules collide with sufficient kinetic energy (threshold energy) and
proper direction, resulting into products. Such collision are called effective OR fruitful collision.
Endothermic reaction and Exothermic reaction.
Minimum potential energy of reactants is less than that of products then the reaction will be
the endothermic.
Minimum potential energy of reactants is more than that of products then the reaction will
be exothermic.
Endothermic reation : DH = Hp-Hr = +ve
DH = Ea-Ear = +ve when Ea > Ear
Exothermic reaction : DH = Hp-Hr = -ve
DH = Ea-Ear = -ve when Ea < Ear
Where Ea = activation energy of forward reaction
Ear = activation energy of reverse reaction
Effect of catalyst :
The main function of catalyst is decrease the activation energy, bring energy barrier down and
increases the rate of reaction.
Equilibrium constant (K) is not changed.
Rate of reaction increases.

212
M.C.Q.
1. Rate of reation is defined as
(a) decrease in the concentration of a reactant
(b) increase in the concentration of a product
(c) change in the concentration of any one of the reactants or products per unit time.
(d) all the above three are correct
2. 2A + B 3C for the reaction instant rate of reaction is...
1 d [A] d [ B] 1 d [ C ] 1 d [A] d [ B] 1 d [ C]
(a) + =+ + (b) - =- =+
2 dt dt 3 dt 2 dt dt 3 dt
d [A] d [ B] d [ C] d [A] d [ B] d [ C]
(c) + 2 =+ = +3 (d) - 2 =- = +3
dt dt dt dt dt dt
3. The rate of reaction of spontaneous reaction is generally very slow. This is due to the fact that..
(a) the equilibrium constant of the reaction is < 1
(b) the activation energy of the reaction is large
(c) the reaction are exothermic
(d) the reaction are endothermic
4. N2 + 3H2 2NH3 For the reaction the rate of change of concentration for hydrogen is -0.3
x 10-4Ms-1. The rate of change of concentration of ammonia is ...
(a) 0.2 10-4 (b) -0.2 10-4 (c) 0.1 10-4 (d) 0.3 10-4
5. For the reaction of 4A + B 2C + D. Which of the following statement is not correct ?
(a) The rate of formation of C and D are equal
(b) The rate of formation of D is one half the rate of consumption of A
(c) The rate of appearance of C is one half the rate of disappearance of B
(d) The rate of disappearance of B is one fourth of the rate of disappearance of A
6. ______ does not affect the rate of reaction.
(a) size of the vessel (b) amount of the reactants
(c) physical state of reactants (d) DH of reaction
7. In the reaction N2O4(g) 2NO2(g) the pressure of N2O4 falls from 0.5 atm to 0.32 atm is 30
minutes, the rate of appearance of NO2(g) is
(a) 0.012 atm min-1 (b) 0.024 atm min-1 (c) 0.006 atm min-1 (d) 0.003 atm min-1
8. In the reaction K1 and K2 are the velocity constants for the forward and backward reaction
respectively. The equilibrium constant is
K1
(a) K = (b) K = K1 K 2 (c) K = K 2 (d) none of the above
K2 K1
1 1
9. For the reaction A + B + C Products, Rate = K [A] 2 [B] 3 [C]. The order of reaction is

11
(a) 3 (b) 1 (c) 5 6 (d) 6
10. For a reaction pA + qB Products. Rate = K[A]m[B]n. Then
(a) (p+q) = (m+n) (b) (p+q) (m+n)
(c) (p+q) > (m+n) (d) (p+q) = (m+n) or (p+q) (m+n)

213
11. A + 2B C + D For a reaction from following data correct rat law =
Mole liter -1
(A) (B) mole lite-1 min-1
1 0.1 0.1 6.0 10-3
2 0.3 0.2 7.2 10-2
3 0.3 0.4 2.88 10-1
4 0.4 0.1 2.4 10-2
(a) Rate = K [A]2[B] (b) Rate = K [A]2[B]2
(c) Rate = K [A][B]2 (d) Rate = K [A][B]
12. In the reaction A + B Products, the doubling of [A], increases the reaction rate to four times,
but doubling of [B] has no effect on the reaction rate. The rate expression is .
(a) Rate = K [A]2 (b) Rate = K [A] (c) Rate = K [A]2[B]2 (d) Rate = K [A][B]
13. A zero order reaction is one whose rate is independent of .
(a) Reaction vessel volume (b) Concentration of reactants
(c) temperature (d) pressure of light
14. The rate constant of a reaction changes when ...
(a) pressure is changed (b) concentration of reactants changed
(c) temperature is changed (d) a catalyst is added
15. Which of the following is a reaction of zero order ?
hJ
(a) H 2 + Cl 2 2HCl (b) 2N 2 O5 4NO 2 + O 2

(c) 2HI H 2 + I 2 (d) H 2 + Br2 2HBr


16. Which of the following is a reaction of fractional order ?
(a) 2N 2 O5 4NO 2 + O 2 (b) 2NO 2 + F2 2NO 2 F

(c) H 2 + Br2 2HBr (d) 2NO + O 2 2NO2


17. A reaction involving two different reactants can never be a .
(a) bimolecular reaction (b) Unimolecular reaction
(c) first order reaction (d) second order reaction
18. For a reaction 3A hProducts, the order of reaction
(a) 3 (b) 1, 2 or 3
(c) zero (d) any value between 1 and 3
19. When concentration of reactant is increased eighteen times the rate becomes two times, the rate
of reaction is

(a) 1 (b) 1 2 (c) 13 (d) 1 4


20. The rate determining step in a reaction is A + 2B C. Doubling the concentration of B would
make the reaction rate...

(a) two times (b) same rate (c) four times (d) 1 4 times

214
21. The rate law of a reaction is rate = K [A]2[B]. On doubling the concentration of both A and
B the rate X will become ...
(a) x3 (b) 8x (c) 4x2 (d) 9x
22. For the reaction CH3COCH3 + I2 + Products, the rate is governed by, rate =
H+
+
K[CH3COCH3] [H ]. The rate order of iodine is = ______.
(a) 3 (b) 2 (c) 1 (d) O
23. If the order of reaction is zero. It means that
(a) rate of reaction is independent of temperature
(b) rate of reaction is independent of the concentration of the reacting species
(c) the rate of formation of activated complex is zero
(d) the rate of decomposition of activated complex is zero
24. The reactions of higher order are rare because
(a) many bady collisions involve very high activation energy
(b) many bady collisions have a low probability
(c) many bady collisions are not energetically favoured
(d) many bady collisions can take place only in the gaseous phase.
25. 2A +2B D + E For the reaction following mechanism has been proposed.
A + 2B 2C +D (slow) A + 2C E (Fast)
The rate law expression for the reaction is
(a) rate = K [A]2[B]2 (b) rate = K [A]2[B]2[C]
(c) rate = K [A][B]2 (d) rate = K [A][B]
26. A2 + B2 2 AB reaction follow the mechanism as given below
(i) A2 2A (fast)
(ii) A + B2 AB + B (slow)
(iii) A + B AB (fast) the order of overall reaction is
(a) 1.5 (b) 2 (c) 0 (d) 1
27. In the sequence of reaction A
K1
, B
K2
, C
K3
then the rate of determining step of
reaction is
(a) A B (b) B C (c) C D (d) A D
28. For the reaction 2A + B Products, reaction rate = K [A][B]2. Concentration of A is doubled
and that of B is halved the rate of reaction will be ...
(a) doubled (b) halved (c) unaffected (d) four times
29. In one reaction concentration of reaction A is incereased by 16 times, the rate increases only two
times. The order of the reaction would be ...

(a) 2 (b) 4 (c) 1 2 (d) 1 4


30. In the reaction A B. When the concentration of A is changed from 0.1 M to 1 M, the rate
of reaction increases by a factor of 100. The order of reaction with respect to A is .
(a) 10 (b) 1 (c) 2 (d) 3

215
31. For the reaction of A + B C + D, doubling the concentration of both the reactants increases
the reaction rate by 8 times and doubling the initial concentration of only B simply doubles the
reaction rate. The rate law for the reaction is
(a) r = K [A][B]2 (b) r = K [A][B] (c) r = K [A][B] (d) r = K [A]2[B]
32. The unit of rate constant for a zero order reaction is ...
(a) litre sec-1 (b) litre mole-1 sec-1 (c) mole litre-1 sec-1 (d) mole sec-1
33. The rate constant of a reaction has same units as the rate of reaction. The reaction is of ...
(a) third order (b) second order (c) first order (d) zero order
34. The rate constant of reaction is 3 10-3 bar-1 sec-1. The order of reaction is ...
(a) 1 (b) 2 (c) 3 (d) 0
35. The dimensions of the rate constant of a third order reaction involve.
(a) only time (b) time and concentration
(c) time and square of concentration (d) only concentration
36. The rate constant of reaction is 5 10 litre mole-3 minite-1. The order of reaction is...
-2 3

(a) 1 (b) 2 (c) 3 (d) 4


37. Which of the following statements is incorrect about the molecularity of a reaction ?
(a) Molecularity of a reaction is the number of molecules in the slowest step.
(b) Molecularity of a reaction is the number of molecules of the reaction present in the balanced
equation.
(c) There is no difference between order and molecularity of a reaction.
(d) Molecularity is always a positive whole number.
38. For a single step reaction A + 2B Products, the molecularity is
(a) zero (b) 1 (c) 2 (d) 3
39. Which of the following statement is false ?
(a) For a zero order reaction, the rate changes with temperature.
(b) Both order and molecularity of a reaction are always the same.
(c) Active mass of 128 g of HI present in a two litre flask is 0.5.
(d) For the first order reaction, the rate of reaction halved as the concentration of a reactant
halved.
40. If a is the initial concentration of the reactant, the time taken for completion of the reaction, it
if is of zero order, will be

(a) K 2a (b) a 2k (c) a k (d) k a


41. The reaction 2O3 3O2 proceeds in two steps as follows.
(i) O3 O2 + O (fast) (ii) O + O3 2O2 (slow)
The rate law expression should be...
(a) r = K[O3]2 (b) r = K[O3]2[O2]-1 (c) r = K[O3][O2] (d) r = K[O3]2[O2]1
42. For reaction of zero order is ...

2.303 [ Ao ]
(a) K = [ Ao ] (b) Kt = [ A ] - [ Ao ] (c) Kt = [ A ] - [ Ao ] (d) K = n
t
t [A]
216
43. For reaction first order is ...

(a) t = K 2.303log
[A] (b) K =
2.303
log
[A]
[A] o t [A] o
2.303 a
(c) [ A ] = [ A ]o .e - Kt (d) K = log
t a+x
44. For the reaction Zero order
(a) t 1 a Co 2 (b) t 1 a Co (c) t 1 a Co -1 (d) t 1 a Co
2 2 2 2

45. For reaction first order


0.693 0.693 0.693 0.693
(a) t 1 = (b) t 1 = (c) t 1 a (d) t 1 a
2 k 2 Co 2 k 2 Co
46. th
Which of the following represents the expression for life of a first order reaction
k 4 2.303 3 2.303 2.303
(a) og (b) og (c) og 4 (d) og 3
2.303 3 k 4 k k
47. If initial concentration is doubled, the time for half reaction is also doubled. The order of reaction
is ...
(a) First (b) Second (c) Third (d) Zero
48. If a is the initial concentration of the reactant, the half life period of the reaction of the nth order
is proportional to ...
(a) an+1 (b) a1-n (c) an (d) an-1
49. For the first order reaction, half life is 14 s. The time required for the initial concentration to
reduce to 1/8th of its value is ...
(a) 28 s (b) 42 s (c) (14)2 s (d) (14)3 s
50. In the first order reaction the concentration of the reactants is reduced to 25% in one hour. The
half life period of the reaction is
(a) 120 min (b) 4 hr (c) 30 min (d) 15 min
51. For the First order reaction with half life is 150 seconds, the time taken for the concentration of
the reactant to fall from m/10 to m/100 will be approximately
(a) 600 s (b) 900 s (c) 500 s (d) 1500 s
52. The half life period of a first order reaction is 15 minutes. The amount of substance left after one
hour will be ...
1 1 1 1
(a) (b) (c) (d)
2 4 8 16
53. For the reaction N2O5 2NO2 + O2 t = 24 hrs. starting with 10 g of N2O5 how many
grams of N2O5 will remain after a period of 96 hours ?
(a) 0.63 g (b) 0.5 g (c) 1.77 g (d) 1.25 g
54. In the first order reaction 75% of reactant disappeared in 1.386 h. Calculate the rate constant
of reaction.
(a) 3.6 10-3 S-1 (b) 2.8 10-4 S-1 (c) 17.2 10-3 S-1 (d) 1.8 10-3 S-1

217
55. The minimum amount of energy required for the reacting molecules to undergo reaction is called :
(a) potential energy (b) internal energy (c) activation energy (d) threshold energy
56. Increase in the concentration of the reactants leads to the change in
(a) heat of reaction (b) threshold energy (c) collision energy (d) activation energy
57. Energy of activation of an exothermic reaction is
(a) zero (b) negative
(c) positive (d) can not be predicated
58. The chemical reactions in which reactants require high amount of activation energy are generally
................
(a) slow (b) fast (c) instantaneous (d) spontaneous
59. The rate of reaction increases with increase of temperature because ...
(a) an increase in the number of activated molecules
(b) an increase in the number of collisions
(c) lowering of threshold energy
(d) activation energy is lowered
60. The activation energy of reaction is equal to
(a) Threshold energy + Energy of the products
(b) Threshold energy - Energy of the reactants
(c) Threshold energy + Energy of the reactants
(d) Threshold energy - Energy of the products
61. Collision theory is most satisfactory for ___________ reaction.
(a) First order (b) second order (c) Bimolecular (d) Any
62. If Ef and Er are the activation energies of the forward and reverse reactions and the reaction is
known to be exothermic then
(a) Ef < Er (b) Ef > Er (c) Ef >>> Er (d) Ef = Er
63. Which of the following does not affect the rate of reaction ?
(a) size of the vessel (b) physical state of reactants
(c) amount of the reactants (d) DH of reaction
64. For a an endothermic reaction, DH represents the enthalpy of reaction. The minimum value for
the energy of activation will be ...
(a) equal to DH (b) zero (c) more than DH (d) less than DH
65. For an endothermic reaction A B. An activation energy of 15 Kcal mole-1 and the enthalpy
change of reaction is 5 Kcal mole-1. The activation energy for the reaction B A is
(a) 10 Kcal mole-1 (b) 20 Kcal mole-1 (c) 15 Kcal mole-1 (d) zero
66. -1
For an exothermic reaction an activation energy of 70 KJ mole and the enthalpy change of
reaction is 30 KJ mole-1. The activation energy for the reverse reaction is ...
(a) 70 KJ mole-1 (b) 30 KJ mole-1 (c) 40 KJ mole-1 (d) 100 KJ mole-1
67. The rate constant of the reaction increases by ...
(a) increasing the temperature (b) increasing the concentration of reactants
(c) carrying out the reaction for longer period (d) adding catalyst

218
68. Which of the following is the expression for Arrhenius equation ?

k 2 Ea 1 1
(a) n = - (b) n k = n A - Ea
k1 R T1 T2 RT

- Ea
(c) k = A.e RT (d) All the above
1
69. The Plot of log K vs helps to calculate
T
(a) Activation energy (b) Rate constant
(c) Reaction order (d) Activation energy and frequency factor
70. At 290 K velocity constant of a reaction was found to be 3.2 10-3. At 300 K, it will be
(a) 1.6 10-3 (b) 6.4 10-3 (c) 3.2 10-4 (d) 3.2 10-2
71. The increase in reaction rate as a result of temperature rise from 10 K to 100 K is ...
(a) 512 (b) 614 (c) 400 (d) 112
72. At 300 K rate constant is 0.0231 min-1, for a reaction. Bt at 320 K rate constant is 0.0693 min-1.
The activation energy of the reaction is
(a) 84 KJ mole-1 (b) 34.84 KJ mole-1 (c) 43.84 KJ mole-1 (d) 30 KJ mole-1
73. The activation energy of a reaction is 9 Kcal mole-1. The increase in the rate constant when its
temperature is raised from 295 to 300 K is approximately
(a) 1.289 times (b) 12.89 times (c) 0.1289 times (d) 25%
74. A reactant A forms two products.
(i) A B activation energy E1
k1

(ii) A C activation energy E2


k2

If E2 = 2E1 then K1, and K2 are related as

(a) K 2 = K1.e E1
E2 E1 E2
(b) K 2 = K1.e (c) K1 = AK 2 .e (d) K1 = 2K 2 .e
RT RT RT RT
75. The activation energys of two reaction are E1 and E2 (E1 > E2). If the temperature of the system
is increased from T1 to T2, the rate constant of the reaction changes from K1 to K21 in the first
reaction and K2 to K21 in second reaction, predict which of the following expression is correct ?

k11 k12 k11 k12 k11 k12 k11 k12


(a) > (b) < (c) = = =0 (d)
k1 k 2 k1 k 2 k1 k 2 k1 k 2
76. The rate of reaction 2x + y Products. Rate = K[x]2[y]. If x is present in large excess, the
order of the reaction is
(a) 3 (b) 2 (c) 1 (d) 0
77. CH3COOEt + H2O H+
CH3COOH + Et OH. Order of reaction is ...
(a) 0 (b) 1 (c) 2 (d) 3
78. In which of the following cases, does the reaction go farthest to completion ?
(a) K = 100 (b) K = 10-2 (c) K = 10 (d) K = 1

219
79. The activation energy of a reaction is zero. The rate constant of the reaction
(a) increase with increase of temperature (b) decrease with increase of temperature
(c) decrease with decrease of temperature (d) is nearly independent of temperature
80. Which of the following is the fast reaction ?
(a) H 2 + Cl 2
H2O
2HCl (b) NO 2 + CO NO + CO 2

(c) CH 3CHO CH 4 + CO (d) 6CO 2 + 6H 2 O


H2O
C6 H12 O6 + 6O 2
81. Oxidation of oxalic acid by acidified KMnO4 is an example of autocatalysis. It is due to which
of the following ?
(a) SO42- (b) MnO42- (c) Mn2+ (d) K+
82. What will be the order of the reaction if doubling the concentration of a reactant increases the
rate by a factor of 4 and trebling the concentration of the reactant by a factor of 9 ?
(a) 1 (b) 2 (c) 3 (d) 0
83. If the half time for a particular reaction is found to be constant and independent of the initial
concentration of the reactants then reaction is of ...
(a) 1 (b) 2 (c) 3 (d) 0
84. The rate of reaction A + B + C Products is given by r = K[A][B] [C]. If A is taken in large
o

excess, the order of the reaction would be


(a) 0 (b) 1 (c) 2 (d) nil
85. Rate of chemical reaction can be kept constant...
(a) by stirring the components (b) by keeping the temperature constant
(c) both of the above (d) none of the above
86. The one which is unimolecular reaction is
1 1 1
(a) HI H 2 + I 2 (b) N 2 O5 N 2 O 4 + O 2
2 2 2
(c) H 2 + Cl 2 2HCl (d) PCl3 + Cl 2 PCl5
87. For the reaction H2(g)+ Br2(g) 2HBr(g) the experimental data suggests, rate = K[H2][Br2].
The molecularity and order of reaction respectively for the reaction is
(a) 2, 2 (b) 2, 1 (c) 1, 2 (d) 1, 1
88. The rate of reaction for Cl3 C CHO + NO CHCl3 + NO + CO is given by equation, rate
= K[Cl3 C CHO][NO]. If concentration is expressed in mole litre-1, the unit of K are
(a) litre2 mole-2 sec-1 (b) mole litre-1 sec-1 (c) litre mole-1 sec-1 (d) sec-1
89. For a reaction 2A + B Products, the active mass of B is kept constant and that of A is
doubled. The rate of reaction will then
(a) increase two times (b) increase four times
(c) decrease two times (d) decrease four times
90. The conversion of A B follows second order kinetics. Doubling the concentration of A will
increase the rate of formation B by a factor of
(a) (b) 2 (c) (d) 4

220
91. Ethyl acetate is hydrolysed in alkaline medium, its order of a reaction and molecularity are
respectively
(a) 1, 1 (b) 1, 2 (c) 2, 1 (d) 2, 2
92. According to the Arrhenius equation a straight line is to be obtained by plotting the logarithm of
the rate constant of a reaction against ...
(a) T (b) log T (c) 1/T (d) log 1/T
93. The given reaction 2FeCl
3 +SnCl2 2FeCl2 +SnCl4 is an example of ________ reaction
(a) first order (b) second order (c) third order (d) none of these

94.
K1
In the reverable reaction 2NO2
N2O4, the rate of disappearance of NO2 is equal to
K2

2K1
(a) K [ NO 2 ]
2
(b) 2k1 [ NO 2 ] - 2k 2 [ N 2 O 4 ]
2

(c) 2k1 [ NO 2 ] - k 2 [ N 2 O 4 ] (d) ( 2k1 - k 2 ) [ NO 2 ]


2

95. If concentration of reactants is increased by x, then rate constant K becomes ..


k k
(a) n (b) (c) k+x (d) k
x x
96. The rate constant is given by equation K = p.z.e-Ea/RT which factor should register a decrease
for the reaction to proceed more rapidly ?
(a) E (b) T (c) Z (d) P
97. For the reaction A + B
k
C. the unit of rate constant is
(a) sec-1 (b) sec-1 mole L-1 (c) sec-1 mole-1 L (d) sec-1 mole-2 L2
98. The rate of the gaseous reaction is equal to K[A][B]. The volume of the vessel is suddenly
reduced to one forth of the initial volume. The rate of reaction would be ...
1 16 1 8
(a) (b) (c) (d)
16 1 8 1
99. For reaction Y2 + 2Z Product, rate controlling step is Y + Z Q. If the concentration
of Z is doubled, the rate of reaction will be
(a) remain the same (b) become four times
(c) become 1.414 times (d) become double
100. The rate law for a reaction given by Rate = K[A]n[B]m. On doubling the concentration of A and
halving the concentration of B, the ratio of the new rate of the earlier rate of the reaction will
be as
1
(a) m+n (b) n-m (c) 2(n-m) (d) 2 ( m + n )
101. The time for half lif of a certain reaction A Products, is one hour. When the initial concentration
of the reactant A is 2 mol L-1 how much time does it take for its concentration to come from
0.50 to 0.25 mole L-1 if it is a zero order reaction ?
(a) 0.25 h (b) 1 h (c) 4 h (d) 0.5 h

221
102. For a first order reaction A Products, the concentration of A changes from 0.1 M to 0.025 M
in 40 minutes. The rate of reaction when the concentration of A is 0.01 M is
(a) 1.73 10-4 M min-1 (b) 1.73 10-5 M min-1
(c) 3.47 10-4 M min-1 (d) 3.47 10-5 M min-1
103. In the reaction 2N2O5 4NO2 + O2, initial pressure is 500 atm and rate constant K is 3.38
10-5 sec-1. After 10 minutes the final pressure of N2O5 is
(a) 490 atm (b) 250 atm (c) 480 atm (d) 420 atm
104. The half life period of a first order reaction is 6.93 minutes. The time required for the completion
of 99% of chemical reaction will be
(a) 230.3 min (b) 23.03 min (c) 46.06 min (d) 460.6 min
105. The rate constants K1 and K2 for two different reactions are 1016.e-2000/T and 1015.e-1000/T
respectively. The temperature at which K1 = K2 is
2000 1000
(a) 1000 K (b) k (c) 2000 K (d) k
2.303 2.303

ANSWER KEY
1 c 26 a 51 c 76 c 101 a
2 b 27 c 52 d 77 b 102 c
3 b 28 b 53 a 78 a 103 a
4 a 29 d 54 b 79 d 104 c
5 c 30 c 55 d 80 d 105 d
6 d 31 d 56 c 81 c
7 a 32 c 57 c 82 b
8 a 33 d 58 a 83 a
9 d 34 b 59 a 84 b
10 d 35 c 60 b 85 d
11 c 36 d 61 c 86 b
12 a 37 c 62 a 87 b
13 b 38 d 63 d 88 c
14 c 39 b 64 c 89 b
15 a 40 c 65 a 90 d
16 c 41 b 66 d 91 d
17 b 42 b 67 a 92 c
18 d 43 c 68 d 93 c
19 c 44 d 69 d 94 b
20 c 45 a 70 b 95 d
21 b 46 c 71 a 96 a
22 d 47 d 72 c 97 c
23 b 48 b 73 a 98 b
24 b 49 b 74 c 99 c
25 c 50 c 75 a 100 c
222
Hints
1. Defination of the rate of reaction

1 d [A] d [ B] 1 d [ C]
2. - =- =+
2 dt dt 3 dt
3. the activation energy of the reaction is large
4. 0.2 104
d [H2 ] 1 d [H2 ] 1 d [ NH3 ]
= - 0.3 10-4 Ms -1 But Rate = - =+
dt 3 dt 2 dt
d [ NH 3 ] 2 d [H2 ] 2
Hence
dt
=-
3 dt 3
( )
= - -0.3 10-4 = 0.2 10-4

1 d [A] d [ B] 1 d [ C] d [ D]
5. \- =- =+ =+
4 dt dt 2 dt dt
6. DH of reaction
7. 0.012 atm min-1
d [ N 2O4 ] 1 d [ NO 2 ]
- =+
dt 2 dt

-
( 0.32 - 0.50 ) = 0.006 = 1 d [ NO2 ] \
d [ NO 2 ]
= 0.012 atm min -1
30 2 dt dt
8. K = K1/K2
1 1 1 1 11
Rate = k [ A ] 2 [ A ] 3 [ c ]
1 1 1
9. \ Order of reaction = + + =
6 2 3 1 6
10. (p+q) = (m+n) or (p+q) (m+n)
11. Rate = K[A][B]2
Keeping [B] constant, [A] is made a 4 times, rate also become 4 times. Hence rate a [A]
Keeping [A] constant, [B] is doubled, rate becomes 4 times. Hence rate a [B]2
\ rate = K[A][B]2
12. rate = K[A]2
[A] doubling, rate becomes four time. Hence rate a [A]2
[B] doubling, no effect on the rate. Hence rate a [B]0
\ rate = K[A]2[B]0
13. concentration of reactants
14. temperature is changed
hn
15. H 2 + Cl 2 2HCl
16. H 2 + Br2 2HBr

223
17. Unimolecular reaction
18. any value between 1 and 3
n
1 k 2 A2 1
19. = Or 2 = (8) n \n =
3 k1 A1 3

four times \rate a [ B]


2
20.

8x V1 = k [ A ] [ B] = x V2 = k [ 2A ] [ 2B] \V2 = 8x
2 2
21.
22. OO \No I2 in the rate law equation.
23. rate of zero order reaction is independent of the concentration of the reacting species
24. many bady collisions have a low probability
25. rate = K [A][B]2 Rate of reaction for slowest step
26. 1.5 From slowest step rate = k [ B2 ][ A ]
From 1st eq. Keq = [ A ] 2 \[ A ] = keq 2 . [ A 2 ] 2
1 1

[A2 ]
1 1 1 1 1
rate = K[B2 ] keq 2
[A 2 ] 2 = k keq 2 [A 2 ] 2 [B2 ] = K1 [A 2 ] 2 [B2 ]
27. C D is lowest
2
B
rate '' = k [ A ][ B] rate '' = k [ 2A ]
2
28. halved
1
1
= k [ A ][ B]
2

2
1
\x '' = x '
2

(1) r = k [ A ] (2) 2r = k [16A ]


n n
29.

2r = K [ A ] 16n
n

2r K [ A ] 6
n n
1
= \2 = 16n \n =
K [A]
n
r 4
30. 2 cocentration increased = 10 times
rate increased = 102 times
\ Order = 2

(i) r = k [ A ] [ B] (ii)8r = k [ 2A ] [ 2B] (iii) 2r = k [ A ] [ 2B]


x y x y x y
31. r = K [A]2[B]

(iii) (i) @ 2 y = 2\y = 1

(ii) (i) @ 2 x = 4\x = 2

224
rate M/s
mole litre-1 sec-1 rate = K [ R ] , K =
n
32. = n =0
[R ]
n
Mn

K = M1- n S-1 \K = M S

rate
33. zero order K = K = rate, when n = 0
[R ]
n

rate bar / s
34. 2 K= = when n = 2 k = bar -1 S-1
[R ]
n
bar n

35. time and square of concentration


rate M /S
K= = = M1- n S-1when n = 3 k = litre3 mole -1 min -1
[R ]
n n
M

rate M /S
36. 4 K= = = M1- n S-1when n = 4 k = liter 3 mole -3 min -1
[R ]
n n
M

37. There is no difference between order and molecularity of a reaction.


38. 3
39. Both order and molecularity of a reaction are always the same.
1
40. a
{
k For Zero order reaction t = k [ A ]o - [ A ] But [ A ]o = a }
a
And when reaction complete [ A ] = 0\t =
k
41. r = k[O3]2[O2]-1 From Slowest step r = k [ O3 ][ O ]

From eq (i) keq = [ O 2 ][ O ] / [ O3 ]

\[ O ] = Keq [ O3 ] / [ O 2 ]2

\r = k [ O3 ] Keq [ O 2 ] = K1 [ O3 ] [ O 2 ]
2 -1

42. Kt = [ Ao ] - [ A ]
43. [ A ] = [ A ]o .e- kt
44. t 1 2 a Co
0.693
45. t 12 =
k
2.303 2.303 ao 2.303 ao
46. log4 t 34 = og = og
k k 3 k ao
ao - ao 4
4

225
47. Zero For Zero order reaction t 12 a Co

48. a1- n t 12 a a1- n


Ao t 12 Ao t 12 Ao
Ao
t 2
\ 3 t 12
1
49. 42 S
2 4 8
T
50. 30 min 100 %
t 12
50
t 12
25% \ T = 2 t 1 2\t 1 2 =
2
M t 12 M t 12 M t 12 M t 12 M
51. 500S
10 20 40 80 180
\ T @ 3 t 1 2 to 4 t 1 2 @ 450 to 600S
1 a
52. After n + t 1 2 amount left = n
16 2
60 1 1
T = n t 1 2 n = = 4 \ Amount left = a =
15 2 16

96 a 10 10
53. 0.63 g T = n t 1 2 n = = 4 \ Amount left = n = 4 = = 0.63
24 2 2 16
2.303 a
54. 2.8 10-4 S-1 K = og = 2.8 10-4 S-1
1.386 60 60 a - 0.75a
55. threshould energy.
56. Collision frequency
57. Positive Activation energy is always Positive
58. Slow
59. an increase in the number of activated molecules.
60. Threshold Energy Energy of the reactants
61. Bimolecular
62. Ef < Er
63. DH of reaction
64. more than DH
65. lo Kcal mole -1 DH = Ea - Ea r \+ 5 = 15 - Ea r \Ea r =10
66. 100 Kj mole -1 DH = Ea - Ea r - 30 = 70 - Ea r \Ea r = 100
67. increasing the temperature.
68. All the above
69. Activation energy and frequency factor.
70. 3.2 104 10k rise, the velocity constant becomes nearly double.
71. 512 Increases of temperature n 10
Increases reaction rate = 29
DT = 100 10 = 90 = 9 10 \ n = 9
\ Increases reaction rate = 29 = 512

226
K2 Ea T2 - T1
43.84 Kj mole og
-1
72. =
K1 2.303R T1T2

0.0693 Ea 320 - 300


og =
0.0231 2.303 8.3 300 320

Ea 20
og 3 = Ea = 43.84
1.901 96000

K2 Ea.DT 9000 5
73. 1.289 times og = = = 0.1104
K1 2.303R T2 T1 2.303 2 300 295

K2 K2
og = 0.1104 = 1.289 K 2 = K1 1.289
K1 K1
E1 - E1 - E2
74. K1 = K 2 A.e RT
K1 = A1.e RT
K 2 = A 2 .e RT

K1 A1 ( E2 - E1 ) / RT E1
= A.e( 1 1 ) = A.e RT
2E - E / RT
= e
K 2 A2
E1
\K1 = K 2 .A.e RT

K11 K12 K11 E1 T2 - T1 K12 E 2 T2 - T1


75. > og = og =
K1 K 2 K1 2.303R T1T2 K 2 2.303R T1T2
Since E1 > E2

K11 K12 K11 K12


\og og >1 OR >
K1 K2 K1 K 2
76. 1 The rate is not depend upon the reactant present in excess
77. 1
78. K = 100
79. is nearly independent of temperature.
- Ea
Ea = 0 \K = A.e RT
= A.eo = A
hn
80. 6CO 2 + 6H 2 O C6 H12 O6 + 6O 2

81. Mn 2+
82. 2 2n = 4 3n = 9 \n = 2
83. 1
84. 1
85. none of the above

227
1
86. N 2 O5 N 2 O 4 + O 2
2
1
87. 2, 1
2
88. litre mole1 sec1
89. increase four times

4 Rate = K [ A ] \rate = K [ 2A ] = 4.K [ A ]


2 2 2
90.

91. 2, 2 CH 3COOC2 H 5 + NaOH CH 3COONa + C2 H 5OH

92. 1
T
93. third order

2K1 [ NO 2 ] - 2K 2 [ N 2 O 4 ]
2 k1
94. For 2NO 2
k
N 2O4 2

1 d [ NO 2 ]
= K1 [ NO 2 ] - K 2 [ N 2 O 4 ]
2
Rate = -
2 dt
-d [ NO 2 ]
= 2K1 [ NO 2 ] - 2K 2 [ N 2 O 4 ]
2
\ rate =
dt
95. K
96. E
97. Sec1 mole1 L \ Second order reaction
16
98. Volume of the vessel is reduced to one foreth
1
Concentration bocomes 4 ttimes

Rate = K [ Y ][ Z] 2
1
99. become 1.414 times

\New Rate = 2.k [ Y ][ Z] 2 = 1.414 K [ Y ][ Z] 2


1 1

m
b
r = k ( 2a )
n
100. 2(n m) r = ka b 1 n m 11

r11 2n a n b m a - m
1
= n m
= 2n.2- m = 2(n - m)
r a b

[ A ]o 2
101. 0.25 h For Zero order reaction K = = =1mol L-1 hr -1
2t 1
2 2 1

[ A ]o - [ A ] 0.50 - 0.25
t= = = 0.25 hr
K 1

228
2.303 0.1
102. 3.47 10-4 M min -1 K = og = 0.03466 min -1
40 0.025

Rate = K [ A ] = 0.03466 0.01 = 3.466 10-4 M min -1


1

2.303 Po 2.303 500


103. 490 atm K= og \3.38 10-5 = og
t Pt 600 Pt

500 500
og = 0.0088 OR = 1.021 OR pt = 490 atm
Pt pt

0.693 0.693
104. 46.06 min K = = = 0.1min -1
t2
1 6.93

2.303 a 2.303 a
t 99% = og = og
k a-x 0.1 a - 0.99a

1
= 23.03 og = 46.03min
0.01
1000 -2000 -1000
105. k k1 = k 2 \1016.e T
= 1015.e T
2.303
-2000 -1000
\10.e T
= 1.e T

2000 1000
\n10 - =-
T T
2000 1000
\ 2.303 - =-
T T
1000
\T = - K
2.303

229
UNIT : 10 SURFACE CHEMISTRY
Important Points
The study of chemistry regarding the boundary separating two bulk states or phases is called
surface chemistry. This boundary surface is known as interface. It is expressed as hyphen
() or slash (/). Dissolution, crystallization, catalysis, metallic corrosion are surface phenomena.

The surface should be completely pure which can be obtained by vacuum generating method
and can be stored also.

In this unit, surface phenomena like adsorption, catalysis, colloid and emulsion are studied.

In adsorption, the substance which is in solid form and on which other gas or liquid is adsorbed
is called adsorbent. The substance that is adsorbed is called adsorbate and the whole phenomenon
is called adsorption. The phenomenon opposite to adsorption is called desorption.

Absorption is such a phenomenon in which there is homogeneous system viz. any coloured
solution but if solid adsorbent like charcoal is added to it then there is decrease in intensity of
the colour which is adsorption. The combined phenomenon of adsorption and absorption is called
sorption. In adsorption the concentration of adsorbate is more than that in the bulk. More porous
the adsorbent more will be adsorption. Adsorption is an exothermic phenomenon.
In adsorption, the residual particles on the surface are responsible for the adsorption that is due
to difference in forces of attraction.
Adsorption is of two types Physical and Chemical. The points of difference between them are
given in the unit.
Adsorption is used in many fields as well as in everyday life viz. To wear gas mask in which there
is adsorbent to save from the poisonous gas like chlorine. Silica gel is used as adsorbent for
keeping the electronic instruments moisture free. In the removal of yellow colour from sugar, the
phenomenon of adsorption is used.

The factors affecting adsorption are (1) nature of adsorbate (2) nature of adsorbent (3) specific
area of adsorbent surface (4) pressure of adsorbed gas (5) temperature. The detailed discussion
about each one is included in the unit.

At constant temperature the graph of pressure of gas adsorbed or concentration is called adsorp-
tion isotherm. There are five different types of isotherms. The study of adsorption isotherm was

x 1 x 1
done by scientist Freundlich and gave the equation = Kp n or log = log K + log p (for
m m n

x 1 x 1
pressure (p)) and = KC n or l log = log K + log C (for concentration (C)) where K and
m m n
n are constants. This was an empirical isotherm and had no scientific base.

230
Langmuir on the basis of kinetic theory of gases gave isotherm equation.

x ap x aC
= , = (where a and b are constants)
m 1 + bp m 1 + bC

The study of Freundlich isotherm can be understood by the study of demonstration experiment in
practicals book. There are many uses of adsorption which are described in the unit.
Catalysis is also a surface phenomenon. Some chemical reactions are slow. To increase their
rates, the substance used in small proportion is called catalyst. This phenomenon is called cataly-
sis. The catalyst is obtained back in original form at the end of the reaction.
There are two types of catalysis (1) Homogeneous and (2) Heterogeneous. In homogeneous
catalysis, catalyst and the reactant are in one phase e.g. hydrolysis of methyl acetate in presence
of H+ (2) In heterogeneous catalysis the catalyst and the reactants are in different phases e.g.
Production of sulphuric acid in presence of V2O5 by contact process. There are many uses of
homogeneous and heterogeneous catalysis as shown in the unit.
The characteristics of catalysis are activity, selectivity that is specific reaction; selection of
specific catalyst e.g. zeolite. The zeolite named ZSM-5 is used to obtain gasoline from alcohol.
Catalyst increases the rate of reaction but does not affect the equilibrium because it affects
equally both the forward and the reverse reaction. Hence, more product is not obtained.
Enzymes are proteins and are necessary for biochemical reaction. For every reaction separate
and specific enzyme works viz. invertase can transform sugar into glucose and fructose. Urease
can decompose urea into ammonia and carbon dioxide.
For enzyme catalysis the lock and key model or induced fit model are proposed. As the lock can
be opened by suitable key, similarly for a particular reaction suitable enzyme will be required. The
enzymes work at the temperature of the body i.e. 298-310 K temperature is considered the best.
Colloid chemistry is also a surface phenomenon. The colloidal solution is called sol. There are two
components called dispersing phase and dispersion medium in it. The particles are of some
particular size so this is a heterogeneous system. Colloids are of two types-Lyophilic and Lyo-
phobic. The colloid which has attraction for solvent (dispersion medium) is used called lyophilic
colloid e.g. gum. The colloid which has repulsion towards the solvent (dispersion medium is called
lyophobic colloid. If water is as a medium then they are respectively called hydrophilic and
hydrophobic. Colloids are of eight types which depend on the dispersing phase and dispersion
medium. This is shown in the unit. Multimolecular, macromolecular and associated colloids are
also known. In associated colloids molecules come nearer and form an association which is called
micelle.

The certain temperature, at which the micelle is formed is called Kraft's temperature (TK). Below
the critical micelle concentration (CMC), it remains in colloidal state and at higher than that
concentration it is changed to solid in the form of precipitates. The formation micelle is obtained
in the cleansing action of soap. Molecules like soap are shown as RCOONa, and their ionization
form will be RCOONa +. From this RCOO, R part combines with organic impurity and drags
inside. It is called tail. The upper charged part COO attracts dust etc. and removes the dirt. It
is called head.

231
The methods of preparation of colloids are as follows :

Condensation method : In these methods, oxidation, reduction, decomposition etc. types of


reactions are associated. In physical methods, excessive cooling is used, and in dispersion meth-
ods-mechanical dispersion (use of colloid mill), electrical dispersion (Bredig arc method) and
peptization are used.

For purification of prepared colloidal solution, a method like dialysis and better method like
electrodialysis can be used. On addition of certain electrolytes to colloid solutions, precipitation
occurs which is called coagulation. The order of concentration for coagulation for iron sol having
positive charge is trivalent > divalent > monovalent for negative ions (anions). Similarly for
arsenious sulphide colloid having negative charge, the order of coagulation remains the same but
positive ions (cations) are used. Amongst the other methods used for purification of colloids are
ultrafiltration and ultracentrifugation.
The properties of colloidal solutions-sol are as follows : (1) Colligative properties (2) Optical
properties (3) Mechanical and (4) Electrical properties.
In colligative properties determination of molecular masses is by osmosis method. In optical
properties- Tyndall effect and in mechanical properties Brownian movement and in electrical
properties, instrument called electrophoresis is used for determination of electrical charge of
colloid. There are two laws given by Hardy and Schulze for the study of coagulation of colloids.
The electrolytes having electric charge opposite to that on the colloid are required for coagulation
of colloid. For colloid having positive or negative electric charge negative or positive ions of the
electrolyte respectively are useful. In concentration, highest concentration of monovalent and less
than that concentration of divalent ion and the least concentration of trivalent produces coagula-
tion.
Emulsions are also colloids, in which both the dispersion medium and the dispersed phase are in
liquid form. They are of two types- Oil / water and water/oil. The examples of water/oil emulsion
are cold cream, butter etc; while in oil/water emulsion the examples are milk, vanishing cream
etc. There are two methods for the test of emulsions (1) Dye test and (2) Dilution test. Demulsification
is the opposite phenomenon.

There are many uses of colloids. Its specific uses are rubber plating, sewage disposal, Cottrell
smoke precipitator, preparation of nano-substances, medicines, as germicides in metallurgy, con-
struction of roads etc. which are described in detail in the unit.

232
M.C.Q.
(1) On which factors interface depends ?
(a) Size of the molecules in the bulk phase.
(b) Wieght of the molecules in the bulk phase.
(c) Numbers of molucules in the bulk phase.
(d) Physical state of molucules in the bulk phase.
(2) Which of the following phenomenon in not involved in surface chemistry ?
(a) Elecrode reactions (b) Dissolution
(c) Heterogeneous catalysis (d) Liquid and its vapour taken in closed vessel.
(3) How much pascal high vacuum is required to achive completely pure surface of metal?
(a) 10-8 to 10-9 (b) 10-8 to 10-10 (c) 10-6 to 10-9 (d) 10-8 to 10-7
(4) Due to adsorption
(a) surface energy increases (b) surface energy becomes zero
(b) surface energy decreases (d) no change occurs in surface energy
(5) Which of the following processes is adsortion phenomenon ?
(a) Soakd of rayon clothes in coloured solution
(b) Contact between silicagel and vapour of water
(c) Contact between anhydrous CaCl2 and vapour of water
(d) H2S gas in contact to water.
(6) Which of the following is good adsorbent ?
(a) Silica gel (b) Alumina (c) Clay (d) All the given
(7) Which type of process adsorption is ?
(a) Isotonic (b) Isochonic (c) Indothermic (d) Exothermic
(8) When adsorption phenomenon occurs complete ?
(a) H < O,G < O,S < O (b) G < O,H < O,S > O

(c) H > O,S > O,G < O (d) G < O,S < O,H > O
(9) Which is correct for adsorption ?
(a) H-T S is negative (b) H is positive

(C) H-T S is positive (d) TS and G=0


(10) Which of the following statement is not true ?
(a) The value of adsorption enthalpy of physical adsorption is less than chemical adsorption.
(b) Physical adsorption occurs due to van der waals forces
(c) Chemical adsorption decreases at high temperacture and low pressure.
(d) Physical adsorption is reversible.

233
(11) At which temperature chemical adsorption occurs ?
(a) At high temperature (b) At very low temperature
(c) At low temperature (d) Temperature does not affect.
(12) Whose value is less than zero during adsoption ?

(a) D G (b) H (c) S (d)All the given

(13) How molecules of gases are deposited on the surface of solid during physical adsorption ?
(a) By electrostatic forces (b) By chemical forces
(c) By gravitational forces (d) By van der waals forces
(14) What is used to prevante electronic instruments clamaged by the moisture ?
(a) Silica gel (b) Zeolite
(c) Chromatographic plate (d) All the given
(15) On which factor adsorption of gas on solid adsorbtion depend ?
(a) On temprature (b) On pressure of gas
(c) On nature of adsorbent (d) All the given
(16) If H2,CH 4,CO2 and NH3 gases are adsorbed by 1 gram charcol at 290 k temperature than
deceasing order of their volume is
(a) H2>CH4>CO2>NH 3 (b) CO2>NH 3>H 2>CH 4
(c) NH 3>CO2>CH4>H 2 (d) CH4>CO 2>NH3>H 2
(17) Which gaseous molecule has highest value of physical adsorption enthalpy ?
(a) H2 (b) N2 (c) H2O (d) He
(18) How magnitude of adsorption gas is expressed ?
m x
(a) (b) m. x (c) (d) x+m
x m
(19) Which is Freundlich adsorption isotherm equation ?
m 1 m n x 1
(a) p n (b) p (c) p n (d) None of these
x x m
x
(20) What will be the value of slope after drawing graph of log log p in Freundlich adsorption
m
isotherm?
1 1 1
(a) (b) (c) (d) -k
p n a
(21) What will be the intercep+ in a graph of Freundlich adsorption isotherm ?
1 1
(a) k (b) log k (c) (d)
a n
(22) Whose value we can get from intercept in the graph of adsorption isotherm ?
1
(a) a and b (b) n and k (c) (d) log k
a
234
(23) Which of the following is not true regarding to Freudlich adsorption isotherm ?
(a) This isotherm is applicable in certain limit of pressure
(b) Constant k and n change with temprature
(c) It shows deviation at low pressure
(d) Freundlich isotherm is empirical, there is no theoritical proof of it.
1
(24) If the value of becomes zero in Freundich adsorption isotherm then adsorption is indepen
n
dent to
(a) pressure (b) temperature (c) quantity (d) a and b
1 x
(25) If the value of is 1 in Freundlich adsorption isotherm then = .............
k n m
(a) (b) kp (c) k (d) none of these
p
(26) On basis of which theory Langmuir derived isotherm equation ?
(a) Thermodynamics (b) Kinetic theory of gases
(c) Collosion theory (d) Wave mechanic theory
(27) Which of the following is Langmuir adsorption isotherm ?
m 1 + bp x ab x ap x ap
(a) = (b) = (c) = (d) =
x ab m 1 + bp m 1 + bp m 1 + bc
(28) How will be Langmuir equation at high pressure ?
x a x x ap x b
(a) = (b) = ap (c) = (d) =
m b m m 1 + bp m a
m 1
(29) What will be the value of slope in graph of according to Langmuir equation ?
x p
1 b a
(a) (b) (c) (d)k
a a b
(30) In Indothermic reaction with the increase of temperature adsorption will be
(a) constsnt (b) increase (c) decrease (d) none of these
(31) Which adsorbent is used in separation of inert gases by Dewars method ?
(a) Vanadium pentoxide (b) Silica gel
(c) Activated charcoal (d) Allumina
(32) Which of the following are adsorption indicators ?
(a) eosin (b) fluorescin (c)methelene blue (d) (a) and (b)
(33) What is not true for catalytic reaction ?
(a) Catalyst increases equally both the rate of forward and reverse reactions.
(b) Catalyst does not effect to equilibrium constant.
(c) Catalyst decreases activation energy
(d) Catalyst increases activation energy of chemical equation.
(34) Which catalyst forms NH3 and CO2 from urea ?
(a) Invertase (b) Celluase (c) Urease (d) Pepcine

235
(35) Which of the following is an example of surface catalysis ?
(a) Inversion of sucrose
(b) Production of ammonia by Habers process
(c) Production of H2SO4 by lead chamber process
(d) Hydrolysis of ester
(36) Which catalyst is used in inversion of sucrose ?
(a) Fe(s) (b) NO (g) (c) H2SO 4 (d) Cl(g)
(37) Which catalyst is used to obtain methanal from water gas ?
(a) Cu (b) ZnO-Cr2O 3 (c) (a) and (b) (d) FeO
(38) Which catalyst is used in the decomption of ozone ?
(a)Cl2(g) (b)Cl(g) (c) O2(g) (d) all the given
(39) Which catalyst is used to prepare propylene oxide from the reaction between propylene and
dioxygen ?
(a) Rh-Pd complex (b) [Rh(CO)2I2]complex (c) Mo(VI)complex (d) Ni-Pd complex
(40) On which factor activity of catalyst depends ?
(a) On the strength of chemical adsorption. (b) On the conecentration of products.
(c) On the concentration of reactants. (d) On the physical state of catalyst
(41) By which name this reaction is also known ?
reaction: 2 SO2 + O2 ( g ) 2 SO3( g )

(a) Shape-selective catalysis (b) Homogeneous catalysis


(c) Enzyme catalysis (d) Surface catalysis
(42) CO(g)+H2(g) x.What is x ?
(a) Methane (b) Methanal (c) Formic acid (d) Formaldehyde
(43) On which factor shape-selective catalysis depends ?
(a) Size of reactant malecules (b) Pore structure of catalyst
(c) Size of product molecules (d) All the given
(44) How many times reaction rate increase by catalyst ?
(a) 1010 to 1020 (b) 102 to 104
(c) 106 to 108 (d) 108 to 1020
(45) What is called colloid system in which dispesing phase and dispersion medium, both are in
solid state ?
(a) Gel (b) Emulsion (c) Sol (d) Aerosol
(46) What is called colloid system in which dispersing phase is gas and dispersion medium is lquid
?
(a) Gel (b) Areosol (c) Emulsion (d) Foam

236
(47) Milk is example of which type of colloid ?
(a)Emulsion (b) Suspension (c) Gel (d)Aerosol
(48) Smoke is which type of colloidal system ?
(a) Gas in salid (b) Solid in gas (c) Gas in gas (d) Gas in liquid
(49) Which of the following is reversible sol ?
(a) Cellulose (b) (c) Mist (d) Gelatin
+ 2+ 3+
(50) Which type of sol sulphar is ? Na > Ba > Fe
Fe3+ > Na + > colloid
(a) Multimolecular Ba 2+ (b) Micelle
(c) Associated colloid (d) Macromolecular colloid
(51) Which of the following is macromolecalar colloid ?
(a) Artificial rubber (b) Protein (c) Nylon (d) All the given
(52) What is called to that temperature at which the formation of micelle takes place ?
(a) Zero temperature (b) Kraff temperatue
(c) Kelvin temperature (d) Absolute temperature
(53) At which condition micelle is formed ?
(a) At concentration higher then critical micelle concentration and lower then kraft temperature.
(b) At concentration higher then critical micelle concentration and higher then kraft temperature.
(c) At concentration lower then critical micelle concentration and lower then kraft temperature.
(d) At concentration lower then critical micelle concentration and higher then kraft temperature.
(54) Which of the following condition is true during the formation of micelle ?
(a) DH = -ve, Ds = -ve (b) DH = +ve, Ds = -ve

(c) DH = +ve, Ds = +ve (d) DH = -ve, Ds = +ve


(55) What is approximate value of CMC for saap ?
(a) 10-9M to 104M (b) 10-3M to 10-4M (c) 10-9M to 10-14M (d) 103M to 105M
(56) Which of the following is physical method for the preparation of collodiad sol ?
(a) coagulation (b) peptization (c) fusion (d) excessive cooling
(57) Which sol is formed due to hydrolysis of FeCl3 ?
(a) FeCl2 (b) Fe(OH)2 (c) Fe2O 3 (d) Fe(OH)3
(58) Which of the following is double deamposition ?
(a) SO2+2H2S 3S+2H2O (b) FeCl3+2H2O Fe(OH)2+3HCl
(c) As2O3+3H2S As 2S3+3H2O (d) All the given
(59) By which method As2S3 sol can be obtain by the reaction between As2O3andH2S ?
(a) Redution (b) Oxidation (c) Hydrolysis (d) Double decomposition
(60) In which of the following method condensation and dispersion are associated ?
(a) Excessive cooling (b) Hydrolysis (c) Bredigs are (d) Peptization

237
(61) Which method is is used to obtain sol of gold and silver ?
(a) Electric dispersion (b) Peptization (c) Excessive cooling (d) Mechanical dispersion
(62) Whose menbranes are used in dialysis ?
(a) Parchment paper (b) Plastic (c) Filter paper (d) Ultrafilter paper
(63) ...............Phenomenon is called reverse to coagulation ?
(a) Flocculation (b) Tyndall (c) Brownian (d) Dialysis
(64) Which of the following is correct order of coagulations for the coagulation of As2S3?
(a) Fe3+ > Ba 2+ > Na + (b) Na + > Ba 2+ > Fe3+

(c) Fe3+ > Na 2+ > Ba 2+ (d) Ba 2+ > Na + > Fe3+


(65) Which of the following is correct order of coagulation ions for the coagulation of Fe(OH)3?
(a) C1- > SO42- > PO43- (b) PO43- > SO42- > C1-

(c) SO42- > C1- > PO43- (d) SO42- > PO43- > C1-
(66) Which is correct order of coagulation of ion necessary for coagulation of calloid sol ?
(a) Monovalent ion < divalent ion <trivalent ion
(b) Divalent ion<trivalent ion<monovalent ion
(c) Trivalent ion<monovalent ion<divalent ion
(d) Trivalent ion<divalent ion<monovalent ion
(67) Which of the following substances contain negative charge in their clloidal solution ?
(a) Arsenious sulphide (b) Platinum
(c) Gold-Silver (d) All the given
(68) Which peptizing agent is used to obtain sol of Fe(OH)3 ?
(a) HCl (b) FeCl2 (c) KCl (d) FeCl3
(69) By the use of which colligative property molecularmass of colloid can be determine ?
(a) Decrease in vapour pressure (b) Elevation in boiling point
(c) Depression infeezing point (d) Osmotic pressure
(70) Which of the following is oil/water emulsion ?
(a) Milk (b)Vanishing cream (c) Butter (d) All the given
(71) Which of the following substance is used as stabilizer in emulsion ?
(a)Protein (b) Gum (c)Agar (d) All the given
(72) Tyndall effect is associated with which property of colloid ?
(a) Mechanical (b) Colligative (c) Optical (d) Electrical
(73) Which method is used for demulsification ?
(a) Sulimation (b) DIstillation (c) Filtration (d) Centrifugation
(74) Surface-tention of lyophobic sol is
(a) less then water (b) greater then water(c) equal to water (d) can not be predict

238
(75) for the preparation of which substances reversible micelle is used ?
(a) Medicines (b) Nano products (c) Rubber plating (d) All the given
(76) Tyndall effect is shown by
(a) Ideal solution (b) Colloidal solution (c) Saturated solution (d)True solution
(77) Which of the following can prepare cationic micelle ?
(a) Sodium stearate (b) Cetyltrimethyl ammonium bromide
(c) Urea (d) Sodium dodesylsulphate
(78) When the rate of physical adsorption will increase ?
(a) By reducing pressure (b) By reducing temperature
(c) By increasing temperature (d) (a) and (c)
(79) According to the adsorption theory of catalysis, the speed of the reaction increases because
(a) Adsorption produces heat which increases the speed of the reaction.
(b) Adsorption lowers the activation energy of the raction
(c) The concentration of reactant molecules at the active centres of the catalyst becomes high
due to adsorption.
(d) In the process of adsorption, the activation energy of the molecules become large.
(80) Which of the following property is true ?
(a) The particles of colloid can diffuse slowly through semipermeable membrance
(b) The particles of colloid can not be settle down under the influence of gravitational force.
(c) The particle of colloid can not be observed by ultramicroscope.
(d) All the given.
(81) In a case of auto-catalysis
(a) reactant act as a catalyst (b) heat ivolved during reaction, act as a catalyst
(c) product act as a catalyst (d) solvant act as a catalyst
(82) When light passes through collodial solution it.......
(a) is reflected (b) is refracted (c) is scatterd (d) does not affect
(83) Which of the following is least effective for flocculation >
(a) k 2 CrO 4 (b) KBr (c) K 2SO 4 (d) K 3 [Fe ( CN )6 ]
(84) 0.25g of starch sol is required to prevent coagulation of 10ml gold sol when 1 ml of 10%
NaCl solution is present. What is gold number of starch sol ?
(a) 0.25 (b) 2.5 (c) 250 (d) 0.025
(85) Which of the following is appropriate ?
(a) Emulsion - butter milk (b) Aerosol - smoke
(c) Foam - fogs (d) Solid sol - cake
(86) Which of the following has minimum value of coagulation for AS2S3 ?
(a) Kcl (b) Bacl2 (c) Nacl (d) Alcl3
239
(87) Surface tention of lyophilic sol is
(a) greater than H2O (b) equal to H2O (c) less than H2O (d) none of these
(88) Which of the following is an example of homogeneous catalysis ?
(a) Acidic hydrolysis of methyl acetate
(b) Catalytic coversion in methanol from watergas
(c) Catalytic coversion in SO3 from SO2 by contract process
(d) Synthesis of NH3 by Habers process
(89) In peptiration reaction
(a) colloid coverts in precipitates
(b) precipitates converts in calloid
(c) true solution is formed from suspention particels.
(d) true solution is formed by dissolving precipitates.
(90) From which of the following effect colloidal system is free ?
(a) gravitational (b) Concentration of electrolyte
(c) heat (d) applied electrical field.

ANSWER KEY
1 A 16 C 31 C 46 D 61 A 76 C
2 D 17 C 32 D 47 A 62 A 77 C
3 A 18 C 33 D 48 A 63 A 78 A
4 B 19 C 34 C 49 D 64 A 79 B
5 B 20 B 35 B 50 A 65 C 80 D
6 D 21 B 36 B 51 D 66 D 81 C
7 D 22 B 37 C 52 B 67 D 82 C
8 A 23 C 38 B 53 B 68 D 83 B
9 A 24 A 39 C 54 C 69 D 84 C
10 C 25 B 40 A 55 B 70 A 85 C
11 A 26 B 41 D 56 D 71 D 86 D
12 D 27 C 42 D 57 D 72 C 87 B
13 D 28 A 43 D 58 C 73 D 88 A
14 A 29 A 44 D 59 D 74 A 89 B
15 D 30 B 45 C 60 C 75 B 90 C

240
UNIT : 11 CLASSIFICATION OF ELEMENTS AND
PERIODICITY IN PROPERTIES
Important Points
Modern Periodic law: The physical and chemical properties of elements are periodic function
of their atomic numbers .
The chemical properties of elements are governed by the number of electrons in the outermost
orbital of atom. Elements with similar electronic a configuration posses similar properties.
Modern Periodic Table: The two terms used to describe the periodic table are period and
group.
v Period: The horizontal rows of the periodic table are known as periods. Each period
starts with filling up of a new quantum shell and continues till the p-orbital of the same
shell is filled up. There are seven periods in the modern periodic table.
v NUMBER OF ELEMENTS IN THE DIFFERENT PERIODS
Period Number Orbitals being filled up Number of elements
1 1s 2
2 2s 2p 8
3 3s 3p 8
4 4s 3d 4p 18
5 5s 4d 5p 18
6 6s 4f 5d 6p 32
7 7s 5f 6d 7p Incomplete

Group: The vertical columns of the periodic table are known as groups. There 18 groups; which
are numbered 1to 18 according to IUPAC recommendations. Elements of same group have
same electronic configuration.
GP-1 Alkalimetals Electronic Configuration ns2 np1 6
GP-2 Alkaline Earthmetals GP-16 Chalcogens
Electronic configurations GP-17 Halogeus
p
S
H 13 14 15 16 17 18
1s 1 2
He
2s Li Be Electro Configuration :
D Inner Transition Metal 2p B C N O F Ne
3s Na Mg (n 1)d 1 10 ns1 2
3 4 5 6 7 8 9 10 11 12 3p Al Si P S Cl Ar
4s K Ca 3d Sc Ti V Cr Ma Fe Ce Ni Cu Zn 4p Ga Ge As Se Br Kr
5s Rb Sr 4d Y Zr Nb Mo Tc Ru Rh Pd Ag Cd 5p In Sn Sb Te I Xe
6s Cs Ba 5d La Hf Ta W Re Os Ir Pt Au Hg 6p Tl Pb Bi Po At Rn
7s Fr Ra 6d Ac Rf Db Sg Bh Hs Mt Ds Uuu Uub 7p - Uuq - Uuh - -

241
Trends in different properties down the group
Property Trend Reason Exception
1) Atomic Number Increases in number of protons in
effective the nucleus of atoms

2) Effective nuclear
Charge

3) Atomic radii The Principal quantum number In transition metal size increases
increases from 1st member to 2nd but size of
Effective nuclear charge is almost 2nd & 3rd are almost equal
constant
Nuclear force of attraction
decreases

4) Transition enthalpy The Principal quantum number (i) Tl > In


increases (ii) Pb > Sn
The nuclear force of attraction on (iii) In transition element decreases
valence electrons decreases from 1st to 2nd member but I.E of
3rd member is higher than 2nd
member

5) Electron gain Atomic size increases therefore Cl > F


enthalpy distance of valence electron from
nucleus increases and nuclear
force of attraction decreases

7) Electron negativity

8) Metallic Character Increase in atomic size, transition


enthalpy tendency to loose
electrons

9) Non metallic Electron gain enthalpy decreases


character
10)Reactivity of metals I.E decreases, decrease the gp Ag, Au, P+

11) Reducing property I.E decreases, decrease the gp (i) Li strongest


of metals Tendency to donate electrons (ii) Au, Hg, Tl, Bi, W, Re & Pb are
less stronger than Ag, Cd, In, Sb,
Mo, Tc & Sn

12) Reactivity of non- Electro negativity decreases< gp


metals decreases

13) Oxidising property Tendency to loose electrons


of non metals

14) M.P & B.P of Lattice enthalpy decreses


metals

15) M.P & B.P of non- Molecular solids


metals

246
Quick glance of properties across the period & down the group
Property Across the Period Down the Group
(1) Effective nuclear charge - -
(2) Atomic Size Constant
(3) Ionisation Enthalpy -
(4) Electron gain Enthalpy -
(5) Electronegativity -
(6) Metallic property -
(7) Oxidising Agent -
(8) Reducing Agent -
(9) Basic character of oxides -
& hydroxides
(10) Acidic character of oxides -
& hydroxides
(11) Thermal stability of -
carbonates, nitrates etc.
(12) Density First - then

Important points to remember


(1) Liquid element Br, Hg, Ga, Cs, Fr
(2) Solid non metal Idodine I
(3) Lightest Metal Li
(4) Heaviest & Highest O.S. Os
(5) Hardest element W
(6) Metalloids B, Si, As, Te
(7) Lowest electronegativity Cs
(8) Highest electronegativity F
(9) Highest DiH He
(10) Lowest DiH Cs
(11) Highest DegH Cl
(12) Highest electronegativity F
(13) Strongest oxidising agest F
(13) Strongest Reducing agent Li
(14) Most reactive liquid metal Cs

247
M.C.Q.
1. The most electronegative element possess the electronic configuration
a) ns 2 np 2 b) ns 2 np 4 c) ns 2 np 5 d) ns 2 np 3
2. The maximum number of electrons in d orbital of an element with atomic number 46 is
a) 10 b) 18 c) 20 d) 19
3. The ionisation enthalpy of Cs is 375.6KJmol1< what is the energy required to convert
[at mass of Cs = 133] 2.66mg o f gaseous Cs completely to Cs+
a) 7.512 J b) 7.512 KJ c) 7512.2 J d) 18782 J
4. The atomic num ber of elements M, N, & P are x, x1, x3. If P is a halogen atom then
the type of bond between N & P is
a) Covalent b) Ionic c) Coordinate d) Metallic
5. In the above question (Question 4) the formula of M & P is
a) MP b) M2 P c) MP 2 d) M2P 3
6. The elem ents X. Y. & Z h ave 2, 3 & 4 electrons in th e o utermo st orb ital respectively.
The element which form most basic oxide is
a) X b) Y c) Z d) None of the above
7. Elem en ts A, B, C & D b elo ng to the 17th gro up . If th e atom ic nu mb ers are y , y x,
y + 4x + 4 & y + 2x + 2 (x = 8) > the element which is violet solid is
a) C b) A c) B d) D
8. An element X belongs to Gp16 & 5th period. Its atomic number is
a) 34 b) 50 c) 52 d) 85
9. The position of an element with atomic number 114 is
a) Period 6 gp 14 b) Period 6 gp 16 c) Period 5 gp 18 d) Period 7 gp 14
10. The total no. o f electrons in the outermost o rbital in element A, B, C, D are 2, 1, 4, & 6
respectively. The elements which belongs to chalcogens is
a) B b) C c) D d) A
11. The io nic radii of isoelectron ic sp ecies are fo un d to be 171p m, 136p m & 140pm
respectively. The isoelectronic species are __________
a) N 3 , O 2 , F b) F , O 2 , N 3 c) O 2 , N 3 , F d) N 3 , F , O 2
12. The size of Mo is very similar to W due to_______
a) Shielding effect b) Actinide contraction
c) Poor Shielding by 4f electrons d) Poor shielding by 4d electrons
13. Choose the correct order ionization energy
a) N > O > F b) F > O > N c) N > O < F d) O > F > N

248
14. The order of ionization energy of K, Ca, & Ba are
a) K > Ca > Ba b) Ca > Ba > K c) Ba > K > Ca d) K > Ba > Ca
15. The element with zero electron gain enthalpy is
a) Argon b) Lithium c) Calcium d) Fluorine
16. Pick the iso electronic species from the following
I. NH 3 II. NH 2
III. CH + 3 IV. H 3 O +
a) ii, iii, iv b) i , ii , iv c) i , ii , iii , iv d) i & iv
17. The element with atomic number 44 belongs
a) d Block b) pBlock c) s Block d) fBlock
18. In the third period there are only eight elements because
a) It is a short period b) The 3d orbitals are absent
c) The d orbitals are absent
d) When n=3, the maximum number of electrons which can be accommodated are eight
19. Cho ose the correct electron ic co nfiguration which has the highest difference between
first & second ionisation enthalpies.
a) 1s 2 2s 2 2p 6 3s 2 3p 6 4s 1 b) 1s 2 2s 2 2p 6 3s 2 3p 6 4s 2
c) 1s 2 2s 2 2p 4 d) 1s 2 2s 2 2p 3
20. The set of quantum numbers for unpaired electron of an element with atomic number 84
are
a) N= 6 , l = 1 , m = +_
1 , ms = +_
1/2 b) N= 5 , l = 3 , m = 0 , m s = +_
1/2
c) N= 6 , l = 0 , m = 0 , m s = +_
1/2 d) N= 6 , l = 3 , m = 2 , m s = +_
1/2
21. The elements with highest ionization enthalpy in a period are
a) Alkaline earth metals b) Halogens
c) Noble gases d) Lanthanides
22. Choose the species which is not isoelectronic
a) Bo 33 b) Co 32 c) No 3 d) So 32
23. The formation of Mg2+ is as follows
I. Mg (g) Mg+ (g) + e 737 KJ mo l 1
II. Mg +(g) Mg 2+(g) + e 1450 KJ mo l 1
The energy required in the second steps is higher because
a) Mg+ is more electropositive b) Mg+ has larger size than Mg
c) Mg+ tends to loose only one electron d) Mg+ has smaller size than Mg

249
24. The first 2nd and 3rd ionizatio n enthalpies o f galliu m are 579KJmo l1 ,1979KJmo l1 &
2962 KJmol1 even though the iii I.P is highest Ga3+ is the most stable because
a) The energy loss is maximum resulting greater stability
b) The size of Ga3+ is smallest
c) Ga 3+ is most reactive
d) It attains a stable configuration
25. The electronic configuration of M3+ is [ Kr ] 4d 10. Its position in the periodic table is
a) Period 4 gp 8 b) Perio d 5 gp 13 c) Perio d 4 gp 18 d) Perio d 5 gp 16
26. The electronic which will exhibit maximum no. of oxidation states
a) 1s 2 2s 2 2p 6 3s 2 3p 5 b) 1s 2 2s 2 2p 6 3s 2 3p 6 4s 2 3d 5
c) [Xe] 4f14 5d 6 6s 2 d) [Ar] 4s 2 4p 4
27. Choose the incorrect order w.r.t properly indicated
a) Electro negativity F > Cl > Br b) Electron affinity Cl > F > Br
c) Oxidizing power F2 > Cl2 > Br2 d) Bond en thalpy F2 > Cl2 > Br2
28. Choose the correct statement
a) As shielding effect increases electro negativity decreases
b) As shielding effect increases electro negativity increases
c) As ionization potential increases metallic property increases
d) As +ve charge on species increases ionic radii increases
29. The electronic configuration which contain metals, non metals & metalloid is______
a) ns 1 & ns 2 b) ns 2 , ns 2(n1)d (110)
c) ns 2 np 6 & ns d) ns 2 np 4 & ns 2 np 5
30. The group in which all the three physical states (solid ,liquid, gas) are observed is
a) gp 17 b) gp 14 c) gp 18 d) gp 15
31. The element which exhibits highest oxidation number is
a) Mn b) Os c) Fr d) I
32. Fo ur elem en ts A, B, C & D. D is n on reactive gas. C is a high ly reactive gas, B is a
so lid & forms ox ide, A is high reactive solid & used to p rep are Lasagne s so lutio n.
Choose the correct sequence of possible atomic no. of elements
a) 12, 18, 9, 11 b) 11, 36, 9, 20 c) 20, 36, 11, 9 d) 9, 18, 11, 20
33. The element with highest electronic affinity belongs to
a) Period 1 gp b) Period 3 gp 17 c) Period 2 gp 17 d) Period 2 gp 16
34. The atomic no. of B = atomic of A+18 , Statements A & B to
a) Same pd & same gp b) Same pd but different gp
c) Different pd but same gp d) Different pd and different gp
250
35. Element B occupies 3rd pd & gp 16
Element C occupies 4 pd & gp 3
The molecular formula of compound formed between B & C is
a) B3C 2 b) C 2B 3 c) CB2 d) B2 C
36. Choose the correct statement w.r.t oxidising property of F
a) It is the strongest oxidising agent because it has highest electron gain enthalpy
b) It is the strongest oxidising agent due to its small size
c) It is the strongest oxidising agent because it has maximum electron negativity
d) It is the strongest oxidising agent due to high lattice enthalpy
37. The name of th e scien tist wh o disco vered the elem ent Unu & its accep ted IUPAC
nameis
a) Mendeleev &Mendelinium b) Seaborg & Seaborgium
c) Mendeleev &Dubinium d) G.T.Seaborg & Mendelinium
38. Which of the following property does not indicate the periodicity of elements
a) Ionization potential b) Neutron/ proton Ratio
c) Bonding behaviour d) Electron negativity
39. Properties of Li are similar to Mg because
a) The size of Li & Mg are different b) The size by charge ratio is similar
c) The charges are same d) Both are reactive
40. From the given set of quantum num bers for th e last electro n o f the ato m, ch oose the
elemen t which is a n on metal. The set of Quantu m nu mbers o f A, B, C & D are given
below
A n =2,l=1,m =0, +_
1 B n=4,l=0,m=0
C n=5,l=2,m= +_
2 D n=6,l=3,m=0
a) D b) C c) B d) A
41. Be shows diagonal relationship with
a) Mg b) Al c) B d) Na
42. Which of the following ions are not isoelectronic with Ar
a) Na + b) Ca+2 c) Cl d) K +
43. The ionisation potential of N > O because
a) Ionisation potential increases with decrease in size
b) N posses stable half filled porbital
c) The screening effect in N > O
d) O is more electropositive than N

251
44. The physical properties of chromium is most closely related to
a) Niobium b) Tungsten c) Titanium d) Calcium
45. The electronic configuration of an element of chalcogen family is
a) [Ar] 3d 10 4s 2 4p 1 b) [Ar] 3d 10 4s 2 4p 4
c) [Ar] 3d 10 4s 2 4p 3n d) [Ar] 3d 10 4s 2 4p 2
46. Choose the incorrect statement
a) An element with high electronegativity always has high electron affinity
b) Electron gain enthalpy is the property of an isolated atom
c) Electronegativity is the property of a bonded atom
d) Both electronegativity & electron affinity are equally proportional to nuclear charge
& inversely proportional to atomic size
47. Choose the ox ide which is most basic CuO, MgO, Al2O 3
& K2 O
a) K 2 O b) MgO c) CuO d) Al2 O 3

48. An elemen t with atomic nu mber 19 will mo st read ily react with the elem ent who se
atomic number is
a) 18 b) 21 c) 20 d) 17
49. If graph is drawn between electro n en thalpy & ato mic num ber from 1 to 60, wh ich of
the following statement will be true
a) Alkali metals are at the maxima & noble gases at the minimum
b) Alkali metals are at the minimum & noble gases at the maxima
c) Transition elements at maxima
d) Maxima & minima are not observed
50. In a period with increase in atomic number, the metallic character of an element
a) Decrease across p d & increases in gp
b) increase across pd & decreases in gp
c) increase across pd & increases in gp
d) Decrease across p d & decreases in gp
51. In elem en t P with electron ic co nfiguration [Ar] 4s1 will com bine with an elem en t of
________ configuration to form a highly soluble ionic solid with high melting point
a) [Ar]4s 2 b) [Ne]3s 2 3p 3 c) [Ne] 3s 2 3p 5 d) [Ar] 4s 2 3d 2
52. In group 14 the lower oxidation state becomes more stable down the group. The reason
is
a) Inert pair effect b) Decreases in ionisation potential
c) Metallic character increases d) Decrease in electron affinity

252
53. Choose the correct option. Hint T=true F = False
I. In the second period atomic radii of Be is 90pm, F is 64pm, & that of Ne is 160pm
II. Atomic radii decreases from Li to Ne
III. The increase in size of Ne is due to presen ce of van derwaals force of attractio n &
presence of covalent bond
IV. In Ne there is absence of covalent bond therefore the radii is vanderwaals radii
V. The order of radii is Metallic > Covalent > Vanderwaals
a) TTFTF b) TTTFF c) TFFTT d) TFFFT
54. Choose correct option
I. Ionisation enthalpy 1/shielding effect
II. Ionisation enthalpy Chemical reactivity
III. Ionisation enthalpy 1/Metallic character
IV. Ionisation enthalpy Effective nuclear charge
a) TFFT b) FFTT c) TTTF d) TFTT
55. Choose the correct option
I. C < N < F < C Second ionisation potential
II. d 5 < p 3 ; d 10< p 6 Half filled order of stability & fully filled orbitals
III. Al2O 3 < SiO 2 < P 2O 3 < SO 2 Acid strength
IV. M 3+ > M2+ > M > M2 Atomic/Ionic radii
a) TFTT b) TTTF c) TTFT d) TTTT
56. Choose the correct option
I. Cs + is the most hydrated than other alkali metal
II. Among the alkali metals, Li has the highest M.P
III. Li is the strongest reducing agent because of low ionisation enthalpy
IV. Li is the stro ngest red ucing agen t becau se th e high io nisatio n p otential is
compensated by high hydration enthalpy
V. Li is resemble to Al
a) FTFTF b) TTFTF c) FFFTF d) TTTFF
57. Choose the correct option
I. NaCl < NaI < NaF < NaBr Io nic character
II. Si < P < C < N Electron egativity
III. BeCl2 < MgCl2 < CaCl2 < BaCl2 Ion ic character
IV. Al3+ < Mg2+ < Na + Ionic mobility
a) FTTF b) TFFT c) FTTT d) FFTT

253
58. Choose the correct option
I. Transition metals are characterised by variable oxidation state
II. Elements of IB & IIB are transition elements
III. Elements of gp1 exhibit only +1 O.S
IV. Group 17 contains only gases
a) TTFF b) TFTF c) TTTF d) TTTT
59. Choose the correct option
I. The ionisation enthalpy of Be > B
II. dBlock elements are known as representative elements
III. Palladium is the only element of fifth period th at h as no electron in fifth energy
level
IV. The second ionisation enthalpy of Al is greater than that of Mg
V. Among Li, Be, B ,C N ; Li has least value of electron gain enthalpy
a) TFTFT b) TFFTT c) TFFFT d) TFTTT
60. Choose the correct option
I. The last electron in case of inner transition elements goes to forbital
II. The electron affinity is highest for fluorine
III. Metallic radius is smaller than covalent radii
IV. Ar has lesser ionisation enthalpy than K
a) TFFT b) TFFF c) TTTF d) TTFF
61. Choose the correct option
I. All halogens exhibit variable oxidation state
II. sBlock elements do not exhibit variable oxidation state
III. the most stable oxidation state of Bi is +3
IV. N exhibits 3, +3 & +5 oxidation state
a) TFTT b) TFFT c) FTTT d) FTTF
62. Choose the correct option
I. O.S o f O in OF 2 is 2
II. Ionisation enthalpy is the minimum amount of energy required to remove an electron
from an atom
III. Screening effect : it is the attraction of electron towards the nuclear
IV. Half filled orbitals are more stable half fully filled orbitals
a) TTTT b) FFFF c) TTFT d) TFFT

254
Match the following
63. Set A Set B
1. Strongest reductant p) silver
2. Fully filled dorbital q) Berkelium
3. Noble metal r) Co pper
4. Actinide s) Iodide ion
t) Sodium ion
a) 1s, 2r, 3p, 4q b) 1t, 2r, 3p, 4q c) 1s, 2t, 3q, 4p d) 1t, 2s, 3q, 4p

64. Set A Set B


1. Liquid non metal p) Lightest metal
2. Metal stored in paraffin q) Cs
3. Most electropositive metal r) KOH
4. Strongest alkali s) K
t) group 17
u) CsOH
a) 1s, 2q, 3s, 4r b) 1t, 2p, 3s, 4r
c) 1t, 2q, 3q, 4r d) 1t, 2p, 3q, 4u
65. Set A Set B
1. C O I) Basic oxide
2. CO 2 K) neutral oxide
3. K 2 O L) Amphoteric oxide
4. Al 2 O 3 M) acidic oxide
5. SiO 2 O) Neutral
a) 1 K, 2 & 5 M, 3 J, 4L b) 1, 2 & 5 M, 3 J, 4L
c) 1K, 2 M, 3L, 4J d) 1J, 2 & 5 M, 3 K, 4L
66. Set A Set B
1. Osmium p) Hardest metal electric
2. Lead q) poor conductor of current
3. Tungsten r) largest size
4. Caesium s) most reactive solid matter
t) highest oxidation state
a) 1 t, 2q , 3p , 4s b) 1 t, 2q , 3p , 4r
c) 1 t, 2s, 3q , 4t d) 1 t, 2q , 3s, 4r

255
67. Set A Set B
1. Diagonal relationship q)attraction towards nucleus
2. Shielding effect r) charge on the nucleus available for other electrons
3. Effective nuclear charge s) similar polarising power
t) Ionisation enthalpy decreases
a) 1s, 2t, 3r b) 1t, 2s, 3r c) 1r, 2s, 3t d) 1s, 2r, 3t
68. Set A Set B
1. Br p) Chalcogen
2. Ba q) alkali metal
3. Se r) alkaline earth metal
4. Rb s) Halogen
a) 1p, 2r, 3s, 4q b) 1 s, 2r, 3p , 4q
c) 1s, 2r, 3q, 4p d) 1 s, 2p , 3r, 4q
69. Set A Set B
1. Hg p) Liquid nonmetal
2. Carbon (Diamond) q) reacts very violently
3. Bromine r) reaction endothermic
4. Caesium & F s) Liquid metal
t) extremely high M.P
a) 1s, 2r, 3p, 4q b) 1 t, 2s, 3p , 4r
c) 1 s, 2t, 3p , 4q d) 1 s, 2t, 3p , 4r
70. Set A Set B
1. Inner transition elements p) 3rd perio d
2. Transition q) s & p Block
3. Typical element r) d Block
4. Representative element s) fBlock
t) pBlock
a) 1r, 2s, 3p, 4q b) 1 s, 2r, 3p , 4q
c) 1q, 2r, 3s, 4t d) 1 s, 2r, 3t, 4q
71. Set A
1. Be < Al p) noble gases
2. Aufbau principle q) pBlock
3. ns 2 np 15 r) Diagonal relationship
4. ns np2 6
s) Block deciding rule
a) 1r, 2s, 3q, 4p b) 1 s, 2r, 3p , 4q
c) 1q, 2r, 3q, 4p d) 1 r, 2q , 3s, 4p
256
72. Set A (Atomic no.) Set B (Position of element)
1. 100 p) dBlock
2. 50 q) sBlock
3. 40 r) lanthanides
4. 11 s) Actinides
a) 1 t, 2s, 3p , 4q b) 1 r, 2s, 3p , 4q
c) 1 t, 2p , 3s, 4q d) 1 r, 2s, 3q , 4p
73. The position of element A, B, C & D are
Element Period Group
A 4 2
B 3 13
C 3 16
D 4 16
74. The molecular formula of the oxide of each element in its highest state are
a) AO 2, B 3O 2, CO, DO b) AO, B2 O 3, CO, DO
c) A 2O, B 2O 3, CO 2, DO 2 d) AO, B2 O 3, CO 3, DO 3
75. With reference of above questio n the oxide which is (i) m ost ion ic (ii) ampho teric (iii)
highest M.P (iv) reacts most readily acid only
a) AO,BO,AO,AO b) AO,CO,BO,AO
c) BO, AO, AO, DO d) DO, AO, BO, CO
Assertio n rea son type
7585 are assertion reason type for each question select the correct cho ice from
the following
a) Statement 1 is true, statement 2 is true & is correct explanation for statement 1
b) Statement 1 is true, statement 2 is true but is not correct explanation for statement 1
c) Statement 1 is true, statement 2 is false
d) Statement 1 is false, statement 2 is true
75.
1. F atom has lesser electron affinity than atom
2. The size of F is very small therefore electron electron repulsion is high
76.
1. The size of X+,X & O are id entical
2. The rem oval of electro n decreases th e size while ad ditio n o f e increases the
size.This is primarily due to decreases increases in electronic repulsion

257
95. Choose the option in which the order is not in accordance to the property indicated.
(a) Al3+ Mg 2+ Na + F- (Increasing ionic size)
(b) B C N O (Increasing first ionisation enthalpy)
(c) I Br FCl (Incresing negative electron gain enthalpy)
(d) Li Na K Rb (Increasing metallic radius)
96. Choose the wrong order
(a) NH 3 PH 3 A 5 H 3 (Acidic)

(b) Li Be B C DiHi

(c) Al2 O 3 MgO Na 2 O K 2 O (Basic)

(d) Li + Na + K + CS+ (Ionic Radius)


97. Position in perodic table
Element Period Group
A 3 2
B 7 10
C 2 16
D 5 13

I. the atomic number of B is


A) 104 B) 108 C) 110 D) 105
II. the type and nature of compound form between A & C is
A) Sulphide and basic B) Oxide and amphoteric
C) Sulphide and neutral D) Oxide and basic
III. Element D is
A) Metal B) Metalloid C) Nonmetal D) Liquid
(a) 1-B, 2-C, 3-A (b) 1-D, 2-A, 3-C (c) 1-C, 2-D, 3-B (d) 1-A, 2-D, 3-A
98. The properties of elements are given below
Element Property
B Liquid and forms strongest alkali
C Nonmetal and shining crystal
D A metal used as catalyst with exceptional electronic configuration
I. The element B is
A) Cs B) Ga C) Fr D) Na

260
ANSWER KEY
1 c 18 b 35 b 52 a 69 c 86 b
2 c 19 a 36 c 53 a 70 b 87 d
3 a 20 a 37 d 54 d 71 a 88 c
4 b 21 c 38 b 55 b 72 a 89 c
5 c 22 d 39 b 56 a 73 d 90 a
6 a 23 d 40 d 57 c 74 a 91 d
7 a 24 d 41 b 58 c 75 a 92 b
8 c 25 b 42 a 59 d 76 d 93 c
9 d 26 c 43 b 60 b 77 b 94 c
10 b 27 d 44 c 61 c 78 d 95 b
11 d 28 a 45 b 62 b 79 a 96 b
12 c 29 d 46 a 63 a 80 b 97 c
13 c 30 a 47 a 64 d 81 d 98 d
14 b 31 b 48 d 65 a 82 b 99 a
15 a 32 c 49 b 66 b 83 a 100 c
16 b 33 b 50 a 67 a 84 c
17 a 34 c 51 c 68 b 85 b

262
UNIT : 12 GENERAL PRINCIPLES AND
PROCESSES OF ISOLATION OF METALS
Important Points
Occurrence of elements: Highly electropositive metals of S-block occur in nature in combined state with
several anions. The lesser electropositive metal like chloride, silicate, carbonate (p & s block) occur as sulphides;
while noble metals like Au, Pt, Ag are dative that is free state.
Abundance:

Ten abundant elements in Earths crust:


Mineral: The chemicalcompounds ormetalcompounds which occur naturally in earths crust are known
as minerals.
Ores: The minerals from which metal can be extracted economically are known as ores.
It can be concluded that all minerals are not ores but all ores are minerals.
Principal Ores of some metals:
Metal Ore Composition
Al Bauxite Al2O3.2H2O
Corundum Al2O3
Diaspora Al2O3.H2O
Cryolite Na3AlF6
China Clay Al2O3.2SiO2.2H2O
Mica K2O.3Al2O3.6SiO2
Feldspar KAlSi3O8
Cu Malachite CuCO3.Cu(OH)2
Azurite 2CuCO3.Cu(OH)2
Copper pyrites CuFeS2
Copper Glance Cu2S
Fe Iron Pyrites FeS2
Haematite Fe2O3
Magnetite Fe3O4
Limonite Fe2O3.3H2O
Siderite FeCO3
Zn Zinc Blend /Sphalerite ZnS
Calamine ZnCO3
Zincite ZnO

263
Different methods of concentration of ores:
1) Gravity separation: Principle used: difference in specific gravity.
2) Magnetic separation: Principle used: difference in magnetic properties.
3) Froth Flotation: Principle used: differential wetting properties.
I. Substances used: Frothing agent & collecting agent like pine oil and Sodium Ethyl Xanthate
respectively.
II. Froth Stabilizers: Cresols and Aniline.
III. Depressant: Used to prevent certain type of particles from forming the froth.
4) Leaching: The ore is treated with chemical to make it soluble. It is also used in concentration of
noble metals like Gold, Silver, which occur in Free State.
Different methods of reduction
Chemical reduction:
Using Carbon: Carbon is useful as a reducing agent because it is very cheap and can be used for
reduction of several metal oxides. The ore is mixed with coke with flux and heated to high temperature.
This is called smelting or pyrometallurgy.
Using more reactive metals: For extraction of lesser electropositive metal, Al powder can be used.
The process is known Aluminothermite or Thermite.
Oxides of Cr, Mn are reduced by Aluminum.
Auto reduction: Reducing agent is not required. Cu2S + Cu2O 4Cu +SO2
Electrolytic reduction: Highly electropositive metals like Na, Mg, Ca etc cannot be reduced using
carbon, carbon monoxide or metal. Therefore these elements are extracted electrolyitcally using fused
salts of the metal. It is mainly used for S-block elements and Al.
Different methods used in refining
The removal of impurities from metals after extraction is known as refining. The refining technique
depends upon the nature of impurity and metal and the use to which the metal is put.
Refining techniques can be classified into 2 types:
(1) Physical (2) Chemical.
Physical:

Method Principle or property Process Metals purified

Liquation Difference in fusibility of metal Heated on a sloping hearth. Impurities Bi, Sn, Pb, Hg.
impurities. remain behind and metal flows down.
Distillation Used for volatile metal. Metal is heated and vapor of pure metal are Zn, Cd,Hg
Difference in volatile character. condensed in a receiver.
Zone Difference in solubility of The circular heater is fitted round a rod of Very high purity
refining impurity in molten and solid impure metal and is slowly moved down the viz Ge, Ga
state. rod.

265
Thermodynamic principles of metallurgy:

The basic concepts of thermodynamics are useful in selection of reducing agent for a particular oxide.

Free energy of changes during reduction process are important. For a reaction to be spontaneous free
energy changes must be negative. Any compound with lower value of standard free energy of
formation is more stable. They are converted to oxide prior to reduction.

DG = DH - TDS DG= Change in free energy

T= Temperature in Kalvin
DS= Enthalpy change
DG 0 = -RTlnK
= -2.303 RT log10 K K= equation constant
R= Universal gas constant
If K value is high, it indicted [P] >> [R] & DG 0 becomes more negative
Eg. 2Mg(s) + O 2(g) 2MgO (s)- DG 1000c = -941KJ/mole

2C (s) + 2CO (g) DG 1000c = -439 KJ/mole

MgO + C Mg + CO

DG = SG p - SG r

= -439 + 941

= +502
Since DG is +ve carbon cannot be used as reducing agent

2Zn + O 2 2ZnO DG 1100c = -360 KJ mol-1

2C + O 2 2CO DG 1100c = -460 KJ mole-1

SG = SG p - SG r

= -460 (- 360)

= -100 KJ mol-1

Reduction is possible.

Electrochemical Principle

Electrode potential DG 0 = - nFE0


This concept is useful in electrolytic reduction. If E0cell is positive the DG 0 is negative. Therefore
reaction is spontaneous.

267
Types of iron preparation Impurities &Properties Uses

Pig iron Obtained from blast 4%C &S ,P Manufacture of cast


furnace iron

Cast iron Blowing hot air in 3%C Gutter pipes, Lamp


mixture of pig posts, casting of
iron,iron scrap Hard & brittle articles like stoves
&coke etc. It does not rust.

Wrought iron Heating cast iron in Purest form of iron.It is Anchors, chains nails
reverberatory extremely tough with high etc
furnace lined with melting point &malleable.
hematite, limestone
is added as flux to
remove S,P,Si
(impurities)

Fe2O3 +C(cast
iron)2Fe +CO

Metal from is
passed furnace is
passed through the
rollers to remove
impurities

272
Manufacture of Zn Continued
ORE

Crushing

Forth flotation if Zn blend Concentration

Rosting, Calcination ZnS + 3O 2 2 ZnO + 2SO 2


D
ZnCO3(s )
ZnO(s ) + CO 2 ( g )
Reduction

Chemical with Carbon Leaching

ZnO(s) + C(s)

Zn (s) + CO (g) ZnO + H 2SO 4

ZnSO 4 + H 2 O
Heated ZnOwith coal ZnSO4 impurities like Cd, Fe, Sb & As
in cylindrical retort (i) Zn Dust (ii) Filter
Zn vapours are cooled and Cdppq ZnSO4 + Soluble Salt of Fe, Sb, AS
Collected (i) Ca(OH)2
(ii) Filter
Impure Zn or spelter Soluble ZnSO4 Pot of hydroxides of Fe, Sb, As

Purification Pure ZnSO4

Electrolysis

Liquidation Distillation Electrolytic Pure Zn 99.9%


refining

273
M.C.Q.
1) The most abundant metal in the earth crust
a) Al b) Fe c) Ca d) Na
2) The impurities associated are
a) Flux b) Gangue c) Slag d) Ore
3) An ore after concentration was found to have basic impurities. The flux which can be used is
a) CaCO3 b) SiO2 c) FeO d) Ca(OH)2
4) Choose the correct statement
a) All ore are mineral b) All mineral are ores
c) Minerals are not ores d) a & b
5) A metal oxide is reduced with a metal, M1 of pd 3 & Gp13. The process is known as ____ & the metal
is
a) Pyrometallurgy &Mg b) Thermite & Ga c) Liquation & Al d) Thermite & Al
6) Diaspore & Corundum are ores of ______ & ______
a) Al & Fe b) Fe & Al c) Al & Al d) Si & Al
7) Choose the correct option
a) Siderite FeCO3 b) Limonite Fe2O3 c) Calamine ZnCO3 d) a & c
8) The salt which is most unlikely to occur as mineral is
a) Bromide b) Sulphate c) Oxide d) Sulphides
9) During concentration of ore by froth flotation, the ore particles float on surface because:
a) Ores are insoluble b) Sulphides ores are lighter
c) The surface is not wetted by H2O d) Difference in densities
10) An ore contains Pbs with impurity Zns. NaCN is added in the froth flotation process ZnS does not form
the froth because
a) NaCN forms a complex of ZnS on surface of ZnS b) ZnS is not wetted by pine oil
c) ZnS is wetted by H2O d) NaCN reacts with ZnS ionic compound
11) The method used for concentration of magnetic ore is
a) Gravity separation b) Froth floatation c) Magnetic separation d) Leaching
12) During froth flotation process the student observed that the froth was disappear after formation. The
student added-----_______to the container to overcome the difficulty
a) Pine oil b) Cresol c) Benzene d) NaCN
13) Silver ore is related with NaCN(aq) to
a) Reduce silver b) Extract pure silver
c) Refine silver d) To remove the impurities

274
14) The principle involved in leaching is
a) Difference in volatility b) Difference in density
c) Difference in solubility d) Soluble complex formation
15) Heating of ore in presence of O2 below its melting point is known as
a) Roasting b) Calcinations c) Smelting d) b & c
16) During electrolysis graphite is used as an electrode & not diamond because
a) Graphite is cheaper b) Graphite is soft
c) Graphite posses free electron while diamond doesnt d) Graphite is non reactive
17) Group 1 & 2 elements are extracted by
a) Thermite process b) Electrolytic method c) Bessomerisation d) Cupellation
18) Hydrometallurgy is used in extraction of
a) Sodium b) Manganese c) Iron d) Silver
19) Several metals are commercially produced by reduction of oxides by carbon. The oxides which can be
reduced with carbon are
a) ZnO & Fe2O3 b) CaO & Cr2O3 c) BaO & U3O8 d) SiO2 & Al2O3
20) Blister copper is
a) Pure copper b) Ore of Cu c) Alloy of Cu d) Impure copper
21) In Hall Heroults process cryolite is added to alumina to
a) Increase the conductivity b) Lower the melting point
c) Increase the mobility of iron d) All of above
22) During extraction of metal charcoal powder is sprinkled on top of molten metal. This is useful in preventing
a) Oxidation of metal b) Formation of alloy c) Reduction d) a & b
23) zone refining is used to obtain metal
a) Pure Cu b) Zirconium c) Ultrapure Si d) Nickel
24) Strongly
D
Metal + _______ Volatile compound
Strongly
Pure Metal

This method is known as


a) Liquation b) VanArkel c) Zone refining d) Distillation
25) ______ metal is purified by Monds process
a) Zr b) Ti c) Ge d) Ni
26) The slag obtained during manufacture of Cu is
a) CaSiO3 b) FeSiO3 c) CuSiO3 d) FeO

275
27) The principle used in zone refining
a) Fractional distillation b) Adsorption
c) Fractional crystallisation d) Chromatographic separation
28) In the reaction 2MO(s) + C(s) M(s) + CO2(g) the entropy of the reaction will
a) Decreases b) Increases
c) Remain constant d) May increases or decreases
29) In electrolytic refining of copper, anode mud contains
a) Earthly impurities b) Zn & Mn c) Noble metal d) Oxides of Cu
30) Zn is extracted from ZnS. The reducing agent used is _________ & method of refining is ______
a) Coke & Electrolysis b) Mg & Liquation
c) Coke & Zone refining d) Coke & fractional distillation
(31 To 40 are match the following sets.)
31) Set 1 Set 2
1) Al p) Haematite
2) Fe q) Nuggets
3) Zn r) Sphalerite
4) Ag s) Feldspar
t) Limonite
a) 1-p, 2-t, 3-r, 4-q b) 1-s, 2-p & t, 3-r, 4-q
c) 1-s, 2-r & t, 3-p, 4-q d) 1-p, 2-t, 3-r, 4-s
32) Set 1 Set 2
1) Pi g iron p) Hard & brittle
2) Cast iron q) Prepared from cast Fe
3) Wrought iron r) Prepared by methyl pig iron
s) Malleable
t) Fe + 4% C
a) 1-t, 2-p & r, 3-q & s b) 1-t, 2-r , 3- s
c) 1-p, 2-r & t, 3-q & s d) 1-p, 2-s & r, 3-t & q
33) Set 1 Set 2
1) Cr2O3 + Al p) electrolysis
2) Zinc q) Bayers process
3) M2O3 + NaOH r) Thermite
D
Soluble M2O3 s) Hall Heroult

a) 1-r, 2-p, 3- s b) 1-r, 2-s, 3- p c) 1-s, 2-p, 3-q d) 1-r, 2-p, 3-q

276
34) Set 1 Set 2
1) Chromatography p) Chemical process
2) Poling q) Difference in solubility
3) Liquation r) Difference in meltingpoint
4) Zone refining s) Low boiling point metal
t) Adsorption
a) 1-r, 2-p, 3-s, 4-q b) 1-q, 2-r, 3-p, 4-s c) 1-t, 2-p, 3-s, 4-q d) 1-p, 2-p, 3-s, 4-q
35) Set 1 Set 2
1) Earthly impurities p) Froth flotation
2) Sulphide ores q) Magnetic separation
3) Bauxide r) Gravity separation
4) Magnetite s) Leaching
a) 1-p, 2-r, 3-s, 4-q b) 1-r, 2-p, 3-s, 4-q c) 1-q, 2-r, 3-p, 4-s d) 1-s, 2-p, 3-r, 4-q
36) Given below are the different temperature reactions & products during extraction of iron in blast furnace
1) 5000-8000 K p) Pig iron
2) 12700 q) Molten slag
3) 21700 r) C + O2 2CO
4) 2170 s) 3Fe2 O3 +CO 2Fe3O4 +CO 2
5) > 2170 0
t) CaO + SiO2 CaSiO3
a) 1-s,2 q,3r,4-p b) 1-s,t ; 2-r,3-q ,4-p
c) 1-r,s ;2-t,3-p,4-q d) 1-s,2-r,3-q,4-ps
37) Set 1 Set 2
1) Bauxite p) Bayers
2) Zinc blend q) Blast Furnace
3) Copper pyrites r) Hall- heroult
4) Haematite s) Bessemerisation
t) Fire Clay cylindrical retort
a) 1-r & p, 2-t, 3-s, 4-q b) 1-p, 2-t, 3-s, 4-q c) 1-r, 2-s, 3-s, 4-q d) 1-r, 2-t, 3-s, 4-q
38) Set 1 Set 2
1) Zn p) Auto mobiles
2) Wrought iron q) Galvanising
3) Steel r) Bell Meta
4) Copper s) Muntz Metal
t) Anchors
a) 1-s, 2-r, 3-t, 4-q b) 1-q & r, 2-t, 3-p, 4-r & s
c) 1-q, 2-s, 3-p, 4-r d) 1-q, 2-t, 3-p, 4-r & s

277
39) Set 1 Set 2
1) Electrolysis p) Wide range of temperature
2) Zone refining q) Adsorption
3) Blast furnace r) Electrode potential
4) Liquation s) Noble gas atmosphere
t) Low meltingpoint
a) 1-r, 2-t, 3-p, 4-t b) 1-r, 2-s, 3-p, 4-t c) 1-r, 2-s & t, 3-p, 4-q d) 1-t, 2-s, 3-p, 4-r
40) Set 1 Set 2
1) Non spontaneous p) DG= 0
2) spontaneous q) K < 1
3) Equilibrium r) DG decreases
s) K > 1
t)
a) 1-q, 2-r & s, 3-p & t b) 1-q, 2-r & s, 3-p c) 1-t, 2-r & s, 3-p d) 1-q, 2-r, 3-p
Questions 41 to 51 are assertion reason type
a) Statement 1 is correct, statement 2 is the correct reason for statement 1
b) Statement 1 is correct, statement 2 is correct but does not give reason of for statement 2
c) Statement 1 is correct, statement 2 is false
d) Statement 1 is false, statement 2 is correct
41) Statement 1) Magnesium is mainly extracted by electrolysis of molten electrolyte and not by
chemicalmethods.
Statement 2) The DiH of Mg is very low & hence it is very difficult to reduce Mg2+(aq)Mg(s)
42) Statement 1) Cu is obtained by ____ . Then by pdt CO2 obtained
Statement 2) The by-product obtained is SO2& is used in manufacture of sulphuric acid.
43) Statement 1) In Bayers process the ore is heated with Conc. Ca(OH)2.
Statement 2) The ore used in Bayers process is atmospheric & is soluble in NaOH therefore ore
is concentrated.
44) Statement 1) Froth flotation is used for concentration of sulphide ores.
Statement 2) Cresol is used as a depressant in froth flotation.
45) Statement 1) Metal oxide can be easily reduced with carbon
D
Statement 2) Metal oxide + C Melted + CO DG = -x KJ

46) Statement 1) DG = -nFE0 is applicable to metallurgy


Statement 2) If cell potential is negative electrolytic reduction of metal ions is possible

278
47) Statement 1) Ultra pure silicon is manufactured by vapour phase refining
Statement 2) Vapour phase refining gives ultra pure metals
48) Statement 1) Ni is converted to Ni(CO)4 in Mond process
Statement 2) Ni(CO)4 is volatiles & the compound can be easily disposable
49) Statement 1) Mercury is purified by distillation
Statement 2) Mercury has low melting point
True / false questions (50-58)
50)
1) Ag & Au are manufacture by hydrometallurgy
2) Fe can be extracted by electrolytic method
3) Mg is extracted from aq MgCl2 by passing electric current
4) Zn is extracted from Zinc blend by using coal or anthracite coal
a) TFFT b) TFTT c) FFFT d) FFTT
51) The metals which can be extracted using carbon/coke
1) Magnesium
2) Iron
3) Potassium
4) Zinc
a) FTTF b) TTFF c) TTFT d) FTFT
52)
1) DG0 = -nFE0
2) The above equation is the principle used in electrolytic reduction for manufacture of metals
D
3) 2Cu2S(s) + 3O2(g) 2Cu2O(s) + 2SO2(g) DS is positive in this reaction

4) DG = DH TDS if free energy change is negative the reaction is spontaneous


a) TFFT b) TTFT c) TTTT d) TFTF
53)
1) Kaolinite is an ore of Al
2) Sphalerite is an ore of sulphide of Cu
3) Malachite is an oxide ore of copper
4) sphalerite is iron carbonate
a) TFTT b) TTFT c) TFFT d) TFFF

279
54)
1) Ore is heated strongly during calcinations to remove all volatile impurities
2) Ore is heated with oxygen during roasting to convert sulphide to oxide
3) Cryolite is added to bauxite in Hall Heroults process to increase solubility of bauxite in ____
4) After leaching of bauxite, it is directly used as an electrolyte
a) TTFF b) TTFT c) TTTF d) TFFT
55)
1. The reduction reaction by accepting electrons is known as electro nation
2. In extraction of Gold and silver by process of leaching K > 1
3. If DH = -1369 KJ/ mol-1, DS= +26 J/mol-1 T = 400K for the reaction
A + B C the reaction is non spontaneous
a) TTT b) TTF c) TFT d) TFF
56)
1. Chewing of cathode occurs in Hall Heroults process
2. When water is added after digesting boxide, CO2 is bobbled to nutrelise the solution
3. HCl can not be replaced in (2) because AlCl3 is formed which is highly soluble
a) FTF b) TTF c) FFF d) FTT
57)
1. Al is used in preparation of parts of airplane and manufacture of alloy alnico.
2. Copper is used in preparation of tubes of boilers, delta metal and muntz metal.
3. Copper and aluminum are used in alloys Duralumin, Aluminum bronze.
4. Cu & Zn are used in manufacture of German Silver.
a) TFTF b) TTFT c) TFFT d) TFTT
58)
1. Wet metallurgical process is used for pyrites ores of lower grade.
2. 2Cl- +2H2O2OH- +H2 +Cl2 The cell potential is -2.186V.This reaction will take place in forward
direction.
3. Pure Zn metal is called spleter.
4. The abundance of Al is highest .Its place is third & is about 8.3% by weight.
a) TFFT b) TTTT c) FTFT d) FFFT

280
Linked Comprehension type:
59) Cassiterite ( SnO 2) and ore of Tin contains FeWO4 & MnWO 4 as impurities as well as small
amounts of sulfur. SnO2 is melted with cold, Lime stone and sand in a reveberatory furnace.
The impure Tin Sn is purified by liquation
I. The ore is concentrated by
A) Froth flotation B) Magnetic separation
C) Leaching D) a & b
II. The impurities of sulfur can be removed by
A) Treatment with NaOH B) Treatment with HCl
C) Roasting D) Calcinations
III. If limestone and sand are added during smelting the nature of impurities are
A) Acidic and basic B) Amphotric C) Acidic D) Basic
(a) i-B, ii-C, iii-C, iv-B (b) i-C, ii-D, iii-D, iv-A
(c) i-A, ii-C, iii-B, iv-C (d) i-A, ii-C, iii-A, iv-B
55) DG 0f for CuS, CaS, SO2, CS 2 & CuO are -49, -1230, -300.4, +63.6 & - 127.2 KJ/ mol-1
respectively.
i. The most stable and unstable compounds are
A) SO2 & CaS B) CaS & CS 2 C) CuS & SO2 D) CaS & SO2
ii. CuS & CaS are reduced with Carbon to give Cu or Ca and CS2. The reaction which will
be spontaneous is
A) Reduction of CuS because DG is positive
B) Reduction of CaS because DG is negative
C) Both are spontaneous because DG is positive
D) Both are non spontaneous because DG is positive
iii. The DG 0 for the reaction 2CuS + 3O2 2CuO + 2SO 2 is ________ & the reaction
is _________
A) 756.8KJ & Spontaneous B) +756.8KJ & non- Spontaneous
C) -378.4KJ & Spontaneous D) +378.4 KJ & non-Spontaneous
(a) i-B, ii-D, iii-A (b) i-D, ii-C, iii-B (c) i-B, ii-D, iii-C (d) i-C, ii-B, iii-D

281
ANSWER KEY
1 A 16 C 31 B 46 C
2 B 17 B 32 A 47 D
3 B 18 D 33 D 48 A
4 A 19 A 34 C 49 A
5 D 20 D 35 B 50 A
6 C 21 B 36 D 51 D
7 D 22 A 37 A 52 B
8 B 23 C 38 D 53 C
9 C 24 B 39 B 54 A
10 A 25 D 40 B 55 B
11 C 26 B 41 A 56 D
12 b 27 C 42 D 57 B
13 D 28 B 43 D 58 A
14 D 29 C 44 C 59 D
15 A 30 D 45 A 60 A

282
Therefore hydrogen is placed in middle of first period.

Isotopes of hydrogen
Sl:No: Name Atomic Atomic No: of No: of occurrence Nuclear
&symbol number mass protons neutrons stability&t1/2
1 Protium 1 1 1 0 Highest Stable
1
1 H 99.9850%
2 Deuterium 1 2 1 1 0.015% Stable
or 12H
3 Tritium or 1 3 1 2 T: 11 H 12.33yrs
3
1 H 1:10 Radioactive
3 3
1 H-> 2 He
0
+ -1 e

Physical properties

Physical properties of isotopes are slightly higher than hydrogen because the mass of isotopes are higher.

Chemical properties: The chemical properties are similar because the electronic configuration is same. The
rate of reactions differs.

Laboratory preparation of hydrogen :

Zn +H2SO4 -ZnSO4 +H2

Zn +NaOH -Na2ZnO2 +H2


(Sodium zincate)

Manufacture of Hydrogen:

Industrial Preparation:

Electrolysis:

Electrolyte Electrodes

a) Acidified water Pt

b) Aqueous Ba(OH)2 Ni

Hydrogen is liberated at the cathode.

284
Manufacture of Hydrogen

Starting H2 O Normal CH3 OH (l)


Material Steam Gas CH4

1270K Ni H2O
Red hot Catalysts
coke Cu2O
50 bar
673K

CO + H2

Synthesis gas
FeCrO 4 as catalyst H2O
673K

CO2 + H2

Bubble through H2O

H2O + CO2 H2CO3

CO2 Dissolve H2 insoluble

Physical properties:

1. Colourless, tasteless & odourless

2. It is diatomic gas, insoluble in water.

3. It is the lightest element & diamagnetic.

285
FORMULA :
W
(1) M = M = Molarity
M V
1

W= Weight of solute
M1 = Molecular mass
V = Volume
W
(2) N = N = Normality
EV
W= Weight of solute
E = Equivalent mass
V = Volume
(3) gL-1 = N E
(4) Equivalent weight for H2O2 = 17
(5) For H2O2, N = 2M
(6) % w/v = The mass of H2O2 in 100 ml solution.
(7) Volume = % w v 3.294
(8) %w/v = 3.4 M
(9) % Volume = 11.2 M
(10) N1V1 = N2V2

(11) ( gL- )1 ( Volume strength )1 + ( gL- )2 ( Volume strength )2 +

( )
gL- ( Volume strength ) = ( g L )
3 2 mixture
( Volume strength )mix

292
M.C.Q.
1. The element which is the biggest source of energy in future is
(a) Monoatomic gas (b) Gaseous non-metal
(c) Liquid nonmetal (d) lightest element
2. Dihydrogen is liberated at the anode by electrolysis of :
(a) Molten sodium hydride (b) Acidified water
(c) Molten sodium chloride (d) Water with Ba(OH)2
3. The conversion of atomic hydrogen to dihydrogen is :
(a) endothermic change (b) Photochemical change
(c) exothermic change (d) Nuclear change
4. The isotope of hydrogen with half-life of 12.33 year is :
(a) Protium (b) Deuterium (c) Tritium (d) b &c
5. Zinc on reaction with ....... liberates a combastible gas.
(a) dil HCl (b) dil KOH (c) H2SO4 (d) a, b, &c
6. Hydrogen gas can be produced from
(a) Water gas (b) producer gas (c) coal gas (d) air
7. When Zn pieces are dropped in NaOH solution H2(g) is obtained and soluble ...... is obtamed
(a) Na2ZnO3 (b) NaZnO2 (c) Na4ZnO3 (d) Na3ZnO2
8. CO (g) + H 2 O(g)
[ x ] CO 2(g) + H 2(g) . x is ......
673k

(a) Fe (b) Pd (c) FeCrO4 (d) V2O5


9. H2 can be obtaioned from mixture of CO2&H2 by bubbling the mixture through
(a) Water (b) Alkaline Ca2Cl2
(c) Conc H2SO4 (d) Hot Nacl solution
10. H 2 + A
673k 200bar
[ Fe] Alkaline gas . A is

(a) Cl2 (b) O2 (c) N2 (d) Na


11. The decay product of tritium is :
(a) 42 He (b) 11 H (c) 21 H (d) 23 He
12. The metal Zn, Al, Mg, &Be are placed in different Test tubes. If NaOH is added, the metal which libaate
Hydrogen gas are
(a) Zn, Al, Mg & Be (b) Zn &Al (c) Mg & Be (d) Zn, Al & Mg

293
13. The gas used in welding & cutting of metal is a strong ......
(a) Reducing agent (b) Oxidising agent
(c) Reducing & oxidising agent (d) Dehydrating agent
14. Hydrogen closely resembles halogens becouse
(a) Strong reducing agent (b) diatomic gas
(c) it is a colourless gas (d) its is reduction potential is o.oo V
15. K w = 1.0 10-14 at 298k because

(a) OH - = H3O + = (1.0 10-14 ) M

(b) OH - = (1.0 10-8 ) M & H3O + = (1.0 10-6 ) M

(c) OH - = H3O + = (1.0 10-7 ) M

(d) OH - = (1.0 10-6 ) M, H3O+ = (1.0 10-8 ) M


16. The type of hybridisation of O in H 2O & H 2 H 2(s) is
(a) sp3, sp3 (b) sp2, sp3 (c) sp3, sp2 (d) sp3, sp
17. The shape of water moleale is bent and not linear because
(a) Bond angle is < 180 (b) sp3 hybridisation
(c) Presence of one lone pair of electron (d) sp2 hybridisation
18. BH + H 2 O BH 2 + + OH - , H 2 O acts as
(a) Base (b) Reducing
(c) acid (d) a &c
19. A metal M belongs to period 3 & group 2 reacts with nitrogen to give compound B. If B is added to
water the products are :
(a) Mg(OH)2 & NH3 (b) Be(OH)2 & NH3
(c) LiOH & NH3 (d) Ca(OH)2 & NH3
20. The Only compound whose density in solid state is less if than liguid is
(a) Water (b) Sodium hydrocide
(c) Nitric acid (d) phosphoras penta chloride
21. Fishes survive in frozen lakes because
(a) Ice floats on water (b) Ice acts as an insulator
(c) The Solubility of CO 2 in water increaze (d) a &b

294
22. Water is most important solvent because it is
(a) polar (b) Non polar
(c) forms H- bond (d) a &c
23. Ice is lighter than water because
(a) Density of ice is greater than water
(b) The volume of ice is more for given mass of water due to H - bonding
(c) Anomalous expansion
(d) Oxygen is electronegative & size is large
24. SiCl 4 + H 2O A + HCl A is .
(a) Si(OH)4 (b) SiO 2 (c) SiO (d) SiCl4 2H 2O
25. MH + H 2 O MOH + H 2 M belongs to
(a) p - block (b) d - block (c) s - block (d) p - block & d - block
26. The hydrogen which are used in catalytic reaction are :
(a) hydrides of p-block (b) hydrides of d-block
(c) interstial hydrides (d) b &c
27. Alkali metal do not form interstial hydrides because
(a) alkali metals loose electron readily. (b) The packing in alkali metals in vay close
(c) Absence of interstitial voids (d) size is large
28. The position of the element which forms deficient hydrides.
(a) Period 2 group 14 (b) Period 2 group 15
(c) Period 2 group 13 (d) Period 6 group 13
29. An element forms electron rich hydride. The elctornic configuration of the element is
(a) [ He] 2s 2 2p 2 (b) [ He] 2s 2 2p1

(c) [ Ne] 2s 2 2p 2 (d) [ He] 2s 2 2p5


30. The set if quantum number for valence electron of an element which from election precise of water is
(a) n = 4 = 2 (b) n = 2 = 1
(c) n = 3 = 0 (d) n = 2 = 0
31. The method which can be used for removal of temporary & permanent hardness of water is
(a) Decantation (b) Distillation
(c) Boiling (d) Filteration
295
32. Which of the following reacts easily with H2O to form hydrogen
(a) HCl (b)KH (c) NH 3 (d) B2H 6
33. It is not advisable to use hard water for washing clothes beacause
(a) Precipitate of sodium salt of fatty acid is formed
(b) Precipitate of sodium salt of sulphonic acid is formed
(c) Precipitate of Magnesium salt of sulphonic acid is formed
(d) Precipitate of Magnesium salt of fatty acid is formed
34. Calgon softens hard water by

(a) Precipation of Ca 2+ & Mg 2+ ions (b) Coagulation of solts

(c) Complexing Ca 2+ & Mg 2+ ions (d) a & c

35. clarks method is used to remove


(a) Temporary hardenes (b) permanent
(c) Hardnes due to soluble SO4-2 of Ca+2, MG+2 (d) Temporary & permanent

36. Na + D 2 O A + B, A & B are ______ & ______respecitively..

(a) NaOH & H 2 O (b) NaOD & D 2

(c) NaOD & D 2 O (d) Na2D & D2

37. The % (mass) of deuterium in heavy water is


(a) 18.0 (b) Cannot be predicted
(c) 11.1 (d) 20.0

38. H 2 O 2 is not used as

(a) Oxidising agent (b) Redusing agent


(c) Catalyst (d) Bleaching
True - False Type

39. 1. H 2 O 2 acts as bleaching agent because of if its oxidising property

2. It is dangerous to used H 2 O 2 is maintenance of environment

3. H 2 O 2 is used is properation of good quality detergents

4. Perhydral is used as a disuifectant


(a) TFTT (b) FTTT (c) TTFF (d) FFTF
296
40. 1. A paper with stain of black Pbs, dipped in H 2 O 2 solution turns white

2. The colour of acidified KMnO4 does not diappear when H 2 O 2 is added

3. A basic solution containinig Fe3+ ion turn blue on addition of H 2 O 2

(a) FFF (b) FTF (c) TFF (d) TTT


41. 1. Tritium can be obtaned from natural source.
2. In ionic hydrides the oxidation state of hydrogen is +1

3. The four atom of oxygen in H 2 O 2 are in the same plane

4. Na 2 CO3 removes temporary & permannent hardenes

(a) TFTT (b) FFFT (c) TFFT (d) TTTF


42. Statement S: The position of hydrogen is not fixed,
Statement R: Hydrogen resemker alkali metals because of its stable +1 oxidation state.
(a) S & R both are correct (b) S is corrct R is correct and explains S.
(c) S is incorrect R is correct (d) S is correct R is incorrect
Solve the problems from question 43-50
Questions 43-50 - Solve the problems.
43. Calculate M, N % w/v, gL-1 of 10 Vol H2O2
M N %w/v gL-1
(a) 0.89 1.78 3.036 30.36
(b) 0.78 2.78 2.036 3.036
(c) 0.92 1.95 2.45 3.5
(d) 0.1 0.78 4.0 3.6
44. 30 ml of acidified solution of H2O2 required 30 ml of 0.1NKMnO4 Calculate strength Volume
strength & molarity.
M gL-1 Volume
(a) 0.06 0.7 0.8
(b) 0.12 0.9 0.6
(c) 0.05 1.7 0.56
(d) 0.1 1.0 0.90
45. 374 g of H2O2 is present in 15 lit solution calculate M, N, % w/v & volume strength.
M N %w/v Volume gL-1
(a) 0.89 1.2 1.49 6.8 22.0
(b) 1.6 1.3 3.49 7.9 23.1
(c) 1.9 1.7 0.49 9.0 22.2
(d) 0.733 1.466 2.49 8.2 24.9

297
46. 2.72 g of H 2O 2 is present in 50ml solution Calculate M, N, Strength gL-1 Volume strength
H2 O 2
(a) M = 10, N = 2.9, g/l = 53.0, Vol = 17.0
(b) M = 1.6, N = 3.2, g/l = 54.4, Vol = 17.92
(c) M = 0.89, N = 1.8, g/l = 52.9, Vol = 18.1
(d) M = 0.90, N = 1.0, g/l = 5.44, Vol = 16.0
47. Calculate % w/v, Volume Strength, M & N of a mixture contaning 800 ml of 2.5% w/v,
700 ml of 4.2 w/v & 500 ml 5.3w/v of H2O 2 Solution
% W/V g/L M N
(a) 3.795 37.95 3.4 2.23
(b) 3.02 3.795 1.116 1.92
(c) 3.795 37.95 1.116 2.23
(d) 2.92 02.89 3.4 1.92
48. 10ml of KMnO 4 Solution is required ti completely oxidise acidic solution of 30ml of 1.5
Volume strength H2O2 Calculate normality of KMnO4 Solution
(a) 0.4 (b) 0.65 (c) 0.19 (d) 0.8
49. 500 ml of 5Vol, 400 ml of 10 Volume & 600 ml of 15 volume solution of H2O2 is mixed
Calculate volume strength, M, N of resulting solution.
Vol % W/V g/l M N
(a) 10.33 3.137 31.37 0.92 1.84
(b) 10.33 31.37 3.137 1.84 0.92
(c) 9.95 3.137 32.w 0. 921.84
(d) 9.95 31.37 31.3 1.8 40.92
50. A
O2
B
O2
C
H 2SO 4
D+E
gives green 1st Prepared by
Colour flame J.L Thenard
A, B, C, D & E are
(a) Ba, BaO, BaO2, BaSO 4, H 2O 2 (b) Na, Na2O, Na 2O 2, Na 2SO 4, H 2O
(c) Ba, Ba2O 2, BaO 2, BaSO 4, H 2O 2 (d) Ca, CaO, CaO2, CaSO 4, H 2O 2

298
HINTS
43. Volume = 11.2 M N = 2M %W / V = 3.4 M
10 = 11.2 M = 2 0.89 = 3.4 0.89
M = 0.89 N = 1.78 gL-1 = 3.036
gm/L = 30.36

44. H2O2 = kMnO4 gm/L = N E V=11.2 M


N1V1 = N2V2 gm/L = 0.1 17 = 11.2 0.05
N1 30 = 0.1 30 gm/L =1.7 = 0.56
N1 = 0.1
Molarity of H2O2 Sol. = 0.05
W
45. M= N = 2M % W / V = 3.4 M Vol =11.2 0.733
M V
1

374
= 2 0.73 = 3.4 0.733 Vol = 8.2
34 15

M = 0.733 = 1.466 % W / V = 2.49


gm/L = 24.9
W
46. M= N = 2M % W / V = 3 .4 M Vol =11.2 1.6
M V
1

2.72
=
34 0.05 = 2(1.6) = 3.4 1.6 Vol = 17.92

M= 1.6 = 3.2 = 5.44


gm/L = 54.4
47. 2.5 800 + 700 4.2 + 500 5.3 = 2000 % W / V
%W/V = 3.795
gm/L = 37.95
%W/V = 3.4 M
3.795 = 3.4 M
M= 1.116
N = 2.232

299
48. H2O2 kMnO4 Volume =11.2 M
1.5 30 = Strength 10 M = 0.40
V = 4.5
N = 2M= 2 0.4
N = 0.8
49. 500 5 + 400 10 + 600 15 = Volume 1500
Volume = 10.33
% W / V = /3.294
%W/V = 3.137
gm/L = 31.37
Volume = 11.2 M
M = 0.92
N = 2M
N = 1.84

Answer Key
1 d 11 d 21 d 31 b 41 b
2 a 12 b 22 d 32 b 42 a
3 c 13 a 23 b 33 d 43 a
4 c 14 b 24 b 34 c 44 c
5 d 15 c 25 c 35 a 45 d
6 a 16 a 26 d 36 b 46 b
7 b 17 b 27 a 37 d 47 c
8 c 18 c 28 c 38 c 48 d
9 a 19 a 29 d 39 a 49 a
10 c 20 a 30 b 40 c 50 a

300
UNIT : 14 S - Block ELEMENTS
Important Points
Group-1 (alkali metals) and group-2 (alkaline earth metals) are included in the s-
block elements of the periodic table. They are known like this because their oxides
and hydroxides are alkaline in nature. Alkali metals possess one and alkaline earth
metals possess two s-electrons. They are highly electropositive metals and form
monovalent cations (M+) and divalent cations (M2+) respectively.
With the increase in atomic number, the physical and chemical properties of alkali
metals show regular trend. The atomic and ionic sizes increase on going down in the
group and ionization enthalpies decrease in alkali metals. The same type of trend is
observed in alkaline earth metals.
The first element of each of these two groups, namely, lithium in group-1 and beryllium
in group-2 show similarities with the element of the next group viz. Li-Mg and Be-
Al. This is called diagonal relationship. In fact, the first element of each group shows
difference with other elements in the same group i.e their behaviour is anomalous.
Alkali elements are bright white, soft metals melting at low temperatures. Li and Na
are obtained by electrolysis. They are very active and their compounds are ionic.
Their oxides and hydroxides are soluble in water and give strong alkaline solution.
Amongst important compounds of sodium are sodium carbonate, sodium hydrogen-
carbonate, sodium hydroxide, NaOH is produced by Castner Kellner process and
sodium carbonate by Solvay ammonia soda process.
The chemistry of alkaline earth metals is similar to that of alkali metals. Even then
some differences are there, because the atomic and ionic sizes of alkaline earth metals
decrease and the charge of the cation increases. Their oxides are less basic than those
of alkali metals.
Amongst the industrially important compounds of sodium are caustic soda, washing
soda and those of calcium are calcium oxide, calcium hydroxide, plaster of Paris,
calcium carbonate and Portland cement. The production of cement can be carried
out by grinding the mixture of lime stone and clay and heating it in rotary kiln. The
clinker obtained is mixed with gypsum (2-3%) which gives fine powder of cement.
All these substances have many uses.
Monovalent sodium and potassium ions and divalent magnesium and calcium ions are
found in larger proportion in biological fluid in a human being. These ions carry out
important biological functions like maintenance of ionic equilibrium and nerve impulse
conduction which is known as sodium-potassium pump.

301
M.C.Q.
(1) The alkali metals are low melting. Which of the following alkali metal is expected to melt if the
room tempreature rises to 300 c ?
(a) Na (b) K (c) Rb (d) Cs
(2) The reducing power of a metal depends on various factors. Suggest the factor which makes Li,
the strongest reducing agent in aqueous solution.
(a) Sublimation enthalpy (b) Ionisation enthalpy
(c) Hydration enthalpy (d) Electron - gain enthalpy
(3) Metal carbonates decompose on heating to give metal oxide and carbondioxide. Which of the
metal carbonates is most stable thermally ?
(a) MgCO3 (b) CaCO3 (c) SrCO3 (d) BaCO3
(4) Which of the following metal hydroxide is the least basic ?
(a) Mg(OH)2 (b) Ca(OH)2 (c) Sr(OH)2 (d) Ba(OH)2
(5) Some of the group - 2 metal halides are covalent and soluble in organic solvents. Among the
following metal halides, the one which is soluble in ethanol is ....
(a) BeCl2 (b) MgCl2 (c) CaCl2 (d) SrCl2
(6) The order of decreasing ionisation enthalpy in alkali metal is
(a) Na > Li > K > Rb (b) Li > Na > K > Rb
(c) Rb > Na > K > Li (d) K < Li < Na < Rb
(7) The solubility of metal halides depends on their nature, lattice enthalpy and hydration enthalpy of
the individual ions. Among the fluorides of alkali metals, the lowest solubility of LiF in water
is due to...
(a) Ionic nature of lithium fluoride (b) High lattice enthalpy
(c) High hydration enthalpy for lithium ion. (d) Low ionisation enthalpy of lithium atom.
(8) Amphoteric hydroxides react with both alkalies and acids. Which of the following Group - 2 metal
hydroxides is soluble in sodium hydroxide ?
(a) Be(OH)2 (b) Mg(OH)2 (c) Ca(OH)2 (d) Ba(OH)2
(9) In the synthesis of sodium carbonate, the recovery of ammonia is done by treating NH4Cl with
Ca(OH)2.The by product obtained in this process is ...
(a) NaCl (b) NaOH (c) CaCl2 (d) NaHCO3
(10) When sodium is dissolved in liquid ammonia, a solution of deep blue colour is obtained. The colour
of the solution is due to ...
(a) sodium ion (b) ammoniated electron
(c) sodium amide (d) ammoniated sodium ion
(11) By adding gypsum to cement...
(a) setting time of cement becomes less. (b) setting time of cement increases
(c) colour of cement becomes light (d) shining surface is obtained

302
(12) Dead burnt plaster is ...
1
(a) CaSO4 (b) CaSO 4 H 2O (c) CaSO 4 H 2O (d) CaSO 4 2H 2O
2
(13) A substance which gives crimson red flame and breaks on heating to give oxygen and a brown
gas is ..
(a) Magnesium nitrate (b) Calcium nitrate (c) Barium nitrate (d) Strontium nitrate
(14) The formula of sodash is ...
(a) Na 2CO3 10H 2O (b) Na 2CO3 2H 2O (c) Na2CO3 (d) Na 2CO3 H 2O
(15) Which of the following compounds are readily soluble in water ?
(a) BeSO4 (b) MgSO4 (c) BaSO4 (d) both (a) and (b)
(16) Identify the correct formula of halides of alkaline earth metals from the following.
(a) BaCl2 2H2O (b) BaCl2 4H 2 O (c) CaCl2 4H 2 O (d) SrCl2 4H 2 O
(17) Which of the following statement is true about Ca(OH)2 ?
(a) It is used in the preparation of bleaching powder.
(b) It is a light blue solid.
(c) It does not possess disinfectant property.
(d) It is used in the manufacture of cement.
(18) Match the elements given in Column - I with the colour they impart to the flame given in Column - II.
Column - I Column - II
(A) Cs (P) Apple green
(B) Sr (Q) Brick red
(C) K (R) Blue
(D) Ca (S) Crimson red
(E) Ba (T) Violet
(a) A-P, B-Q, C-S, D-R, E-T (b) A-Q, B-P, C-R, D-S, E-T
(c) A-R, B-S, C-T, D-Q, E-P (d) A-S, B-R, C-Q, D-P, E-T
(19) When water is added to compound (A) of calcium, solution of compound (B) is formed. When
CO2 is passed into the solution, it turns milky due to the formation of compound (C). If
excess of carbon dioxide is passed into the solution, milkiness disappears due to the formation of
compound (D). Identify the compound (D).
(a) CaO (b) Ca(OH)2 (c) CaCO3 (d) Ca(HCO3)2
(20) Which alkali metal emits longest wavelength light in Flame test ?
(a) Na (b) K (c) Cs (d) Li

303
(21) Which of the following is not known ?
(a) KO3 (b) KO4 (c) KO2 (d) K2O2
(22) Which of the following acts as reducing as well as oxidising agent ?
(a) NaNO3 (b) Na2O2 (c) Na2O (d) KNO3
(23) The salt that is added to table salt to make it flow freely in rainy season is ...
(a) KCl (b) KI (c) Ca3(PO4)2 (d) Na3PO4
(24) Which of the following alkaline earth metal sulphates is least soluble in water ?
(a) MgSO4 (b) CaSO4 (c) BaSO4 (d) SrSO4
(25) The hydration energy of Mg2+ is greater than that of ....
(a) Al3+ (b) Be2+ (c) Na+ (d) Mg3+
(26) The active constituent of bleaching powder is ...
(a) Ca(OCl)2 (b) Ca(OCl)Cl (c) Ca(C1O2)2 (d) Ca(C1O2)Cl
(27) KO2 is used in oxygen cylinders in space and submarines because it...
(a) absorbs CO2 and increases O2 content (b) eliminates moisture
(c) produces ozone (d) None of the above
(28) A metal M readily forms water soluble sulphate MSO4, water insoluble hydroxide and oxide MO
which becomes inert on heating. The hydroxide is soluble in NaOH. The metal M is...
(a) Be (b) Ca (c) Mg (d) Sr
(29) Which of the following is sparingly soluble in water ?
(a) NaOH (b) KOH (c) LiOH (d) RbOH
(30) Photo electric effect is maximum in ...
(a) Cs (b) K (c) Na (d) Li
(31) Among the following compounds of cement which is present in the highest amount ?
(a) Ca2SiO4 (b) Al2O3 (c) Ca3SiO5 (d) Ca3Al2O6
(32) Which pair of the following chlorides do not impart colour to the flame ?
(a) BeCl2 and SrCl2 (b) BeCl2 and MgCl2
(c) BaCl2 and CaCl2 (d) MgCl2 and CaCl2
(33) The sequence of hydration enthalpy in following ion is ...

(a) Rb + > K + > Cs + > Na + > Li + (b) Li+ > Na+ > K+ > Rb+ > Cs+

(c) K + >Na + > Rb + > Cs + > Li + (d) Cs + >Rb + > K + > Na + > Li +

304
(34) In case of alkali metals, the covalent character increases in the order :
(a) MI > MBr > MCl < MF (b) MF < MCl < MBr < MI
(c) MBr < MCl < MI < MF (d) MF < MBr < MCl < MI
(35) Among the following the least thermally stable is ?
(a) K2CO3 (b) Na2CO3 (c) BaCO3 (d) Li2CO3
(36) Which of the following oxides is amphoteric in nature ?
(a) MgO (b) BeO (c) CaO (d) BaO
(37) Which of the following characteristics is not related to alkali metals ?
(a) Their ions are iso electronic with noble gases.
(b) low melting point (c) low electronegativity (d) high ionisation energy
(38) Fill in the blanks with proper option given below for the following statement.
All the halides of alkaline earth metals with exception of are ionic in nature.
(a) Barium halide (b) Strontium halide (c) Beryllium halide (d) Calcium halide
(39) K2CO3 can not be prepared by solvay ammonia process because ...
(a) K2CO3 is fairly soluble in water. (b) It has no water of crystallization.
(c) KHCO3 is highly soluble in water. (d) K2CO3 decomposes in H2O.
(40) The reaction of Cl2 with X gives bleaching powder X is ....
(a) CaO (b) Ca(OH)2 (c) Ca(OCl)2 (d) Ca(O3Cl)2
(41) Which of the following alkaline earth metal sulphates has hydration enthalpy higher than the lattice
enthalpy ?
(a) SrSO4 (b) MgSO4 (c) CaSO4 (d) BaSO4
(42) A compound (A) on heating gives a colourless gas and a residue that is dissolved in water to
obtain (B). Excess of CO2 is bubbled through aqueous solution of B, (C) is formed, which is
recovered in the solid form. Solid (C) on gentle heating gives back (A). The compound is ...
(a) CaCO3 (b) K2CO3 (c) Na2CO3 (d) CaSO 4 2H 2O
(43) For alkaline metal, which of the following trends is incorrect ?
(a) Hydration enthalpy : Be > Mg > Ca > Sr
(b) Second Ionization enthalpy: Be > Mg > Ca > Sr
(c) Density : Sr > Be > Mg > Ca
(d) Atomic size : Sr > Ca > Mg > Be
(44) Which of the following compounds is most stable ?
(a) LiCl (b) LiI (c) LiBr (d) LiF

305
(45) Flame test is not given by ...
(a) Be (b) Sr (c) K (d) Ca
(46) The alkaline earth metals forming ionic oxides are ...
(a) MgO (b) BeO (c) CaO (d) (a) and (c)
(47) The basic character of the oxides MgO, SrO, K2O, NiO and Cs2O increases in the order :
(a) MgO > SrO > K2O > NiO > Cs2O (b) Cs2O < K2O < MgO < SrO < NiO
(c) NiO < MgO < SrO < K2O < Cs2O (d) K2O < NiO < MgO < SrO < Cs2O
(48) Which of the following are arranged in increasing order of solubilities ?
(a) CaCO3 < KHCO3 < NaHCO3 (b) NaHCO3 < KHCO3 < CaCO3
(c) KHCO3 < NaHCO3 < CaCO3 (d) CaCO3 < NaHCO3 < KHCO3
(49) The compound insoluble in aceticacid is ...
(a) Calcium oxide (b) Calcium carbonate
(c) Calcium oxalate (d) Calcium hydroxide
(50) Which of the following has the lowest melting point ?
(a) LiCl (b) KCl (c) NaCl (d) RbCl
(51) The correct order of decreasing ionic character is ...
(a) BeCl2 > MgCl2 > CaCl2 > BaCl2 (b) BeCl2 > MgCl2 > BaCl2 > CaCl2
(c) BeCl2 > BaCl2 > MgCl2 > CaCl2 (d) BaCl2 > CaCl2 > MgCl2 > BeCl2
(52) The highest lattice energy corresponds to ...
(a) MgO (b) CaO (c) SrO (d) BaO
(53) How many of the following s -block elements do not give characteristic colours in the flame test ?
Li , Be, Ca, Ba, Sr, Mg, Na, K, Ba
(a) 3 (b) 4 (c) 2 (d) 5
(54) How many of the following sulphates of metals dissolve in the water ?
SrSO4, K2SO4, BeSO4, Li2SO4, MgSO4, BaSO4, Na2SO4, CaSO4, Rb2SO4
(a) 6 (b) 4 (c) 3 (d) 5
(55) How many of the following hydroxides is/are amphoteric in character ?
CsOH, LiOH, Ca(OH)2, Be(OH)2, Mg(OH)2, Sr(OH)2, Ba(OH)2, KOH, NaOH.
(a) 1 (b) 4 (c) 5 (d) 3
(56) Out of Li, Na, K, Rb and Cs how many of them directly form superoxides on heating with oxygen ?
(a) 5 (b) 2 (c) 3 (d) 4

306
(57) How many of the following metals when heated in an atmosphere of N2 gas form nitrides ?
Li, Na, K, Rb, Cs, Mg, Ca, Sr, Ba
(a) 9 (b) 5 (c) 3 (d) 6
(58) Which of the following is not correct for workfunction of Na+ ions in human body ?
(a) An important role in nerve signal transmission.
(b) Control of flow of water between cell membrane.
(c) For transport of sugar and amino acid in cell.
(d) They activate the enzyme.
(59) Which of the following is not correct ?

(a) 2Li 2 O
heat
673k
Li 2 O 2 + 2Li (b) 2K 2 O
heat
673k
K 2 O 2 + 2K

(c) 2Na 2O
heat
673k
Na 2O 2 + 2Na (d) 2Rb 2O
heat
673k
Rb 2O 2 + 2Rb

(60) Which of the following has maximum lattice energy ?


(a) Li2O (b) Na2O (c) MgO (d) BaO
(61) Which of the following statements is/are correct for when alkali metals are dissolved in liquid
ammonia. We get...
(a) A blue solution in case of dilute alkali metal ammonia solution.
(b) If we increase cocentration of metal in ammonia then the blue colour starts changing and
finally changes to that of bronze colour.
(c) The blue colour of the solution of alkali metal in liquid ammonia is due to excitation of
free ammoniated electrons to higher energy levels.
(d) All the above are correct.
(62) Which of the following pairs of elements possess diagonal relationship ?
(a) Li and Mg (b) Li and Al (c) Na and Mg (d) Cs and Ba
(63) Fill in the blanks with suitable option.
The important ingredients of potland cement are dicalcium silicate % , tricalcium
silicate ... % and tricalcium aluminate % respectively.
(a) 26 %, 51%, 11% (b) 51%, 26%, 11%
(c) 11%, 51%, 26% (d) 26%, 11%, 51%
(64) A sample of portaland cement contain 23% SiO2, 3% Al2O3 and 2% Fe2O3 then what would
be its silica module (n) ?
(a) 3.83 (b) 28 (c) 21.73 (d) 4.6

307
(65) For a good quality of cement, the ratio of silica (SiO2) and alumina (Al2O3) must be between...
(a) 3 to 5 (b) 2.5 to 4 (c) 6 to 7.5 (d) 4 to 5.5
(66) Chloro phyll and heamoglobin are complex of and respectivelly..
(a) Mg2+ and Ca2+ (b) Na+ and K+ (c) Mg2+ and Fe2+ (d) Cl and Fe2+
(67) Which of the following is the component of most of the kidney stones ?
(a) (COO)2Ca (b) (COONa)2 (c) (COO)2 Ba (d) (COO)2Mg
(68) Which of the following metal ions plays an important role in muscle contraction ?
(a) K+ (b) Mg2+ (c) Na+ (d) Ca2+
(69) White enamel of our teeth is ...
(a) Ca3(PO4)2 (b) CaCl2 (c) CaF2 (d) CaBr2
(70) Two mole of magnesium nitride on reacting with an excess of water gives :
(a) One mole of ammonia (b) Two moles of nitric acid
(c) Four moles of ammonia (d) Three moles of ammonia
(71) Which of the following metal is used in windows of X - ray tubes ?
(a) Be (b) Mg (c) Ba (d) Al
(72) Which of the following is not a Mg ore ?
(a) Magnesite (b) Gypsum (c) Dolomite (d) Carnalite
(73) The difference of water molecules in gypsum and plaster of paris is ...

5 1 1
(a) (b) 2 (c) (d) 1
2 2 2

(74) Which of the following exists in polymeric form ?


(a) AlCl3 (b) SiC (c) BeCl2 (d) B2H6
(75) The electronic cofiguration of metal M is ls2 2s2 2p6 3s2. The formula of its oxide would be ...
(a) MO (b) M2O (c) M2O3 (d) MO2
(76) The formula of carnalliteis ...

(a) KCl MgCl2 2H 2O (b) K 2 O Al2 O3 6H 2 O

(c) KCl g MgCl2 g 6H 2 O (d) Na 2 B4 O7 10H 2 O

(77) When NaOH is made, the gas released at the cathode is ...
(a) Cl2 (b) H2 (c) O2 (d) H2O

308
(78) Choose proper option for matching column - I and column - II
Column - I Column - II
(A) NaOH (P) Photo electric cells
(B) Na2CO3 (Q) Coolant in nuclear reactors
(C) Liquid Na (R) SO2 absorber
(D) Caesium (S) Detergent
(a) A-R, B-S, C-Q, D-P (b) A-P, B-Q, C-R, D-S
(c) A-Q, B-P, C-R, D-S (d) A-S, B-Q, C-P, D-R
(79) In the electrolytic separation of Li, KCl is added to LiCl ...
(a) To increase the conductivity of LiCl
(b) To lower the fusion temperature of the mixure
(c) To decrease the conductivity of LiCl
(d) Both (a) and (b)
(80) Molecular formula of Glaubers salt is ...

(a) MgSO 4 7H 2O (b) FeSO 4 7H 2O (c) CuSO 4 5H 2O (d) Na 2SO 4 10 H 2 O


(81) The name Blue John is given to which of the following compounds.
(a) CaH2 (b) CaF2 (c) Ca3(PO4)2 (d) CaO
(82) The wire of flash bulb is made of ...
(a) Mg (b) Cu (c) Ba (d) Ag
(83) 30 gm of Mg and 30 gm of O2 are reacted and the residual mixure contains ...
(a) 60 gm of MgO only (b) 40 gm of MgO and 20 gm of O2
(c) 45 gm of MgO and 15 gm of O2 (d) 50 gm of MgO and 10 gm of O2
(84) Which of the following is not correct ?
(a) 4LiNO3(s) 2Li2O(s) + 4NO2(s) + O2(g) (b) 2NaNO3(s) 2NaNO2(s) + O2(g)
(c) The oxides Li2O and MgO do not give super oxides by combining with more oxygen.
(d) Lithium hydrogen carbonate is obtained in solid form.
(85) The number and type of bonds between two carbon atoms in calcium carbides are ...
(a) one sigma, one pi. (b) one sigma, two pi.
(c) two sigma, one pi. (d) two sigma, two pi
(86) Which of the following salts are composed of isoelectronic cations and anions ?
(a) NaCl (b) MgF2 (c) CaS (d) (b) and (c) both

309
(87) Which are correct statements for Be and Al ?
(a) Both are rendered passive by con.HNO3
(b) Both have sp - hybridization in their compounds.
(c) Both form acidic oxides
(d) Both form hydrides
(88) Identify the correct statement ?
(a) Gypsum contains a lower percentage of calcium than plaster of paris
(b) Gypsum is obtained by heating plaster of paris
(c) Plaster of paris can be obtained by hydration of gypsum
(d) Plaster of paris is obtained by partial oxidation of gypsum.
(89) Which of the following metal is used in the test of elements in organic compounds by
Lassigne test ?
(a) Li (b) Na (c) K (d) Rb
(90) The setting time of dicalcium silicates is
(a) 28 days (b) 1 year (c) 1 week (d) 24 hour

Assertion - Reason type Questions


The questions given below contains statement - 1 (Assertion) and statement - 2 (Reason) Each
question has four choices (a), (b),(c) and (d) out of which only one is correct. Choose the correct
option as under.
(a) Statement -1 is true, statement - 2 is true
Statement - 2 is a correct explanation for statement - 1
(b) Statement - 1 is true, statement - 2 is true;
Statement - 2 is not a correct explanation for statement - 1
(c) Statement - 1 is true, statement - 2 is false
(d) Statement - 1 is false, statement - 2 is true.
(91) Statement-1 Alkali metals dissolve in ammonia to give blue solutions.

Statement-2 Alkali metals in liquid ammonia give solvated species of the type [M(NH3 ) X ]+

(92) Statement-1 Sodium metal is softer than potassium metal


Statement-2 Metalic bonding in Potassium is weaker than inSodium.
(93) Statement-1 Be(OH)2 is solublein HCl and NaOH
Statement-2 Be(OH)2 is amphoteric in nature

310
(94) Statement-1 Be Forrns [BeF4 ]2 but Al forrms [AlF6 ]3

Statement-2 Be doesnot have dorbitals in the Valence shell but Al has.


(95) Statement - 1 Li CO3 and Na2CO3 are therally unstable.
2

Statement - 2 Both the carbonates are salts of large cations and large anions.
(96) Statement - 1 Metallic character of alkali metals increases on going down a group from top to
bottom.
Statement - 2 Ionisation enthalpy of alkali metals increases on going down from top to bottom.
(97) Statement - 1 Superoxides of alkali metals are diamagnetic.
Statement - 2 Superoxides contain the ion O2 which has one unpaired electron.
(98) Statement - 1 Alkali metals donot impart colour to the flame.
Statement - 2 Their ionization enthalpies are very low.
(99) Statement - 1 Sodium cannot be obtained by chemical reduction of its ore.
Statement - 2 Sodium is one of the strongest reducing agent.
(100) Statement - 1 Beryllium hydroxide becomes soluble in excess alkali forming beryllate ion [Be(OH)4]2-
Statement - 2 Beryllium ion has greater tendency to form complexes.
(101) The half life period of isotopes223Fr is ....
(a) 21 hour (b) 21 second (c) 21 minute (d) 21 day
(102) Sylvine is the mineral of ...
(a) K (b) Na (c) Li (d) Rb
(103) Which pump is important in biological reaction in human body ?
(a) Ca-Mg Pump (b) K-Fe Pump (c) Na-K Pump (d) Fe-Ca Pump
(104) Withrite is which type of salt of Barium ?
(a) Carbonate (b) Sulphate (c) Chloride (d) Phosphate
(105) Silastine is the mineral of ...
(a) Ca (b) Ra (c) Ba (d) Sr
(106) Which of the following reaction does not occur in solvay ammonia soda process ?

(a) (NH 4 ) 2 CO3 + H 2O + CO 2 2NH 4 HCO3

(b) 2KHCO3

K 2 CO3 + H 2 O + CO 2

D
(c) 2NaHCO3 Na 2CO3 + H 2O + CO 2

(d) 2NH 4 Cl + Ca(OH)2 2NH3 + CaCl2 + H 2 O

311
(107) What will be final weight of 286 gm Na2CO3.10H2O by Heating at 373 K ?
(a) 206 gm (b) 162 gm (c) 186 gm (d) 124 gm
(108) The order of decreasing polarity in the compounds CaO, CsF, KCl, MgO is
(a) CaO, CsF, KCl, MgO (b) MgO, KCl, CaO, CsF,
(c) KCl, CaO, CsF, MgO (d) CsF, KCl, CaO, MgO
(109) Which is not an ore of Ca ?
(a) Lime stone (b) Flurospar (c) Dolomite (d) Epsomsalt
(110) A certain metal is used to prepare an antacid, this metal accidently catches fire which can not
be put out by using CO2 based extiguishers. The metal is
(a) C (b) Ca (c) Mg (d) Na

ANSWER KEY
(1) d (16) a (31) c (46) d (61) d (76) c (91) b (106) b
(2) c (17) a (32) b (47) c (62) a (77) b (92) d (107) d
(3) d (18) c (33) b (48) d (63) a (78) a (93) a (108) d
(4) a (19) d (34) b (49) c (64) d (79) d (94) a (109) d
(5) a (20) b (35) d (50) a (65) b (80) d (95) c (110) c
(6) b (21) b (36) b (51) a (66) c (81) b (96) c
(7) b (22) c (37) d (52) a (67) a (82) a (97) d
(8) a (23) c (38) c (53) c (68) d (83) d (98) d
(9) c (24) c (39) c (54) a (69) c (84) d (99) a
(10) b (25) c (40) b (55) a (70) c (85) b (100) b
(11) b (26) a (41) b (56) c (71) a (86) d (101) c
(12) a (27) a (42) a (57) b (72) b (87) a (102) a
(13) d (28) a (43) b (58) d (73) d (88) a (103) b
(14) c (29) c (44) d (59) a (74) c (89) b (104) a
(15) d (30) a (45) a (60) c (75) a (90) a (105) d

312
Hints
(1) Melting point decreases as the strength of metallic bonding decreases with increasing size of the
atom.
(2) Due to small size of the Li+, its hydration enthalpy is the highest and hence Li is the strongest
reducing agent.
(3) Thermal stability of metal carbonates increases as the electropositive character of the metal or
the basicity of the metal hydroxide increases from Be(OH)2 to Ba(OH)2, Thus, BaCO3 is the
most stable.
(4) As the ionisation enthalpy increases from Mg Ba the M - O bond becomes weaker and weaker
down the group, and hence basicity increases down the group. Thus, Mg(OH)2 is least basic.
(6) Ionisation enthalpy decreases as the atomic size, increases , so, Li > Na > K > Rb
(7) Due to small size of Li+ and F ions, lattice enthalpy is much higher than hydration enthaalpy and
hence LiF is least soluble among alkali metal Fluorides.
(8) Be(OH)2 being amphoteric dissolves in NaOH.

(9) 2NH 4 Cl(s) + Ca(OH) 2(aq) 2NH3(g) + CaCl2(aq) + H 2 O(1)

D
(14) 2Sr(NO3 )2 2SrO + O 2 + 4NO 2 brown gas. Sr gives Crimson red flame.
(24) The solubility decreases as we move from CaSO4 to BaSO4.
(32) The electrons of Be and Mg are so strongly bonded that they do not get excited in flame.
(35) Li2CO3 is not so stable towards heat, being small in size, it is decompose into Li2O and CO2.
(41) The hydration enthalpy of Be2+ and Mg2+ ions is more than the lattice enthalpy and so they are
soluble in water.
D
(42) CaCO3(s) CO 2(g) + CaO(s)

(A)
CaO(s) + H 2 O(1) Ca(OH) 2(aq)

(B)
Ca(OH) 2(aq) + 2CO 2(g) Ca(HCO3 ) 2(aq)

(C)
D
Ca(HCO3 ) 2(aq) CaCO3(s) + CO 2(g) + H 2 O(l)

(A)
(44) Because of the small size of Li and F, LiF has highest lattice enthalpy and hence most stable.
(47) Alkali metal oxides are most basic followed by alkaline earth metal oxides while transition metal
oxides are least basic. Amongst alkali and alkaline earth metal oxides, basicity increases down
the group. Thus, Cs2O is more basic than K2O and SrO is more basic than MgO. Therefore,
the overall order is :
NiO < MgO < SrO < K2O < Cs2O

313
(48) The solubility of bicarbonates of alkali metals increases down the group but alkaline earth metal
carbonates are insoluble in H2O.
(49) CaO, CaCO3 and Ca(OH)2 are all bases and hence must dissolve in acetic acid to form calcium
acetate only calcium oxalate does not dissolve in CH3COOH.
(53) 2 (Be, Mg)
(54) 6 [K2SO4, BeSO4, Li2SO4, MgSO4, Na2SO4, Rb2SO4]
(55) 1 [Be(OH)2]
(56) 3 [K, Rb, Cs]
(57) 5 [Li, Mg, Ca, Sr, Ba]
(59) Lithium does not form peroxide.
%SiO 2 23
(64) silica mod ule h = = = 4.6
%Al2 O3 + %Fe2 O3 2 + 3

(70) Mg 3 N 2 + 6H 2O 3Mg(OH)2 + 2NH3


1mole 2 moles
2 mole 4 moles
(71) Gypsum = CaSO4 . 2H2O
1
Plaster of paris = CaSO 4 H 2 O
2

1 1
So, difference = 2 =1
2 2

(83) 2Mg + O 2 2MgO


2 24 32gm 2 40gm

2x24 gm 32gm 2 40gm


Here, 48 gm of Mg requires 32 gm of O2 to form 80 gm MgO
so, 30 gm of Mg requires 20 gm of O2 to form 50 gm MgO
so, 10 gm of O2 is remain.

(85) Ca 2+ C- C- 1s and 2p CaC2 : Calcium Carbide

(97) correct statement : 1


Superoxides of alkali metals are paramagnetic.
(98) correct statement : 1
Alkali metals impart characteristic colour to the bunsen flame.

(107) Na 2CO3 10H 2O(s)


373K

Na 2CO3 H 2 O(s)
+ 9H 2O(g)
286 gm 124 gm

(108) According to Fajan rules, polarity increases as the size of the cation increases.

314
NOTES

315
NOTES

316

You might also like